You are on page 1of 212

Ths. L Ths. L Ths. L Ths.

L V VV V n n n n on on on on
www.MATHVN.com
www.DeThiThuDaiHoc.com
MC LC
Trang
PHN I I S
CHNG I MNH & TP HP -------------------------------------------------------------------- 1
A MNH --------------------------------------------------------------------------------------------- 1
B TP HP ---------------------------------------------------------------------------------------------- 6
CHNG II HM S BC NHT & BC HAI ----------------------------------------------------- 12
A I CNG V HM S ------------------------------------------------------------------------ 12
Dng ton 1. Tm tp xc nh ca hm s ------------------------------------------------------ 13
Dng ton 2. Tnh n iu ca hm s --------------------------------------------------------- 16
Dng ton 3. Xt tnh chn l ca hm s ------------------------------------------------------- 18
B HM S BC NHT ------------------------------------------------------------------------------- 20
C HM S BC HAI ---------------------------------------------------------------------------------- 25
CHNG III PHNG TRNH & H PHNG TRNH ---------------------------------------- 36
A I CNG V PHNG TRNH ------------------------------------------------------------- 36
B PHNG TRNH BC NHT ------------------------------------------------------------------- 38
C PHNG TRNH BC HAI ---------------------------------------------------------------------- 43
Dng ton 1. Gii v bin lun phng trnh bc hai ------------------------------------------ 43
Dng ton 2. Du ca s nghim phng trnh bc hai ---------------------------------------- 44
Dng ton 3. Nhng bi ton lin quan n nh l Vit --------------------------------------- 47
Dng ton 4. Phng trnh bc cao quy v phng trnh bc hai ----------------------------- 52
Dng ton 5. Phng trnh cha n trong du tr tuyt i ------------------------------------ 57
Dng ton 6. Phng trnh cha n di du cn ---------------------------------------------- 59
D H PHNG TRNH BC NHT NHIU N ----------------------------------------------- 73
E H PHNG TRNH BC HAI HAI N S -------------------------------------------------- 80
CHNG IV BT NG THC & BT PHNG TRNH ------------------------------------- 106
A BT NG THC --------------------------------------------------------------------------------- 106
Dng ton 1. Chng minh BT da vo nh ngha v tnh cht ---------------------------- 108
Dng ton 2. Chng minh BT da vo BT Cauchy ---------------------------------------- 113
Dng ton 3. Chng minh BT da vo BT Bunhiacpxki -------------------------------- 122
Dng ton 4. Chng minh BT da vo BT Cauchy Schwarz ----------------------------- 125
Dng ton 5. Chng minh BT da vo phng php ta vct ------------------------ 126
Dng ton 6. ng dng BT gii phng trnh --------------------------------------------- 127
PHN II HNH HC
CHNG I VCT & PHP TON ------------------------------------------------------------------- 141
A VCT & CC PHP TON TRN VCT ------------------------------------------------- 141
Dng ton 1. i cng v vct ----------------------------------------------------------------- 143
www.MATHVN.com
www.DeThiThuDaiHoc.com
Dng ton 2. Chng minh mt ng thc vct ------------------------------------------------ 147
Dng ton 3. Xc nh im tha ng thc vct --------------------------------------------- 156
Dng ton 4. Phn tch vct Chng minh thng hng Song song ---------------------- 164
Dng ton 5. Tm mun Qu tch im im c nh ----------------------------------- 177
B H TRC TA --------------------------------------------------------------------------------- 180
Dng ton 1. Ta vct Biu din vct --------------------------------------------------- 181
Dng ton 2. Xc nh im tha mn iu kin cho trc ----------------------------------- 183
Dng ton 3. Vct cng phng v ng dng ------------------------------------------------- 185
CHNG II TCH V HNG & NG DNG ---------------------------------------------------- 190
A GI TR LNG GIC CA MT CUNG GC BT K --------------------------------- 190
B TCH V HNG CA HAI VCT ---------------------------------------------------------- 194
Dng ton 1. Tch v hng Tnh gc Chng minh v thit lp vung gc ----------- 195
Dng ton 2. Chng minh ng thc Bi ton cc tr --------------------------------------- 201
C H THC LNG TRONG TAM GIC ------------------------------------------------------- 207


www.MATHVN.com
www.DeThiThuDaiHoc.com


PHN I

I S



www.MATHVN.com
www.DeThiThuDaiHoc.com
cng hc tp mn Ton 10 tp I Ths. L Vn on


"Cn c b thng minh" Page - 1 -
Chng









C CC C Mnh
Mnh l mt cu khng nh ng hoc mt cu khng nh sai.
Mt mnh khng th va ng, va sai.
C CC C Mnh ph nh
Cho mnh P.
Mnh "khng phi P" c gi l mnh ph nh ca P v k hiu l P .
Nu P ng th P sai, nu P sai th P ng.
C CC C Mnh ko theo
Cho mnh P v Q.
Mnh "Nu P th Q" c gi l mnh ko theo v k hiu l: P Q.
Mnh P Q ch sai khi P ng v Q sai.
Lu rng: Cc nh l ton hc thng c dng P Q. Khi :
P l gi thit, Q l kt lun.
P l iu kin c Q.
Q l iu kin cn c P.
C CC C Mnh o
Cho mnh ko theo P Q. Mnh Q P c gi l mnh o ca mnh P Q.
C CC C Mnh tng ng
Cho mnh P v Q.
Mnh "P nu v ch nu Q" c gi l mnh tng ng v k hiu l P Q.
Mnh P Q ng khi v ch khi c hai mnh P Q v Q P u ng.
Lu rng: Nu mnh P Q l 1 nh l th ta ni P l iu kin cn v c Q.
Mnh cha bin
Mnh cha bin l mt cu khng nh cha bin nhn gi tr trong mt tp X no m
vi mi gi tr ca bin thuc X ta c mt mnh .
C CC C K hiu v
"x X, P(x)".
"x X, P(x)".
Mnh ph nh ca mnh "x X, P(x)" l "x X, P(x) ".
Mnh ph nh ca mnh "x X, P(x)" l "x X, P(x) ".
Php chng minh phn chng
Gi s ta cn chng minh nh l: A B
Cch 1. Ta gi thit A ng. Dng suy lun v cc kin thc ton hc bit
chng minh B ng.
Cch 2. (Chng minh phn chng) Ta gi thit B sai, t chng minh A
sai. Do A khng th va ng va sai nn kt qu l B phi ng.
A MNH
MNH MNH MNH MNH TP HP TP HP TP HP TP HP
1
www.MATHVN.com
www.DeThiThuDaiHoc.com
Ths. L Vn on Phn i S


Page - 2 - "All the flower of tomorrow are in the seeks of today"
BI TP P DNG
Bi 1. Bi 1. Bi 1. Bi 1. Trong cc cu di y, cu no l mnh , cu no l mnh cha bin ?
a/ S 11 l s chn. b/ Bn c chm hc khng ?
c/ Hu l mt thnh ph ca Vit Nam. d/ 2x 3 l mt s nguyn dng.
e/ 2 5 0 < . f/ 4 x 3 = .
g/ Hy tr li cu hi ny !. h/ Paris l th nc .
i/ Phng trnh
2
x x 1 0 = c nghim. k/ 13 l mt s nguyn t.
Bi 2. Bi 2. Bi 2. Bi 2. Trong cc mnh sau, mnh no l ng ? Gii thch ?
a/ Nu a chia ht cho 9 th a chia ht cho 3. b/ Nu a b th
2 2
a b .
c/ Nu a chia ht cho 3 th a chia ht cho 6. d/ S ln hn 2 v nh hn 4.
e/ 2 v 3 l hai s nguyn t cng nhau. f/ 81 l mt s chnh phng.
g/ 5 > 3 hoc 5 < 3. h/ S 15 chia ht cho 4 hoc cho 5.
Bi 3. Bi 3. Bi 3. Bi 3. Trong cc mnh sau, mnh no l ng ? Gii thch ?
a/ Hai tam gic bng nhau khi v ch khi chng c din tch bng nhau.
b/ Hai tam gic bng nhau khi v ch khi chng ng dng v c mt cnh bng nhau.
c/ Mt tam gic l tam gic u khi v ch khi chng c hai ng trung tuyn bng nhau v c
mt gc bng 60
0
.
d/ Mt tam gic l tam gic vung khi v ch khi n c mt gc bng tng ca hai gc cn li.
e/ ng trn c mt tm i xng v mt trc i xng.
f/ Hnh ch nht c hai trc i xng.
g/ Mt t gic l hnh thoi khi v ch khi n c hai ng cho vung gc vi nhau.
h/ Mt t gic ni tip c ng trn khi v ch khi n c hai gc vung.
Bi 4. Bi 4. Bi 4. Bi 4. Trong cc mnh sau, mnh no l ng ? Gii thch ? Pht biu cc mnh thnh li ?
a/
2
x , x 0 R . b/
2
x , x x R .
c/
2
x , 4x 1 0 = . d/
2
n , n n N .
e)
2
x , x x 1 0 = R . f/
2
x , x 9 x 3 R .
g/
2
x , x 3 x 9 R . h/
2
x , x 5 x 5 < < R .
i/
2
x , 5x 3x 1 R . k/
2
x , x 2x 5 R l hp s.
l/
2
n , n 1 N khng chia ht cho 3. m/
*
n , n(n 1) N l s l.
n/
*
n , n(n 1)(n 2) N chia ht cho 6. o/
*
n , N
3
n 11n chia ht cho 6.
Bi 5. Bi 5. Bi 5. Bi 5. in vo ch trng t ni "v" hay "hoc" c mnh ng ?
a/ 4............ 5 < .
b/ ab 0 khi a 0............ b 0 = = = .
c/ ab 0 khi a 0............b 0 .
d/ ab 0 khi a 0............ b 0............a 0............ b 0 < < .
e/ Mt s chia ht cho 6 khi v ch khi n chia ht cho 2 cho 3.
f/ Mt s chia ht cho 5 khi v ch khi ch s tn cng ca n bng 0 bng 5.
Bi 6. Bi 6. Bi 6. Bi 6. Cho mnh cha bin
( )
P x , vi x R. Tm x
( )
P x l mnh ng ?
a/
( )
x
2
P x : " x 5 4 0" = . b/
( )
2
P x : " x 5x 6 0" = .
www.MATHVN.com
www.DeThiThuDaiHoc.com
cng hc tp mn Ton 10 tp I Ths. L Vn on


"Cn c b thng minh" Page - 3 -
c/
( )
2
P x : " x 3x 0" . d/
( )
P x : " x x" .
e/
( )
P x : " 2x 3 7 " . f/
( )
2
P x : " x x 1 0" .
Bi 7. Bi 7. Bi 7. Bi 7. Nu mnh ph nh ca cc mnh sau:
a/ S t nhin n chia ht cho 2 v cho 3.
b/ S t nhin n c ch s tn cng bng 0 hoc bng 5.
c/ T gic T c hai cnh i va song song va bng nhau.
d/ S t nhin n c c s bng 1 v bng n.
Bi 8. Bi 8. Bi 8. Bi 8. Nu mnh ph nh ca cc mnh sau:
a/
2
x : x 0 R b/
2
x : x x R .
c/
2
x : 4x 1 0 = . d/
2
x : x x 7 0 R .
e/
2
x : x x 2 0 < R . f/
2
x : x 3 = R .
g/
2
n , n 1 N khng chia ht cho 3. h/
2
n , n 2n 5 N l s nguyn t.
i/
2
n , n n N chia ht cho 2. k/
2
n , n 1 N l s l.
Bi 9. Bi 9. Bi 9. Bi 9. Pht biu cc mnh sau, bng cch s dng khi nim "iu kin cn", "iu kin ":
a/ Nu mt s t nhin c ch s tn cng l ch s 5 th n chia ht cho 5.
b/ Nu a b 0 th mt trong hai s a v b phi dng.
c/ Nu mt s t nhin chia ht cho 6 th n chia ht cho 3.
d/ Nu a b = th
2 2
a b = .
e/ Nu a v b cng chia ht cho c th a b chia ht cho c.
Bi 10. Bi 10. Bi 10. Bi 10. Pht biu cc mnh sau, bng cch s dng khi nim "iu kin cn", "iu kin ":
a/ Trong mt phng, nu hai ng thng phn bit cng vung gc vi mt ng thng th ba
th hai ng thng y song song vi nhau.
b/ Nu hai tam gic bng nhau th chng c din tch bng nhau.
c/ Nu t gic T l mt hnh thoi th n c hai ng cho vung gc vi nhau.
d/ Nu t gic H l mt hnh ch nht th n c ba gc vung.
e/ Nu tam gic K u th n c hai gc bng nhau.
Bi 11. Bi 11. Bi 11. Bi 11. Pht biu cc mnh sau, bng cch s dng khi nim "iu kin cn v ":
a/ Mt tam gic l vung khi v ch khi n c mt gc bng tng hai gc cn li.
b/ Mt t gic l hnh ch nht khi v ch khi n c ba gc vung.
c/ Mt t gic l ni tip c trong ng trn khi v ch khi n c hai gc i b nhau.
d/ Mt s chia ht cho 6 khi v ch khi n chia ht cho 2 v cho 3.
e/ S t nhin n l s l khi v ch khi n
2
l s l.
Bi 12. Bi 12. Bi 12. Bi 12. Chng minh cc mnh sau bng phng php phn chng:
a/ Nu a b 2 < th mt trong hai s a v b nh hn 1.
b/ Mt tam gic khng phi l tam gic u th n c t nht mt gc nh hn 60
0
.
c/ Nu x 1 v y 1 th x y xy 1 .
d/ Nu bnh phng ca mt s t nhin n l mt s chn th n cng l mt s chn.
e/ Nu tch ca hai s t nhin l mt s l th tng ca chng l mt s chn.
f/ Nu 1 t gic c tng cc gc i din bng 2 gc vung th t gic ni tip c ng trn.
g/ Nu
2 2
x y 0 = th x 0 = v y 0 = .

www.MATHVN.com
www.DeThiThuDaiHoc.com
Ths. L Vn on Phn i S


Page - 4 - "All the flower of tomorrow are in the seeks of today"
BI TP RN LUYN
Bi 13. Bi 13. Bi 13. Bi 13. Trong cc cu sau, cu no l mnh , cu no khng l mnh ? Nu l mnh th n l
mnh ng hay sai ?
a/ Cc em c vui khng ?
b/ Cm hc sinh ni chuyn trong gi hc !
c/ Phng trnh
2
x x 0 = c hai nghim dng phn bit.
d/
5
2 1 l mt s nguyn t.
e/ 2 l mt s v t.
f/ Thnh ph H Ch Minh l th ca nc Vit Nam.
g/ Mt s t nhin chia ht cho 2 v 4 th s chia ht cho 8.
h/ Nu
2003
2 1 l s nguyn t th 16 l s chnh phng.
Bi 14. Bi 14. Bi 14. Bi 14. Vit mnh ph nh ca mi mnh sau v xt xem mnh ph nh ng hay sai ?
a/ 3,15 < . b/ 125 0 .
c/ 3 l s nguyn t. d/ 7 khng chia ht cho 5.
e/ l s hu t. f/ 1794 chia ht cho 3.
g/ 2 l s hu t. h/ Tng 2 cnh 1 ln hn cnh th 3.
Bi 15. Bi 15. Bi 15. Bi 15. Pht biu thnh li cc mnh sau v xt tnh ng sai ca cc mnh :
a/
2
x , x 0 R . b/
2
n , n n = N .
c/ n , n 2n N . d/ x , x 0 < R .
e/ x , 1, 2 x 2,1 < < N . f/
2
n , n 1 N chia ht cho 3.
Bi 16. Bi 16. Bi 16. Bi 16. Cc mnh sau y ng hay sai ? Gii thch ? Vit mnh ph nh ca chng ?
a/
2
n , n 2 = . b/
2
x , x x R .
c/
2
x , x x R . d/
2
n , n n N .
e/
2
n , n n N . f/
2
x , x x 1 0 R .
g/
2
x , x x 1 0 R h/
2
n , n 1 N khng chia ht cho 3.
i/
2
n , n 1 N khng chia ht cho 3. j/
2
n , n 1 N chia ht cho 4.
Bi 17. Bi 17. Bi 17. Bi 17. Cho mnh cha bin
( )
2
P x : " x x" = . Xc nh tnh ng sai ca cc mnh sau:
( ) ( ) ( ) ( ) ( )
P 0 ; P 1 ; P 1 ; " x , P x "; " x , P x " R R .
Bi 18. Bi 18. Bi 18. Bi 18. Cho mnh cha bin
( )
3
P x : " x 2x 0" = . Xc nh tnh ng sai ca cc mnh sau:
( ) ( )
( )
( ) ( )
P 0 ; P 2 ; P 2 ; " x , P x "; " x , P x " R R .
Bi 19. Bi 19. Bi 19. Bi 19. Cc mnh sau ng hay sai ? Nu sai hy sa li c mt mnh ng ?
a/
2
x 1 x 1 = = . b/ 2001 l s nguyn t.
c/
2
x , x x R . d/
2 2
x , x y 2xy R .
e/
2
x , x x N . f/
2
n , n n 1 7 N .
b/ ABCD l hnh vung ABCD l hnh bnh hnh.
c/ ABCD l hnh thoi ABCD l hnh ch nht.
d/ T gic MNPQ l hnh vung Hai ng cho MP v NQ bng nhau.
e/ Hai tam gic bng nhau Chng c din tch bng nhau.
www.MATHVN.com
www.DeThiThuDaiHoc.com
cng hc tp mn Ton 10 tp I Ths. L Vn on


"Cn c b thng minh" Page - 5 -
Bi 20. Bi 20. Bi 20. Bi 20. Dng bng chn tr hy chng minh:
a/
( )
( )
A B A B = . b/
( )
A B A A
l
=
l
l
.
c/
( )
( ) ( )
A B A B B A = = . d/
( ) ( )
A B B A B
l
=
l
l
.
e/
( ) ( )
A B A B = . f/
( ) ( )
A B A B = .
i/
( ) ( ) ( )
A B C A B A C
l l
=
l l
l l
. j/
( )
( )
A B C A B C
l
=
l
l
.
Bi 21. Bi 21. Bi 21. Bi 21. Vi n l s t nhin l, xt nh l: " Nu n l s t nhin l th
2
n 1 chia ht cho 8". nh l
trn c vit di dng
( ) ( )
P n Q n .
a/ Hy xc nh mnh
( )
P n v
( )
Q n .
b/ Pht biu nh l trn bng cch s dng thut ng "iu kin " v " iu kin cn".
Bi 22. Bi 22. Bi 22. Bi 22. Cho nh l: " Nu n l s t nhin th
3
n n chia ht cho 3". nh l trn c vit di dng
( ) ( )
P n Q n .
a/ Hy xc nh mnh
( )
P n v
( )
Q n .
b/ Pht biu nh l trn bng cch s dng thut ng "iu kin " v " iu kin cn".
c/ Chng minh nh l trn.
Bi 23. Bi 23. Bi 23. Bi 23. S dng thut ng "iu kin " pht biu cc nh l sau:
a/ Nu mt t gic l hnh bnh hnh th n c hai ng cho ct nhau ti trung im ca mi
ng.
b/ Nu mt hnh thoi c hai ng cho bng nhau th n l hnh vung.
c/ Nu
( )
2
ax bx c 0, a 0 = c
2
b 4ac 0 th phng trnh c 2 nghim phn bit.
d/ Nu x 2 th
2
x 4 .
Bi 24. Bi 24. Bi 24. Bi 24. S dng thut ng "iu kin cn" pht biu cc nh l sau:
a/ Nu x 5 th
2
x 25 .
b/ Nu hai gc i nh th chng bng nhau.
c/ Nu hai tam gic bng nhau th din tch ca chng bng nhau.
d/ Nu a l s t nhin v a chia ht cho 6 th a chia ht cho 3.
Bi 25. Bi 25. Bi 25. Bi 25. Cho hai mnh , mnh A: "a v b l hai s t nhin l" v mnh B: " a b l s chn".
a/ Pht biu mnh A B . Mnh ny ng hay sai ?
b/ Pht biu mnh B A . Mnh ny ng hay sai ?
Bi 26. Bi 26. Bi 26. Bi 26. Chng minh cc mnh sau bng phng php phn chng.
a/ Nu tng ca 99 s bng 100 th c t nht mt s ln hn 1.
b/ Nu a v b l cc s t nhin vi tch a.b l th a v b l cc s t nhin l.
c/ Cho a, b, c R . C t nht mt trong ba ng thc sau l ng:

2 2 2 2 2 2
a b 2bc; b c 2ac; c a 2ab .
d/ Vi cc s t nhin a v b, nu
2 2
a b chia ht cho 8 th a v b khng th ng thi l s l.
e/ Nu nht 25 con th vo trong 6 ci chung th c t nht 1 chung cha nhiu hn 4 con th.
Bi 27. Bi 27. Bi 27. Bi 27. Cho nh l: " Nu a v b l hai s nguyn dng v mi s u chia ht cho 3 th
2 2
a b cng
chia ht cho 3". Hy pht biu v chng minh nh l o ca nh l trn (nu c), ri dng thut
ng "iu kin cn v " gp c hai nh l thun v o.

www.MATHVN.com
www.DeThiThuDaiHoc.com
Ths. L Vn on Phn i S


Page - 6 - "All the flower of tomorrow are in the seeks of today"






















































(
////////// //////////
l
l
l


+


C CC C Tp hp
Tp hp l mt khi nim c bn ca ton hc, khng nh ngha.
Cch xc nh tp hp.
+ Lit k cc phn t: vit cc phn t ca tp hp trong hai du mc { }.
+ Ch ra tnh cht c trng cho cc phn t ca tp hp.
Tp rng: l tp hp khng cha phn t no, k hiu .
C CC C Tp hp con Tp hp bng nhau
Tp hp con:
( )
A B x A x B .
+ A A, A .
+ A, A .
+ A B, B C A C .
Tp hp bng nhau:
A B
A B
B A
'
1
1
=
!
1
1
+
. Nu tp hp c n phn t
n
2 tp hp con.
C CC C Mt s tp hp con ca tp hp s thc R
Tp hp con ca R:
*
N N Z R .
Khong:
+
( )
a; b x / a x b = < < R
+
( )
a; x / a x = < R
+
( )
; b x / x b = < R
on:

a; b x / a x b
l
=
l
l
R
Na khong:
+
)
a; b x / a x b

= <

R
+
(
a; b x / a x b
l
= <
l
l
R
+
)
a; x / a x

R
+
(
; b x / x b
l
=
l
l
R
C CC C Cc php ton tp hp
Giao ca hai tp hp: A B { x x A v x B }.
Hp ca hai tp hp: A B { x x A hoc x B }.
Hiu ca hai tp hp: A \ B { x x A v x B }.
Phn b: Cho B A th \
A
C B A B = .
A B
( )
////////// //////////
a
b
+

)
////////// //////////

a
b
+


+ ////////// (

+ ////////// [
////////// //////////
l
l
l


+


+ ) //////////

+
] //////////
A B
D
A
B
A
B
B TP HP
www.MATHVN.com
www.DeThiThuDaiHoc.com
cng hc tp mn Ton 10 tp I Ths. L Vn on


"Cn c b thng minh" Page - 7 -
BI TP P DNG
Bi 28. Bi 28. Bi 28. Bi 28. Vit mi tp hp sau bng cch lit k cc phn t ca n.
a/
( )( )

2 2
A x 2x 5x 3 x 4x 3 0 = = R .
b/
( )( )

2 3
B x x 10x 21 x x 0 = = R .
c/
( )( )

2 2
C x 6x 7x 1 x 5x 6 0 = = R .
d/

2
D x 2x 5x 3 0 = = Z .
e/

E x x 3 4 2x ; 5x 3 4x 1 = < < N .
f/

F x x 2 1 = Z .
g/

G x x 5 = < N .
h/

2
H x x x 3 0 = = R .
i/
a
1 1
K x Q x , a N
32 2
' '
1 1
1 1
= =
! !
1 1
1 1
+ +
.
Bi 29. Bi 29. Bi 29. Bi 29. Vit mi tp hp sau bng cch ch r tnh cht c trng cho cc phn t ca n:
a/ A 0; 1; 2; 3; 4 = . b/ B 0; 4; 8; 12; 16 = .
c/ C 3 ; 9; 27; 81 = . d/ D 9; 36; 81; 144 = .
e/ E 2; 3; 5; 7; 11 = . f/ F 3; 6; 9; 12; 15 = .
g/

G 0; 3; 8;15; 24; 35; 48; 63 = . h/
1 1 1 1 1
H 1; ; ; ; ;
3 9 27 81 234
' '
1 1
1 1
=
! !
1 1
1 1 + +
.
i/
1 1 1 1 1
I ; ; ; ;
2 6 12 20 30
' '
1 1
1 1
=
! !
1 1
1 1 + +
. j/
2 3 4 5 6
J ; ; ; ;
3 8 15 24 35
' '
1 1
1 1
=
! !
1 1
1 1 + +
.
k/

K 4; 3; 2; 1; 0;1;2; 3; 4; 5 = . l/

L 3, 8,15, 24, 35, 48, 63 = .
m/
2 3 4 5 6 7 8
M 1, , , , , , ,
3 5 7 9 11 13 15
' '
1 1
1 1
=
! !
1 1
1 1 + +
. n/

N 3, 4, 7,12,19, 28, 39, 52 = .
o/

O 0, 3, 2 2, 15, 2 6, 35, 4 3, 63 = . p/
1 2 3 4 5 6 7 8 9
P 0, , , , , , , , ,
2 3 4 5 6 7 8 9 10
' '
1 1
1 1
=
! !
1 1
1 1 + +
.
q/ Q = Tp tt c cc im thuc ng trung trc ca on thng AB.
r/ R = Tp tt c cc im thuc ng trn tm I cho trc v c bn knh bng 5.
Bi 30. Bi 30. Bi 30. Bi 30. Trong cc tp hp sau y, tp no l tp rng ?
a/

A x x 1 = < Z . b/

2
B x x x 1 0 = = R .
c/

2
C x x 4x 2 0 = = . d/

2
D x x 2 0 = = .
e/

2
E x x 7x 12 0 = = N . f/

2
F x x 4x 2 0 = = R .
Bi 31. Bi 31. Bi 31. Bi 31. Tm tt c cc tp con, cc tp con gm hai phn t ca cc tp hp sau:
a/

A 1; 2 = . b/

B 1; 2; 3 = .
www.MATHVN.com
www.DeThiThuDaiHoc.com
Ths. L Vn on Phn i S


Page - 8 - "All the flower of tomorrow are in the seeks of today"
c/

2
C x 2x 5x 2 0 = = R . d/

2
D x x 4x 2 0 = = .
Bi 32. Bi 32. Bi 32. Bi 32. Trong cc tp hp sau, tp no l tp con ca tp no ?
a/
( )

2
A 1; 2; 3 , B x x 4 , C 0; , D x 2x 7x 3 0 = = < = = = N R .
b/ A =Tp cc c s t nhin ca 6; B =Tp cc c s t nhin ca 12.
c/ A =Tp cc hnh bnh hnh; B =Tp cc hnh ch nht;
C=Tp cc hnh thoi; D=Tp cc hnh vung.
d/ A =Tp cc tam gic cn; B =Tp cc tam gic u;
C= Tp cc tam gic vung; D=Tp cc tam gic vung cn.
Bi 33. Bi 33. Bi 33. Bi 33. Tm A B; A B; A\ B; B\ A vi:
a/

A 2, 4, 7, 8, 9,12 ; B 2, 8, 9,12 = = .
b/

A 2, 4, 6, 9 ; B 1, 2, 3, 4 = = .
c/

2
A x 2x 3x 1 0 ; B x 2x 1 1 = = = = R R .
d/ A = Tp cc c s ca 12 ; B = Tp cc c s ca 18.
e/
( )( )( )

2
A x x 1 x 2 x 8x 15 0 = = R ; B =Tp cc s nguyn t c 1 ch s.
f/
( )( )

2 2 2
A x x 4 ; B x 5x 3x x 2x 3 0 = < = = Z Z .
g/ A =
( )( )

x
2 2
x x 9 x 5 6 0 = N ; B ={ x N/x l s nguyn t, x 5}.
Bi 34. Bi 34. Bi 34. Bi 34. Tm tt c cc tp hp X sao cho:
a/

1, 2 X 1, 2, 3, 4, 5 .
b/

1, 2 X 1, 2, 3, 4 = .
c/

X 1, 2, 3, 4 , X 0, 2, 4, 6, 8 .
Bi 35. Bi 35. Bi 35. Bi 35. Xc nh cc tp hp A, B sao cho:
a/

; A B 0,1, 2, 3, 4 A \ B 3, 2 ; B \ A 6, 9,10 = = = .
b/

; A B 1, 2, 3 A \ B 4, 5 ; B \ A 6, 9 = = = .
Bi 36. Bi 36. Bi 36. Bi 36. Xc nh A B; A B; A\ B; B\ A v biu din chng trn trc s, vi:
a/ A 4; 4 , B 1; 7
l l
= =
l l
l l
. b/
(
A 4; 2 , B 3; 7
l l
= =
l l
l l
.
c/
( )
A 4; 2 , B 3; 7
l
= =
l
l
. d/
( )
A ; 2 , B 3;
l
= =
l
l
.
e/
) ( )
A 3; , B 0; 4

= =

. f/
( ) ( )
A 1; 4 , B 2; 6 = = .
Bi 37. Bi 37. Bi 37. Bi 37. Xc nh A B C; A B C

v biu din chng trn trc s, vi:
a/
( ) ( )
A 1; 4 , B 2; 6 , C 1;2
l
= = =
l
l
. b/
( ) ( )
A ; 2 , B 3; , C 0;4
l
= = =
l
l
.
c/
( ) (
A 0; 4 , B 1, 5 , C 3;1
l l
= = =
l l
l l
. d/
( ) ( )
A ; 2 , B 2; , C 0; 3
l
= = =
l
l
.
e/
( ) ( )
A 5;1 , B 3; , C ; 2
l
= = =
l
l
. f/
( ( ) )
A 2; 5 , B 0; 9 , C ; 6
l
= = =
l
l
.
Bi 38. Bi 38. Bi 38. Bi 38. Chng minh rng:
a/ Nu A B th A B A = . b/ Nu A C v B C th
( )
A B C .
c/ Nu A B A B = th A B = . d/ Nu A B v A C th
( )
A B C .
Bi 39. Bi 39. Bi 39. Bi 39. Mi hc sinh lp 10A
1
u chi bng hoc bng chuyn. Bit rng c 25 bn chi bng ,
www.MATHVN.com
www.DeThiThuDaiHoc.com
cng hc tp mn Ton 10 tp I Ths. L Vn on


"Cn c b thng minh" Page - 9 -
20 bn chi bng chuyn v 10 bn chi c hai mn th thao ny. Hi lp 10A
1
c bao nhiu hc
sinh ?
Bi 40. Bi 40. Bi 40. Bi 40. Trong mt trng THPT, khi 10 c: 160 em hc sinh tham gia cu lc b Ton, 140 tham gia
cu lc b Tin, 50 em tham gia c hai cu lc b. Hi khi 10 c bao nhiu hc sinh ?
Bi 41. Bi 41. Bi 41. Bi 41. Mt lp c 40 HS, ng k chi t nht mt trong hai mn th thao: bng v cu lng. C 30
em ng k mn bng , 25 em ng k mn cu lng. Hi c bao nhiu em ng k c hai
mn th thao ?
Bi 42. Bi 42. Bi 42. Bi 42. Cho cc tp hp

A a, b, c, d ; B b, d, e ; C a, b, e = = = . Chng minh cc h thc
a/
( ) ( ) ( )
A B \ C A B \ A C = . b/
( ) ( ) ( )
A \ B C A \ B A \ C = .
Bi 43. Bi 43. Bi 43. Bi 43. Tm cc tp hp A v B. Bit rng:

A \ B 1, 5, 7, 8 = ;

A B 3, 6, 9 = v

A B x 0 x 10 = < N .
Bi 44. Bi 44. Bi 44. Bi 44. Cho cc tp hp:

A 1, 2, 3, 4, 5, 6, 7, 8, 9 ; B 1, 2, 3, 4 ; C 2, 4, 6, 8 = = = . Hy xc nh:
( )

A A A
C B, C C, C B C .
Bi 45. Bi 45. Bi 45. Bi 45. Cho cc tp hp

A x 3 x 2 , B x 0 x 7 = = < R R

, C x x 1 = < R
v

D x x 5 = R .
a/ Dng k hiu on, khong, na khong vit li cc tp hp trn.
b/ Biu din cc tp hp A, B, C v D trn trc s. Ch r n thuc phn no trn trc s.
Bi 46. Bi 46. Bi 46. Bi 46. Xc nh mi tp hp sau v biu din chng trn trc s
a/
( ) ( )
5; 3 0; 7 . b/
( ) ( )
1; 5 3; 7 .
c/
( )
\ 0; R . d/ \ 0;1
l
l
l
R .
e/
( ) ( )
; 3 2; . f/
( )
1; 3 0; 5
l

l
l
.


BI TP RN LUYN
Bi 47. Bi 47. Bi 47. Bi 47. Vit cc tp hp sau bng phng php lit k
a/
( )( )
2 2
A x / 2x x 2x 3x 2 0 = = b/

2
B n / 3 n 30 = < < N .
c/

4 2
C x / x 5x 6 0 = = R . d/

2
D n / 0 n 30 = < < Z .
Bi 48. Bi 48. Bi 48. Bi 48. Vit cc tp sau bng phng php nu ra tnh cht c trng
a/

A 1, 2, 3, 4, 5, 6, 7, 8, 9 = . b/

A 0, 2, 4, 6, 8,10 = .
c/

A 3, 2, 1, 0,1, 2, 3 = . d/

A 1, 4, 7,10,13,16,19 = .
e/

A 1, 2, 4, 8,16, 32, 64,128, 256, 512 = . f/ Tp hp cc s chn.
g/ Tp hp cc s l. h/ ng phn gic trong ca

ABC.
i/ ng trn tm I, bn knh R. j/ ng trn ng knh AB.
k/

A 2,1, 6,13, 22, 33, 46, 61 = . l/

A 3, 8, 24, 35, 48, 63, 80, 99 = .
m/
1 2 3 4 5 6
A 0, , , , , ,
3 9 19 33 73 99
' '
1 1
1 1
=
! !
1 1
1 1 + +
. n/
2 10 17 26 37 10
A ,1, , , , ,
3 7 9 11 13 3
' '
1 1
1 1
=
! !
1 1
1 1 + +
.
Bi 49. Bi 49. Bi 49. Bi 49. Cho tp hp

A 1, 2, 3, 4 = .
www.MATHVN.com
www.DeThiThuDaiHoc.com
Ths. L Vn on Phn i S


Page - 10 - "All the flower of tomorrow are in the seeks of today"
a/ Lit k tt c cc tp hp con c 3 phn t ca A.
b/ Lit k tt c tp con c 2 phn t ca A.
c/ Lit k tt c cc tp con ca A.
Bi 50. Bi 50. Bi 50. Bi 50. Biu din cc tp hp sau thnh cc khong
a/

A x / 2 x 3 = < < R . b/

B x / x 4 = R .
c/
2
C x / 3
x 1
' '
1 1
1 1
1 1
=
! !
1 1

1 1
1 1 + +
R . d/
5
D x / 4
x 7
' '
1 1
1 1
1 1
=
! !
1 1

1 1
1 1 + +
R .
Bi 51. Bi 51. Bi 51. Bi 51. Xt cc quan h " " gia cc tp hp sau
a/

A 1, 2, 3, 4, 5 = v

B n / 0 n 5 = Z .
b/
( )( )
2
A x / x x 2 x 1 0 = = Z v

2
B x / x x 2 0 = = R .
c/

A x / 2 x 4 = < < R v

B x / 4 x 3 = < < N .
Bi 52. Bi 52. Bi 52. Bi 52. Cho

A 1, 2, 3, 4, 5 = v

B 1, 3, 5, 7, 9,11 = . Hy tm:
a/ C A B = . b/ C A B = .
c/
( ) ( )
C A B \ A B = . d/
( ) ( )
C A \ B B \ A = .
Bi 53. Bi 53. Bi 53. Bi 53. Cho

A x / 1 x 5 = < R v

B x / 0 x 7 = < R . Hy tm tm hp C tha:
a/ C A B = . b/ C A B = .
c/
( ) ( )
C A B \ A B = . d/
( ) ( )
C A \ B B \ A = .
Bi 54. Bi 54. Bi 54. Bi 54. Cho

A x / 3 x 3 = < < R ,

B x / 2 x 3 = < R v

C x / 0 x 4 = R .
Hy tm tp hp D tha:
a/
( )
D A B C = . b/
( )
D A B C = .
c/
( )
D A B C = . d/
( )
D A B C = .
e/
( )
D A B \ C = . f/
( ) ( )
D A \ B A \ C = .
g/
( ) ( )
D B \ A C\ A = . h/
( )
D B \ A \ C = .
i/
( )
D B \ A C = . j/
( )
D B C \ A = .
Bi 55. Bi 55. Bi 55. Bi 55. Cho

A x / 5 x hay x 5 = R ,

B x / 10 x 4 = < < R v

C x / 1 x 9 = < R . Hy tm tp hp D tha:
a/
( )
D A B C = . b/
( )
D A B C = .
c/
( )
D A B C = . d/
( )
D A B C = .
e/
( )
D A B \ C = . f/
( ) ( )
D A \ B A \ C = .
g/
( ) ( )
D B \ A C\ A = . h/
( )
D B \ A \ C = .
i/
( )
D B \ A C = . j/
( )
D B C \ A = .
Bi 56. Bi 56. Bi 56. Bi 56. Cho
1
A x / 2
x 2
' '
1 1
1 1
1 1
=
! !
1 1

1 1
1 1 + +
R v

B x / x 1 1 = < R . Hy tm cc tp hp:
( ) ( )
A B, A B, A \ B B \ A .
www.MATHVN.com
www.DeThiThuDaiHoc.com
cng hc tp mn Ton 10 tp I Ths. L Vn on


"Cn c b thng minh" Page - 11 -
Bi 57. Bi 57. Bi 57. Bi 57. Chng minh rng
a/ A B C . b/ B A C .
c/ A B B A = . d/
( ) ( )
A B C A B C = .
e/ A B B A B = . f/ A B A .
g/ A B B . h/ A B B A = .
i/
( ) ( )
A B C A B C = . j/ A B B B A = .
k/ A\ B A . l/ B\ A B .
m/ A B A B . n/
( ) ( ) ( )
A B C A B A C = .
o/
( ) ( ) ( )
A B C A B A C = . p/
( )
A \ B A \ A B = .
r/ A\ B A B = . s/ Nu A B th A B A = .
Bi 58. Bi 58. Bi 58. Bi 58. Xc nh mi tp hp s sau v biu din chng trn trc s
a/
( ) ( )
3; 3 1; 0 . b/
( ) ( )
; 0 0;1 .
c/
( )
2; 2 1; 3
l

l
l
. d/
( ) ( )
3; 3 \ 0; 5 .
e/
( ) ( )
5; 5 \ 3; 3 . f/
( ) ( )
2; 3 \ 3; 3 .
g/

A x x 3 = R . h/

B x x 5 = < R .
Bi 59. Bi 59. Bi 59. Bi 59. Xc nh cc tp hp A B, A B v biu din chng trn trc s
a/
( ) ( )
A 1; 5 , B 3; 2 3; 7
l
= =
l
l
. b/
( ) ( ) ( ) ( )
A 5; 0 3; 5 , B 1;2 4; 6 = = .
c/

A x x 1 2 , B x x 1 3 = < = < R R .
Bi 60. Bi 60. Bi 60. Bi 60. Cho hai tp hp A v B. Bit tp hp B khc rng, s phn t ca tp B gp i s phn t ca
tp A B v A B c 10 phn t. Hi tp A v B c bao nhiu phn t. Hy xt cc trng
hp xy ra v dng biu Ven minh ha.
Bi 61. Bi 61. Bi 61. Bi 61. Trong 100 hc sinh lp 10, c 70 hc sinh ni c ting Anh, 45 hc sinh ni c ting Php
v 23 hc sinh ni c c hai ting Anh v Php. Hi c bao nhiu hc sinh khng ni c hai
ting Anh v Php.
Bi 62. Bi 62. Bi 62. Bi 62. Tm phn b ca tp hp cc s t nhin trong tp hp cc s nguyn ?

www.MATHVN.com
www.DeThiThuDaiHoc.com
Ths. L Vn on Phn i S


Page - 12 - "All the flower of tomorrow are in the seeks of today"
Chng































BA DNG TON THNG GP
DNG 1. Tm tp xc nh ca hm s.
DNG 2. Xt tnh n iu ca hm s.
DNG 3. Xt tnh chn l ca hm s.
A I CNG V HM S

C CC C nh ngha
Cho D , D R . Hm s f xc nh trn D l mt qui tc t tng ng mi s x D
vi mt v ch mt s y R.
x c gi l bin s (i s), y c gi l gi tr ca hm s f ti x. K hiu:
( )
y f x = .
D c gi l tp xc nh ca hm s.

( )
T y f x x D = = c gi l tp gi tr ca hm s.
C CC C Cch cho hm s
Cho bng bng.
Cho bng biu .
Cho bng cng thc
( )
y f x = .
Tp xc nh ca hm s
( )
y f x = l tp hp tt c cc s thc x sao cho biu thc
( )
f x c
ngha.
C CC C th ca hm s
th ca hm s
( )
y f x = xc nh trn tp D l tp hp tt c cc im
( ) ( )
M x; f x trn
mt phng to vi mi x D .
Ch : Ta thng gp th ca hm s
( )
y f x = l mt ng. Khi ta ni
( )
y f x = l
phng trnh ca ng .
C CC C Tnh chn l ca hm s
Cho hm s
( )
y f x = c tp xc nh D.
Hm s f c gi l hm s chn nu x D th x D v
( ) ( )
f x f x = .
Hm s f c gi l hm s l nu x D th x D v
( ) ( )
f x f x = .
Lu :
+ th ca hm s chn nhn trc tung Oy lm trc i xng.
+ th ca hm s l nhn gc to O lm tm i xng.
HM S BC NHT HM S BC NHT HM S BC NHT HM S BC NHT V BC HAI V BC HAI V BC HAI V BC HAI
2
www.MATHVN.com
www.DeThiThuDaiHoc.com
cng hc tp mn Ton 10 tp I Ths. L Vn on


"Cn c b thng minh" Page - 13 -














BI TP P DNG
Bi 64. Bi 64. Bi 64. Bi 64. Tnh gi tr ca cc hm s sau ti cc im ch ra
a/
( )
f x 5x = . Tnh
( ) ( ) ( ) ( )
f 0 , f 2 , f 2 , f 3 .
b/
( )
2
x 1
f x
2x 3x 1

=

. Tnh
( ) ( ) ( ) ( )
( )
f 2 , f 0 , f 2 , f 3 , f 2 .
c/
( )
f x 2 x 1 3 x 2 = . Tnh
( ) ( ) ( ) ( )
( ) ( )
f
1
f 2 , f 0 , f 2 , f 3 , , f 3 , f 1 2
2
1


( )
.
d/
( )



2
2
khi x 0
x 1
f x x 1 khi 0 x 2
x 1 khi x 2
'
1
1
<
1
1

1
1
!
=
1
1
1
1

1
1+
. Tnh
( ) ( ) ( ) ( )
( )
f 2 , f 0 , f 2 , f 3 , f 2 .
Bi 65. Bi 65. Bi 65. Bi 65. Tm tp xc nh ca cc hm s sau
a/ y 2 4x = . b/
2
y x 4x 15 = . c/
3
2x 3x 1
y
2013

= .
d/
2x 1
y
3x 2

. e/
x 3
y
5 2x

. f/
4
y
x 4
=

.
g/
2
x
y
x 3x 2
=

. h/
2
x 1
y
2x 5x 2

=

. i/
2
3x
y
x x 1
=

.
j/
3
x 1
y
x 1

. k/
( )( )
2
2x 1
y
x 2 x 4x 3

=

. l/
4 2
1
y
x 2x 3
=

.
Bi 66. Bi 66. Bi 66. Bi 66. Tm tp xc nh ca cc hm s sau
Dng ton 1. Tm tp xc nh ca hm s
Tm tp xc nh D ca hm s y = f(x) l tm tt c nhng gi tr ca bin s x sao cho biu
thc f(x) c ngha: D={
( )
x f x R } c ngha.
Ba trng hp thng gp khi tm tp xc nh
+ Hm s
( )
( )

P x
y
Q x
= iu kin xc nh
( )
Q x 0 .
+ Hm s
( )
y P x = iu kin xc nh
( )
P x 0 .
+ Hm s
( )
( )

P x
y
Q x
= iu kin xc nh
( )
Q x 0 .
Lu
+ i khi ta s dng phi hp cc iu kin vi nhau.
+ iu kin hm s xc nh trn tp A l A D .
+
A 0
A.B 0
B 0
'
1
1

!
1
1
+

www.MATHVN.com
www.DeThiThuDaiHoc.com
Ths. L Vn on Phn i S


Page - 14 - "All the flower of tomorrow are in the seeks of today"
a/ y 2x 3 = . b/ y 2x 3 = . c/ y 4 x x 1 = .
d/
1
y x 1
x 3
=

. e/
( )
1
y
x 2 x 1
=

. f/ y x 3 2 x 2 = .
g/
( )
5 2x
y
x 2 x 1

=

. h/
1
y 2x 1
3 x
=

. i/
2
1
y x 3
x 4
=

.
Bi 67. Bi 67. Bi 67. Bi 67. Tm tham s m hm s xc nh trn tp D c ch ra
a/ trn
2
2x 1
y , D
x 6x m 2

= =

R.
b/ trn
2
3x 1
y , D
x 2mx 4

= =

R.
c/
( )
trn y x m 2x m 1, D 0; = = .
d/
( )
trn
x m
y 2x 3m 4 , D 0;
x m 1

= =

.
e/
( )
trn
x 2m
y , D 1; 0
x m 1

= =

.
f/
( )
trn
1
y x 2m 6, D 1; 0
x m
= =

.
g/
( )
trn
1
y 2x m 1 , D 1;
x m
= =

.


BI TP RN LUYN
Bi 68. Bi 68. Bi 68. Bi 68. Tm tp xc nh ca cc hm s sau
a/ y x 3 = . b/
2
y x 4 = .
c/
3 2
y x 3x 4x 5 = . d/
2
2x 3x 1
y
5

= .
e/
2
x 3x 6
y
2

=

. f/ y x 11 = .
g/ y 9x 40 23x 13 = . h/ y x 1 x 3 100 41x = .
Bi 69. Bi 69. Bi 69. Bi 69. Tm tp xc nh ca cc hm s sau
a/
2
x x 1
y
x

= . b/
x 2
y
x 1

. c/
x 3
y
x 1

.
d/
3x 5
y
3x 2

=

. e/
x 1
y
2x 1

. f/
1
y
2x 2
=

.
g/
x 3
y
x 7

. h/
2
y x 2
x 9
=

. i/
3
y x 1
x 1
=

.
www.MATHVN.com
www.DeThiThuDaiHoc.com
cng hc tp mn Ton 10 tp I Ths. L Vn on


"Cn c b thng minh" Page - 15 -
j/
2
x 3x 1
y
2x 1

=

. k/
1 x
y
2x 11 1 x
=

. l/
1 1
y
2x 1 6x 2
=

.
m/
10 11
y
13 9x 6x 7
=

. n/
( )( )
2x
y
2 x 3 x
=

. o/
( )( )
2
2x 4x 7
y
2 3x 2 4x

=

.
p/
1 1
y .
32x 0,25 25 0,5x
=

. q/
2
5
y
x 6x 25
=

. r/
2
3
y
14x 49 x

=

.
s/
2
x 2
y
x 2x 3

=

. t/
2
x 2012
y
2x 6x 4

=

. u/
2
x
y
x 4x 5
=

.
v/
( )( )
2
2x 1
y
x 1 2x 3x 1

=

. x/
2
4 2
3x x 1
y
x x 6

=

. y/
2
4 2
3x 1
y
x 9x 8

=

.
Bi 70. Bi 70. Bi 70. Bi 70. Tm tp xc nh ca cc hm s sau
a/ y x = . b/
2
y x = . c/ y x 1 = .
d/ y 4 3x = . e/ y x 10 = . f/ y 2x 9 = .
g/
3
y 0,1x 5 = . h/
3
y 2, 6x 3,14 = . i/
3
y x 2 = .
j/ y 1 x 1 x = . k/ y 2x 1 1 2x = . l/ y 15x 3 = .
m/ y 3x 25 x 1 = . n/ y 13 4x 7x 22 = . o/
3 3 2
y x x = .
p/
3 3 2 3
y 1 x x x = . q/
1
y
x
= . r/
3x
y
x 1
=

.
s/
1 2x
y
4x 8

=

. t/
x 1
y
3x 10 10 3x
=

. u/
4x x
y
7x 1 3 4 28x
=

.
v/
1 2
y
2 x 3x 18
=

. w/
0,2x 25
y
0,7x 0,7 8 0,8x
=

. x/
3 3 2
1 1
y
x 1 x
=

.
y/
3 3 2 2
x 10x
y
x 1 x 4

=

. z/
2
1
y
x x 1
=

. /
2
2x
y
4x 8x 120
=

.
Bi 71. Bi 71. Bi 71. Bi 71. Gii cc phng trnh v cc bt phng trnh sau
a/
2
x 6x 8 0 = . b/
2
x x 1 0 = .
c/
2
x 5x 14 0 . d/
2
3x 4x 1 0 .
e/
( )
2
3x 2 5 . f/
( )
2
0, 5x 1 1 .
g/ x 1 2 2x 0 = . h/ 1 x 2x 2 0 .
i/ x 3 2x 1 0 = . j/
( )( )
2 6x 3x 5 3x 1 0 = .
k/
2 2
4x 11x 7 19x 36x 77 0 . l/
2 2
9x 6x 1 4 10x 25x 0 .
m/ x 3 2x 1 0 . n/ x x 0 .
o/
( )
2
x x 2 1x 0 . p/
4 2
x 3x x 0 .
q/
3 6 3 2
x x 11x 0 . r/
2
x 1 x .

www.MATHVN.com
www.DeThiThuDaiHoc.com
Ths. L Vn on Phn i S


Page - 16 - "All the flower of tomorrow are in the seeks of today"



















BI TP P DNG
Bi 72. Bi 72. Bi 72. Bi 72. Xt s bin thin ca cc hm s sau trn cc khong ch ra
a/ y 2x 3 trn = R. b/ y x 5 trn = R.
c/
( )

2
y x 10x 9 trn 5; = . d/
( )

2
y x 2x 1 trn 1; = .
e/
( ) ( )

2
y x 4x trn ; 2 , 2; = . f/
( ) ( )

2
y x 6x 8 trn 10; 2 , 3;5 = .
g/
( ) ( )

2
y 2x 4x 1 trn ;1 , 1; = . h/
( ) ( )

4
y trn ; 1 , 1;
x 1
=

.
i/
( ) ( )

3
y trn ;2 , 2;
2 x
=

. j/
( )

1 x
y trn ;1
1 x

.
k/
( ) ( )

x
y trn ; 7 , 7;
x 7
=

. l/
f
y x 1 trn D = .
m/
f
y x 3 trn D = . n/
f
y x 3 trn D = .
o/
f
y 2 x 1 trn D = . p/
( ) ( )

2
x
y trn 0;1 , 1;
x 1
=

.
Bi 73. Bi 73. Bi 73. Bi 73. Vi gi tr no ca m th cc hm s sau ng bin hoc nghch bin trn tp xc nh (hoc
trn tng khong xc nh)
a/
( )
y m 2 x 5 = . b/
( )
y m 1 x m 2 = .
c/
m
y
x 2
=

. d/
m 1
y
x

= .

Dng ton 2. Xt chiu bin thin ca hm s (Tnh n iu hm s)
Cho hm s
( )
f x xc nh trn K.
Hm s
( )
y f x = ng bin trn
( ) ( )
1 2 1 2 1 2
K x , x K : x x f x f x < <
( ) ( )
2 1
1 2 1 2
2 1
f x f x
x , x K : x x 0
x x

.
Hm s
( )
y f x = nghch bin trn
( ) ( )
1 2 1 2 1 2
K x , x K : x x f x f x <

( ) ( )
2 1
1 2 1 2
2 1
f x f x
x , x K : x x 0
x x

<

.
Lu : Mt s trng hp, ta c th lp t s
( )
( )
1
2
f x
f x
so snh vi s 1, nhm a v kt qu
( ) ( ) ( ) ( )

1 2 2 1
f x f x hay f x f x < <

www.MATHVN.com
www.DeThiThuDaiHoc.com
cng hc tp mn Ton 10 tp I Ths. L Vn on


"Cn c b thng minh" Page - 17 -
BI TP RN LUYN
Bi 74. Bi 74. Bi 74. Bi 74. Xt tnh ng bin v tnh nghch bin ca hm s trn tng khong tng ng
a/ y x 2013 trn = R. b/ y 2x 3 trn = R.
c/
( )

2
y x 4x 2 trn 2; = . d/
( )

2
y 2x 4x 1 trn ;1 = .
e/
( )

2
x
y x 1 trn 1;
2
= . f/
( )

2
y 4x x 3 trn 2; = .
g/
( )

2
y 5 x 6x trn ; 3 = . h/
2
y x trn ,

= R R .
i/
2
y x trn ,

= R R . j/
2
y 2x trn = R.
k/
2
y x 4x 1 trn = R. l/
( ) ( )

1
y trn 3; 2 , 2; 3
x 1
=

.
m/
( )

2
y trn 1;
1 x
=

. n/
( )

1
y trn 3;
x 3
=

.
o/
( )

1
y trn 2;
x 2
=

. p/
( )

5x
y trn 2;
x 2
=

.
q/
( ) ( )

x 1
y trn ; 1 , 1;
x 1

. r/
( ) ( )

2x 1
y trn ; 3 , 3;
x 3

.
s/
( ) ( )

2
2x
y trn 0;1 , 1;
x 1
=

. t/
( )

1
y 2 trn 2;
x 2
=

.
u/
y
y 5 x trn D = . v/
y
y x 2 trn D = .
w/
( )
y x x trn 0; = . x/
3
y
y x trn D = .
y/
y
y x 3 trn D = . z/
y
y 2x 5 trn D = .
/
y
y 2 x 3 trn D = . /
y
y x 3 2 x 2 trn D = .
Bi 75. Bi 75. Bi 75. Bi 75. Cho hm s
( )
y f x 2 x 2 1 x = = .
a/ Tm tp xc nh ca hm s.
b/ Xt tnh n iu ca hm s.
c/ Tm gi tr ln nht v gi tr nh nht ca hm s trn
1 1
;
4 2
l
l
l
l
.
Bi 76. Bi 76. Bi 76. Bi 76. Cho hm s
( )
y f x 5 x 2 x 4 = = .
a/ Tm tp xc nh ca hm s.
b/ Xt tnh n iu ca hm s.
c/ Lp bng bin thin ca hm s.
d/ V th hm s.
Bi 77. Bi 77. Bi 77. Bi 77. Cho hm s
( )
1
y f x
x 1
= =

.
a/ Tm tp xc nh ca hm s.
b/ Chng minh hm s gim trn tng khong xc nh ca n.
c/ Lp bng bin thin v v th hm s.

www.MATHVN.com
www.DeThiThuDaiHoc.com
Ths. L Vn on Phn i S


Page - 18 - "All the flower of tomorrow are in the seeks of today"

















BI TP P DNG
Bi 78. Bi 78. Bi 78. Bi 78. Xt tnh chn l ca cc hm s sau
a/
2
y 3x 1 = . b/
3
y 6x = . c/
( ) ( )
2014 2014
y 2x 2 2x 2 = .
d/
4 2
y x 4x 2 = . e/
3
y 2x 3x = . f/
( )
2
y x 1 = .
g/
2
y x x = . h/
2
x
y
x 1
=

. i/ y x 2 x 2 = .
j/
2
y 4x 5 x 3 = . k/
4
y 5x 3 x 8 = . l/
2
4
x 4
y
x

= .
m/ y 2x 1 2x 1 = . n/
x 1 x 1
y
x 1 x 1

=

. o/
2
y 2x x = .
p/ y 2x 9 = . q/ y 2 x 2 x = . r/
2
y 25 4x = .
s/
2 2
y x x x x = . t/
1
y x 2
2 x
=

. v/
x 2 x 2
y
x

= .
Bi 79. Bi 79. Bi 79. Bi 79. Vi gi tr no ca tham s m th hm s
( ) ( )
3
y f x x x 2 2m 1 = = l hm s l.
Bi 80. Bi 80. Bi 80. Bi 80. Tm tham s m hm s
( ) ( )
4 3 2 2
y f x x m m 1 x x mx m = = l hm s chn.

Dng ton 3. Xt tnh chn l ca hm s
xt tnh chn l ca hm s
( )
y f x = , ta tin hnh lm cc bc sau
Bc 1. Tm tp xc nh D ca hm s v xt xem D c l tp i xng hay khng.
Bc 2. Nu D l tp i xng th so snh
( )
f x vi
( )
f x (x bt k thuc D).
+ Nu
( ) ( )
f x f x , x D = th hm s
( )
y f x = l hm s chn.
+ Nu
( ) ( )
f x f x , x D = th hm s
( )
y f x = l hm s l.
O Lu
Tp i xng l tp tha mn iu kin: x D th x D .
Nu x D m
( ) ( )
f x f x th
( )
y f x = l hm s khng chn, khng l
www.MATHVN.com
www.DeThiThuDaiHoc.com
cng hc tp mn Ton 10 tp I Ths. L Vn on


"Cn c b thng minh" Page - 19 -
BI TP RN LUYN
Bi 81. Bi 81. Bi 81. Bi 81. Xt tnh chn l ca cc hm s sau
a/
2
y 7x 1 = . b/
3
y 4x x = . c/
4
y x 3x 2 = .
d/
4 2
y x 2x 1 = . e/
( ) ( )
2012 2012
y x 1 x 1 = . f/
2
x 2
y
x

= .
g/
2
5x
y
x 1
=

. h/
2
3
x
y
3x x
=

. i/
3
x 3x
y
2x

= .
j/
4 2
x x 1
y
x

= . k/ y 3x 1 = . l/ y x 3 3 x = .
m/
2
y 4 x = . n/
2 2
y x x x x = . o/
2
y x 1 x 1 1 x = .
p/
1
y x 1
1 x
=

. q/
2
1 x 1 x
y
x

= . r/
3
3x
y
4 x 4 x
=

.
s/ y 3x 2 3x 2 = . t/ y x 2 x 2 = . u/ y 4x 3 4x 3 = .
v/
2
y 3x 2 x 11 = . x/
4
y x 2 x 5 = . z/
x 1 x 1
y
2013x

= .
Bi 82. Bi 82. Bi 82. Bi 82. Xt tnh chn l ca cc hm s sau
a/
( )
x 1 x 1
y f x
x 1 x 1

= =

. b/
( )
4 2
3x x 5
y f x
x 1

= =

.
c/
( )
2
3x
y f x
2 x
= =

. d/
( )
( )
2
2
x x 2
y f x
x 2

= =

.
e/
( )



x 2 khi x 1
y f x 0 khi 1 x 1
x 2 khi x 1
'
1

1
1
1
= = < <
!
1
1
1
1
+
. f/
( )



3
3
x 1 khi x 1
y f x 0 khi 1 x 1
x 1 khi x 1
'
1

1
1
1
= = < <
!
1
1
1
1
+
.

www.MATHVN.com
www.DeThiThuDaiHoc.com
Ths. L Vn on Phn i S


Page - 20 - "All the flower of tomorrow are in the seeks of today"





























BI TP P DNG
Bi 83. Bi 83. Bi 83. Bi 83. V th ca cc hm s sau
a/ y 2x 7 = . b/ y 3x 5 = .
c/
x 3
y
2

= . d/
5 x
y
3

= .
e/
x khi x 1
y 1 khi 1 x 2
x 1 khi x 2
'
1
1
1
1
= < <
!
1
1
1
1+
. f/
2x 2 khi x 1
y 0 khi 1 x 2
x 2 khi x 2
'
1 <
1
1
1
=
!
1
1
1
1+
.
g/ y 3x 5 = . h/ y 2 x 1 = .
i/
1 5
y 2x 3
2 2
= . j/ y x 2 1 x = .
k/ y x x 1 = . l/ y x x 1 x 1 = .
Bi 84. Bi 84. Bi 84. Bi 84. Tm ta giao im ca cc cp ng thng sau bng phng php th v bng php tnh
a/ y 3x 2; y 2x 3 = = . b/
( )
y 3x 2; y 4 x 3 = = .
c/ y 2x; y x 3 = = . d/
x 3 5 x
y ; y
2 3

= = .
e/ y x 3; y 5x 3 = = . f/ x y 1; x 2y 4 0 = = .
B HM S BC NHT

C CC C Hm s bc nht
( )
y ax b, a 0 = .
Tp xc nh: D= R.
S bin thin:
Khi a 0 : hm s ng bin (tng) trn R.
Khi a 0 < : hm s nghch bin (gim) trn R.
th l ng thng c h s gc bng a, ct trc tung ti im
( )
B 0; b .
O Lu rng: Cho hai ng thng d : y ax b = v d' : y a ' x b' = .
d song song vi d' a a' = v b b' .
d trng vi d' a a' = v b b' = .
d ct d' a a ' .
C CC C Hm s
( )
y ax b , a 0 = .



b
ax b khi x
a
y ax b
b
(ax b) khi x
a
'
1
1

1
1
1
= =
!
1
1
<
1
1
1+

O Lu rng: v th hm s
( )
y ax b , a 0 = ta c th v hai ng
thng y ax b = v y ax b = , ri xo i hai phn ng
thng nm pha di trc honh.
www.MATHVN.com
www.DeThiThuDaiHoc.com
cng hc tp mn Ton 10 tp I Ths. L Vn on


"Cn c b thng minh" Page - 21 -
Bi 85. Bi 85. Bi 85. Bi 85. Trong mi trng hp sau, hy tm gi tr tham s m th hm s
( )
y 2x m x 1 = :
a/ i qua gc ta O. b/ i qua im
( )
M 2; 3 .
c/ Song song vi ng thng y 2.x = . d/ Vung gc vi ng thng y x = .
Bi 86. Bi 86. Bi 86. Bi 86. Xc nh tham s a v b th ca hm s y ax b = :
a/ i qua hai im
( )
A 1; 20 v
( )
B 3; 8 .
b/ i qua hai im
( )
A 1; 3 v
( )
B 1; 2 .
c/ i qua hai im
2
A ; 2
3
1



( )
v
( )
B 0;1 .
d/ i qua hai im
( )
A 4;2 v
( )
B 1;1 .
e/ i qua im
( )
A 1; 1 v song song vi ng thng y 2x 7 = .
f/ i qua im
( )
A 3; 4 v song song vi ng thng x y 5 0 = .
g/ i qua im
( )
M 4; 3 v song song vi ng thng
2
d : y x 1
3
= .
h/ i qua im im
( )
M 3; 5 v im N l giao im ca hai ng thng
1
d : y 2x = v
ng thng
2
d : y x 3 = .
i/ Ct ng thng
1
d : y 2x 5 = ti im c honh bng 2 v ct ng thng
2
d : y 3x 4 = ti im c tung bng 2.
j/ Song song vi ng thng
1
y x
2
= v i qua giao im ca hai ng thng
1
y x 1
2
= v y 3x 5 = .
k/ Qua im
( )
H 1; 3 v ct trc honh ti im K c honh l 4.
l/ Ct trc honh ti im A c honh bng 2 v song song vi ng thng 3x 4y 36 = .
m/ i qua gc ta O v vung gc vi ng thng y x = .
n/ i qua im
( )
A 1;1 v vung gc vi ng thng y x 1 = .
Bi 87. Bi 87. Bi 87. Bi 87. Trong mi trng hp sau, tm cc gi tr ca tham s m sao cho ba ng thng sau y phn
bit (khng c im chung) v ng qui.
a/ y 2x; y x 3; y mx 5 = = = .
b/
( )
y 5 x 1 ; y mx 3; y 3x m = = = .
c/
( )
y 2x 1; y 8 x; y 3 2m x 2 = = = .
d/
( )
y 5 3m x m 2; y x 11; y x 3 = = = .
e/
( )

2
y x 5; y 2x 7; y m 2 x m 4 = = = .
Bi 88. Bi 88. Bi 88. Bi 88. Tm im sao cho ng thng sau lun i qua d m ly bt c gi tr no (im c nh th)
a/ y 2mx 1 m = . b/ y mx 3 x = .
c/
( )
y 2m 5 x m 3 = . d/
( )
y m x 2 = .
e/
( )
y 2m 3 x 2 = . f/
( )
y m 1 x 2m = .
www.MATHVN.com
www.DeThiThuDaiHoc.com
Ths. L Vn on Phn i S


Page - 22 - "All the flower of tomorrow are in the seeks of today"
Bi 89. Bi 89. Bi 89. Bi 89. Vi gi tr no ca m th hm s sau ng bin ? nghch bin ?
a/
( )
y 2m 3 x m 1 = . b/
( )
y 2m 5 x m 3 = .
c/ y mx 3 x = . d/
( )
y m x 2 = .
Bi 90. Bi 90. Bi 90. Bi 90. Tm cc cp ng thng song song trong cc ng thng cho sau y ?
1
d : 3y 6x 1 0 = .
2
d : y 0, 5x 4 = .
3
x
d : y 3
2
= .
4
d : 2y x 6 = .
5
d : 2x y 1 = .
6
d : y 0, 5x 1 = .
Bi 91. Bi 91. Bi 91. Bi 91. Vi gi tr no ca m th th ca cc cp hm s sau song song vi nhau
a/
( )
y 3m 1 x m 3; y 2x 1 = = .
b/
( ) ( )
y m x 2 ; y 2m 3 x m 1 = = .
c/
( )

2 m 2
m 3m 5m 4
y x ; y x
1 m m 1 3m 1 3m 1


= =

.
Bi 92. Bi 92. Bi 92. Bi 92. Tm qu tch (tp hp im) ca cc im sau
a/
( )
M m 1; 2m 1 . b/
( )
2
M 3 2m; 4m 1 .
c/
( )
2
M m 2; m 4 . d/
( )
2
M 3 m; 4m 2m 1 .
Bi 93. Bi 93. Bi 93. Bi 93. nh tham s m hai ng thng ct nhau. Khi , tm qu tch giao im ca hai th.
a/
1
d : y 2x m = v
2
d : y 1 = . b/
1
d : y x 2m = v
2
d : y 1 = .
Bi 94. Bi 94. Bi 94. Bi 94. nh tham s m din tch tam gic OAB tha mn iu kin cho trc (O l gc ta )
a/
( ) ( )

2
OAB
A 0; m , B 1; 0 , S 9

= . b/
( ) ( )

2
OAB
A 0; 2 , B 3m ; 0 , S 18

= .
c/
( ) ( )

OAB
A 0; m , B m; 0 , S 8

= . d/
( ) ( )
2
OAB
A 0; 2m 1 , B m 2; 0 , S 2

= .
Bi 95. Bi 95. Bi 95. Bi 95. nh tham s m ng thng d chn trn hai trc ta tam gic c din tch cho trc.
a/ d : y x 2m, S 1

= = . b/
2
2
d : y x m , S 25
3

= = .
Bi 96. Bi 96. Bi 96. Bi 96. Gii cc h phng trnh sau bng phng php th
a/
y 2x 3
4x 2y 4
'
1 =
1
!
1 =
1
+
b/
3 y 0
3x 2y 3 0
'
1 =
1
!
1 =
1
+

c/
y 3x 5
2x y 1
'
1 =
1
!
1 =
1
+
d/
2x y 2 0
6x 3y 6 0
'
1 =
1
!
1 =
1
+

Bi 97. Bi 97. Bi 97. Bi 97. Cho th hm s y 3 2x = .
a/ Kho st v v th hm s trn.
b/ Xc nh cc giao im ca th trn vi ng thng
1
y x 1
2
= .
www.MATHVN.com
www.DeThiThuDaiHoc.com
cng hc tp mn Ton 10 tp I Ths. L Vn on


"Cn c b thng minh" Page - 23 -
BI TP RN LUYN
Bi 98. Bi 98. Bi 98. Bi 98. Kho st v v th hm s sau
a/ y 2x 3 = . b/ y 2x 7 = . c/ y 6 x = .
d/
4
y x 1
3
= . e/
x 1
y
4

= . f/
3 2x
y
5

= .
g/ y 2 = . h/ y 3 = . i/ y x = .
j/ y x = . k/ y 2x = . l/ y 2x = .
Bi 99. Bi 99. Bi 99. Bi 99. V th ca cc hm s sau
a/


x 2 khi x 2
y
1 khi x 2
'
1
1
=
!
1
1
+
. b/


2x 1 khi x 1
y
1
x 1 khi x 1
2
'
1
1
11
=
!
1
<
1
1
1+
c/ y 2x 3 = .
d/
3
y x 1
4
= . e/ y x 2 = . f/ y x 2x = .
g/ y 2x 2x = . h/ y x 2 1 = . i/ y x 3 2x 1 = .
j/ y x 1 5 x = . k/ y x 2 3x 4 6x 4 = . l/ y 11x 8 2 9x 2x 9 = .
Bi 100. Bi 100. Bi 100. Bi 100. Xc nh ta giao im ca cc cp ng thng sau bng th v bng php tnh.
a/
1
d : y 2 3x = v
2
d : y 4x 12 = . b/
1
d : y 3x 2 = v
2
5
d : y
4
= .
c/
1
d : y 5x 2 = v
2
3 3
d : y 1 x
2 4
1

=


( )
. d/
1
d : y x 3 = v
2
d : y x 1 = .
Bi 101. Bi 101. Bi 101. Bi 101. Xc nh tham s a v b th ca hm s y ax b = :
a/ i qua hai im
( ) ( )
A 1; 2 , B 99; 2 .
b/ i qua hai im
( ) ( )
A 1; 3 , B 2; 4 .
c/ i qua hai im
( ) ( )
A 3;2 , B 5; 2 .
d/ i qua hai im
( ) ( )
A 100;1 , B 50;1 .
e/ i qua hai im
( ) ( )
A 1; 3 , B 1; 4 .
f/ i qua
( )
A 3; 4 v c h s gc l 2.
g/ Song song vi ng thng d : y 3x 2 = v i qua im
( )
M 2; 3 .
h/ Song song vi ng thng y 7x 2013 = v i qua im
( )
N 1; 2 .
i/ i qua im
( )
A 1; 3 v vung gc vi ng thng d : 2x y 1 0 = .
j/ i qua im
( )
A 2; 1 v vung gc vi ng thng d : y 1 = .
k/ i qua im
( )
M 1; 4 v ct trc tung ti im N c tung bng 2 .
l/ Ct trc tung ti im E c tung bng 3 v ct trc honh ti F c honh l 1.
m/ Ct trc tung ti im A c tung bng 3 v vung gc vi ng thng
1
d : y x
2
= .
n/ i qua im
( )
A 2; 30 v im B l giao im ca hai ng thng 14x y 2 0 = v
www.MATHVN.com
www.DeThiThuDaiHoc.com
Ths. L Vn on Phn i S


Page - 24 - "All the flower of tomorrow are in the seeks of today"
y 2x 26 = .
Bi 102. Bi 102. Bi 102. Bi 102. Chng minh rng b ba ng thng trong cc trng hp sau ng qui.
a/
1
d : y x 2 = .
2
d : y 2x 1 = .
3
d : y 3x = .
b/
1
d : y x 1 = .
2
d : y 2 = .
3
d : y 3 x = .
c/
1
d : 3x y 7 0 = .
2
d : 3x 2y 8 0 = .
3
d : y 2x 3 = .
d/
1
d : 5x 4y 6 0 = .
2
d : y 2x 3 = .
3
d : 2y 3x 4 0 = .
Bi 103. Bi 103. Bi 103. Bi 103. Tm tham s m b ba ng thng sau ng qui.
a/
1
d : y x 1 = .
2
d : y x m = .
3
d : y 3x = .
b/
1
d : y 2x = .
2
d : y x 3 = .
3
d : y mx 5 = .
c/
1
d : y 2x 3 = .
2
d : y x 5 = .
( )
3
d : y 1 m x 2 = .
Bi 104. Bi 104. Bi 104. Bi 104. Tm im c nh ca h th cc hm s
a/ y mx 3 = . b/ y 2mx 1 m = .
c/
( )
y m 1 x 6m 2014 = . d/ y mx 5m 2 = .
e/
( ) ( )
4 5m x 3m 2 y 3m 4 0 = . f/ mx y 3m 7 0 = .
g/
( ) ( )
m 2 x m 3 y m 8 0 = . h/
( )
2 2
y m 1 x 2m 3 = .
Bi 105. Bi 105. Bi 105. Bi 105. Tm qu tch (tp hp im) ca cc im sau
a/
( )
M 2m 1; 2m 7 . b/
( )
M m 5; 4m 3 .
c/
( )
3 2
M 2m 7; m 3m 6m 1 . d/
( )
2
M 2; m m .
e/
( )
3
M 3m ; 3 . f/
( )
2
M 5 5m; 3m 10 .
Bi 106. Bi 106. Bi 106. Bi 106. nh tham s m hai ng thng ct nhau. Khi , tm qu tch giao im ca hai th
a/
( )
1
d : y m 1 x 3 = .
2
d : y m = .
b/
1
d : y mx 2m 4 = .
2
d : y 3x 2m = .
Bi 107. Bi 107. Bi 107. Bi 107. nh tham s m din tch tam gic OAB tha mn iu kin cho trc (O l gc ta )
a/
( ) ( )

OAB
A 0; 2m , B m; 0 , S 5

= . b/
( ) ( )

2
OAB
A 0; 3m 2 , B 2 m 1;0 , S 15

= .
Bi 108. Bi 108. Bi 108. Bi 108. nh tham s m ng thng d chn trn hai trc ta tam gic c din tch cho trc.
a/ d : y 2x m, S 10

= = . b/
( )
d : y m 1 x 2, S 16

= = .
Bi 109. Bi 109. Bi 109. Bi 109. Cho hm s y 2x 3 = c th l ng thng d.
a/ Kho st v v th hm s.
b/ Xc nh hm s c th l ng thng i xng vi ng thng d qua trc tung.
Bi 110. Bi 110. Bi 110. Bi 110. Cho hm s y 2 x 2x 1 = .
a/ Kho st v v th hm s trn.
b/ Da vo th, bin lun theo m s nghim ca phng trnh: 2 x 2x 1 m = .

www.MATHVN.com
www.DeThiThuDaiHoc.com
cng hc tp mn Ton 10 tp I Ths. L Vn on


"Cn c b thng minh" Page - 25 -






















































b
I ;
2a 4a
1


( )
: nh
nh
b
I ;
2a 4a
1


( )

Trc i xng
C CC C Dng hm s:
( )
2
y ax bx c, a 0 = .
C CC C Tp xc nh: D= R.
C CC C S bin thin:
Khi a 0 Khi a 0 <

x

b
2a



x

b
2a



y



4a




y
4a






C CC C th: l mt parabol c nh
b
I ;
2a 4a
1

( )
, nhn ng thng
b
x
2a
= lm trc i xng,
hng b lm ln trn khi a 0 , xung di khi a 0 < .
C CC C Cc bc v parabol
( ) ( )
2
P : y ax bx c, a 0 = .
Bc 1. Xc nh to nh
b
I ;
2a 4a
1

( )
.
Bc 2. Xc nh trc i xng
b
x
2a
= v hng b lm ca parabol.
Bc 3. Xc nh mt s im c th ca parabol (chng hn, giao im ca parabol vi cc
trc to v cc im i xng vi chng qua trc trc i xng).
Bc 4. Cn c vo tnh i xng, b lm v hnh dng parabol v parabol.
Hnh dng parabol
( ) ( )
2
P : y ax bx c, a 0 = .





b
2a


b
2a



4a


4a


O
O
y y
x
x
Khi a 0 Khi a 0 <
C HM S BC HAI
www.MATHVN.com
www.DeThiThuDaiHoc.com
Ths. L Vn on Phn i S


Page - 26 - "All the flower of tomorrow are in the seeks of today"






































C CC C Mt s bi ton thng gp
Bi ton 1. Tm ta giao im ca hai th
( )
y f x = v
( )
y g x = .
Xt phng trnh honh giao im
( ) ( ) ( )
f x g x = .
Nu phng trnh
( )
c n nghim
( )
n 1 th th
( )
y f x = v
( )
y g x = ct
nhau ti n im phn bit.
Nu phng trnh
( )
c ng 1 nghim th th
( )
y f x = tip xc (c mt im
chung) vi th
( )
y g x = .
Nu phng trnh
( )
v nghim, th th
( )
y f x = v
( )
y g x = khng c im
chung (khng ct nhau).
tm ta giao im, ta thay nghim x vo
( )
y f x = hoc
( )
y g x = c honh y
Bi ton 2. Tm im c nh ca h th
( ) ( )
m
C : y f x, m = khi m thay i
Gi
( ) ( ) ( ) ( )

o o m o o
M x ; y C , m y f x , m , m 1 = .
Bin i
( )
1 v mt trong hai dng
Dng 1:
( ) ( )

A 0
1 Am B 0, m 2a
B 0
'
1 =
1
=
!
1 =
1
+
.
Dng 2:
( ) ( )

2
A 0
1 Am Bm c 0, m B 0 2b
C 0
'
1
=
1
1
1
= =
!
1
1
= 1
1
+
.
Gii h
( )
2a hoc
( )
2b ta tm c ta
( )
o o
x ; y ca im c nh.
Bi ton 3. Qu tch im M (tp hp im) tha tnh cht
Bc 1. Tm iu kin nu c ca tham s m tn ti im M.
Bc 2. Tnh ta im M theo tham s m.
C cc trng hp sau xy ra:
Trng hp 1.
( )
( )
x f m
M
y g m
'
1
=
1
1
!
1
=
1
1+

Kh tham s m gia x v y, ta c h thc gia x v y c lp vi m c dng:
( )
F x, y 0 = , c gi l phng trnh qu tch.
Trng hp 2.
( )
x a
M
y g m
'
1 =
1
1
!
1 =
1
1+
vi a l hng s.
Khi , im M nm trn ng thng x a = .
Trng hp 3.
( )
x f m
M
y b
'
1
=
1
1
!
1 =
1
1+
vi b l hng s.
Khi , im M nm trn ng thng y b = .
www.MATHVN.com
www.DeThiThuDaiHoc.com
cng hc tp mn Ton 10 tp I Ths. L Vn on


"Cn c b thng minh" Page - 27 -
2
2 8
y x x 2
3 3
=



























Hnh 1 Hnh 2
o th cn tm l hp ca hai phn trn (th d hnh 2).
Lu :
( )
2
Parabol P : y ax bx c = , ta cn nh:
nh
b
I ;
2a 4a
1


( )
, trc i xng
b
x
2a
= .
2
y x 2 x 1 =
Bc 3. Tm gii hn qu tch.
Da vo iu kin (nu c) ca m ( bc 1), ta tm c iu kin ca x
hoc y tn ti im
( )
M x; y . l gii hn ca qu tch.
Bc 4. Kt lun
Tp hp im M c phng trnh
( )
F x, y 0 = (hoc x a = hoc y b = )
vi iu kin ca x, y nu c ( bc 3).
Bi ton 4. V th hm s cha du tr tuyt i
V hm th hm s
( ) ( )
2
y f x ax bx c , a 0 = = .
Bc 1. V Parabol
( )
2
P : y ax bx c = .
Bc 2. Suy ra th hm s
( ) ( )
2
y f x ax bx c , a 0 = = , nh sau:
o Gi nguyn phn th
( )
P pha trn trc honh Ox.
o Ly i xng phn th
( )
P pha di trc Ox qua trc Ox.
o th cn tm l hp hai phn trn. (th d hnh 1)

















V hm th hm s
( ) ( )
2
y f x ax b x c, a 0 = = .
Bc 1. V Parabol
( )
2
P : y ax bx c = .
Bc 2. Suy ra th hm s
( ) ( )
2
y f x ax b x c, a 0 = = , nh sau:
o Gi nguyn phn
( )
P bn phi trc tung Oy, b phn bn tri trc tung.
o Ly i xng phn bn phi trc tung trn qua trc tung Oy.
y
x x
O
2
2
3

2
3


1 2 3

1 1 3
1
4
www.MATHVN.com
www.DeThiThuDaiHoc.com
Ths. L Vn on Phn i S


Page - 28 - "All the flower of tomorrow are in the seeks of today"
BI TP P DNG
Bi 111. Bi 111. Bi 111. Bi 111. Xt s bin thin v v th ca cc hm s sau
a/
2
y 2x 6x 3 = . b/
2
y x 2x = . c/
2
y x 2x 3 = .
d/
2
1
y x 2x 6
5
= . e/
2
y x 2x 2 = . f/
2
1
y x 2x 2
2
= .
g/
2
y x 4x 4 = . h/
2
y x 4x 1 = . i/
2
y x 2 = .
k/
2
y x = . l/
( )
2
y x 3 = . m/
( )
2
y x 1 = .
Bi 112. Bi 112. Bi 112. Bi 112. Tm to giao im ca cc cp th ca cc hm s sau
a/
2
y x 1; y x 2x 1 = = . b/
2
y x 3; y x 4x 1 = = .
c/
2
y 2x 5; y x 4x 4 = = . d/
2 2
y x 2x 1; y x 4x 4 = = .
e/
2 2
y 3x 4x 1; y 3x 2x 1 = = . f/
2 2
y 2x x 1; y x x 1 = = .
Bi 113. Bi 113. Bi 113. Bi 113. Xc nh parabol
( )
P bit
a/
( )
2
P : y ax bx 2 = i qua im
( )
A 1; 0 v c trc i xng
3
x
2
= .
b/
( )
2
P : y ax 4x c = c trc i xng l l ng thng x 2 = v ct trc honh ti
im
( )
M 3; 0 .
c/
( )
2
P : y ax bx 3 = i qua im
( )
A 1; 9 v c trc i xng x 2 = .
d/
( )
2
P : y 2x bx c = c trc i xng l ng thng x 1 = v ct trc tung ti im
( )
M 0; 4 .
e/
( )
2
P : y ax 4x c = i qua hai im
( ) ( )
A 1; 2 , B 2; 3 .
f/
( )
2
P : y ax 4x c = c nh l
( )
I 2; 1 .
g/
( )
2
P : y ax 4x c = c honh nh l 3 v i qua im
( )
A 2;1 .
h/
( )
2
P : y ax bx c = i qua im
( )
A 0; 5 v c nh
( )
I 3; 4 .
i/
( )
2
P : y ax bx c = i qua im
( )
A 2; 3 v c nh
( )
I 1; 4 .
j/
( )
2
P : y ax bx c = i qua im
( )
A 1;1 v c nh
( )
I 1; 5 .
k/
( )
2
P : y ax bx c = i qua cc im
( ) ( ) ( )
A 1;1 , B 1; 3 , O 0; 0 .
l/
( )
2
P : y ax bx c = i qua cc im
( ) ( ) ( )
A 0; 1 , B 1; 1 , C 1;1 .
m/
( )
2
P : y ax bx c = i qua cc im
( ) ( ) ( )
A 1; 1 , B 0; 2 , C 1; 1 .
n/
( )
2
P : y x bx c = i qua im
( )
A 1; 0 v nh I c tung bng 1.
o/
( )
2
P : y ax bx c = c nh l
( )
I 3; 1 v ct Ox ti im c honh l 1.
Bi 114. Bi 114. Bi 114. Bi 114. Kho st v v th ca hm s sau
a/
2
y x 2 x 1 = . b/
2
y 3x 6 x 4 = .
c/
( )
y x x 2 = . d/
2
y x 2 x 1 = .
www.MATHVN.com
www.DeThiThuDaiHoc.com
cng hc tp mn Ton 10 tp I Ths. L Vn on


"Cn c b thng minh" Page - 29 -
e/


2
2
x 2 khi x 1
y
2x 2x 3 khi x 1
'
1
<
1
1
=
!
1

1
1+
. f/


2
2x 1 khi x 0
y
x 4x 1 khi x 0
'
1
1
1
=
!
1 <
1
1+
.
g/


2
2x khi x 0
y
x x khi x 0
'
1 <
1
1
=
!
1
1
1+
. h/
2
y 2x 2x = .
i/
2
2 8
y x x 2
3 3
= . j/
2
1
y x 2 x 1
2
= .
Bi 115. Bi 115. Bi 115. Bi 115. Lp bng bin thin, ri tm gi tr ln nht (GTLN max) v gi tr nh nht (GTNN min)
ca hm s trn min xc nh c ch ra.
a/
2
y x x trn 1; 3
l
=
l
l
. b/
2
y 2x 3x trn 4; 6
l
=
l
l
.
c/
2
y 3x 6x trn 5; 2
l
=
l
l
. d/
2
y x 5x 4 trn 1; 2
l
=
l
l
.
e/
2
y x 5x 3 trn 1; 3
l
=
l
l
. f/
)

2
y 3x 6x trn 3;

.
g/
(

2
y x 5x trn ; 3
l
=
l
l
. h/
( )

2
y 2x 2.x trn ; 1 1;
l
=
l
l
.
Bi 116. Bi 116. Bi 116. Bi 116. V th ca hm s
2
y x 5x 6 = . Hy s dng th bin lun theo tham s m, s
im chung ca parabol
2
y x 5x 6 = v ng thng y m = .
Bi 117. Bi 117. Bi 117. Bi 117. Cho Parabol
( )
2
P : y x 2x 3 = .
a/ Kho st v v th ca parabol trn.
b/ Da vo th, bin lun s nghim ca ca phng trnh
2
x 2x m 0 = .
c/ Vit phng trnh ng thng d vung gc vi ng thng : y 2x 1 = v i qua nh
ca parabol
( )
P .
Bi 118. Bi 118. Bi 118. Bi 118. Cho Parabol
( )
2
P : y x x 2 = .
a/ Kho st v v th ca hm s
( )
P .
b/ Tm tham s m phng trnh
2
x x m 2 0 = c duy nht 1 nghim.
Bi 119. Bi 119. Bi 119. Bi 119. nh tham s m cc cp th sau khng ct nhau; ct nhau ti hai im phn bit.
a/
( )
2
1
P : y x 2x 4 = v
( )
2
2
P : y x 2x m = .
b/
( )
2
1
P : y mx mx m = v
( ) ( )
2
2
P : y x 2 m x 3 = .
Bi 120. Bi 120. Bi 120. Bi 120. nh tham s m cc cp th sau tip xc nhau (c duy nht mt im chung)
a/
( )
2
1
1
P : y x x 1
2
= v
( )
2
2
P : y x x m = .
b/
( )
2 2
1
P : y x mx m = v
( )
2
2
P : y x 5mx 6 = .
Bi 121. Bi 121. Bi 121. Bi 121. Cho Parabol
( )
2
P : y x 3x 2 = v ng thng d : y mx 2 = .
a/ Kho st s bin thin v v th hm s
( )
P .
b/ Tm tham s m hai th ca hai hm s tip xc nhau (c duy nht mt im chung),
ct nhau ti hai im phn bit.
c/ Bin lun theo m s nghim ca phng trnh
2
x 3x 3 2m 0 = .
www.MATHVN.com
www.DeThiThuDaiHoc.com
Ths. L Vn on Phn i S


Page - 30 - "All the flower of tomorrow are in the seeks of today"
Bi 122. Bi 122. Bi 122. Bi 122. Tm im c nh ca h th cc hm s
a/
( )
2
y m 1 x 2mx 3m 1 = . b/
( ) ( )
2
y m 2 x m 1 x 3m 4 = .
c/
2
y mx 2mx 1 = . d/
( )
2 2 2
y m x 2 m 1 x m 1 = .
e/
( )
3
y m 1 x m 2 = . f/
3
y mx mx 2 = .
Bi 123. Bi 123. Bi 123. Bi 123. Chng minh rng vi mi m, th ca mi hm s sau lun ct trc honh ti hai im phn
bit v nh I ca th lun chy trn mt ng thng c nh.
a/
2
2
m
y x mx 1
4
= . b/
2 2
y x 2mx m 1 = .
Bi 124. Bi 124. Bi 124. Bi 124. Tm qu tch nh ca cc Parabol sau
a/
2
y x mx 1 = . b/
( )
2 2 3 2
y mx 2m x m 2m 3, m 0 = .
Bi 125. Bi 125. Bi 125. Bi 125. nh tham s m cp th ct nhau ti hai im phn bit. Khi , tm qu tch trung im
ca giao im ca hai th
a/
( ) ( )
P : y x x 2 , d : y m = = .
b/
( )

2
P : y x 2mx m, d : y 3 x = = .
Bi 126. Bi 126. Bi 126. Bi 126. V th hm s v da vo th bin lun theo m s nghim ca phng trnh
a/
2 2
y 2x 10x 12 , 2x 10x 12 m = = .
b/
2 2
y x 4 x 3, x 4 x 3 m = = .
c/
2 2
y x 3 x 8 , x 3 x 8 m = = .
d/
2 2
1 2 8 1 2 8
y x x , x x m
3 3 3 3 3 3
= = .
Bi 127. Bi 127. Bi 127. Bi 127. Bin lun theo m s nghim ca phng trnh
a/
2
x x x 2 m = . b/
2
x 3x 2 m = .
c/
( )( )
x 2 x 1 m 0 = . d/
2
x 2 x 3 m 0 = .
e/
2
x x 3 4 m 0 = . f/
2 3
x 3x x 2 m 5 0 = .
g/
( )( )
x 1 1 x 2m 0 = . h/
2
2x 3 x 1 m 0 = .
Bi 128. Bi 128. Bi 128. Bi 128. Tm tham s m phng trnh sau c k nghim phn bit
a/
( )( )

2 2
m x x 1 m x x 0, k 4 = = .
b/
( )( )

2 2
x 2x m x 4x 2 m 0, k 4 = = .
c/
( ) ( )

4 3 2 2
x 2x 2m 1 x 2 m 1 x m m 0, k 4 = = .
Bi 129. Bi 129. Bi 129. Bi 129. nh tham s m bt phng trnh sau c nghim
a/
( )( )
2
x 3x m x 1 x 2 . b/ 2x m 5 x .
c/
2
x 6x 5 m < . d/ 2x 4 3 x m .
www.MATHVN.com
www.DeThiThuDaiHoc.com
cng hc tp mn Ton 10 tp I Ths. L Vn on


"Cn c b thng minh" Page - 31 -
Bi 130. Bi 130. Bi 130. Bi 130. Cho hm s
( ) ( ) ( ) ( )
2
m
P : y 2 m x 3m 1 x 2m, C = .
a/ Kho st s bin thin v v th hm s
( )
P khi m 1 = , gi l
( )
1
C .
b/ Chng minh rng h th
( )
m
C lun i qua im c nh.
c/ nh tham s m th hm s
( )
m
C nhn ng thng y 2x 1 = lm tip tuyn.
d/ Da vo th
( )
1
C , bin lun theo m s nghim ca phng trnh:

( )
2
x 2x 3 2 m 1 0 = .
Bi 131. Bi 131. Bi 131. Bi 131. Cho Parabol
( )
2
P : y x 1 = .
a/ Kho st v v th
( )
P .
b/ Xc nh im M trn
( )
P on OM l ngn nht.
c/ Chng minh rng khi OM ngn nht th ng thng OM vung gc vi tip tuyn ti M
ca
( )
P .
Bi 132. Bi 132. Bi 132. Bi 132. Cho ng thng d : y 2x 1 2m = v Parabol
( )
P i qua im
( )
A 1; 0 v c nh
( )
S 3; 4 .
a/ Lp phng trnh v v Parabol
( )
P .
b/ Chng minh rng d lun i qua mt im c nh.
c/ Chng minh rng d lun ct
( )
P ti hai im phn bit.
Bi 133. Bi 133. Bi 133. Bi 133. Cho Parabol
( ) ( )
2
P : y f x x 4x 3 = = v ng thng
( )
d : y g x mx 1 = = .
a/ Kho st s bin thin v v
( )
P .
b/ nh m
( )
P v d tip xc nhau.
c/ Cho m ty . Chng minh:
( ) ( )
2
m 8m 8
f x g x , x
4

R.
Bi 134. Bi 134. Bi 134. Bi 134. Cho
( )
2
m
P : y x 3mx 5 = .
a/ Tm tham s m hm s c gi tr nh nht bng 4.
b/ Tm qu tch nh ca
( )
m
P .
c/ Tm m
( )
m
P c duy nht mt im chung vi Ox.
d/ Khi m 1 = , vit phng trnh tip tuyn vi th ti im c honh bng 1.
e/ nh tham s m ng thng d : y x 2 = ct
( )
m
P ti hai im phn bit A, B sao
cho OA vung gc vi OB. Tnh din tch tam gic OAB.
Bi 135. Bi 135. Bi 135. Bi 135. Cho
( ) ( )
2
m
P : y x m 1 x m 6 = .
a/ nh m Parabol i qua im
( )
A 1; 2 .
b/ Kho st s bin thin v v th
( )
P ca hm s khi m 3 = .
c/ Chng minh
( )
m
P lun i qua mt im c nh.
d/ Chng minh: x R th khong cch t nh ca
( )
m
P n Ox khng nh hn 6.

www.MATHVN.com
www.DeThiThuDaiHoc.com
Ths. L Vn on Phn i S


Page - 32 - "All the flower of tomorrow are in the seeks of today"
BI TP RN LUYN
Bi 136. Bi 136. Bi 136. Bi 136. Xc nh trc i xng, ta nh, cc giao im vi trc tung v trc honh ca parabol
a/
2
y 2x x 2 = . b/
2
y 3x 6x 4 = . c/
2
y 2x x 2 = .
d/
2
1
y x 2x 6
5
= . e/
2
1
y x 2x 1
2
= . f/
2
y 2x 2 = .
Bi 137. Bi 137. Bi 137. Bi 137. Xt s bin thin v v th ca cc hm s sau
a/
2
y x = . b/
2
y x 1 = . c/
2
y x 1 = .
d/
( )
2
y x 1 = . e/
( )
2
y x 1 = . f/
2
y x 2x 2 = .
g/
2
y 2x 6x 3 = . h/
2
y 4x 2x 6 = . i/
2
y 3x 6x 4 = .
k/
2
1
y x 2x 1
2
= . l/
2
y 2x 2 = . m/
2
y x 3x = .
Bi 138. Bi 138. Bi 138. Bi 138. Kho st v v th hm s
a/
2
y x 2x 1 = . b/
2
y y x 2 x 1 = = . c/
2
y x 4 x 3 = .
d/
2
1
y x 2 x 1
2
= . e/
2
y 2x 10x 12 = . f/
2
1 21
y x 5x
2 2
= .
Bi 139. Bi 139. Bi 139. Bi 139. Lp bng bin thin, ri tm gi tr ln nht (GTLN max) v gi tr nh nht (GTNN min)
ca hm s trn min xc nh c ch ra.
a/
2
y x 6x 1 trn 2; 7
l
=
l
l
. b/
2
y 6x 3x 4 trn 1; 2
l
=
l
l
.
c/
2
y x 5x 4 trn 1; 2
l
=
l
l
. d/
2
y x 3x 5 trn 3; 2
l
=
l
l
.
e/
( )

2
y 2x x 5 trn ; 3 4;
l
=
l
l
. f/
)

2
y 3x 4x trn 1;

.
g/
( )

2
y 2x 3 trn ; 6 5;
l
=
l
l
. h/
(

2
y 3x 6x trn ; 2
l
=
l
l
.
Bi 140. Bi 140. Bi 140. Bi 140. Xc nh Parabol
( ) ( )
2
P : y f x ax bx c = = trong cc trng hp sau, bit:
a/ Qua im
( )
A 8; 0 v c nh
( )
I 5;12 .
b/ Qua im
( )
A 3; 6 v c nh
( )
I 1; 4 .
c/ Qua im
( )
A 1; 2 v c nh
4 25
I ;
7 8
1



( )
.
d/ Qua im
( )
A 2; 3 v c nh
( )
I 1; 4 .
e/ C nh
( )
I 3; 6 v i qua im
( )
M 1; 10 .
f/ Qua ba im
( ) ( ) ( )
A 0; 1 , B 1; 1 , C 1;1 .
g/ Qua ba im
( )

3 7
A 1; , B 1; , C 2; 2
2 2
1 1





( ) ( )
.
h/ Qua ba im
( ) ( ) ( )
A 0; 3 , B 1; 2 , C 1;16 .
i/ Qua ba im
( ) ( ) ( )
A 2; 7 , B 1; 2 , C 3;2 .
j/ Qua im
( )
A 1;16 v ct trc honh ti hai im c honh l 1 v 5.
www.MATHVN.com
www.DeThiThuDaiHoc.com
cng hc tp mn Ton 10 tp I Ths. L Vn on


"Cn c b thng minh" Page - 33 -
k/ th nhn ng thng
4
x
3
= lm trc i xng v i qua hai im
( ) ( )
A 0; 2 , B 1; 7 .
l/ C trc i xng l x 2 = , i qua im
( )
A 1; 4 v c nh thuc ng thng y 2x 1 =
m/ C trc i xng l x 1 = , ct trc tung ti im c tung bng 1 v ch c mt giao im
vi trc honh.
Bi 141. Bi 141. Bi 141. Bi 141. Tm Parabol
( )
2
P : y ax bx 2 = trong cc trng hp sau:
a/ Parabol
( )
P i qua
( )
M 1; 5 v
( )
N 2; 8 .
b/ Parabol
( )
P i qua
( )
A 3; 4 v c trc i xng l
3
x
2
= .
c/ Parabol
( )
P c nh l
( )
I 2; 2 .
d/ Parabol
( )
P i qua
( )
B 1; 6 v c tung nh l
1
4
.
Bi 142. Bi 142. Bi 142. Bi 142. Tm im c nh ca h th
a/
2
y mx 2mx 3m = . b/
( )
2 2 2
y m x 2 m 1 x m = .
c/
( )
2
y m 1 x 2x 3m = . d/
2
y mx 2x m = .
e/
( )
3
y m 2 x m 2 = . f/
( )
3 2
y mx 2mx x 2 x m = .
Bi 143. Bi 143. Bi 143. Bi 143. Tm ta giao im ca cc ng sau:
a/
( )

2
d : y x 2 P : y x = = . b/
( )

2
d : y 2x 3 P : y x = = .
c/
( )

2
d : y x 1 P : y 2x = = . d/
( )
2
d: x y 1 0, P : y x 4x 3 0 = = .
e/
( )

2
d : 2x y 11 0 P : y x 6x 5 0 = = . f/
( )
2
d: x 2 y 0, P : 2y x 2x 8 0 = = .
Bi 144. Bi 144. Bi 144. Bi 144. Xc nh hm s
2
y ax bx c = trong cc trng hp sau
a/ i qua im
( )
A 0;1 v tip xc vi ng thng y x 1 = ti im
( )
M 1; 0 .
b/ i qua im
( )
A 0;1 v tip xc vi hai ng y x 1 = v ng y 2x 1 = .
c/ i qua im
( )
A 2; 3 v tip xc vi hai ng y 2x 7 = v ng y 4x 4 = .
d/ ia qua hai im
( ) ( )
A 0; 2 , B 2; 8 v tip xc vi trc honh Ox.
e/ Hm s t cc tiu bng 2 v th hm s ct ng thng y 2x 6 = ti hai im c
tung tng ng bng 2 v 10.
Bi 145. Bi 145. Bi 145. Bi 145. Cho cc hm s
( ) ( )
1
P : y 2x x 2 = v
( ) ( )( )
2
P : y x 1 x 2 = .
a/ V cc th hm s
( )
1
P v
( )
2
P trn cng mt h trc ta v tm giao im ca chng.
b/ nh a, b, c hm s
2
y ax bx c = c cc i bng 8 v th ca n qua giao im
ca
( )
1
P v
( )
2
P .
Bi 146. Bi 146. Bi 146. Bi 146. Cho Parabol
( )
2
P : y x 6x 5 = v ng thng d : y ax 1 2a = .
a/ Kho st v v th
( )
P v d trn cng mt h trc ta .
b/ Chng minh rng d lun i qua im c nh.
www.MATHVN.com
www.DeThiThuDaiHoc.com
Ths. L Vn on Phn i S


Page - 34 - "All the flower of tomorrow are in the seeks of today"
c/ Bng th v php ton. Chng minh
2
x 6x 5 ax 1 2a = lun c nghim.
Bi 147. Bi 147. Bi 147. Bi 147. Cho
( )
2
1
P : y x 4x 3 = v
( )
2
2
P : y x 2x 3 = .
a/ V
( )
1
P v
( )
2
P trn cng mt h trc ta .
b/ Tm ta giao im ca chng bng th v php tnh.
c/ nh m ng thng d : y m = ct mi th ti hai im phn bit.
d/ Gi s d ct
( )
1
P ti hai im phn bit A, B v d ct
( )
2
P ti hai im C, D. Tnh di
on AB, CD theo m.
e/ Tm m AB CD = .
Bi 148. Bi 148. Bi 148. Bi 148. Cho
( )
2
1
P : y x 4x 2 = v
( )
2
2
P : y x = .
a/ V
( )
1
P v
( )
2
P trn cng mt h trc ta .
b/ Bng php tnh, chng minh rng hai Parabol trn tip xc nhau.
c/ Gi A l tip im. Lp phng trnh ng thng d i qua A v song song vi ng thng
: y 2x 2013 = .
d/ ng thng d ct
( )
1
P ti M v ct
( )
2
P ti N. Tm ta im M v N. Chng minh rng A
l trung im ca MN.
e/ Vit phng trnh tip tuyn chung ca
( )
2
1
P : y x 4x 2 = v
( )
2
3
P : y x x 1 = .
Bi 149. Bi 149. Bi 149. Bi 149. Cho Parabol
( )
2
P : y x 6x 5 = .
a/ Kho st v v th hm s
( )
P .
b/ Gi A v B l giao im ca
( )
P v Ox
( )
A B
x x < . Vit phng trnh ng thng d i
qua A v c h s gc bng 1, ng thng qua B v vung gc vi d.
c/ Gi C l giao im ca d v . Chng minh rng ABC vung cn.
Bi 150. Bi 150. Bi 150. Bi 150. nh tham s m cc cp th sau khng ct nhau, ct nhau ti hai im phn bit
a/
( )
2
1
P : y 2x 3x 5 = v
( )
2
2
P : y 6x 9x 2m = .
b/
( )
2
1
P : y x 3mx 5m = v
( )
2
2
P : y 3x 5x m = .
Bi 151. Bi 151. Bi 151. Bi 151. nh tham s m cc cp th sau tip xc nhau (c mt im chung duy nht)
a/
( )
2
1
1
P : y x x 1
2
= v
( )
2
2
P : y x x m = .
b/
( )
2
1
1
P : y x x 4
3
= v
( )
2
2
2
P : y x x m
3
= .
c/
( )
2
1
P : y x x 3 = v
( )
2
2
P : y x x 2m = .
Bi 152. Bi 152. Bi 152. Bi 152. Cho
( )
2
1
P : y x x 1
2
= .
a/ Kho st s bin thin v v th hm s
( )
P .
b/ Vit phng tnh ng thng d i qua
( )
A 2; 0 v c h s gc k. Bin lun theo k s giao
im ca d v
( )
P .
c/ Mt ng thng i qua
( )
B 2; 0 v ct
( )
P theo mt dy cung nhn B lm trung im.
Tm phng trnh ng thng .
www.MATHVN.com
www.DeThiThuDaiHoc.com
cng hc tp mn Ton 10 tp I Ths. L Vn on


"Cn c b thng minh" Page - 35 -
Bi 153. Bi 153. Bi 153. Bi 153. Cho
( )
2
P : y x x 2 = .
a/ Kho st s bin thin v v th hm s
( )
P .
b/ Vit phng trnh ng thng d i qua im
( )
M 1; 1 c h s gc
1
2
. Tm ta giao
im A, B ca d v
( )
P .
c/ Cho im
( )
E 0; 2 . Chng minh rng
0
AEB 90 = .
Bi 154. Bi 154. Bi 154. Bi 154. nh tham s m hai ng thng ct nhau. Khi tm qu tch giao im ca hai th.
a/
( )

2
P : y x 5x 6 d : y 2m 1 = = .
b/
( )

2
P : y mx 3x 2m d : y mx 2 = = .
Bi 155. Bi 155. Bi 155. Bi 155. Cho
( ) ( )
P : y x 4 x 2 = .
a/ Bin lun theo m s giao im ca
( )
P v d : x y m 0 = .
b/ Trong trng hp d ct
( )
P ti hai im M, N. Tm qu tch trung im I ca MN.
Bi 156. Bi 156. Bi 156. Bi 156. Cho
( )
2
P : y ax bx c = .
a/ Xc nh hm s ca
( )
P qua im
( )
A 0; 3 v tip xc vi ng thng
( )
y 3x 1 =
ti im B v c honh bng 1.
b/ Cho ng thng d i qua im
( )
C 0; 2 v h s gc l m. Bin lun theo m s giao im
ca d v
( )
P .
c/ Trong trng hp d ct
( )
P ti hai im M, N. Tm qu tch trung im I ca on MN.
Bi 157. Bi 157. Bi 157. Bi 157. Cho
( )
2
P : y x 2x 3 = .
a/ Chng minh rng ng thng d : y mx = lun ct
( )
P ti hai im phn bit M, N. Tm
qu tch trung im on MN.
b/ Vi gi tr no ca m th hai tip tuyn ca
( )
P ti M, N vung gc nhau.
Bi 158. Bi 158. Bi 158. Bi 158. nh tham s m cc bt phng trnh sau c nghim
a/ 2 x m x 1 . b/
2
2 x m 2mx x 2 < .
Bi 159. Bi 159. Bi 159. Bi 159. Cho hm s
( )

2
y ax bx c P = .
Tm a, b, c tho iu kin c ch ra.
Kho st s bin thin v v th
( )
P ca hm s va tm c.
Tm m ng thng d ct
( )
P ti hai im phn bit A v B. Xc nh to trung im I
ca on AB.
a/
( )
P c nh
1 3
S ;
2 4
1

( )
v i qua im
( )
A 1;1 ; d : y mx = .
b/
( )
P c nh
( )
S 1;1 v i qua im
( )
A 0; 2 ; d : y 2x m = .

www.MATHVN.com
www.DeThiThuDaiHoc.com
Ths. L Vn on Phn i S


Page - 36 - "All the flower of tomorrow are in the seeks of today"
Chng











































Bi 160. Bi 160. Bi 160. Bi 160. Tm iu kin xc nh ca mi phng trnh v gii phng trnh
a/ . b/ .
c/ . d/ .
e/ . f/ .
2x 3 4x 3
2 2

=
5 5
3x 12
x 4 x 4
=

1 1
5x 15
x 3 x 3
=

2
1 1
x 9
x 1 x 1
=

2 2
3x 15
x 5 x 5
=

( )
2 2 4 2
x 1 x 1 3
x x 1 x x 1 x x x 1

=

C CC C Phng trnh mt n .
l mt nghim ca nu l mt mnh ng.
Gii phng trnh l tm tt c cc nghim ca phng trnh .
Khi gii phng trnh ta thng tm iu kin xc nh ca phng trnh.
O Lu
Khi tm iu kin xc nh (TX) ca phng trnh, ta thng gp cc trng hp sau
o Nu trong phng trnh c cha biu thc th cn iu kin .
o Nu trong phng trnh c cha biu thc th cn iu kin .
o Nu trong phng trnh c cha biu thc th cn iu kin .
Cc nghim ca phng trnh l honh cc giao im ca th hai hm
s v .
C CC C Phng trnh tng ng, phng trnh h qu
Cho hai phng trnh c tp nghim S
1
v
khi v ch khi .
khi v ch khi .

( ) ( ) ( )
f x g x , 1 =
o
x
( )
1
( ) ( )
o o
" f x g x " =
( )
1
P x
( )
P x 0
( )
P x
( )
P x 0
( )
1
P x
( )
P x 0
( ) ( )
f x g x =
( )
y f x =
( )
y g x =
( ) ( ) ( )

1 1
f x g x 1 =
( ) ( ) ( )

2 2
f x g x 2 =
( ) ( )
1 2
1 2
S S =
( ) ( )
1 2
1 2
S S
C CC C Php bin i tng ng
Nu mt php bin i phng trnh m khng lm thay i iu kin xc nh ca
n th ta c mt phng trnh tng ng. Ta thng s dng cc php bin
i sau
+ Cng hai v ca phng trnh vi cng mt biu thc.
+ Nhn hai v ca phng trnh vi mt biu thc c gi tr khc 0.
Khi bnh phng hai v ca mt phng trnh, ni chung ta c mt phng
trnh h qu. Khi ta phi kim tra li loi b nghim ngoi lai.
A I CNG V PHNG TRNH
PHNG TRNH V H PHNG TRNH V H PHNG TRNH V H PHNG TRNH V H PHNG TRNH PHNG TRNH PHNG TRNH PHNG TRNH
3
www.MATHVN.com
www.DeThiThuDaiHoc.com
cng hc tp mn Ton 10 tp I Ths. L Vn on


"Cn c b thng minh" Page - 37 -
g/ . h/ .
i/ .
Bi 161. Bi 161. Bi 161. Bi 161. Tm iu kin xc nh ca mi phng trnh v gii phng trnh
a/ . b/ .
c/ . d/ .
e/ . f/ .
g/ . h/ .
i/ . j/ .
k/ . l/ .
m/ . n/ .
o/ . p/ .
q/ . r/ .
s/ . t/ .
Bi 162. Bi 162. Bi 162. Bi 162. Tm iu kin xc nh ca mi phng trnh v gii phng trnh
a/ . b/ .
c/ . d/ .
Bi 163. Bi 163. Bi 163. Bi 163. Tm iu kin xc nh ca mi phng trnh v gii phng trnh
a/ . b/ .
c/ . d/ .
Bi 164. Bi 164. Bi 164. Bi 164. Tm iu kin xc nh ca mi phng trnh v gii phng trnh
a/ . b/ .
c/ . d/ .
Bi 165. Bi 165. Bi 165. Bi 165. Tm cc tham s m cc cp phng trnh sau y tng ng nhau
a/ v .
b/ v .
c/ v .
d/ v . e/ v .
9 x 11 x
2
2009 2011

=
15 x 17 x 19 x
3
2010 2012 2014

=
x 2014 x 2012 x 2010 x 2007 x 2009 x 2011
2007 2009 2011 2014 2012 2010

=
x 1 x 3 x 1 = x 5 x 2 x 5 =
2x 1 x 2
x 3 x 3

=

2
2x 8
x 1 x 1
=

1 1 x x 2 = x 1 2 x =
1
2x 1
x
=
x 3
x 1 x 1
=

x 1 x 1 = x 1 1 x =
2
4 x 3
2x 3
x 1 x 1

=

2
2
x 1
3x x 1
2x 1

x 2
x 1 x 3
=

2
2x 3
x 1
x 4

x 3 x x 3 3 =
2
x 2 x 3 x 4 =
2
x x 1 4 x 1 =
2
3x 1 4
x 1 x 1

=

2
x 3x 4
x 4
x 4

=

2
3x x 2
3x 2
3x 2

=

( )
2
x 3 x 3x 2 0 =
( )
2
x 1 x x 2 0 =
x 1
x 2
x 2 x 2
=

2
x 4 x 3
x 1
x 1 x 1

=

x 2 x 1 = x 1 x 2 =
2 x 1 x 2 = x 2 2x 1 =
x
x
x 1 x 1
=

x 2
x 2
x 1 x 1


=

x
x
2 x 2 x
=

x 1
1 x
x 2 x 2


=

( )
2
x 1 0 =
( )
2
mx 2m 1 x m 0 =
x 2 0 =
mx
3m 1 0
x 3
=

2
x 9 0 =
( ) ( )
2
2x m 5 x 3 m 1 0 =
( )
3x 2 0 =
( )
m 3 x m 4 0 = x 2 0 =
( )
2 2
m x 3x 2 mx 2 0 =
www.MATHVN.com
www.DeThiThuDaiHoc.com
Ths. L Vn on Phn i S


Page - 38 - "All the flower of tomorrow are in the seeks of today"

















BI TP P DNG
Bi 166. Bi 166. Bi 166. Bi 166. Gii v bin lun cc phng trnh sau theo tham s m
a/ . b/ .
c/ . d/ .
e/ . f/ .
g/ . h/ .
i/ . j/ .
Bi 167. Bi 167. Bi 167. Bi 167. Gii v bin lun cc phng trnh sau theo tham s m
a/ . b/ .
c/ . d/ .
Bi 168. Bi 168. Bi 168. Bi 168. Gii v bin lun cc phng trnh sau theo tham s m
a/ . b/ .
c/ . d/ .
e/ . f/ .
g/ . h/ .
Bi 169. Bi 169. Bi 169. Bi 169. Gii v bin lun cc phng trnh sau theo tham s m
a/ . b/ .
c/ . d/ .
mx 5 =
( )
m 1 x m 1 =
( )
2m 1 x m 3 =
( )
m 1 x 2m 2 =
( )
m x 2 3x 1 =
( )
m 1 x 2x m 3 =
( )( )
m 1 x 2 3m 1 =
( )( )
2
m 1 x 1 m 1 =
( ) ( )
m 3 x m m 1 6 =
( ) ( )
2m 3 x m 2m 5 3 =
2mx 2x m 4 =
( )
m x 4 5x 2 =
( )
m x 3 x m =
( )( )
m 1 x 2 3m 1 =
( )
m m 2 x m 2 =
( )
2 2
m 3m x m 9 =
( )
2
m 3m 2 x m 2 =
( )
2
m x 1 mx 1 =
( )
2 2
m m x 2x m 1 =
( ) ( )
2
m m 1 x m m 1 =
( ) ( )( )
2
m 1 x m m 1 m 2 =
( ) ( )
m x m 3 m x 2 6 =
x m 1 m 1 =
( )
m 1 x m 1 =
x m 1
x 1
x 1

x m mx
3x 2
3x 2 3x 2

=

B PHNG TRNH BC NHT ax b =

Xt phng trnh bc nht:
H s Kt lun
c nghim duy nht


v nghim

nghim ng vi mi x

( )
ax b 0 1 =
a 0 ( )
1
b
x
a
=
a 0 =
b 0
( )
1
b 0 =
( )
1
O OO O Ch : Khi a 0 th c gi l phng trnh bc nht mt n.
( )
1
www.MATHVN.com
www.DeThiThuDaiHoc.com
cng hc tp mn Ton 10 tp I Ths. L Vn on


"Cn c b thng minh" Page - 39 -
Bi 170. Bi 170. Bi 170. Bi 170. Gii v bin lun cc phng trnh sau theo tham s
a/ . b/ .
c/ . d/ .
e/ . f/ .
g/ . h/ .
Bi 171. Bi 171. Bi 171. Bi 171. Gii v bin lun cc phng trnh sau theo tham s
a/ . b/ .
c/ . d/ .
e/ . f/ .
Bi 172. Bi 172. Bi 172. Bi 172. Gii v bin lun cc phng trnh sau theo tham s
a/ . b/ .
c/ . d/ .
Bi 173. Bi 173. Bi 173. Bi 173. Cho phng trnh: .
a/ Tm tham s m phng trnh c nghim.
b/ Tm tham s m phng trnh c nghim duy nht .
c/ Tm m l nghim ca phng trnh .
Bi 174. Bi 174. Bi 174. Bi 174. Cho phng trnh .
a/ Tm tham s m phng trnh c nghim.
b/ Tm tham s m phng trnh c nghim duy nht .
c/ Tm m l mt nghim ca phng trnh .
Bi 175. Bi 175. Bi 175. Bi 175. Tm tham s m cc phng trnh sau y v nghim.
a/ . b/ .
c/ . d/ .
mx 2
m 4 m 4

=

( )
2
m 2 x
m 4
2m 3 m 1


=

( )
2
mx m 1
m x
2m 5 2m 5

=

2 2
m x 2mx m 1
1
m 5 m 5

=

( )
2
x ab x bc x b
3b, a,b,c 1
a 1 c 1 b 1

=

( )

x a x b
b a a, b 0
a b

=
( )
,
x b c x c a x a b
3 a,b,c 0
a b c

=
( ) ( )
a ab 2 x a 2b b 2 x =
3
m
x 1
=

2m 1
m 3
x 2

mx 2m
2
x 2

( )
m 1 x m 2
m
x 3

=

( )
2
m 3 x 6
m
x 1

=

2 2
m x m
m
x 1

x m 2x = 3x m x m 2 =
2mx 3 5 = mx 2 x m =
( )( ) ( )

2
m 1 x 2 1 m =
( )

( )
x 3 =
x 3 =
( )

( ) ( )

2 2
m m x 2x m 1 =
( )

( )
x 0 =
x 1 =
( )

( ) ( )
m 1 x x 2 0 =
( ) ( )
2
m x 1 2 2x m 4 =
x m x 2
2
x 2 x

=

x 1 m 1
x x

=
www.MATHVN.com
www.DeThiThuDaiHoc.com
Ths. L Vn on Phn i S


Page - 40 - "All the flower of tomorrow are in the seeks of today"
Bi 176. Bi 176. Bi 176. Bi 176. nh tham s tp nghim ca cc phng trnh sau l .
a/ . b/ .
c/ . d/ .
e/ . f/ .
g/ . h/ .
j/ . l/ .
Bi 177. Bi 177. Bi 177. Bi 177. Tm tham s m cc phng trnh sau y c nghim
a/ . b/ .
c/ . d/ .
e/ . f/ .
g/ vi . h/ .
Bi 178. Bi 178. Bi 178. Bi 178. Tm tham s m cc phng trnh sau y c nghim duy nht
a/ . b/ .
c/ . d/ .
e/ . f/ .
Bi 179. Bi 179. Bi 179. Bi 179.
***
nh cc tham s m cc phng trnh sau y c nghim
a/ . b/ .
c/ . d/ .
e/ . f/ .
g/ . h/ .
i/ . j/ .
k/ . l/ .
Bi 180. Bi 180. Bi 180. Bi 180. Tm phng trnh c nghim nguyn
a/ . b/ .
b/ . d/
c/ . d/ .

R
( )
m 2 x m 1 = mx 3 3x m =
2
3mx 1 x 9m =
( ) ( )
2
m x 1 2 mx 2 =
( )
2
m 2m 3 x m 1 =
( ) ( )
2
m mx 1 2m 2x 1 =
( )( ) ( )
2
mx 2 x 1 mx m x =
( )( ) ( )
2
mx 2 x 1 mx m x =
2ax b 4 bx 5x a =
( ) ( )
2
m 1 x 2m 5 x 2 m =
( )
2
m x 1 x m =
( )
2 2
m x m x m =
( )
2
m 2 x 2m x 3 =
( )
m x m x m 2 =
( ) ( )
2
m x 1 m x 3m 2 =
( )
2 2
m m x 2x m 1 =
( )
2
m x 1 4x 5m 4 = x 0
( )
m 3 x 2
m 1
x 2

=

( )
2
m m 1 x m 1 =
( ) ( )
2
m mx 1 2m 2x 1 =
x 2 x 3
0
x m x 1

=

x m x 2
2
x 2 x

=

2x m x 1 = mx 2 x 4 =
x 1 2x 3 m =
( )
2 x m 1 x m 3 =
2
mx
m x 2m 1
x 1
=

( )
2
a x x 1 2a
, x 0
a 1 a 1 a 1

=

3x m 2x 5m 3
x 1
x 1 x 1

=

2mx 1 m 1
2 x 1
x 1 x 1

=

( ) ( )
2 2
2m 1 x 3 2m 3 x m 2
4 x 4 x

=

2
x x 2m
x x 1 m x 1
x 1

=

x m x 3
x 1 x 2

=

x m x 1
2
x 1 x m

=

( )
m 1 x m 2
m
x 3

=

x x
x m x 1
=

m Z
( )
m 2 x m 1 =
( )
m 1 x 2x m 3 =
( )
m x 3 x m =
( )( )
m 1 x 2 3m 1 =
( ) ( )
2m 3 x m 2m 5 3 =
( )
3m 2 x m 4mx 2m 5 =
www.MATHVN.com
www.DeThiThuDaiHoc.com
cng hc tp mn Ton 10 tp I Ths. L Vn on


"Cn c b thng minh" Page - 41 -
BI TP RN LUYN
Bi 181. Bi 181. Bi 181. Bi 181. Gii v bin lun phng trnh sau theo tham s m
a/ . b/ .
c/ . d/ .
e/ . f/ .
g/ . h/ .
i/ . j/ .
Bi 182. Bi 182. Bi 182. Bi 182. Gii v bin lun phng trnh sau theo tham s m
a/ . b/ .
c/ . d/ .
e/ . f/ .
g/ . h/ .
i/ . j/ .
k/ . l/ .
m/ . n/ .
o/ . p/ .
q/ . r/ .
s/ . t/ .
u/ . v/ .
w/ . x/ .
y/ . z/ .
/ . / .
Bi 183. Bi 183. Bi 183. Bi 183. Gii v bin lun phng trnh sau theo tham s m
a/ . b/ .
c/ . d/ .
e/ . f/ .
g/ . h/ .
i/ . j/ .
mx m 1 =
( )
m 1 x 2m 1 =
( )
m 3 x 2m 4 =
( )
m 1 x 2m 2 =
( )
m x 1 3m 2 =
( )
m x 4 5x 2 =
( )
3m 1 x m 2x 1 =
( )( )
m 1 x 4 2x 3 =
( )
2
m 1 x m 3m 2 =
( ) ( )
2m 3 x m 2m 5 3 =
mx 3 3x m =
( )
m 1 x 2x m 3 =
( )
3m 1 x m 2x 1 =
( )( )
2
m 1 x 1 m 1 =
( )
2
m m 1 x m 1 =
( )
2
2m m 3 x m 1 =
2
m x 3 9x m =
( )
2
m 2 x 2m x 3 =
( ) ( )
m mx 1 4m 3 x 3 =
2 2
m x 4m 3 x m =
( ) ( )
2
m 1 x m 2m 5 x 2 =
( )
3 2
m x 1 m x 1 =
( ) ( )
3 m 1 x 4 2x 5 m 1 =
( ) ( )
2 m 1 x m x 1 2m 3 =
( ) ( )
m 4x m 3 4mx 2m 6 =
( ) ( )
2
m x m 4 m x m 4x =
mx 1 m 1
x 2x
2 3

=
( )
mx 1 mx 1
m 1 x 2x
3 2

=
( ) ( )
2
m 2 x m 1 4x 2 8x =
( ) ( )
2
m x m 5x 2 6 1 x =
( ) ( )
2
m x 1 m x 3m 2 =
2 2
m x 4m 3 m 6mx 5x =
( ) ( )
2
x 1 m 2x 1 m x 2 0 =
( ) ( )
2 2
m x 1 3mx m 3 x 1 =
( )
2
a ax b 4ax b 5 =
( )
a 2 ax 2b ax =
( ) ( )
2 3 2
a ax 2b a b x a =
2 2 2 2
a x 2ab b x a b =
x 2m 1 2m 1 =
x 1 m
x x

=
x m m
2x 1
2x 1 2x 1

=

( )
x 2m x 4 0 =
( )
mx 1 x 1 0 =
( )
m 1 x
2m 1
2m 3 2m 3


=

( )
2mx m 1
mx
3m 1 3m 1

=

( )
m 1 x
3mx
m m 1

2
m x x 1 2a
m 1 m 1 a 1

=

2m 3
m 4 0
x 3

www.MATHVN.com
www.DeThiThuDaiHoc.com
Ths. L Vn on Phn i S


Page - 42 - "All the flower of tomorrow are in the seeks of today"
k/ . l/ .
m/ . n/ .
o/ . p/ .
q/ . r/ .
s/ . t/ .
u/ . v/ .
x/ . y/ .
Bi 184. Bi 184. Bi 184. Bi 184. Gii v bin lun cc phng trnh sau theo tham s
a/ . b/ .
c/ . d/ .
e/ . f/ .
g/ . h/ .
i/ . j/ .
Bi 185. Bi 185. Bi 185. Bi 185. nh tham s m cc phng trnh sau v nghim
a/ . b/ .
c/ . d/ .
e/ . f/ .
Bi 186. Bi 186. Bi 186. Bi 186. nh tham s phng trnh sau c tp nghim l .
a/ . b/ .
c/ . d/ .
e/ . f/ .
Bi 187. Bi 187. Bi 187. Bi 187. nh tham s m phng trnh sau c nghim
a/ . b/ .
c/ d/ .
Bi 188. Bi 188. Bi 188. Bi 188. nh tham s m phng trnh sau c nghim duy nht
a/ . b/ .

( )
mx m 2
4
2x 1

=

( )
2
m 1 x 10
m 1
x 2

=

x 2 x 1
x m x 1

=

m
2
mx 3
=

3mx 2 5m
m
x 2 x 2

=

x m x 2
2
x 1 x

=

5x m 4x 3
1
x 2 x

=

5x m 4x 3
3
2x 1 x

=

5m 2
3
mx 1

x 2 x
x 3 x m

=

x 2 3
2
x m x 1

=

5 3
2x m 4 mx
=

ax b x b
x a x a

=

x a x b x c
3
b c c a a b

=

x 2m x 1 = m 4x x 3m =
mx 2x 1 x = mx 3x x m =
3x m 2x 2m = 3m x 5x 4m =
mx 2 x 4 = mx x 1 x 2 =
mx 1 2x m 3 = ax b bx a =
( )
2
4m 2 x 1 2m x =
( ) ( )
2
m 1 x 2 4m 9 x m =
x 2 x
x 3 x m

=

x m 2
1
m 1 x

x m x 2
2
x 1 x

=

x 1 x
x m 1 x m 2

=

R
2 2
m x m 2 m 4x =
( )
2
m x 1 9x m 6 =
2 2
m x 4m 3 x m =
3 2
m x mx m m =
ax b 6x 2bx a =
( ) ( )
a x 1 b 2x 1 x 2 =
2 2
m x 4x m m 2 =
( )
2
m x m x m =
( ) ( )
2
m x 1 4x 3m 2, x 0 =
2x m x m 1
1
x 1 x

=

x 2 x 1
x m x 1

=

m
2
mx 3
=

www.MATHVN.com
www.DeThiThuDaiHoc.com
cng hc tp mn Ton 10 tp I Ths. L Vn on


"Cn c b thng minh" Page - 43 -






































BI TP P DNG
Bi 189. Bi 189. Bi 189. Bi 189. Gii v bin lun phng trnh bc hai
a/ . b/ .
c/ . d/ .
e/ . f/ .
( )
2
x 2 m 1 x 2m 5 0 =
2
2x 12x 15m 0 =
( ) ( )
2
m 1 x 2 m x 1 0 =
( )
2
mx 2 m 3 x m 1 0 =
( )( ) ( )
2
2m 1 m 2 x 5m 4 x 3 0 =
( )
2 2
2m 5m 2 x 2mx 2 0 =
Dng ton 1. Gii v bin lun phng trnh =
2
ax bx c 0
C PHNG TRNH BC HAI
( )
2
ax bx c 0, a 0 + + =

C CC C Cch gii


Kt lun

c 2 nghim phn bit .

c nghim kp .

v nghim

Nhm nghim
Nu th c hai nghim l v .
Nu th c hai nghim l v .


( ) ( )

2
ax bx c 0, a 0 =
2
b 4ac =
0
( )

1,2
b
x
2a

=
0 =
( )

b
x
2a
=
0 <
( )

a b c 0 =
( )
x 1 =
c
x
a
=
a b c 0 =
( )
x 1 =
c
x
a
=
C CC C nh l Vit
Hai s l cc nghim ca phng trnh bc hai khi v ch
khi chng tho mn cc h thc v .
1 2
x , x
2
ax bx c 0 =
1 2
b
S x x
a
= =
1 2
c
P x x
a
= =
gii v bin lun phng trnh ta cn xt cc trng hp c th xy ra
ca h s a:
Nu th tr v gii v bin lun phng trnh bc nht .
2
ax bx c 0 =
a 0 = bx c 0 =
Nu th ta xt cc trng hp ca bit s nh trn.

a 0
www.MATHVN.com
www.DeThiThuDaiHoc.com
Ths. L Vn on Phn i S


Page - 44 - "All the flower of tomorrow are in the seeks of today"
g/ . h/ .
Bi 190. Bi 190. Bi 190. Bi 190. nh tham s m cc phng trnh sau y c nghim
c/ . d/ .
g/ . h/ .
k/ . l/ .
o/ . p/ .
BI TP RN LUYN
Bi 191. Bi 191. Bi 191. Bi 191. Gii v bin lun phng trnh bc hai
a/ . b/ .
c/ . d/ .
e/ . f/ .
g/ . h/ .
Bi 192. Bi 192. Bi 192. Bi 192. nh tham s m cc phng trnh sau y c nghim
a/ . b/ .
c/ . d/ .
e/ . f/ .
g/ . h/ .
i/ . j/ .





















( )
2
m 3 x x 2m 1 0 =
( )( ) ( )
2
m 1 m 2 x 2m 3 x 1 0 =
( )
2 2
x 2 m 2 x 2m 4m 5 0 =
2
mx x 1 0 =
( ) ( )
2
x 2m 1 x m m 1 0 =
( )
2
x m 2 x 1 m 0 =
( ) ( )
2
3mx 4 6m x 3 m 1 0 =
( ) ( )
x 1 m 1 x 2 0
l
=
l
l
( )( )
mx 2 2mx x 1 0 =
( )
2
mx 1 2m x m 4 0 =
( )
2
4x 2 m 3 x 3 0 =
( ) ( )
2
m 1 x 2 m x 1 0 =
( )( ) ( )
2
2m 1 m 2 x 5m 4 x 3 0 =
( )
2
m 3 x x 2m 1 0 =
2
x 5x 3m 1 0 =
( ) ( )
2
m 1 x 2 m 1 x m 2 0 =
( )
2
m 1 x 2x 1 0 =
( ) ( )
2
m 2 x m 1 x m 0 =
2
2x 3x m 1 0 =
( ) ( )
2
m 1 x 2 m 1 x m 0 =
( )
2 2 2
x m 1 x m 2 0 =
( ) ( )
2
m 1 x 2m 1 x m 2 0 =
( ) ( )
2
m 2 x 2 m 3 x m 5 0 =
( ) ( )
2 2
m 1 x 2 m 3 x 1 0 =
( ) ( )
2
m m 1 x 2m 1 x 1 0 =
( )
2
mx 2 m 3 x m 1 0 =
( ) ( )
2 2
m 5x 36 x 2 m 4 x 1 0 =
( ) ( )
mx 3 m 1 x 3 0
l
=
l
l
Dng ton 2. Du ca nghim s ca phng trnh
( ) ( )
=
2
1 ax bx c 0, a 0
O c hai nghim tri du . O c hai nghim cng du .
O c hai nghim dng . O c hai nghim m .
( )
1 P 0 <
( )
1
0
P 0
'
1
1

!
1
1
+
( )
1
0
P 0
S 0
'
1
1
1
1

!
1
1
1
1+
( )
1
0
P 0
S 0
'
1
1
1
1

!
1
1
< 1
1+
- Lu : Trong cc trng hp trn nu yu cu hai nghim phn bit th > 0.
www.MATHVN.com
www.DeThiThuDaiHoc.com
cng hc tp mn Ton 10 tp I Ths. L Vn on


"Cn c b thng minh" Page - 45 -
BI TP P DNG
Bi 193. Bi 193. Bi 193. Bi 193. Tm tham s m cc phng trnh sau c hai nghim phn bit tri du
a/ . b/ .
c/ . d/ .
e/ . f/ .
g/ . h/ .
Bi 194. Bi 194. Bi 194. Bi 194. nh tham s m cc phng trnh sau c hai nghim dng phn bit
a/ . b/ .
c/ . d/ .
e/ . f/ .
Bi 195. Bi 195. Bi 195. Bi 195. nh tham s m cc phng trnh sau c hai nghim m phn bit
a/ . b/ .
c/ . d/ .
e/ . f/ .
Bi 196. Bi 196. Bi 196. Bi 196. Cho phng trnh: . Tm tham s m
a/ Phng trnh c duy nht mt nghim m.
b/ Phng trnh c hai nghim m phn bit.
Bi 197. Bi 197. Bi 197. Bi 197. Cho phng trnh: . Tm tham s m
a/ Phng trnh c duy nht mt nghim dng.
b/ Phng trnh c hai nghim dng phn bit.
Bi 198. Bi 198. Bi 198. Bi 198. Cho phng trnh: . Tm tham s m
a/ Phng trnh v nghim
b/ Phng trnh khng c nghim dng.
Bi 199. Bi 199. Bi 199. Bi 199. Cho phng trnh: . Tm tham s m
a/ Phng trnh v nghim.
b/ Phng trnh khng c nghim m.
Bi 200. Bi 200. Bi 200. Bi 200. Cho phng trnh: . Tm tham s m
a/ Phng trnh c hai nghim duy nht v hai nghim m.
b/ Phng trnh c t nht mt nghim dng.
Bi 201. Bi 201. Bi 201. Bi 201. Cho phng trnh: . Tm tham s m
a/ Phng trnh c nghim.
b/ Phng trnh c hai nghim phn bit. (HD: t ).
2
x 5x 3m 1 0 =
2
2x 12x 15m 0 =
2
x 4x m 1 0 =
2
mx mx m 2 0 =
( )
2
2
m 2 x x m 3 0 =
( )
2 2
2m m 1 x 2x m 0 =
( ) ( )
2
m 1 x 2 m x 1 0 =
( ) ( )
2
m 1 x 2 m 4 x m 1 0 =
( )
2 2 2
mx m 2m 2 x m 5m 6 0 =
( )
2
mx 2 3m x 6 0 =
( ) ( )
2 2
m 1 x 2m 2m 1 x 2m 0 =
( )
2
x 2m 1 x m 0 =
( )
2
m 1 x 2mx m 0 =
( )
2 2
m 1 x mx 1 0 =
( )
2 2
x 3m 2 x 2m 3m 1 0 =
2 2
m x mx 6 0 =
2
x 2mx 4m 1 0 =
( )
2
x 1 m x 2 m 0 =
( ) ( )
2
m 1 x 2 m 2 x m 2 0 =
( )
2
2m 1 x 2x 1 0 =
( )

2
mx 2mx m 1 0 =
( )

( )

( ) ( ) ( )

2
m 1 x 2 m 3 x m 0 =
( )

( )

( ) ( ) ( )

2
2
m 2 x m 1 x m 0 =
( )

( )

( ) ( ) ( )

2 2
m 1 x m 1 x 2m 2 0 =
( )

( )

( ) ( ) ( )

2
x 2 mx m 3 x m 3 0
l
=
l
l
( )

( )

( ) ( )

2
x 1 x 4x 1 m 0 =
( )

( )
t x 1 =
www.MATHVN.com
www.DeThiThuDaiHoc.com
Ths. L Vn on Phn i S


Page - 46 - "All the flower of tomorrow are in the seeks of today"
Bi 202. Bi 202. Bi 202. Bi 202. Bin lun theo m s nghim m, s nghim dng ca cc phng trnh sau
.
Bi 203. Bi 203. Bi 203. Bi 203. nh m phng trnh c nghim tha:
a/ .
b/ .
c/ .
d/ .
e/ .
f/ .
BI TP RN LUYN
Bi 204. Bi 204. Bi 204. Bi 204. Tm tham s m cc phng trnh sau c hai nghim phn bit tri du
a/ . b/ .
c/ . d/ .
e/ . f/ .
g/ . h/ .
Bi 205. Bi 205. Bi 205. Bi 205. nh tham s m cc phng trnh sau c hai nghim dng phn bit
a/ . b/ .
c/ . d/ .
e/ . f/ .
Bi 206. Bi 206. Bi 206. Bi 206. nh tham s m cc phng trnh sau c hai nghim m phn bit
a/ . b/ .
c/ . d/ .
e/ . f/ .
Bi 207. Bi 207. Bi 207. Bi 207. Cho phng trnh: . Tm tham s m
a/ Phng trnh c t nht mt nghim m.
b/ Phng trnh c hai nghim tri du.
c/ Phng trnh c duy nht mt nghim dng.
Bi 208. Bi 208. Bi 208. Bi 208. Cho phng trnh: . Tm tham s m
a/ Phng trnh c ba nghim dng phn bit.
b/ Phng trnh c hai nghim dng phn bit.
Bi 209. Bi 209. Bi 209. Bi 209. Bin lun theo m s nghim m, s nghim dng ca cc phng trnh sau
( )
2 2 2
mx m 3m 1 x 2m 3m 1 0 =
1 2
x , x

2 2
1 2
x 2mx m 0 x x 1 = < <
( )

2 2
1 2
x 2 m 1 x m 1 0 2 x x = < <
( )

2
1 2
x m 2 x m 1 0 x 3 x = < <
( )

2
1 2
x 2m 3 x 3m 1 0 x 1 x =

2
1 2
mx 2mx m 3 0 x x 4 =
( )

2
1 2
m 1 x 2mx m 0 3 x x =
2
x 2x m 0 =
( )
2 2
x 2 m 1 x m 0 =
( )
2
mx m 1 x m 1 0 =
( )
2
mx 2 m 3 x m 1 0 =
2 2
x x m 1 0 =
2 2
x 2mx m 2m 0 =
( )
2
m 2 x 2x m 2 0 =
( ) ( )
2
m 1 x 2 m 1 x m 2 0 =
( )
2 2
x 1 3m x 2m 2m 0 =
( )
2 2
x 5 2m x m 5m 6 0 =
( )
2
x 2m 3 x m 3 0 =
2 2
x 4mx 3m 0 =
2
x 3x m 1 0 =
( )
2
mx 2 m 1 x m 2 0 =
( )
2 2
x 3m 1 x 2m m 0 =
( )
2 2
mx 2m m 1 x 2m 1 0 =
( )
2
x 2 m 1 x 3m 1 0 =
( )
2 2
x 1 4m x 3m m 0 =
( )
2
mx 4m 1 x 4m 2 0 =
2
mx x m 1 0 =
( ) ( ) ( )

2 2
m 4 x 2 m 1 x 1 0 =
( )

( )

( )

( ) ( ) ( ) ( )

2
x 1 m 1 x m 1 x 4 0
l
=
l
l
( )

( )

www.MATHVN.com
www.DeThiThuDaiHoc.com
cng hc tp mn Ton 10 tp I Ths. L Vn on


"Cn c b thng minh" Page - 47 -
.
Bi 210. Bi 210. Bi 210. Bi 210. nh m phng trnh c nghim tha:
a/ .
b/ .
c/ .
d/ .
e/ .
f/ .
































BI TP P DNG
Bi 211. Bi 211. Bi 211. Bi 211. Gi x
1
, x
2
l cc nghim ca phng trnh. Khng gii phng trnh, hy tnh
.
a/ . b/ .
( )
2 2 2
mx m m 1 x m m 0 =
1 2
x , x

2 2
1 2
x 2x m 2m 0 x 2 x = < <
( )

2 2
1 2
2x m 3 x m 3m 2 0 x x 0 = < <
( )

2 2
1 2
2x m 6 x m 3m 0 1 x x =
( )

2 2
1 2
mx 2m m 1 x 2m 1 0 x x 5 =
( ) ( )

2 2 2
1 2
m 1 x m m 1 x m m 0 4 x x = <
( ) ( )

2 2 2 2
1 2
m 2m x 2 m m 1 x m 1 0 x 2 x =
( )( )

2 2 3 3 4 4
1 2 1 2 1 2 1 2 1 2 2 1
A x x , B x x , C x x , D x x , E 2x x 2x x = = = = =
2
x x 5 0 =
2
2x 3x 7 0 =
Dng ton 3. Nhng bi ton lin quan n nh l Vit

C CC C Biu thc i xng ca cc nghim s
Ta s dng cng thc biu din cc biu thc i xng
ca cc nghim x
1
, x
2
theo S v P. Chng hn nh:

C CC C H thc ca cc nghim c lp i vi tham s
tm h thc ca cc nghim c lp i vi tham s ta tm
(S, P c cha tham s m).
Kh tham s m gia S v P ta tm c h thc gia x
1
v x
2
.

1 2 1 2
b c
S x x ; P x x
a a
= = = =
( ) ( )
( )
( ) ( )
2 2
2
1 2 1 2 1 2
2
2 2 2
1 2 1 2 1 2
2
3 3 2
1 2 1 2 1 2 1 2
x x x x 4x x S 4P
x x x x 2x x S 2P
x x (x x ) x x 3x x S S 3P
........................

= =

= =

= =
l
l

1 2 1 2
b c
S x x ; P x x
a a
= = = =
C CC C Lp phng trnh bc hai
Nu phng trnh bc hai c cc nghim u v v th phng trnh bc hai c dng:
, trong
2
x Sx P 0 =
S u v
P uv
'
1 =
1
!
1 =
1
+
www.MATHVN.com
www.DeThiThuDaiHoc.com
Ths. L Vn on Phn i S


Page - 48 - "All the flower of tomorrow are in the seeks of today"
c/ . d/ .
e/ . f/ .
Bi 212. Bi 212. Bi 212. Bi 212. nh tham s m phng trnh c mt nghim cho trc. Tnh nghim cn li
a/ .
b/ .
c/ .
d/ .
Bi 213. Bi 213. Bi 213. Bi 213. Tm hai s c
a/ Tng l 19 v tch l 84. b/ Tng l 5 v tch l .
c/ Tng l v tch l 16. d/ Tng l 12 v tch l 32.
Bi 214. Bi 214. Bi 214. Bi 214. Tm tui ca mt hc sinh, bit rng sau 7 nm na tui ca em s bng bnh phng s tui
ca em cch y 5 nm.
Bi 215. Bi 215. Bi 215. Bi 215. Tm di ba cnh ca mt tam gic vung bit cnh di nht hn cnh th hai l 2m v cnh
th hai hn cnh ngn nht l 23m.
Bi 216. Bi 216. Bi 216. Bi 216. Mt ming t hnh ch nht c chiu di gp i chiu rng. Nu tng chiu rng thm 3m v
chiu di tng 4m th din tch ming t tng gp i. Hi kch thc ming t lc u ?
Bi 217. Bi 217. Bi 217. Bi 217. Tm di ba cnh ca mt tam gic vung c chu vi bng 30m, bit hai cnh gc vung hn
km nhau 7m ?
Bi 218. Bi 218. Bi 218. Bi 218. nh m phng trnh bc hai c nghim x
1
, x
2
tha ng thc theo sau
a/ .
b/ .
c/ .
d/ .
e/ .
f/ .
g/ .
h/ .
i/ .
j/ .
Bi 219. Bi 219. Bi 219. Bi 219. Cho phng trnh: .
a/ Xc nh tham s m phng trnh c hai nghim phn bit.
b/ Vi gi tr no ca m th phng trnh c hai nghim v tch ca chng bng 8. Tm cc
nghim trong trng hp .
Bi 220. Bi 220. Bi 220. Bi 220. Cho phng trnh: .
a/ Xc nh tham s m phng trnh c nghim kp v tm nghim kp .
2
3x 10x 3 0 =
2
x 2x 15 0 =
2
2x 5x 2 0 =
2
3x 5x 2 0 =
( )

2
1
2x m 3 x m 1 0, x 3 = =
( )

2
1
mx m 2 x m 1 0 x 2 = =
( ) ( )

2
1
m 3 x 2 3m 1 x m 3 0 x 2 = =
( )

2
1
4 m x mx 1 m 0 x 1 = =
24
10

2 2 2
1 2
x mx 7 0 x x 10 = =

2
2 1
x 2x m 2 0 x x 2 = =
( )

2 2 2
1 2
x m 1 x m 6 0 x x 10 = =
( ) ( ) ( )

2
1 2 1 2
m 1 x 2 m 1 x m 2 0 4 x x 7x x = =

2
1 2
x 4x m 3 0 x x 2 = =
( ) ( )

2
1 2
x m 3 x 2 m 2 0 x 2x = =
( )

2
1 2
x m 5 x m 6 0 2x 3x 13 = =
( )

2
1 2
4x m 3 x 24 0 x 2x 1 = =

2
1 2
x 2mx 3m 2 0 2x 3x 1 = =
( )

2 2
1 2
x 2 m 1 x m 2m 4 0 x 2x = =
( ) ( )

2 2
x 2m 3 x m 2m 0 =
( )

( )

( ) ( )

2 2
mx m 3 x m 0 =
( )

www.MATHVN.com
www.DeThiThuDaiHoc.com
cng hc tp mn Ton 10 tp I Ths. L Vn on


"Cn c b thng minh" Page - 49 -
b/ Vi gi tr no ca m th phng trnh c hai nghim x
1
v x
2
tha .
Bi 221. Bi 221. Bi 221. Bi 221. Cho phng trnh: .
a/ Chng t rng vi phng trnh c hai nghim phn bit m.
b/ Xc nh m phng trnh c hai nghim x
1
v x
2
tha . Tnh cc
nghim trong trng hp .
Bi 222. Bi 222. Bi 222. Bi 222. Cho phng trnh: . Vi gi tr no ca tham s m th
phng trnh c hai nghim x
1
v x
2
m . Tnh cc nghim trong trng hp .
Bi 223. Bi 223. Bi 223. Bi 223. Cho phng trnh: . Xc nh m
a/ Phng trnh c hai nghim phn bit.
b/ Phng trnh c mt nghim bng 2. Tnh nghim kia.
c/ Phng trnh c tng bnh phng cc nghim bng 2.
Bi 224. Bi 224. Bi 224. Bi 224. Cho phng trnh: .
a/ Tm m phng trnh c nghim . Tnh nghim cn li.
b/ Khi phng trnh c hai nghim x
1
, x
2
. Tm h thc gia x
1
, x
2
c lp i vi m.
c/ Tm m phng trnh c hai nghim x
1
, x
2
tho: .
Bi 225. Bi 225. Bi 225. Bi 225. Cho phng trnh: .
a/ Tm m phng trnh

c mt nghim bng bnh phng nghim kia.
b/ Tm m phng trnh

c mt nghim bng 1. Tnh nghim cn li.
Bi 226. Bi 226. Bi 226. Bi 226. Cho phng trnh: . Tm tham s m phng trnh c hai
nghim phn bit x
1
v x
2
tha:
a/ Nghim ny hn nghim kia mt n v.
b/ .
c/ C hai nghim tri du sao cho nghim m c gi tr tuyt i ln hn nghim dng.
Bi 227. Bi 227. Bi 227. Bi 227. Cho phng trnh: .
a/ Tm tham s m phng trnh c nghim.
b/ Tm tham s m phng trnh c nghim tha:
/ . / .
/ . / nh nht.
Bi 228. Bi 228. Bi 228. Bi 228. Cho phng trnh: .
( )

1 2
13
x x
4
=
( ) ( )

2 2
9x 2 m 1 x 1 0 =
m 2
( )

( )

1 2
x x 4 =
( ) ( )
2
m 1 x 3m 1 x 2m 2 0 =
1 2
x x 3 =
( ) ( ) ( )

2
m 1 x 2 m 1 x m 2 0 =
( )

( )

( )

( ) ( )

2 2
x 2 m 1 x m 3m 0 =
( )
x 0 =
( )

( )

2 2
1 2
x x 8 =
( ) ( )

2 2 3
x m 3m x m 0 =
( )

( )

( )
2
x m 5 x m 6 0 =
1 2
2x 3x 13 =
( ) ( )

2
x 2 m 1 x 2m 10 0 =
( )

( )

1 2
2 2
x x
2
x x
=
2 1
2x x 8 =
( )
1 2 1 2
x x 2 x x 5
2 2
1 2 1 2
P 10x x x x =
( ) ( )

2
x 2 m 1 x m 3 0 =
www.MATHVN.com
www.DeThiThuDaiHoc.com
Ths. L Vn on Phn i S


Page - 50 - "All the flower of tomorrow are in the seeks of today"
a/ Tm tham s m phng trnh lun c hai nghim phn bit .
b/ Tm m phng trnh c hai nghim tri du v c gi tr tuyt i bng nhau.
Bi 229. Bi 229. Bi 229. Bi 229. Tm tham s m phng trnh: c hai nghim sau cho nghim ny bng
bnh phng nghim kia ?
Bi 230. Bi 230. Bi 230. Bi 230. Tm cc gi tr dng ca m sao cho phng trnh: c hai nghim
tri du v c gi tr tuyt i l nghch o ca nhau.
Bi 231. Bi 231. Bi 231. Bi 231. Cho phng trnh: ( l tham s).
a/ Chng minh phng trnh c nghim vi mi .
b/ Tm tng bnh phng cc nghim ca phng trnh t GTLN, GTNN.
Bi 232. Bi 232. Bi 232. Bi 232. Tm m phng trnh: c hai nghim tha
.
Bi 233. Bi 233. Bi 233. Bi 233. Cho hai phng trnh: v . Tm tham s m
phng trnh c nghim gp i nghim ca phng trnh .
Bi 234. Bi 234. Bi 234. Bi 234. Tm tham s m cc phng trnh sau c nghim x
1
v x
2
. Tm h thc ca x
1
v x
2
c lp
vi tham s m.
a/ . b/ .
c/ . d/ .
e/ . f/ .
g/ . h/ .
Bi 235. Bi 235. Bi 235. Bi 235. Cho phng trnh: . Tm tham s m phng trnh:
a/ C t nht mt nghim dng. b/ C nghim s tha mn: .

BI TP RN LUYN
Bi 236. Bi 236. Bi 236. Bi 236. Gi x
1
, x
2
l cc nghim ca phng trnh. Khng gii phng trnh, hy tnh
.
a/ . b/ .
c/ . e/ .
Bi 237. Bi 237. Bi 237. Bi 237. nh tham s m phng trnh c mt nghim cho trc. Tnh nghim cn li
a/ .
b/ .
c/ .
( )
m R
( )

2
15
x x m 0
4
=
( )
2 2
2x m 2 x 7 m =
( )

2 2
2x 2x sin 2x cos =
( )

( )

( )
2 2
x 2 2m 1 x 3m 6m
0
x 2

=

1 2
x 2x 16 =
( )

2
x 8x 4m 0 1 =
( )

2
x x 4m 0 2 =
( )
1
( )
2
( )
2
mx m 3 x 2m 1 0 =
( ) ( )
2
m 4 x 2 m 2 x m 1 0 =
( ) ( )
2
m 2 x m 4 x 2 m 0 =
( ) ( )
2
m 1 x 2 m 2 x m 3 0 =
( )
2 2
x m 1 x m 4 0 =
( )
2
mx 2 m 1 x m 3 0 =
( )
2 2
x 2m 3 x m 4 0 =
( )
2
mx 2m 3 x m 4 0 =
( )
2
mx 2 m 1 x m 1 0 =
1 2
x 1 x < <
( )( )

2 2 3 3 4 4
1 2 1 2 1 2 1 2 1 2 2 1
A x x , B x x , C x x , D x x , E 2x x 2x x = = = = =
2
x 5x 3 0 =
2
2x 8x 7 0 =
2
8x 7x 3 0 =
2
3x 2x 3 0 =
( )

2
1
2m 1 x 4x 4m 3 0 x 1 = =
( )

2 2
1
m 4 x x m 4m 1 0 x 1 = =
( ) ( )

2
1
m 1 x 2 m 1 x m 2 0 x 2 = =
www.MATHVN.com
www.DeThiThuDaiHoc.com
cng hc tp mn Ton 10 tp I Ths. L Vn on


"Cn c b thng minh" Page - 51 -
d/ .
Bi 238. Bi 238. Bi 238. Bi 238. nh m phng trnh bc hai c nghim x
1
, x
2
tha ng thc theo sau
a/ .
b/ .
c/ .
d/ .
e/ .
f/
c/ .
d/ .
Bi 239. Bi 239. Bi 239. Bi 239. Tm tham s m phng trnh c hai nghim x
1
v x
2
. Khi hy tm h thc gia chng c
lp vi tham s m.
a/ . b/ .
c/ . d/ .
e/ . f/ .
Bi 240. Bi 240. Bi 240. Bi 240. Cho phng trnh: .
a/ Xc nh tham s m phng trnh c hai nghim tri du v tng hai nghim ny
bng .
b/ Vi gi tr no ca m th phng trnh c nghim kp ? Tm nghim kp ?
Bi 241. Bi 241. Bi 241. Bi 241. Cho phng trnh: .
a/ Tm tham s m phng trnh c hai nghim x
1
,x
2
.
b/ Tm h thc x
1
v x
2
c lp vi tham s m.
c/ Tnh theo m gi tr ca biu thc .
d/ Tm tham s m phng trnh c mt nghim gp 3 ln nghim kia.
e/ Lp phng trnh bc hai c cc nghim l .
Bi 242. Bi 242. Bi 242. Bi 242. Tui ca anh hin nay gp i tui ca em, bit rng sau 48 nm na tui ca anh bng bnh
phng s tui ca em hin nay. Hi tui ca em hin nay ?
Bi 243. Bi 243. Bi 243. Bi 243. Chu vi mt hnh thoi bng 34cm , hiu hai ng cho bng 7cm. Tnh di hai ng cho ?
Bi 244. Bi 244. Bi 244. Bi 244. Mt ming t hnh vung. Nu tng mt cnh thm 30m th c ming t mi hnh ch nht
c din tch gp 3 ln din tch lc u. Hi cnh ca ming t lc u ?
Bi 245. Bi 245. Bi 245. Bi 245. Tm di ba cnh ca mt tam gic vung bit chu vi v din tch ca tam gic ln lt bng
120m v 480m
2
.

( )

2 2
1
x 2 m 1 x m 3m 0 x 0 = =
( )

2 2 2 2
1 2
x 2m x x m 3 0 x x 25 = =
( ) ( )

2 2 2
1 2
m 1 x 2 m 1 x m 2 0 x x 2 = =
( )

2 2 2 2
1 2
x 2 m 1 x m 3m 4 0 x x 20 = =
( )

2
1 2
1 1
2x m 3 x m 1 0 3
x x
= =

2 1 2
2 1
x x 8
x mx m 1 0
x x 5
= =
( ) ( )

2
1 1
x m 2 x m m 3 0 x 2x 1 = =

2
1 2
x 2x m 0 x 3x = =
( )

2
1 2
x 2 m 1 x 2m 5 0 2x 3x 1 = =
( )
2
mx 2m 1 x m 2 0 =
( ) ( )
2
m 2 x 2 4m 1 x 2m 5 0 =
( ) ( )
2
3m
m 2 x 2m 1 x 0
4
=
( )
2
3 m 1 x 4mx 2m 1 0 =
( )
2
mx m 4 x m 1 0 =
( ) ( )
2
m 1 x 2 m 2 x m 4 0 =
( ) ( ) ( )

2
m 2 x 2m 1 x 2 0 =
( )

3
( )

( ) ( )

2
x 2 2m 1 x 3 4m 0 =
( )

3 3
1 2
A x x =
( )

2 2
1 2
x , x
www.MATHVN.com
www.DeThiThuDaiHoc.com
Ths. L Vn on Phn i S


Page - 52 - "All the flower of tomorrow are in the seeks of today"






















































Dng ton 4. Phng trnh trng phng
Phng trnh quy v phng trnh bc hai
( )
=
4 2
ax bx c 0, a 0

C CC C Cch gii phng trnh trng phng: .

C CC C S nghim ca phng trnh trng phng
xc nh s nghim ca ta da vo s nghim ca phng trnh v du ca chng
O Phng trnh v nghim
O Phng trnh c 1 nghim
O Phng trnh c 2 nghim
O Phng trnh c 3 nghim c 1 nghim bng 0 v nghim cn li dng.
O Phng trnh c 4 nghim c 2 nghim dng phn bit.
C CC C Phng trnh quy v phng trnh bc hai
O Dng 1. vi .
t .
Phng trnh tr thnh: m bit cch gii.
O Dng 2. .
t .
Phng trnh tr thnh: vi .
O Dng 3. (phng trnh i xng).

( )
4 2
ax bx c 0, a 0 =
( )
( )


2
4 2
2
t x , t 0
ax bx c 0 1
at bt c 0 2
'
1
=
1
1
=
!
1
=
1
1+
( )
1
( )
2
( )
1
( )
( )
( )
2 vo nghie m
2 co nghiem kep am
2 co 2 nghie m am

( )
1
( )
( )
2 co nghiem kep bang 0
2 co 1 nghiem bang 0, nghiem con la i am

( )
1
( )
( )
2 co nghiem kep dng
2 co 1 nghie m dng va 1 nghiem am

( )
1
( )
2
( )
1
( )
2
( )( )( )( )
x a x b x c x d K = a b c d =
( )( ) ( )( )
t x a x b x c x d t ab cd = =
( )
2
t cd ab t K 0 =
( ) ( )
4 4
x a x b K =

a b a b b a
t x x a t , x b t
2 2 2

= = =
4 2 2 4
2t 12 t 2 K 0 =
a b
2

=
( )

4 3 2
ax bx cx bx a 0 a 0 =
V khng l nghim nn chia hai v ca phng trnh cho , ta c:
.
t vi .
Phng trnh tr thnh:
x 0 =
2
x
( )

2
2
1 1
a x b x c 0
x x
1 1

=


( ) ( )

1 1
t x hay t x
x x
1

= =

( )
t 2
( )

( )

2
at bt c 2a 0, t 2 =
www.MATHVN.com
www.DeThiThuDaiHoc.com
cng hc tp mn Ton 10 tp I Ths. L Vn on


"Cn c b thng minh" Page - 53 -















BI TP P DNG
Bi 246. Bi 246. Bi 246. Bi 246. Gii phng trnh sau
a/ . b/ .
c/ . d/ .
e/ . f/ .
Bi 247. Bi 247. Bi 247. Bi 247. Gii cc phng trnh sau
a/ . b/ .
c/ . d/ .
e/ . f/ .
Bi 248. Bi 248. Bi 248. Bi 248. Gii cc phng trnh sau
a/ . b/ .
c/ . d/ .
e/ . f/
Bi 249. Bi 249. Bi 249. Bi 249. Gii cc phng trnh sau
a/ . b/ .
c/ . d/ .
Bi 250. Bi 250. Bi 250. Bi 250. Gii cc phng trnh sau
a/ . b/ .
c/ . d/ .
e/ . f/ .
Bi 251. Bi 251. Bi 251. Bi 251. Gii cc phng trnh sau
a/ . b/ .
c/ . d/ .
Bi 252. Bi 252. Bi 252. Bi 252. Gii cc phng trnh sau
4 2
x 3x 4 0 =
4 2
x 5x 4 0 =
4 2
x 5x 6 0 =
4 2
3x 5x 2 0 =
4 2
x x 30 0 =
4 2
x 7x 8 0 =
3 2
2x 7x 7x 2 0 =
3 2
x x x 2 4x 1 =
3 2
2x 7x 28x 12 0 =
3 2
2x 9x 12x 4 0 =
3 2
x x 4 0 =
3 2
2x 5x 1 0 =
( )( )( )
x x 1 x 1 x 2 3 =
( )( )( )( )
x 1 x 2 x 3 x 4 3 =
( )( )( )( )
x 1 x 3 x 5 x 7 297 =
( )( )( )( )
x 2 x 3 x 1 x 6 36 0 =
( )( )( )( )
4x 1 12x 1 3x 2 x 1 28 =
( ) ( )( )
2
x 1 2x 1 2x 3 18 0 =
( )
4
4
x x 1 97 =
( ) ( )
4 4
x 3 x 5 2 =
( ) ( )
4 4
x 4 x 6 2 =
( ) ( )
4 4
x 3 x 5 16 =
4 3 2
x x 4x x 1 0 =
4 3 2
x 10x 26x 10x 1 0 =
4 3 2
x x 4x 5x 25 0 =
4 3 2
x 2x x 2x 1 0 =
4 3 2
2x 21x 74x 105x 50 0 =
4 3 2
6x 35x 62x 35x 6 0 =
4 3 2
x 5x x 21x 18 0 =
4 3 2
x x 7x x 6 0 =
4 3 2
x 2x 4x 5x 6 0 =
4 3 2
3x 2x 6x x 2 0 =


Gii phng trnh tm cc nghim cn li (nu c)
( )

www.MATHVN.com
www.DeThiThuDaiHoc.com
Ths. L Vn on Phn i S


Page - 54 - "All the flower of tomorrow are in the seeks of today"
a/ . b/ .
c/ . d/ .
e/ . f/ .
g/ . h/ .
Bi 253. Bi 253. Bi 253. Bi 253. Tm tham s m cc phng trnh sau c ba nghim phn bit
a/ . b/ .
c/ . d/ .
e/ . f/ .
Bi 254. Bi 254. Bi 254. Bi 254. Cho phng trnh:
a/ Tm tham s m phng trnh v nghim.
b/ Tm tham s m phng trnh c hai nghim phn bit.
c/ Tm tham s m phng trnh c bn nghim phn bit.
Bi 255. Bi 255. Bi 255. Bi 255. Cho phng trnh:
a/ Tm tham s m phng trnh v nghim.
b/ Tm tham s m phng trnh c nghim duy nht.
c/ Tm tham s m phng trnh c hai nghim phn bit.
d/ Tm tham s m phng trnh c ba nghim phn bit.
e/ Tm tham s m phng trnh c bn nghim phn bit.
Bi 256. Bi 256. Bi 256. Bi 256. Cho phng trnh:
a/ Tm tham s m phng trnh v nghim.
b/ Tm tham s m phng trnh c nghim duy nht.
c/ Tm tham s m phng trnh c hai nghim phn bit.
d/ Tm tham s m phng trnh c ba nghim phn bit.
e/ Tm tham s m phng trnh c bn nghim phn bit.
Bi 257. Bi 257. Bi 257. Bi 257. Cho phng trnh:
a/ Tm tham s m phng trnh v nghim.
b/ Tm tham s m phng trnh c nghim duy nht.
c/ Tm tham s m phng trnh c hai nghim phn bit.
d/ Tm tham s m phng trnh c ba nghim phn bit.
e/ Tm tham s m phng trnh c bn nghim phn bit.
( ) ( )
2
2 2
x x 4 x x 12 0 =
3 2
x 1 x 1 x 1
1 0
x 1 x 1 x 1
1 1


=



( ) ( )
5 4 3 2
x x x x x 1 0 =
6 5 4 3
x x 13x 14x x 1 0 =
( ) ( )
2
2 2 2
x 1 3x x 1 2x 0 =
( )
2
2 3 2
x 3x 6 2x 3x 12x 0 =
( ) ( )
2
2 2 2
x 4x 8 3x x 4x 8 2x 0 =
( ) ( ) ( )
2 2
2 2
x x 1 x x 1 2 x 1 =
3 2
x 3x 2 mx m 2 =
( )
3 2
x 2m 1 x mx m 0 =
( ) ( )
3 2
x 2 m 1 x 7m 2 x 4 6m 0 =
( ) ( )
3 2
mx m 4 x 4 m x m 0 =
( )
3 2 2
x 1 m x 3mx 2m 0 =
( )
3 2 2
x 2mx 4 3m x 4m 0 =
( ) ( )

4 2 2
x 1 2m x m 1 0 =
( )

( )

( )

( ) ( )

4 2
mx 2 m 1 x m 2 0 =
( )

( )

( )

( )

( )

( ) ( )

4
4
x 2 x 82 m =
( )

( )

( )

( )

( )

( )( )( ) ( )
x x 1 x 2 x 3 1 m 0 =
( )

( )

( )

( )

( )

www.MATHVN.com
www.DeThiThuDaiHoc.com
cng hc tp mn Ton 10 tp I Ths. L Vn on


"Cn c b thng minh" Page - 55 -
Bi 258. Bi 258. Bi 258. Bi 258. Cho phng trnh:
a/ Tm tham s m phng trnh v nghim.
b/ Tm tham s m phng trnh c nghim duy nht.
c/ Tm tham s m phng trnh c hai nghim.
d/ Tm tham s m phng trnh c ba nghim phn bit.
e/ Tm tham s m phng trnh c t nht hai nghim.
Bi 259. Bi 259. Bi 259. Bi 259. Tm tham s m phng trnh c ba nghim phn bit
sao cho biu thc t gi tr nh nht.
Bi 260. Bi 260. Bi 260. Bi 260. Cho phng trnh:
a/ Tm tham s m phng trnh ch c ng hai nghim.
b/ Tm tham s m phng trnh c ba nghim phn bit.
c/ Tm tham s m phng trnh c t nht ba nghim phn bit.

BI TP RN LUYN
Bi 261. Bi 261. Bi 261. Bi 261. Gii cc phng trnh sau
a/ . b/ .
c/ . d/ .
e/ . f/ .
Bi 262. Bi 262. Bi 262. Bi 262. Gii cc phng trnh sau
a/ . b/ .
c/ . d/ .
e/ . f/ .
Bi 263. Bi 263. Bi 263. Bi 263. Gii cc phng trnh sau
a/ . b/ .
c/ . d/ .
Bi 264. Bi 264. Bi 264. Bi 264. Gii cc phng trnh sau
a/ . b/ .
c/ . d/ .
e/ . f/ .
g/ . h/ .
( )

3 2
x 2mx 2mx 1 0 =
( )

( )

( )

( )

( )

3 2
x 3mx 3x 3m 2 0 =
1 2 3
x , x , x
2 2 2
1 2 3
P x x x =
( )( ) ( )

2 2 2
x 2mx 3m x 1 0 =
( )

( )

( )

4 2
x 15x 16 0 =
4 2
x 26x 25 0 =
4 2
2x 452x 450 0 =
4 2
x x 6 0 =
4 2
x x 20 0 =
4 2
x 80x 81 0 =
( )( )( )( )
x 5 x 6 x 8 x 9 40 =
( )( )( )( )
x 7 x 5 x 4 x 2 72 =
( )( )( )( )
x 2011 x 2013 x 2015 x 2017 4020 =
( ) ( )( )
2
6x 5 3x 2 x 1 35 0 =
( )( )( )( )
2x 1 x 1 x 3 2x 3 9 0 =
( ) ( )( )
2
6x 7 3x 1 x 1 6 0 =
4 3 2
x 4x 6x 4x 1 0 =
4 3 2
x 2x 7x 4x 4 0 =
4 3 2
x 3x 4x 3x 1 0 =
4 2 2
x 3x 14x 6x 4 0 =
3 2
x 8x 8x 1 0 =
3 2
x 5x 8x 4 0 =
3
x 2x 5x 6 0 =
3 2
2x 5x 1 0 =
( ) ( ) ( )
3 3 3
3
x 1 x x 1 x 2 =
( ) ( )
4 4
x 6 x 4 82 =
( )
4
4
1
2x 1 2x
27
=
4 3 2
3x 10x 3x 10x 3 0 =
www.MATHVN.com
www.DeThiThuDaiHoc.com
Ths. L Vn on Phn i S


Page - 56 - "All the flower of tomorrow are in the seeks of today"
Bi 265. Bi 265. Bi 265. Bi 265. Gii cc phng trnh sau
a/ . b/ .
c/ . d/ .
Bi 266. Bi 266. Bi 266. Bi 266. Tm tham s m phng trnh: c bn nghim
phn bit.
Bi 267. Bi 267. Bi 267. Bi 267. Tm tham s m phng trnh: c 4 nghim phn bit.
Bi 268. Bi 268. Bi 268. Bi 268. Cho phng trnh:
a/ Tm tham s m phng trnh v nghim.
b/ Tm tham s m phng trnh c ba nghim phn bit.
c/ Tm tham s m phng trnh c t nht ba nghim phn bit.
Bi 269. Bi 269. Bi 269. Bi 269. Cho phng trnh:
a/ Tm tham s m phng trnh v nghim.
b/ Tm tham s m phng trnh c nghim duy nht.
c/ Tm tham s m phng trnh c bn nghim phn bit.
Bi 270. Bi 270. Bi 270. Bi 270. nh tham s m phng trnh: c t nht hai nghim.
Bi 271. Bi 271. Bi 271. Bi 271. nh tham s m phng trnh: c ba nghim phn bit
, ng thi ba nghim ny tha mn ng thc: .
Bi 272. Bi 272. Bi 272. Bi 272. nh tham s m phng trnh: c 4 nghim phn bit.
Bi 273. Bi 273. Bi 273. Bi 273. Cho phng trnh:
a/ Tm tham s m phng trnh v nghim.
b/ Tm tham s m phng trnh c t nht mt nghim.
c/ Tm tham s m phng trnh c hai nghim phn bit.







( ) ( )
4 2
2 2 2 4
x x 1 10x x x 1 9x 0 =
( ) ( ) ( )
2 2
2 3
3 x x 1 2 x 1 5 x 1 =
( ) ( ) ( )
2 2
2 3
2 x x 1 7 x 1 13 x 1 =
( ) ( ) ( )
2 2
2 2
x x 1 x x 1 2 x 1 =
( ) ( )( )
4 2
x 3m 14 x 4m 12 2 m 0 =
( )
4 2 2
m 1 x mx m 1 0 =
( ) ( )

4 2 2 4 2
x 2 m 2 x m 4m 0 =
( )

( )

( )

( )

4 3
x 2x x m 0 =
( )

( )

( )

( )
3 2
1 m x 2mx mx 1 0 =
( )
3 2 2
x 2mx 4 3m x 4m 0 =
1 2 3
x , x , x
2 1 3 2
x x x x =
( )( )
2 2
x 2x m x 3x 4m 0 =
( )

2
3x mx 2
0
x m

=

( )

( )

( )

www.MATHVN.com
www.DeThiThuDaiHoc.com
cng hc tp mn Ton 10 tp I Ths. L Vn on


"Cn c b thng minh" Page - 57 -




































BI TP P DNG
Bi 274. Bi 274. Bi 274. Bi 274. Gii cc phng trnh sau
a/ . b/ .
c/ d/ .
e/ . f/ .
Bi 275. Bi 275. Bi 275. Bi 275. Gii cc phng trnh sau
a/ . b/ .
c/ . d/ .
e/ . f/ .
Bi 276. Bi 276. Bi 276. Bi 276. Gii cc phng trnh sau
2x 1 x 3 = 4x 7 2x 5 =
4x 7 4x 7 =
2
2x 3x 5 5x 5 =
2
x 4x 5 4x 17 =
2
4x 17 x 4x 5 =
2
x 6x 9 2x 1 = 5x 1 2x 3 =
3x 4 x 2 =
2 2
3x 2x 6 x =
2 2
x 2x 2x x 2 =
2 2
x 4x 5 2x 3x 5 =
Dng ton 5. Phng trnh cha n trong du tr tuyt i


C CC C nh ngha v tnh cht
.
. .
. .
. .
C CC C Cch gii
gii phng trnh cha n trong du gi tr tuyt i ta tm cch kh du gi tr tuyt i,
bng cch:
Dng nh ngha hoc tnh cht ca gi tr tuyt i.
Bnh phng hai v.
t n ph.
O Dng 1.




A khi A 0
A
A khi A 0
'
1
1
=
!
1 <
1
+
A 0, A
A.B A . B =
2
2
A A =
A B A B A.B 0 = A B A B A.B 0 =
A B A B A.B 0 = A B A B A.B 0 =
( ) ( )
( )
( ) ( )
( )
( ) ( )
( )
( ) ( )
( ) ( )
1 2
C C
f x 0
g x 0
f x g x
f x g x f x g x
f x 0
f x g x
f x g x
'
1

1
1 '
1

!
1
1 1 =

1 1
1 1+
= =
!

'
1 1
<
1 1
1

1 =
!
1
1+ 1
=
1
1+
O Dng 2.
O Dng 3. : dng phng php chia khong gii.
( ) ( ) ( ) ( )
( ) ( )
( ) ( )
1 2
C C
2 2 f x g x
f x g x f x g x
f x g x

l l
= =

l l
l l
=

( ) ( ) ( )
a f x b g x h x =
www.MATHVN.com
www.DeThiThuDaiHoc.com
Ths. L Vn on Phn i S


Page - 58 - "All the flower of tomorrow are in the seeks of today"
a/ . b/ .
c/ . d/ .
e/ . f/ .
g/ . h/ .
Bi 277. Bi 277. Bi 277. Bi 277. Gii cc phng trnh sau
a/ . b/ .
c/ . d/ .
e/ . f/ .

BI TP RN LUYN
Bi 278. Bi 278. Bi 278. Bi 278. Gii cc phng trnh sau
a/ . b/ .
b/ . b/ .
c/ . d/ .
e/ . f/ .
g/ . h/ .
Bi 279. Bi 279. Bi 279. Bi 279. Gii cc phng trnh sau
a/ . b/ .
c/ . d/ .
e/ . f/ .
Bi 280. Bi 280. Bi 280. Bi 280. Gii cc phng trnh sau
a/ . b/ .
c/ . d/ .
e/ . f/ .
g/ . h/ .
Bi 281. Bi 281. Bi 281. Bi 281. Gii cc phng trnh sau
a/ . b/ .
c/ . d/ .


2
x 3 x 2 0 =
2
x 2x x 1 1 0 =
2
x 2x 5 x 1 5 0 =
2
4x 4x 2x 1 1 0 =
1 2x x 1 x 2 = x 2 x 3 4 =
x 3 7 x 10 =
2
2x 5 2x 7x 5 0 =
x 1 2x 1 3x = x 1 x 2 x 3 14 =
x 1 x 2x 3 2x 4 =
1
2x 3
x
=
3
x 3
x 4 1
=

3x x 2
x x 1

x 2 x 1 = 2 x 1 x 2 =
2x 3 x 5 =
2
x 3 x 4x 3 =
2
x 2 3x x 2 =
2
4x 1 x 2x 4 =
2
3x 5 2x x 3 =
2
x 5x 4 x 4 =
2
x 4x 5 4x 17 =
2
5x 16
x 4x 2
3

=
2
4x 12x 9 3x 2 = 3x 1 2x 3 =
x 3 2x 1 =
2 2
x x 2 x 2x =
2 2
x 2x x 5x 6 =
2 2
x x 2 2x 3x 5 =
2
x x 2 8 0 =
2
x 6x x 3 10 0 =
2
x 4x 3 x 2 0 =
2
x 2x 5 x 1 7 0 =
x x 1 2 x 2 =
2 2
x 2x 3 x 2x 3 =
2 2
2x 3x 3 2x 8x 3 0 = x 1 x 2 3 =
x 2
2
x 2

2
2
1 1
x 10 2 x
x x
=
2
2
2x 4
x 4x 4
3 0
x 1 x 2x 1


=

2
x 1 1 2x 1
x x x x 1

=

www.MATHVN.com
www.DeThiThuDaiHoc.com
cng hc tp mn Ton 10 tp I Ths. L Vn on


"Cn c b thng minh" Page - 59 -














































Dng ton 6. Phng trnh cha n di du cn

Phng php gii
gii phng trnh cha n di du cn ta tm cch kh du cn, bng cch:
Nng ly tha hai v.
t n ph.
Lu rng: Khi thc hin cc php bin i cn ch iu kin cc cn c xc nh
O Dng 1.
O Dng 2.
O Dng 3.
O Dng 4. .
t vi .
a phng trnh trn v h phng trnh vi hai n l u v v.
O Dng 5. .
t
O Dng 6.
Ta c
Thay vo , ta c: .
O Dng 7. vi
Bin i v dng: .
Bnh phng, gii phng trnh h qu.
O Dng 8. Nhn thm lng lin hip
D on nghim v dng nhn lng lin hip xut hin nhn t chung.
Cc cng thc thng dng:

( ) ( )
( )
( ) ( )
2
g x 0
f x g x
f x g x
'
1

1
1
=
!
l 1
=
1
l
l 1+
( ) ( )
( ) ( ) ( )
( ) ( )
f x 0 hay g x 0
f x g x
f x g x
'
1

1
1
=
!
1
=
1
1+
( ) ( )
( )

2
t f x , t 0
af x b x c 0
at bt c 0
'
1
= 1
1
=
!
1
=
1
1+
( ) ( ) ( )
f x g x h x =
( ) ( )
u f x , v g x = = u, v 0
( ) ( ) ( ) ( ) ( )
f x g x f x .g x h x =
( ) ( )
t f x g x , t 0 =
( )

3 3 3
A B C =
( )
( ) ( ) ( )
( )

3 3
3 3 3 3 3 3
A B C A B 3 AB A B C = =
3 3 3
A B C =
( )

( )
3
A B 3 ABC C =
( ) ( ) ( ) ( )
f x g x h x k x =
( ) ( ) ( ) ( )
( ) ( ) ( ) ( )
f x h x g x k x
f x .h x g x .k x

( ) ( ) ( ) ( )
f x h x k x g x =
Biu thc Biu thc lin hip Tch







A B A B
A B
3 3
A B
3 3 3 2
A AB B
A B
3 3
A B
3 3 3 2
A AB B
A B
www.MATHVN.com
www.DeThiThuDaiHoc.com
Ths. L Vn on Phn i S


Page - 60 - "All the flower of tomorrow are in the seeks of today"
BI TP P DNG
Bi 282. Bi 282. Bi 282. Bi 282. Gii cc phng trnh sau
a/ . b/ .
c/ . d/ .
e/ . f/ .
g/ . h/ .
i/ . j/ .
Bi 283. Bi 283. Bi 283. Bi 283. Gii cc phng trnh sau
a/ . b/ .
c/ . d/ .
e/ . f/ .
Bi 284. Bi 284. Bi 284. Bi 284. Gii cc phng trnh sau
a/ . b/ .
c/ . d/ .
e/ . f/ .
g/ . h/ .
Bi 285. Bi 285. Bi 285. Bi 285. Gii cc phng trnh sau
a/ . b/ .
c/ . d/ .
e/ . f/ .
g/ . h/ .
Bi 286. Bi 286. Bi 286. Bi 286. Gii cc phng trnh sau
a/ .
b/
c/ .
2x 3 x 3 = 5x 10 8 x =
x 2x 5 4 =
2
x x 12 8 x =
2
x 2x 4 2 x =
2
3x 9x 1 x 2 =
2
3x 9x 1 x 2 =
2
x 3x 10 x 2 =
( )
2 2
x 3 x 4 x 9 =
2
x 4x 3 2x 5 =
2 2
x 6x 9 4 x 6x 6 =
( )( )
2
x 3 8 x 26 x 11x =
( )( )
2
x 4 x 1 3 x 5x 2 6 =
( )( )
2
x 5 2 x 3 x 3x =
2 2
x x 11 31 =
( )( )
2
x 2x 8 4 4 x x 2 0 =
x 1 x 1 1 = 3x 7 x 1 2 =
2 2
x 9 x 7 2 =
2 2
3x 5x 8 3x 5x 1 1 =
3 3
1 x 1 x 2 =
2 2
x x 5 x 8x 4 5 =
3 3
5x 7 5x 13 1 =
3 3
9 x 1 7 x 1 4 =
( )( )
x 3 6 x 3 x 3 6 x =
( )( )
2x 3 x 1 3x 2 2x 3 x 1 16 =
( )( )
x 1 3 x x 1 3 x 1 =
( )( )
7 x 2 x 7 x 2 x 3 =
( )( )
x 1 4 x x 1 4 x 5 =
2
3x 2 x 1 4x 9 2 3x 5x 2 =
2
2
1 x x x 1 x
3
=
2
x 9 x x 9x 9 =
x 2 x 1 x 2 x 1 2 =
x 1 2 x 2 x 2 4 x 2 3 0 =
2x 4 2 2x 5 2x 4 6 2x 5 14 =
www.MATHVN.com
www.DeThiThuDaiHoc.com
cng hc tp mn Ton 10 tp I Ths. L Vn on


"Cn c b thng minh" Page - 61 -
d/ .
e/ .
Bi 287. Bi 287. Bi 287. Bi 287. Gii cc phng trnh
a/ . b/ .
c/ . d/ .
Bi 288. Bi 288. Bi 288. Bi 288. Gii cc phng trnh
a/ x 3 3x 1 2 x 2x 1 = .
b/
2 2
x 3x 2 x 3 6x 2 x 2x 3 = .
c/ .
d/ .
Bi 289. Bi 289. Bi 289. Bi 289. Gii cc phng trnh
a/ .
b/ .
c/ .
d/ .
e/ .
f/ .
g/ .
h/ .
Bi 290. Bi 290. Bi 290. Bi 290. Gii cc phng trnh sau
a/ . b/ .
c/ . d/ .
e/ . f/ .
g/ . h/ .
Bi 291. Bi 291. Bi 291. Bi 291. nh tham s m phng trnh c hai nghim phn bit.
Bi 292. Bi 292. Bi 292. Bi 292. nh tham s m phng trnh c hai nghim phn bit.
Bi 293. Bi 293. Bi 293. Bi 293. nh tham s m phng trnh c nghim duy
nht.

x 5 4 x 1 x 2 2 x 1 1 =
2x 2 2x 1 2 2x 3 4 2x 1 3 2x 8 6 2x 1 4 =
3 3 3
x 1 x 2 x 3 0 =
3 3 3
2x 1 x 1 3x 2 =
3 3 3
x 5 x 6 2x 11 =
3 3 3
x 1 3x 1 x 1 =
3
2
x 1
x 1 x x 1 x 3
x 3

2 2 2 2
2x 1 x 3x 2 2x 2x 3 x x 2 =
2 2
x 12 5 3x x 5 =
( )
2 2 2 2
3x 5x 1 x 2 3 x x 1 x 3x 4 =
2
x 2 4 x 2x 5x 1 =
2
2
1 x 2x x
x 1 x

=

3 2 2 3 3 3
x 2 x 1 2x 2x 1 =
( )
2 2
x x 1 x 2 x 2x 2 =
3
x 24 12 x 6 =
( )
( )
2
2
x x 4 1 1 x =
( )
2 2 2
x 3 x 2 x 1 2 x 2 =
( )
3 3
4x 1 x 1 2x 2x 1 =
2 2
x 1 2x x 2x =
( )
2 2
x 1 x 2x 3 x 1 =
2
4 x 1 1 3x 2 1 x 1 x =
2
2 2x 4 4 2 x 9x 16 =
2 2
x 1 2x x 2x =
( )
2 2
x 4x x 2 x 2x 4 =
2
2x 6x m x 1 =
2
x x m x 3 =
( ) ( )
2
2 2 2
2x 1 x 2 m 1 x m 3m =
www.MATHVN.com
www.DeThiThuDaiHoc.com
Ths. L Vn on Phn i S


Page - 62 - "All the flower of tomorrow are in the seeks of today"
BI TP RN LUYN
Bi 294. Bi 294. Bi 294. Bi 294. Gii cc phng trnh sau (a v dng c bn)
a/ . b/ .
c/ . d/ .
e/ . f/ .
g/ . h/ .
i/ . j/ .
k/ . l/ .
m/ . n/ .
o/ . p/ .
q/ . r/ .
s/ . t/ .
Bi 295. Bi 295. Bi 295. Bi 295. Gii cc phng trnh sau (dng hng ng thc)
a/ . b/ .
c/ . d/ .
e/ . f/ .
Bi 296. Bi 296. Bi 296. Bi 296. Gii cc phng trnh sau (bnh phng hai v)
a/ . b/ .
c/ . d/ .
e/ . f/ .
g/ . h/ .
i/ . j/ .
k/ . l/ .
m/ . n/ .
o/ . p/ .
Bi 297. Bi 297. Bi 297. Bi 297. Gii cc phng trnh sau (a v tch)
x 1 x 3 = x 2 4 x =
2x 2x 1 7 = 3 x 3x 5 =
x 4x 3 2 =
2
x x x =
2
x 1 x 1 =
2
5 x x 1 =
2
x 2 x 4x 3 =
2
x 1 x 1 =
2
x 4 x 2 = 16x 17 8x 23 =
2
x 4x 2x 2 =
2
x 3x 2 2x 1 =
2
x 4x 3 2x 5 =
2
3x 5x 1 1 4x =
2 2
x 2x 1 x 2x 1 =
2 2
7 x x x 5 3 2x x =
2 x 2 2 x 1 x 1 4 =
2
x 3x 2 x 3 =
x 3 4 x 1 x 8 6 x 1 1 = x 8 6 x 1 x 3 4 x 1 5 0 =
2x 4 2 2x 5 2x 4 6 2x 5 4 0 = 2x 2 2 2x 3 4 2x 6 6 2x 3 =
x 3
x 2 x 1 x 2 x 1
2

= 21x 63 7 10 4 3x 9 0 =
2x 3 2x 2 1 = x 4 2x 6 1 =
3x 7 x 1 2 = 11 x x 1 2 =
2 2
x 9 x 7 2 = x x 5 5 =
3x 5 2x 3 x 2 = x 2 x 1 2x 3 =
x 3 7 x 2x 8 = 2 x 7 x 3 2x =
5x 1 3x 2 2x 1 = 5x 1 x 1 2x 4 =
x 2 2x 3 3x 5 = x 4 1 x 1 2x =
3x 4 2x 1 x 3 = x 2x 1 x 2x 1 2 =
www.MATHVN.com
www.DeThiThuDaiHoc.com
cng hc tp mn Ton 10 tp I Ths. L Vn on


"Cn c b thng minh" Page - 63 -
a/ . b/ .
c/ . d/ .
e/ . f/ .
g/ . h/ .
i/
2 3 3 3
x 1 x 2 1 x 3x 2 = . j/
3 3 2 2 3 3
x 1 x x x x = . Chia x.
k/
2
x 3 2x x 1 2x x 4x 3 = . l/
4x
x 3 4 x
x 3
=

chia x 3 .
Bi 298. Bi 298. Bi 298. Bi 298. Gii cc phng trnh sau
a/ . b/ .
c/ . d/ .
e/ . f/ .
g/ . h/ .
i/ . j/ .
Bi 299. Bi 299. Bi 299. Bi 299. Gii phng trnh sau (t n ph)
a/ . b/ .
c/ . d/ .
e/ . f/ .
g/ . h/ .
i/ . j/ .
k/ . l/ .
m/ . n/ .
o/ . p/ .
q/ . r/ .
s/ . t/ .
u/ . v/ .
Bi 300. Bi 300. Bi 300. Bi 300. Gii phng trnh (nhn lng lin hip)
a/ . b/ .
2
x
3x 2 1 x
3x 2
=

2
x x 1 x x x =
2
x 10x 21 3 x 3 2 x 7 6 =
2
x x 2 2 x 2 2 x 1 =
2 2
x 3x 2 x 3 x 2 x 2x 3 =
( ) ( )
2
x x 1 x x 2 2 x =
2 2 2
x 8x 15 x 2x 15 x 9x 18 =
2 2
2x 8x 6 x 1 2x 2 =
3 3
x 34 x 3 1 =
3 3 2 2 3
2 x x 2 x x 4 =
3 3 3 3
1 x 1 x 2 24 x 5 x 1 = =
4 4
18 x x 1 3 =
2x 4 x 1 0 = x 2 x 1 0 =
5x 2 x 3 0 = 2 x 1 3 x 1 0 =
6x 3 3 2 2x 1 5 0 = 25x 5 5 1 5x 3 0 =
2 2
x x 11 31 =
3x 1 x
2 1
x 3x 1

3
x 7 x 1 =
3
2 x 1 x 1 =
3
x 3 x 1 =
( )( )
2
x 5 2 x 3 x 3x =
( )
2 2
2 1 x x 2x 1 x 2x 1 =
( )( )
2
x 4 x 1 3 x 5x 6 4 =
2 2
3x 5x 8 3x 5x 1 1 =
2 2
x 3x 3 x 3x 6 3 =
2 2 2
3x 6x 16 x 2x 2 x 2x 4 =
2
2
1 x x x 1 x
3
=
3 3
2x 1 1
2
x 1 2 2x
=

2
3 x 1 1 4 2
3x 9 x 9 x

=
( )( ) ( )
x 1
x 3 x 1 4 x 3 3
x 3

( )( ) ( )
x 2
x 1 x 2 2 x 1 8
x 1

2 2
x 4 x 2 3x 4 x =
2 2
x 17 x x 17 x 9 =
( )( )
x 1 x 3 2 x 1 x 3 4 2x =
2
x 4 x 4 2x 12 2 x 16 =
2
2x 3 x 1 3x 2 2x 5x 3 16 =
2
3x 2 x 1 4x 9 2 3x 5x 2 =
x 3
4x 1 3x 2
5

=
2 2
4 1 3
x
x x x x x x
=

www.MATHVN.com
www.DeThiThuDaiHoc.com
Ths. L Vn on Phn i S


Page - 64 - "All the flower of tomorrow are in the seeks of today"
c/ . d/ .
e/ . f/ .
g/ . h/ .
i/ . j/ .
k/ . l/ .
Bi 301. Bi 301. Bi 301. Bi 301. Gii cc phng trnh sau (bnh phng hai v)
a/ .
b/ .
c/ .
d/ .
Bi 302. Bi 302. Bi 302. Bi 302. Gii cc phng trnh sau (khng mu mc)
a/ . b/ .
c/ . d/ .
e/ . f/ .
g/ . h/ .
i/ . j/ .
Bi 303. Bi 303. Bi 303. Bi 303. Tm nghim nguyn ca phng trnh: .
Bi 304. Bi 304. Bi 304. Bi 304. nh m c nghim duy nht.
Bi 305. Bi 305. Bi 305. Bi 305. nh tham s m cc phng trnh sau c nghim
a/ .
b/ .
c/ .
d/ .
e/ .

1 1 3
x
1 1 x 1 1 x
=

1
x x 1
x
=
2
2
5
x 1 x
2 x 1
=

2 2
4 1 3
x
x x x x x x
=

( ) ( )
( )
2
2
4 x 1 2x 10 1 3 2x =
( )
( )
2
2
2x x 9 2 9 2x =
2
2
40
x x 16
x 16
=

3x
3x 1 1
3x 10
=

3x 2
2x 4 2 2 x
3

=
( )( )
1 x 1 1 x 1 2x =
x x 1 x 4 x 9 0 =
2 2 2 2
2x 1 x 3x 2 2x 2x 3 x x 2 =
2 2 2 2
x 2 x 7 x x 3 x x 8 =
2 2 2 2
3x 7x 3 x 2 3x 5x 1 x 3x 4 =
2
4x 1 4x 1 1 =
2
x 2 4 x x 6x 11 =
1 1
x x x 2
2 4
=
3
2 x 1 x 1 =
3 3 3
x 1 x 2 x 3 0 =
( ) ( )
2
x 1 3 x 2 x 3 2 x 1 =
( )
2
x 2 x 1 x 1 x x x 0 =
2 2
x 2x 2x 1 3x 4x 1 =
( ) ( )
3 3
2 2
1 1 x 1 x 1 x 2 1 x
l
l
=
l
l
( ) ( )
4
x 1 x 2 x 1 x 1 2 x 1 x =
2
x 3x 13 x 2 36 0 =
( ) ( )
3
4
x 1 x 2m x 1 x 2 x 1 x m =
( )( )
7 x 2 x 7 x x 2 m =
( )( )
1 x 8 x 1 x 8 x m =
( )( )
x 1 3 x x 1 3 x m =
2
5 x x 1 x 6x 5 m =
( ) ( ) ( )( )
2 2
3 3
3
2 x 7 x 7 x 2 x m =
www.MATHVN.com
www.DeThiThuDaiHoc.com
cng hc tp mn Ton 10 tp I Ths. L Vn on


"Cn c b thng minh" Page - 65 -
BI TP QUA CC K THI
Bi 306. Bi 306. Bi 306. Bi 306. Cao ng Hi Quan nm 1996
Gii phng trnh:
3 3 3
2x 1 x 1 3x 2 0 = .
S:
2
x
3
= .
Bi 307. Bi 307. Bi 307. Bi 307. Cao ng Hi Quan Tp. H Ch Minh nm 1999
Cho phng trnh:
( )
x 4 x 4 x x 4 m = .
1/ Gii phng trnh
( )
khi m 6 = .
2/ Tm tham s m phng trnh
( )
c nghim.
S: / 1 x 4 = . / 2 m 6 . p dng phng php hm s.
Bi 308. Bi 308. Bi 308. Bi 308. Cao ng S Phm Nh Tr Mu Gio TW1 nm 2000
Gii phng trnh: 1 x 1 6 x = .
Bi 309. Bi 309. Bi 309. Bi 309. Cao ng Kim St pha Bc nm 2000
Gii phng trnh:
3 3
3 3
7 x x 5
6 x
7 x x 5

=

.
Bi 310. Bi 310. Bi 310. Bi 310. Cao ng Giao Thng nm 2000
Gii phng trnh:
4 4
8 x 89 x 5 = .
Bi 311. Bi 311. Bi 311. Bi 311. Cao ng S Phm H Ni khi A nm 2001
Gii phng trnh:
2
x 2 x 2 2 x 4 2x 2 = .
Bi 312. Bi 312. Bi 312. Bi 312. Cao ng S Phm K Thut Vinh nm 2001
Gii phng trnh:
2
x x 7 7 = .
Bi 313. Bi 313. Bi 313. Bi 313. Cao ng S Phm Th Dc TWII nm 2002
Cho phng trnh:
( )

2 2
x 4 x m 0 = .
1/ Gii phng trnh
( )
khi m 2 = .
2/ nh m phng trnh
( )
c nghim.
Bi 314. Bi 314. Bi 314. Bi 314. Cao ng Xy dng s 3 nm 2002
Gii phng trnh:
3
x 3 1 x = .
Bi 315. Bi 315. Bi 315. Bi 315. Cao ng S Phm Nha Trang nm 2002
Gii phng trnh:
( )( )
x 2 5 x x 2 5 x 4 = .
Bi 316. Bi 316. Bi 316. Bi 316. Cao ng S Phm Bn Tre khi A nm 2002
www.MATHVN.com
www.DeThiThuDaiHoc.com
Ths. L Vn on Phn i S


Page - 66 - "All the flower of tomorrow are in the seeks of today"
Gii phng trnh: 5x 1 3x 2 x 1 0 = .
Bi 317. Bi 317. Bi 317. Bi 317. Cao ng Giao Thng nm 2003
Gii phng trnh:
3 3 3
2x 1 2x 2 2x 3 0 = .
Bi 318. Bi 318. Bi 318. Bi 318. Cao ng Ti Chnh K Ton IV nm 2003
Gii phng trnh: x x 1 x 2 = .
Bi 319. Bi 319. Bi 319. Bi 319. Cao ng S Phm Tp. H Ch Minh khi A nm 2004
Gii phng trnh:
( )
x 3
x 2 x 1 x 2 x 1
2

= .
Bi 320. Bi 320. Bi 320. Bi 320. Cao ng S Phm Mu Gio TW1 nm 2004
Gii phng trnh:
2
x 4x 3 2x 5 = .
Bi 321. Bi 321. Bi 321. Bi 321. Cao ng S Phm H Ni nm 2005
Gii phng trnh:
2 2 2
x 4x 5 x 4x 8 4x x 1 = .
Bi 322. Bi 322. Bi 322. Bi 322. Cao ng S Phm Qung Nam nm 2005
Gii phng trnh:
( )
2 2
x 2 x 3 x 2x 3 = .
Bi 323. Bi 323. Bi 323. Bi 323. Cao ng S Phm Qung Ngi nm 2005
Gii phng trnh:
( )
2
x 3 x 5x 4 2x 6 = .
Bi 324. Bi 324. Bi 324. Bi 324. Cao ng Xy Dng s 3 Cao ng Cng ng Vnh Long khi A, B nm 2005
Gii phng trnh: 3x 1 8 x 1 = .
Bi 325. Bi 325. Bi 325. Bi 325. i hc khi D nm 2005
Gii phng trnh: 2 x 2 2 x 1 x 1 4 = .
Bi 326. Bi 326. Bi 326. Bi 326. D b 2 khi D i hc nm 2002
Gii phng trnh:
2
x 4 x 4 2x 12 2 x 16 = .
Bi 327. Bi 327. Bi 327. Bi 327. D b 1 i hc khi B nm 2005
Gii phng trnh: 3x 3 5 x 2x 4 = .
Bi 328. Bi 328. Bi 328. Bi 328. D b 1 i hc khi D nm 2004
Cho phng trnh:
2 2 2 3
5
x m x 4 2 m 0
3
1

=


( )
. Chng minh rng vi mi m 0
th phng trnh cho c nghim.
Bi 329. Bi 329. Bi 329. Bi 329. Cao ng Truyn Hnh Tp. H Ch Minh nm 2007
Gii phng trnh:
2 2
7 x x x 5 3 2x x = .
S: x 1 = .
www.MATHVN.com
www.DeThiThuDaiHoc.com
cng hc tp mn Ton 10 tp I Ths. L Vn on


"Cn c b thng minh" Page - 67 -
Bi 330. Bi 330. Bi 330. Bi 330. Cao ng S Phm Tp. H Ch Minh nm 2001
Xc nh tham s m phng trnh:
( )( )
2
x 6x m x 5 1 x 0 = c nghim.
Bi 331. Bi 331. Bi 331. Bi 331. i hc Quc Gia H Ni khi D nm 1997 1998
Gii phng trnh: 16x 17 8x 23 = .
S: x 4 = .
Bi 332. Bi 332. Bi 332. Bi 332. Hc Vin Ngn Hng nm 1999 2000
Gii phng trnh:
2
x 4x 2 2x = .
S: x 2 = .
Bi 333. Bi 333. Bi 333. Bi 333. i hc Dc H Ni nm 1999 2000
Gii phng trnh:
( )
2 2
x 3 10 x x x 12 = .
S: x 3 = .
Bi 334. Bi 334. Bi 334. Bi 334. i hc Y Dc Tp. HCM h trung cp nm 1999 2000
Gii phng trnh:
( ) ( )
2
x 1 3 x 2 x 3 2 x 1 = .
S: x 5 = . p dng BT Bunhiacpxki.
Bi 335. Bi 335. Bi 335. Bi 335. Hc Vin K Thut Qun S nm 1999 2000
Gii phng trnh:
2
3x 2 x 1 4x 9 2 3x 5x 2 = .
S: x 2 = . (C th gii theo phng php hm s).
Bi 336. Bi 336. Bi 336. Bi 336. i hc Ngoi Thng H Ni nm 1999 2000
Gii phng trnh:
2 2
3 x x 2 x x 1 = .
S:
1 5
x
2

= . t
2
t x x = .
Bi 337. Bi 337. Bi 337. Bi 337. i hc Nng Nghip I nm 1999 2000
Gii phng trnh:
2
x 2x 5 x 1 2 = .
S: x 1 = . VT 2 nn du " = " xy ra khi x 1 = .
Bi 338. Bi 338. Bi 338. Bi 338. Hc Vin K Thut Qun S nm 2001
Gii phng trnh:
( )
3 2 x 2 2x x 6 = .
Bi 339. Bi 339. Bi 339. Bi 339. i hc Xy Dng nm 2001
Gii phng trnh:
2
x 6x 6 2x 1 = .
Bi 340. Bi 340. Bi 340. Bi 340. i hc M a Cht nm 2001
Gii phng trnh:
2 2
x 4 x 2 3x 4 x = .
Bi 341. Bi 341. Bi 341. Bi 341. Hc Vin Bu Chnh Vin Thng nm 2001
www.MATHVN.com
www.DeThiThuDaiHoc.com
Ths. L Vn on Phn i S


Page - 68 - "All the flower of tomorrow are in the seeks of today"
Gii phng trnh:
x 3
4x 1 3x 2
5

= .
Bi 342. Bi 342. Bi 342. Bi 342. i hc Ngn Hng khi D i hc Quc Gia H Ni khi B, D nm 2001
Gii phng trnh:
2
4x 1 4x 1 1 = .
Bi 343. Bi 343. Bi 343. Bi 343. Hc Vin Ngn Hng khi A i hc Quc Gia H Ni khi A nm 2001
Gii phng trnh:
( )
2 2
x 3x 1 x 3 x 1 = .
Bi 344. Bi 344. Bi 344. Bi 344. i hc Ngoi Ng nm 2001
Gii phng trnh:
( )( )
x 1 4 x x 1 4 x 5 = .
Bi 345. Bi 345. Bi 345. Bi 345. i hc Dn Lp Ngoi Ng Tin hc Tp. H Ch Minh nm 2001
Gii phng trnh:
( )( )
2
x 3 1 x 5 x 2x 7 = .
Bi 346. Bi 346. Bi 346. Bi 346. i hc Bch Khoa H Ni khi A, D nm 2001
Gii phng trnh:
2 2
2x 8x 6 x 1 2x 2 = .
Bi 347. Bi 347. Bi 347. Bi 347. i hc Thy Sn H Ni nm 2001
Gii phng trnh:
x 5
x 2 2 x 1 x 2 2 x 1
2

= .
Bi 348. Bi 348. Bi 348. Bi 348. i hc Quc Gia Tp. H Ch Minh khi D nm 1998 1999
Gii phng trnh: x 9 5 2x 4 = .
S: x 0 = .
Bi 349. Bi 349. Bi 349. Bi 349. i hc Quc Gia H Ni khi B nm 1998 1999
Gii v bin lun phng trnh: x 1 1 x m = .
S:

2 4
1
2 m 2 x 4m m
2
m 2 m 2 VN
'
1
1
=
1
1
!
1
1
<
1
1+

Bi 350. Bi 350. Bi 350. Bi 350. i hc Hu khi A, V nm 1998 1999
Gii phng trnh:
2
x 1 x 1 = .
S:
1 5
x 1 x
2

= = .
Bi 351. Bi 351. Bi 351. Bi 351. i hc Hu khi D nm 1998 1999
Gii phng trnh:
2
4 x x 2 = .
S: x 2 x 0 = = .
Bi 352. Bi 352. Bi 352. Bi 352. i hc Kinh T Quc Dn nm 1998 1999
Cho phng trnh
( )( ) ( )
1 x 8 x 1 x 1 8 m = = .
www.MATHVN.com
www.DeThiThuDaiHoc.com
cng hc tp mn Ton 10 tp I Ths. L Vn on


"Cn c b thng minh" Page - 69 -
1/ Gii phng trnh
( )
khi m 3 = .
2/ Tm tham s m phng trnh
( )
c nghim.
S: / /
9
1 x 1 x 8 2 3 m 3 2
2
= = .
Bi 353. Bi 353. Bi 353. Bi 353. i hc Thng Mi nm 1998 1999
Gii phng trnh:
2 2
x 3x 3 x 3x 6 3 = .
S: x 1 x 2 = = .
Bi 354. Bi 354. Bi 354. Bi 354. i hc Ngoi Thng nm 1998 1999
Vi gi tr no ca m th phng trnh:
3 3
1 x 1 x m = .
S: 0 m 2 < .
Bi 355. Bi 355. Bi 355. Bi 355. i hc Dn lp Tn c Thng nm 1998 1999
Gii phng trnh:
2 2 2
x x 7 x x 2 3x 3x 19 = .
S: x 2 x 1 = = . t
2
t x x 2 = .
Bi 356. Bi 356. Bi 356. Bi 356. i hc M a Cht nm 1998 1999
Gii v bin lun phng trnh: x a x a a = (vi a l tham s).
Bi 357. Bi 357. Bi 357. Bi 357. i hc Quc Gia Tp. H Ch Minh khi A t 2 nm 1997 1998
Vi gi tr no ca m th phng trnh
( )( )
3 x 6 x 3 x 6 x m = c nghim ?
S:
6 2 9
m 3
2

. Dng phng php hm s.


Bi 358. Bi 358. Bi 358. Bi 358. i hc Ngoi Thng Tp. H Ch Minh khi A nm 1997 1998
Gii phng trnh:
2 2
x 15 3x 2 x 8 = .
S: x 1 = . Phng php hm s.
Bi 359. Bi 359. Bi 359. Bi 359. i hc Y Dc Tp. H Ch Minh nm 1997 1998
Cho phng trnh:
( )

2
x 9 x x 9x m = .
1/ Gii phng trnh
( )
khi m 9 = .
2/ Xc nh tham s m phng trnh
( )
c nghim.
S: / /
9 65 9
1 x 0 x 9 x 2 m 10
2 4

= = = .
Bi 360. Bi 360. Bi 360. Bi 360. Hc Vin K Thut Qun S nm 1997 1998
Cho phng trnh:
( ) ( ) ( )

3
4
x 1 x 2m x 1 x 2 x 1 x m = .
www.MATHVN.com
www.DeThiThuDaiHoc.com
Ths. L Vn on Phn i S


Page - 70 - "All the flower of tomorrow are in the seeks of today"
1/ Gii phng trnh
( )
khi m 1 = .
2/ Tm gi tr ca tham s m phng trnh
( )
c mt nghim duy nht.
S: / /
1
1 x 2 m 1 m 0
2
= = = .
Bi 361. Bi 361. Bi 361. Bi 361. i hc Tng Hp Tp. H Ch Minh nm 1991 1992
Cho phng trnh:
( )( ) ( ) ( )

x 1
x 3 x 1 4 x 3 m
x 3


1/ Gii phng trnh m 3 = .
2/ Vi gi tr no ca m th phng trnh
( )
c nghim ?
S:
( )
/ /
x 1
t x 3 , 1 x 1 5 x 1 33 2 m 4
x 3

= = =

.
Bi 362. Bi 362. Bi 362. Bi 362. i hc S Phm Tp. H Ch Minh nm 1991 1992
Gii phng trnh:
3 3
x 34 x 3 1 = .
S: x 61 x 30 = = .
Bi 363. Bi 363. Bi 363. Bi 363. i hc khi B nm 2004
Xc nh m phng trnh sau c nghim:

( )
2 2 4 2 2
m 1 x 1 x 2 2 1 x 1 x 1 x = .
S: 2 1 m 1 (gii bng phng php hm s).
Bi 364. Bi 364. Bi 364. Bi 364. i hc khi D nm 2005
Gii phng trnh: 2 x 2 2 x 1 x 1 4 = .
S: x 3 = .
Bi 365. Bi 365. Bi 365. Bi 365. i hc khi B nm 2006
Tm tham s m phng trnh:
2
x mx 2 2x 1 = c hai nghim thc phn bit.
S:
9
m
2
.
Bi 366. Bi 366. Bi 366. Bi 366. i hc khi D nm 2006
Gii phng trnh:
2
2x 1 x 3x 1 0 = .
S: x 1; x 2 2 = = .
Bi 367. Bi 367. Bi 367. Bi 367. D b 1 i hc khi B nm 2006
Gii phng trnh:
2
3x 2 x 1 4x 9 2 3x 5x 2 = .
S: x 2 = .
www.MATHVN.com
www.DeThiThuDaiHoc.com
cng hc tp mn Ton 10 tp I Ths. L Vn on


"Cn c b thng minh" Page - 71 -
Bi 368. Bi 368. Bi 368. Bi 368. D b 2 i hc khi D nm 2006
Gii phng trnh:
2
x 2 7 x 2 x 1 x 8x 7 1 = .
S: x 5, x 4 = = . a v PT tch
( )( )
x 1 2 x 1 7 x 0 = .
Bi 369. Bi 369. Bi 369. Bi 369. i hc khi A nm 2007
Tm tham s m phng trnh sau c nghim thc:
4 2
3 x 1 m x 1 2 x 1 = .
S:
1
1 m
3
< . t
4
x 1
t , 0 t 1
x 1

= <

. PT
2
3t 2t m = . Dng PP hm s.
Bi 370. Bi 370. Bi 370. Bi 370. i hc khi B nm 2007
Chng minh rng vi mi gi tr dng ca tham s m, phng trnh sau c hai nghim thc
phn bit:
( )
2
x 2x 8 m x 2 = .
S: PT
( )( )
3 2
x 2
x 2 x 6x 32 m 0
'
1
1
1

!
1 =
1
1+
. Dng phng php hm s.
Bi 371. Bi 371. Bi 371. Bi 371. D b 2 i hc khi B nm 2007
Tm m phng trnh sau c ng 1 nghim: x
4 4
x 13 m x 1 0 = .
S:
3
m m 12
2
= . Dng phng php hm s.
Bi 372. Bi 372. Bi 372. Bi 372. D b 1 i hc khi D nm 2007
Tm m phng trnh sau c ng 2 nghim: x 3 2 x 4 x 6 x 4 5 m = .
S: 2 m 4 < . t t x 4 0 = .
Bi 373. Bi 373. Bi 373. Bi 373. i hc khi A nm 2008
Tm cc gi tr ca tham s m phng trnh sau c ng hai nghim thc phn bit
4 4
2x 2x 2 6 x 2 6 x m = .
S:
4
2 6 2 6 m 3 2 6 < . Dng phng php hm s.
Bi 374. Bi 374. Bi 374. Bi 374. i hc khi A nm 2009
Gii phng trnh:
3
2 3x 2 3 6 5x 8 0 = .
S: x 2 = .
Bi 375. Bi 375. Bi 375. Bi 375. i hc khi B nm 2010
Gii phng trnh:
2
3x 1 6 x 3x 14x 8 0 = .
S: x 5 = .
Bi 376. Bi 376. Bi 376. Bi 376. Ton Hc Tui Tr Thng 3 nm 2005
www.MATHVN.com
www.DeThiThuDaiHoc.com
Ths. L Vn on Phn i S


Page - 72 - "All the flower of tomorrow are in the seeks of today"
Gii phng trnh:
( )
2
x 2004 x 1 1 x
1

=


( )
.
S: x 0 = t y 1 x = .
Bi 377. Bi 377. Bi 377. Bi 377. Ton Hc Tui Tr Thng 9 nm 2007
Gii phng trnh:
2 2
x x 1 x x 1 2 = .
Bi 378. Bi 378. Bi 378. Bi 378. Tuyn chn hc sinh gii tnh Qung Bnh 21/12/2004
Gii phng trnh:
2
2x 6x 1 4x 5 = .
Bi 379. Bi 379. Bi 379. Bi 379. Tuyn sinh vo lp 10 chuyn Ton i hc S Phm H Ni I nm 1997 1998
Gii phng trnh:
( )
2
x 1 2 x 1 x 1 1 x 3 1 x = .
HD: a phng trnh v h c mt phng trnh tch s:

2 2 2 2
u 2u v v 3uv u v v 3uv 2v 0 = =

( )( )
u v v u v 2u 0 = .




www.MATHVN.com
www.DeThiThuDaiHoc.com
cng hc tp mn Ton 10 tp I Ths. L Vn on


"Cn c b thng minh" Page - 73 -
































BI TP P DNG
Bi 380. Bi 380. Bi 380. Bi 380. Gii cc h phng trnh sau bng cch dng nh thc
a/ b/
c/ d/
e/ f/
Bi 381. Bi 381. Bi 381. Bi 381. Gii cc h phng trnh sau
a/ b/
5x 4y 3
7x 9y 8
'
1 =
1
!
1 =
1
+
2x y 11
5x 4y 8
'
1 =
1
!
1 =
1
+
3x y 1
6x 2y 5
'
1 =
1
!
1 =
1
+
( )
( )
2 1 x y 2 1
2x 2 1 y 2 2
'
1
= 1
1
1
!
1
=
1
1
1+
3 2
x y 16
4 3
5 3
x y 11
2 5
'
1
1
=
1
1
1
!
1
1
=
1
1
1+

x
3x y 1
5 2y 3
'
1
=
1
1
!
1
=
1
1+
1 8
18
x y
5 4
51
x y
'
1
1
=
1
1
1
!
1
1
=
1
1
1+
10 1
1
x 1 y 2
25 3
2
x 1 y 2
'
1
1
=
1
1

1
!
1
1
=
1
1

1+
D H PHNG TRNH BC NHT NHIU N
C CC C H phng trnh bc nht hai n
Tnh cc nh thc: .
Xt D Kt qu


H c nghim duy nht .


hoc
H v nghim

H c v s nghim

gii h phng trnh bc nht hai n ta c th dng cc cch gii bit nh: phng php
th, phng php cng i s.


( )

1 1 1 2 2 2 2
1 1 2 2
2 2 2
a x b y c
a b 0, a b 0
a x b y c
'
1 =
1

!
1 =
1
+

1 1 1 1 1 1
1 2 2 1 x 1 2 2 1 y 1 2 2 1
2 2 2 2 2 2
a b c b a c
D a b a b , D c b c b , D a c a c
a b c b a c
= = = = = =
D 0

y
x
D
D
x ; y
D D
1

= =


( )
D 0 =
x
D 0
y
D 0
x y
D D 0 = =
C CC C H phng trnh bc nht nhiu n
Nguyn tc chung gii cc h phng trnh nhiu n l kh bt n a v
cc phng trnh hay h phng trnh c s n t hn. kh bt n, ta cng c
th dng cc phng php cng i s, phng php th nh i vi h phng
trnh bc nht hai n.
www.MATHVN.com
www.DeThiThuDaiHoc.com
Ths. L Vn on Phn i S


Page - 74 - "All the flower of tomorrow are in the seeks of today"
c/ d/
e/ f/
Bi 382. Bi 382. Bi 382. Bi 382. Gii cc h phng trnh sau
a/
2x 3y 2z 4
4x 2y 5z 6
2x 5y 3z 8
'
1
=
1
1
1
=
!
1
1
= 1
1
+
b/
3x 2y z 2
5x 3y 2z 10
2x 2y 3z 9
'
1
=
1
1
1
=
!
1
1
= 1
1
+

c/
x 2y z 12
2x y 3z 18
3x 3y 2z 9
'
1
=
1
1
1
=
!
1
1
= 1
1
+
d/
x y z 7
3x 2y 2z 5
4x y 3z 10
'
1
=
1
1
1
=
!
1
1
= 1
1
+

e/ f/
g/
x y z 0
3x 2y 4z 17
5x y 7z 22
'
1
=
1
1
1
=
!
1
1
= 1
1
+
h/
2x y z 3
x 2y z 6
x y 2z 7
'
1
=
1
1
1
=
!
1
1
= 1
1
+

Bi 383. Bi 383. Bi 383. Bi 383. Gii v bin lun cc h phng trnh sau
a/ b/
c/ d/
e/ f/
Bi 384. Bi 384. Bi 384. Bi 384. Trong cc h phng trnh sau hy
Gii v bin lun. Tm h c nghim duy nht l nghim nguyn.
a/ b/
c/ d/
Bi 385. Bi 385. Bi 385. Bi 385. Trong cc h phng trnh sau hy
Gii v bin lun.
Khi h c nghim , tm h thc gia x, y c lp i vi m
27 32
7
2x y x 3y
45 48
1
2x y x 3y
'
1
1
=
1
1

1
!
1
1
=
1
1

1+
2 x 6 3 y 1 5
5 x 6 4 y 1 1
'
1
=
1
1
!
1
=
1
1+
2 x y x y 9
3 x y 2 x y 17
'
1
=
1
1
!
1
=
1
1+
4 x y 3 x y 8
3 x y 5 x y 6
'
1
=
1
1
!
1
=
1
1+
3x y z 1
2x y 2z 5
x 2y 3z 0
'
1 =
1
1
1
=
!
1
1
= 1
1
+
x 3y 2z 8
2x y z 6
3x y z 6
'
1 =
1
1
1
=
!
1
1
= 1
1
+
mx (m 1)y m 1
2x my 2
'
1 =
1
!
1 =
1
+
( )
( ) ( )
mx m 2 y 5
m 2 x m 1 y 2
'
1
=
1
1
!
1
=
1
1+
( )
( )
m 1 x 2y 3m 1
m 2 x y 1 m
'
1
=
1
1
!
1
=
1
1+
( ) ( )
( ) ( )
m 4 x m 2 y 4
2m 1 x m 4 y m
'
1
=
1
1
!
1
=
1
1+
( )
2 2
m 1 x 2y m 1
m x y m 2m
'
1
=
1
1
!
1
=
1
1+
mx 2y m 1
2x my 2m 5
'
1 =
1
!
1 =
1
+
m Z
2 2
(m 1)x 2y m 1
m x y m 2m
'
1 =
1
1
!
1 =
1
1+
mx y 1
x 4(m 1)y 4m
'
1 =
1
!
1 =
1
+
mx y 3 3
x my 2m 1 0
'
1 =
1
!
1 =
1
+
x my 1
x y m
'
1 =
1
!
1 =
1
+
( )
x, y
www.MATHVN.com
www.DeThiThuDaiHoc.com
cng hc tp mn Ton 10 tp I Ths. L Vn on


"Cn c b thng minh" Page - 75 -
a/ b/
c/ d/
Bi 386. Bi 386. Bi 386. Bi 386. Gii v bin lun cc h phng trnh sau
a/ b/
c/ d/
e/ f/
Bi 387. Bi 387. Bi 387. Bi 387. nh tham s m h phng trnh sau c nghim duy nht
a/ b/
Bi 388. Bi 388. Bi 388. Bi 388. nh tham s h phng trnh sau c v s nghim
a/ b/
c/ d/
Bi 389. Bi 389. Bi 389. Bi 389. nh tham s m h phng trnh sau v nghim
a/ b/
Bi 390. Bi 390. Bi 390. Bi 390. nh tham s h phng trnh sau c nghim
a/ b/
c/ d/
Bi 391. Bi 391. Bi 391. Bi 391. Gii bi ton bng cch lp h phng trnh
a/ Tm hai s bit tng ca chng bng 188 v nu ly s ln chia cho s nh ta c thng
bng 5 v s d bng 2.
b/ S cng nhn hai x nghip t l vi 2 v 3. Nu s cng nhn x nghip I tng 80 ngi
v s cng nhn x nghip II tng 40 ngi th s cng nhn mi hai x nghip t l vi 3
v 4. Hi s cng nhn lc u mi x nghip ?
c/ Tm mt s gm hai ch s bit: nu em s chia cho tng s ca hai ch s ta c
thng l 6; nu em cng tch ca hai ch s vi 25 ta c s o li.
mx 2y m 1
2x my 2m 5
'
1 =
1
!
1 =
1
+
( )
( )
6mx 2 m y 3
m 1 x my 2
'
1
=
1
1
!
1
=
1
1+
( )
mx m 1 y m 1
2x my 2
'
1
=
1
1
!
1 =
1
1+
x my 3
mx 4y m 4
'
1 =
1
!
1 =
1
+
ax y b
3x 2y 5
'
1 =
1
!
1 =
1
+
y ax b
2x 3y 4
'
1 =
1
!
1 =
1
+
ax y a b
x 2y a
'
1 =
1
!
1 =
1
+
( ) ( )
( ) ( )
a b x a b y a
2a b x 2a b y b
'
1
=
1
1
!
1
=
1
1+
2 2
ax by a b
bx ay 2ab
'
1
=
1
1
!
1 =
1
1+
2
2
ax by a b
bx b y 4b
'
1
=
1
1
!
1
=
1
1+
2
mx y m
x my m
'
1 =
1
1
!
1 =
1
1+
( )
( )
6mx 2 m y 3
m 1 x my 2
'
1
=
1
1
!
1
=
1
1+
x y 1
4x 4y m 1
'
1 =
1
!
1 =
1
+
( )
4x my m 1
m 6 x 2y 3 m
'
1 =
1
1
!
1 =
1
1+
ax y a b
x 2y a
'
1 =
1
!
1 =
1
+
2 2
2
a x by a b
bx b y 2 4b
'
1
=
1
1
!
1
=
1
1+
x my 1
mx 3my 2m 3
'
1 =
1
!
1 =
1
+
mx y 1
x my 1
'
1 =
1
!
1 =
1
+
( ) ( )
mx 2y m
m 1 x m 1 y 1
'
1 =
1
1
!
1 =
1
1+
( )
mx my m 1
m 1 x 2my m 1
'
1 =
1
1
!
1 =
1
1+
( ) ( )
( ) ( )
3
2 3 4
m m 1 x m m 1 y m 2
m 1 x m 1 y m 1
'
1
=
1
1
!
1
=
1
1+
( ) ( )
( ) ( )
2 2 2 2 2
a b x a b y 2a
a b x a b y 2a
'
1
=
1
1
!
1
=
1
1+
www.MATHVN.com
www.DeThiThuDaiHoc.com
Ths. L Vn on Phn i S


Page - 76 - "All the flower of tomorrow are in the seeks of today"
d/ Hai cng nhn phi lm mt s dng c bng nhau trong cng mt thi gian. Ngi I mi
gi lm tng 2 dng c nn cng vic hon thnh trc 2 gi. Ngi II mi gi lm tng 4
dng c nn cng vic hon thnh trc 3 gi v cn lm thm 6 dng c. Tnh s dng c
mi cng nhn phi lm v thi gian phi hon thnh cng vic ?
BI TP RN LUYN
Bi 392. Bi 392. Bi 392. Bi 392. Dng nh thc gii cc h phng trnh sau
a/ b/
c/ d/
e/ f/
Bi 393. Bi 393. Bi 393. Bi 393. Gii cc h phng trnh sau
a/
3x 4y 5z 12
4x 2y 7z 7
5x 6y 4z 12
'
1
=
1
1
1
=
!
1
1
= 1
1
+
b/
1 2 3
1 2 3
1 2 3
0, 3x 4, 7x 2, 3x 4, 9
2,1x 3, 2x 4, 5x 7, 6
4, 2x 2, 7x 3, 7x 5, 7
'
1
=
1
1
1
=
!
1
1
= 1
1
+

c/
1 2 3
1 2 3
1 2 3
x 2x 3x 2
2x 7x x 5
3x 3x 2x 7
'
1
=
1
1
1
=
!
1
1
= 1
1
+
d/
x 3y 4z 3
3x 4y 2z 5
2x y 2z 4
'
1
=
1
1
1
=
!
1
1
= 1
1
+

e/
2x 3y z 4
3x 2y 3z 9
4x 5y 8z 15
'
1
=
1
1
1
=
!
1
1
= 1
1
+
f/
2x y 2z 4
4x 3y 3z 4
6x 5y 4z 4
'
1
=
1
1
1
=
!
1
1
= 1
1
+

g/
x 2y 3z 4
3x y 3z 7
x 3y 3z 3
'
1
=
1
1
1
=
!
1
1
= 1
1
+
h/
x 2y z 2
3x y z 6
x 3y 3z 2
'
1
=
1
1
1
=
!
1
1
= 1
1
+

Bi 394. Bi 394. Bi 394. Bi 394. Gii v bin lun h phng trnh theo tham s
a/ b/
c/ d/
e/ f/
g/ h/
2x y 3
3x y 7
'
1 =
1
!
1 =
1
+
7x y 4
3x 2y 1
'
1 =
1
!
1 =
1
+
4y 2x 1 0
x 2y 6
'
1 =
1
!
1 =
1
+
2x y 5
y x 1 0
'
1 =
1
!
1 =
1
+
2x y 1 0
2y 4x 2 0
'
1 =
1
!
1 =
1
+
5x y 6 0
x 5y 6
'
1
=
1
1
!
1
=
1
1+
x my 1
x y m
'
1 =
1
!
1 =
1
+
ax y 3 0
x ay 3a
'
1 =
1
!
1 =
1
+
2
2x my m
x y 2 0
'
1
=
1
1
!
1 =
1
1+
mx y 1 0
x my 2 0
'
1 =
1
!
1 =
1
+
2
x my 1
mx y m
'
1 =
1
1
!
1 =
1
1+
my x 3 0
mx 4y m 4
'
1 =
1
!
1 =
1
+
( )
( )
m 1 x y m 2
m 1 x 2y m 5
'
1
=
1
1
!
1
=
1
1+
( )
( )
2a 1 x y 1
x a 1 y 1 0
'
1
=
1
1
!
1
=
1
1+
www.MATHVN.com
www.DeThiThuDaiHoc.com
cng hc tp mn Ton 10 tp I Ths. L Vn on


"Cn c b thng minh" Page - 77 -
i/ j/
Bi 395. Bi 395. Bi 395. Bi 395. Gii v bin lun h phng trnh theo tham s
a/ b/
c/ d/
Bi 396. Bi 396. Bi 396. Bi 396. nh tham s m h phng trnh sau c nghim duy nht
a/ b/
c/ d/
e/ f/
Bi 397. Bi 397. Bi 397. Bi 397. nh tham s m h phng trnh c nghim duy nht v tm h thc c lp gia x v y
a/ b/
c/ d/
Bi 398. Bi 398. Bi 398. Bi 398. nh tham s cc h phng trnh sau c v s nghim
a/ b/
Bi 399. Bi 399. Bi 399. Bi 399. nh tham s m cc h phng trnh sau v nghim
a/ b/
Bi 400. Bi 400. Bi 400. Bi 400. nh tham s m cc h phng trnh sau c nghim
a/ b/
Bi 401. Bi 401. Bi 401. Bi 401. Tm tham s m h phng trnh c nghim duy nht tha .
( )
mx my m 1
m 1 x 2my m 1
'
1 =
1
1
!
1 =
1
1+
( )
( )
2
2m x 3 m 1 y 3
m x y 2y 2
'
1
=
1
1
!
1
=
1
1+
ax y b
bx y a
'
1 =
1
!
1 =
1
+
( ) ( )
( ) ( )
a b x a b y a
2a b x 2a b y b
'
1
=
1
1
!
1
=
1
1+
( ) ( )
( ) ( )
2 2
m m 1 x m 2m 2 y m 2m 3
m 1 x m 1 y m 1
'
1
=
1
1
!
1
=
1
1+
( ) ( )
( ) ( )
3
2 3 4
m m 1 x m m 1 y m 2
m 1 x m 1 y m 1
'
1
=
1
1
!
1
=
1
1+
( )
( )
m 1 x 8y 4m 0
mx m 3 y 1 3m 0
'
1
=
1
1
!
1
=
1
1+
( ) ( )
( ) ( )
m 5 x 2m 3 y 3m 2
3m 10 x 5m 6 y 2m 4
'
1
=
1
1
!
1
=
1
1+
( )( )
( ) ( )
m x m 1 y 2 m 1
m 3 x 2 y 2 2m 4
'
1
=
1
1
!
1
=
1
1+
x 2y m
mx my m 1
'
1 =
1
1
!
1 =
1
1+
x my 1
mx y 2m
'
1 =
1
1
!
1 =
1
1+
( )
2
2
mx 2y m
x m 3 y m 1
'
1
=
1
1
!
1
=
1
1+
mx y 3
x my 2m 1
'
1 =
1
!
1 =
1
+
( )
mx y 1
x 4 m 1 y 4m
'
1 =
1
1
!
1 =
1
1+
mx y 2m
x my m 1
'
1 =
1
!
1 =
1
+
( )
( )
6mx 2 m y 3
m 1 x my 2
'
1
=
1
1
!
1
=
1
1+
( )
mx 4y 2m 3
m 1 x 6y
'
1 =
1
1
!
1 =
1
1+
( ) ( )
( ) ( )
2 2 2 2 2
a b x a b y a
a b x a b y a 1
'
1
=
1
1
!
1
=
1
1+
4x 2y 5
2x y m 1
'
1 =
1
!
1 =
1
+
( )
( )
2 3
5
m x 2 m y 4 m
mx 2m 1 y m 2
'
1
=
1
1
!
1
=
1
1+
( )
( )
2m 1 x y 1
x m 1 y 1 0
'
1
=
1
1
!
1
=
1
1+
( ) ( )
( ) ( )
2 2 2
m 3 x m 3 y 2m
m 9 x m 9 y 2m
'
1
=
1
1
!
1
=
1
1+
( )
2
x m 2 y 2m 2
mx 2y m 2
'
1
=
1
1
!
1
=
1
1+
x y
www.MATHVN.com
www.DeThiThuDaiHoc.com
Ths. L Vn on Phn i S


Page - 78 - "All the flower of tomorrow are in the seeks of today"
Bi 402. Bi 402. Bi 402. Bi 402. Cho h phng trnh
a/ Tm tham s m h phng trnh c nghim duy nht.
b/ Gi s h c nghim duy nht th khi m bng my ?
c/ nh tham s m nghim ca h c dng .
Bi 403. Bi 403. Bi 403. Bi 403. nh tham s m cc h phng trnh sau c nghim sao cho biu thc
c gi tr nh nht
a/ b/
Bi 404. Bi 404. Bi 404. Bi 404. nh tham s m nguyn cc h sau c nghim nguyn
a/ b/
c/ d/
e/ f/
g/ h/
Bi 405. Bi 405. Bi 405. Bi 405. nh m nguyn h c nghim nguyn dng
a/ b/
Bi 406. Bi 406. Bi 406. Bi 406. Cho h phng trnh . Tm tt c cc gi tr ca a v b sao cho h phng
trnh c nghim duy nht .
Bi 407. Bi 407. Bi 407. Bi 407. nh b h phng trnh c nghim
Bi 408. Bi 408. Bi 408. Bi 408. Gi s h c nghim. Chng minh rng: .
Bi 409. Bi 409. Bi 409. Bi 409. Tm mt s c hai ch s, bit hiu ca hai ch s bng 3. Nu vit cc ch s theo th t
ngc li th c mt s bng
4
5
s ban u tr i 10.
( )
( )

2
mx 4y m 4
x m 3 y 2m 3
'
1
=
1
1

!
1
=
1
1+
( )

( )
x, y x y
( )

( )

x 5 4t
t
y t
'
1 =
1

!
1 =
1
+
R
( )
x, y
2 2
P x y =
2x y 5
2y x m
'
1 =
1
!
1 =
1
+
x 2y 4 m
2x y 3m 3
'
1 =
1
!
1 =
1
+
( )
m Z
2mx 3y m
x y m 1
'
1 =
1
!
1 =
1
+
mx y 3
x my 2m 1
'
1 =
1
!
1 =
1
+
mx y 2m
x my m 1
'
1 =
1
!
1 =
1
+
( ) ( )
mx 2y m
m 1 x m 1 y 1
'
1 =
1
1
!
1 =
1
1+
( )
mx y 1
x 4 m 1 y 4m
'
1 =
1
1
!
1 =
1
1+
mx 2y 3 0
3mx y 4m 0
'
1 =
1
!
1 =
1
+
( )
2
2
mx 2y m 2
m 1 x y m 1
'
1 =
1
1
!
1
=
1
1+
( ) ( )
( )
m 1 x 3m 1 y 2 m
2x m 2 y 4 0
'
1
=
1
1
!
1
=
1
1+
mx y 3 0
x my 2m 1 0
'
1 =
1
!
1 =
1
+
( )
2 2
m 1 x 2y m 1
m x y m 2m
'
1
=
1
1
!
1
=
1
1+
ax by 3
ax by ab 3
'
1
=
1
1
!
1
=
1
1+
( ) ( )
x, y 1,1 =
2
ax y 3b
a :
x ay b b
'
1 =
1
1

!
1 =
1
1+
R
ax by c
bx cy a
cx ay b
'
1
=
1
1
1
=
!
1
1
= 1
1
+
3 3 3
a b c 3abc =
www.MATHVN.com
www.DeThiThuDaiHoc.com
cng hc tp mn Ton 10 tp I Ths. L Vn on


"Cn c b thng minh" Page - 79 -
Bi 410. Bi 410. Bi 410. Bi 410. Ba c Lan, Hng v Thy cng thu mt loi o ging nhau. S o ca Lan thu trong 1 gi t
hn tng o ca Hng v Thy thu trong 1 gi l 5 o. Tng s o ca Lan thu trong 4 gi
v Hng thu trong 3 gi nhiu hn s o ca Thy thu trong 5 gi l 30 o. S o ca Lan
thu trong 2 gi cng vi s o ca Hng thu trong 5 gi v s o ca Thy thu trong 3 gi
tt c c 76 o. Hi trong mt gi mi c thu c my o ?
Bi 411. Bi 411. Bi 411. Bi 411. Mt cng ty c 85 xe ch khch gm hai loi, xe ch c 4 khch v xe ch c 7 khch.
Dng tt c s xe , ti a cng ty ch mt ln c 445 khch. Hi cng ty c my xe
mi loi ?
Bi 412. Bi 412. Bi 412. Bi 412. Mt ch ca hng bn l mang 1 500 000 ng n ngn hng i tin xu tr li cho ngi
mua. ng ta i c tt c 1450 ng tin xu cc loi 2000 ng, 1000 ng v 500 ng.
Bit rng s tin xu loi 1000 ng bng hai ln hiu ca s tin xu loi 500 ng vi s tin
xu loi 2000 ng. Hi mi loi c bao nhiu ng tin xu ?
Bi 413. Bi 413. Bi 413. Bi 413. Mt gia nh c bn ngi ln v ba tr em mua v xem xic ht 370 000 ng. Mt gia nh
khc c hai ngi ln v hai tr em cng mua v xem xic ti rp ht 200 000 ng. Hi gi
v ngi ln v gi v tr em l bao nhiu ?
Bi 414. Bi 414. Bi 414. Bi 414. Nu ly mt s c hai ch s chia cho tch hai ch s ca n th c thng l 12 v d l 18.
Nu ly tng bnh phng cc ch s ca s cng vi 9 th c s cho. Hy tm s ?
Bi 415. Bi 415. Bi 415. Bi 415. Mt on xe ti ch 290 tn xi mng cho mt cng trnh xy p thy in. on xe c 57
chic gm ba loi, xe ch 3 tn, xe ch 5 tn v xe ch 7,5 tn. Nu dng tt c xe 7,5 tn ch
ba chuyn th c s xi mng bng tng s xi mng do xe 5 tn ch ba chuyn v xe 3 tn ch
hai chuyn. Hi s xe mi loi ?






























www.MATHVN.com
www.DeThiThuDaiHoc.com
Ths. L Vn on Phn i S


Page - 80 - "All the flower of tomorrow are in the seeks of today"






















































E H PHNG TRNH BC HAI HAI N S

C CC C H gm 1 phng trnh bc nht v 1 phng trnh bc hai
T phng trnh bc nht rt mt n theo n kia.
Th vo phng trnh bc hai a v phng trnh bc hai mt n.
S nghim ca h tu theo s nghim ca phng trnh bc hai ny.
C CC C H i xng loi 1 vi v .
Nhn dng: khi ta hon v (i ch) x v y th v khng thay i.
Phng php gii:
t v .
a h phng trnh v h vi cc n S v P.
Gii h ta tm c S v P.
Tm nghim bng cch gii phng trnh .
C CC C H i xng loi 2
Nhn dng: khi hon v gia x v y th bin thnh v ngc li.
Phng php gii:
Tr v v theo v ta c:
Bin i v phng trnh tch:
Lc :
Gii cc h trn ta tm c nghim ca h .
C CC C H ng cp
( )
( )
( )
f x, y 0
I
g x, y 0
'
1
=
1
1
!
1
=
1
1+
( ) ( )
f x, y f y, x =
( ) ( )
g x, y g y, x =
( )
f x, y
( )
g x, y
S x y = P xy =
( )
I
( )
II
( )
II
( )
x, y
2
X SX P 0 =
( )
( ) ( )
( ) ( )
f x, y 0 1
I
f y, x 0 2
'
1
=
1
1
!
1
=
1
1+
( )
1
( )
2
( )
1
( )
2
( )
( ) ( ) ( )
( ) ( )
f x, y f y, x 0 3
I
f x, y 0 1
'
1
=
1
1

!
1
=
1
1+
( )
3
( ) ( ) ( )
( )
x y
3 x y .g x, y 0
g x, y 0

( )
( )
( )
( )
f x, y 0
x y
I
f x, y 0
g x, y 0
'
1
=
1 1
!

1 =

1
1+

'
1
=
1
1

1
=
1

1+
( )
I
( )

2 2
1 1 1 1
2 2
2 2 2 2
a x b xy c y d
I
a x b xy c y d
'
1
=
1
1
!
1
=
1
1+
Gii h khi (hoc ).
Khi , t . Th vo h ta c h theo t v x. Kh x ta tm
c phng trnh bc hai theo t. Gii phng trnh ny ta tm c t, t
tm c .
O Lu : Ngoi cc cch gii thng thng ta cn s dng phng php bt
ng thc, phng php hm s, lng gic ha (hc lp 11 v 12).
x 0 = y 0 =
x 0 y tx =
( )
I
( )
x, y
www.MATHVN.com
www.DeThiThuDaiHoc.com
cng hc tp mn Ton 10 tp I Ths. L Vn on


"Cn c b thng minh" Page - 81 -
BI TP P DNG
Bi 416. Bi 416. Bi 416. Bi 416. Gii cc h phng trnh sau
a/ b/
c/ d/
e/ f/
g/ h/
i/ j/
Bi 417. Bi 417. Bi 417. Bi 417. Gii cc h phng trnh sau
a/ b/
c/ d/
e/ f/
Bi 418. Bi 418. Bi 418. Bi 418. Gii cc h phng trnh sau
a/ b/
c/ d/
e/ f/
Bi 419. Bi 419. Bi 419. Bi 419. Gii cc h phng trnh sau
a/ b/
c/ d/
2 2
x 4y 8
x 2y 4
'
1
=
1
1
!
1 =
1
1+
2
x xy 24
2x 3y 1
'
1
=
1
1
!
1 =
1
1+
( )
2
x y 49
3x 4y 84
'
1
1 =
1
!
1
=
1
1+
2 2
x 3xy y 2x 3y 6 0
2x y 3
'
1
=
1
1
!
1 =
1
1+
( )
3x 4y 1 0
xy 3 x y 9
'
1 =
1
1
!
1 =
1
1+
2x 3y 2
xy x y 6 0
'
1 =
1
!
1 =
1
+
2
y x 4x
2x y 5 0
'
1
=
1
1
!
1 =
1
1+
2 2
2x 3y 5
3x y 2y 4
'
1 =
1
1
!
1 =
1
1+
2 2
2x y 5
x xy y 7
'
1 =
1
1
!
1 =
1
1+
2 2
4x 3xy y 1
2x y 1 0
'
1
=
1
1
!
1 =
1
1+
( )
2 2
x xy y 11
x y xy 2 x y 31
'
1 =
1
1
!
1 =
1
1+
2 2
x y 4
x xy y 13
'
1 =
1
1
!
1 =
1
1+
2 2
xy x y 5
x y x y 8
'
1 =
1
1
!
1 =
1
1+
x y 13
y x 6
x y 6
'
1
1
=
1
1
!
1
1
=
1
1+
3 3 3 3
x x y y 17
x y xy 5
'
1
=
1
1
!
1 =
1
1+
4 2 2 4
2 2
x x y y 481
x xy y 37
'
1
=
1
1
!
1
=
1
1+
2
2
x 3x 2y
y 3y 2x
'
1
=
1
1
!
1
=
1
1+
2 2
2 2
x 2y 2x y
y 2x 2y x
'
1
=
1
1
!
1
=
1
1+
3
3
x 2x y
y 2y x
'
1
=
1
1
!
1
=
1
1+
y
x 3y 4
x
x
y 3x 4
y
'
1
1
=
1
1
1
!
1
1
=
1
1
1+
2
2
2
2
y 2
3y
x
x 2
3x
y
'
1

1
=
1
1
1
!
1
1
=
1
1
1+
2
2
1
2x y
y
1
2y x
x
'
1
1
=
1
1
1
!
1
1
=
1
1
1+
2 2
2 2
2x 4xy y 1
3x 2xy 2y 7
'
1
=
1
1
!
1
=
1
1+
2
2 2
y 3xy 4
x 4xy y 1
'
1
=
1
1
!
1
=
1
1+
2 2
2 2
3x 5xy 4y 38
5x 9xy 3y 15
'
1
=
1
1
!
1
=
1
1+
2 2
2 2
x 2xy 3y 9
x 4xy 5y 5
'
1
=
1
1
!
1
=
1
1+
www.MATHVN.com
www.DeThiThuDaiHoc.com
Ths. L Vn on Phn i S


Page - 82 - "All the flower of tomorrow are in the seeks of today"
e/ f/
Bi 420. Bi 420. Bi 420. Bi 420. Gii v bin lun cc h phng trnh sau
a/ b/
c/ d/
Bi 421. Bi 421. Bi 421. Bi 421. Gii v bin lun cc h phng trnh sau
a/ b/
c/ d/
Bi 422. Bi 422. Bi 422. Bi 422. Gii v bin lun cc h phng trnh sau
a/ b/
c/ d/
Bi 423. Bi 423. Bi 423. Bi 423. Gii v bin lun cc h phng trnh sau
a/ b/
c/ d/
BI TP RN LUYN
Bi 424. Bi 424. Bi 424. Bi 424. Gii cc h phng trnh sau
a/
2 2
4x 3xy y 1
2x y 1 0
'
1
=
1
1
!
1 =
1
1+
b/
2 2
x y 6x 2y 0
x y 8 0
'
1
=
1
1
!
1 =
1
1+

c/
2 2
x 2xy y x y 6
x 2y 3
'
1
=
1
1
!
1 =
1
1+
d/
2 2
2 2
4x 3xy y 1
4x 4x y 1
'
1
=
1
1
!
1 =
1
1+

Bi 425. Bi 425. Bi 425. Bi 425. Gii cc h phng trnh sau
a/
2 2
x y xy 4
x y xy 2
'
1
=
1
1
!
1 =
1
1+
b/
( )
2 2
x y 2 xy 2
x y 6
'
1
=
1
1
!
1 =
1
1+

2 2
2 2
3x 8xy 4y 0
5x 7xy 6y 0
'
1
=
1
1
!
1
=
1
1+
2 2
2 2
x 3xy y 1
3x xy 3y 13
'
1
=
1
1
!
1
=
1
1+
2 2
x y 6
x y m
'
1 =
1
1
!
1 =
1
1+
2 2
x y m
x y 2x 2
'
1 =
1
1
!
1 =
1
1+
2 2
3x 2y 1
x y m
'
1 =
1
1
!
1 =
1
1+
2 2
4x 3xy y 1
2x y m 1
'
1
=
1
1
!
1
1
1+
2 2
x y xy m
x y 3 2m
'
1 =
1
1
!
1 =
1
1+
2 2 2
x y m 1
x y xy 2m m 3
'
1 =
1
1
!
1 =
1
1+
( )( )
( )
x 1 y 1 m 5
xy x y 4m
'
1
=
1
1
!
1
=
1
1+
2 2
x y xy 4
x y xy m
'
1
=
1
1
!
1 =
1
1+
2
2
x 3x my
y 3y mx
'
1
=
1
1
!
1
=
1
1+
( ) ( )
( ) ( )
2 2
2 2
x 3 4y m 3 4m
y 3 4x m 3 4m
'
1
=
1
1
!
1
=
1
1+
( )
( )
2
2
xy x m y 1
xy y m x 1
'
1
=
1
1
!
1
=
1
1+
2 2
2 2
2x y 3x m
2y x 3y m
'
1
=
1
1
!
1
=
1
1+
( )
2 2
2 2
x mxy y m
x m 1 xy my m
'
1
=
1
1
!
1
=
1
1+
2
2
xy y 12
x xy m 26
'
1
=
1
1
!
1
=
1
1+
2 2
2
x 4xy y m
y 3xy 4
'
1
=
1
1
!
1
=
1
1+
2 2
2 2
2x 3xy 2y 4
x 5xy 3y m
'
1
=
1
1
!
1
=
1
1+
www.MATHVN.com
www.DeThiThuDaiHoc.com
cng hc tp mn Ton 10 tp I Ths. L Vn on


"Cn c b thng minh" Page - 83 -
c/
2 2
x xy y 3
x xy y 1 0
'
1
=
1
1
!
1 =
1
1+
d/
2 2
x y xy 3
x y y x 2
'
1 =
1
1
!
1 =
1
1+

e/
3 3
x y 1
x y 17
'
1 =
1
1
!
1 =
1
1+
f/
x y 13
y x 6
x y 5
'
1
1
=
1
1
!
1
1
=
1
1+

g/
2 2
1 1
5
x y
1 1
13
x y
'
1
1
=
1
1
1
!
1
1
=
1
1
1+
h/
2 2
x y
18
y x
x y 12
'
1
1
=
1
1
!
1
1
=
1
1+

i/
x y 2
x y xy 1
'
1
=
1
1
!
1
=
1
1+
j/
x y 4
x y xy 4
'
1
=
1
1
!
1
=
1
1+

k/
x 1 y 4
x y 7
'
1
=
1
1
!
1
=
1
1+
l/
x y 1 1
x y 2 2y 2
'
1
=
1
1
!
1
=
1
1+

Bi 426. Bi 426. Bi 426. Bi 426. Gii cc h phng trnh sau
a/
2
2
x 3x 2y
y 3y 2x
'
1
=
1
1
!
1
=
1
1+
b/
2
2
x 13x 4y
y 13y 4x
'
1
=
1
1
!
1
=
1
1+

c/
2
2
xy x 1 y
xy y 1 x
'
1
=
1
1
!
1
=
1
1+
d/
2 2
2 2
2x y 3x 2
2y x 3y 2
'
1
=
1
1
!
1
=
1
1+

e/
2
2
2x y 4y 5
2y x 5x 5
'
1
=
1
1
!
1
=
1
1+
f/
2 2
2 2
x 2y 2x y
y 2x 2y x
'
1
=
1
1
!
1
=
1
1+

g/
2x y 1 2
2y x 1 2
'
1
=
1
1
!
1
=
1
1+
h/
x 1 y 1
x y 1 1
'
1
=
1
1
!
1
=
1
1+

i/
x 1 y 1 4
y 1 x y 4
'
1
=
1
1
!
1
=
1
1+
j/
x 1 y 2 3
y 1 x 2 3
'
1
=
1
1
!
1
=
1
1+

Bi 427. Bi 427. Bi 427. Bi 427. Gii cc h phng trnh sau
a/
2 2
2 2
3x 2xy y 11
x 2xy 3y 17
'
1
=
1
1
!
1
=
1
1+
b/
2 2
2 2
2x 4xy y 1 0
3x 2xy 2y 7
'
1
=
1
1
!
1
=
1
1+

c/
2
2 2
y 3xy 4 0
x 4xy y 1
'
1
=
1
1
!
1
=
1
1+
d/
2 2
2 2
x 2xy 3y 9
x 4xy 5y 5
'
1
=
1
1
!
1
=
1
1+

e/
2 2
2 2
3x 8xy 4y 0
5x 7xy 6y 0
'
1
=
1
1
!
1 =
1
1+
f/
2 2
2 2
2x 3xy 2y 4 0
x 5xy 3y 1 0
'
1
=
1
1
!
1 =
1
1+

g/
2 2
x 2xy 3y 0
x x y y 2
'
1
=
1
1
!
1
=
1
1+
h/
2 2
4 2 2 4
x xy y 7
x x y y 21
'
1
=
1
1
!
1
=
1
1+

Bi 428. Bi 428. Bi 428. Bi 428. nh tham s m h sau c nghim
www.MATHVN.com
www.DeThiThuDaiHoc.com
Ths. L Vn on Phn i S


Page - 84 - "All the flower of tomorrow are in the seeks of today"
a/
2 2
x y m
xy 1
'
1
=
1
1
!
1 =
1
1+
b/
2 2
x y m
x y 2
'
1
=
1
1
!
1 =
1
1+

c/
3 3
x y 1
x y 1 3m
'
1 =
1
1
!
1 =
1
1+
d/
x y 1
x x y y 1 3m
'
1
=
1
1
!
1
=
1
1+

e/
( )( )
2 2
x y 2x 2y 11
xy x 2 y 2 m
'
1
=
1
1
!
1
=
1
1+
f/
( )( )
( )
2 2
xy x 4 y 4 m
x y 4 x y 5
'
1
=
1
1
!
1
=
1
1+

Bi 429. Bi 429. Bi 429. Bi 429. nh tham s m h c nghim duy nht
a/
x y
m
y x
x y 1
'
1
1
=
1
1
!
1
1
=
1
1+
b/
2 2
x y xy 3
x y m
'
1 =
1
1
!
1 =
1
1+

c/
2 2
2 2
1 1
x y 2m 4
x y
1 1
x y m
x y
'
1
1
=
1
1
1
!
1
1
=
1
1
1+
d/
( )( )
x y 2 2 x 1 y 1
x y xy m
'
1
=
1
1
!
1 =
1
1+

e/
2 2
x xy y m 6
2x xy 2y m
'
1
=
1
1
!
1 =
1
1+
f/
2 2
x xy y m
x y y x m 1
'
1 =
1
1
!
1 =
1
1+

g/
( )
( )
2
2
xy x m y 1
xy y m x 1
'
1
=
1
1
!
1
=
1
1+
h/
2 2
x xy y m
x y xy 1 2m
'
1 =
1
1
!
1 =
1
1+

Bi 430. Bi 430. Bi 430. Bi 430. Gii cc h phng trnh sau
a/
2 2
x xy y 3
x xy y 1 0
'
1
=
1
1
!
1 =
1
1+
b/
2 2
3 3
2x y xy 15
8x y 35
'
1
=
1
1
!
1
=
1
1+

c/
( )
2 2
x y xy 11
x y 3 x y 28
'
1 =
1
1
!
1 =
1
1+
d/
2 2
xy x y 3 0
x y x y xy 6
'
1 =
1
1
!
1 =
1
1+

e/
( )
3 3
x y 2
xy x y 2 0
'
1
=
1
1
!
1
=
1
1+
f/
( )
3 3
x y 7
xy x y 2 0
'
1
=
1
1
!
1
=
1
1+

g/
3 3 3 3
x y x y 17
x y xy 5
'
1
=
1
1
!
1 =
1
1+
h/
( )
3 3
x y 2
xy x y 2
'
1
=
1
1
!
1
=
1
1+

i/
( )
2 2
4 4
x y xy 78
x y 97
'
1
=
1
1
!
1
=
1
1+
j/
( ) ( )
2 2
x y x y 8
x x 1 y y 1 12
'
1
=
1
1
!
1
=
1
1+

k/
( )( )
2 2
x 2x y 2y 11
xy x 2 y 2 24
'
1
=
1
1
!
1
=
1
1+
l/
( ) ( )
( )( )
2 2
x 1 y 2 9
xy x 2 y 4 5 0
'
1
1 =
1
!
1
=
1
1+

m/
3
3
x 2x y
y 2y x
'
1
=
1
1
!
1 =
1
1+
n/
3
3
x 2x y
y 2y x
'
1
=
1
1
!
1 =
1
1+

www.MATHVN.com
www.DeThiThuDaiHoc.com
cng hc tp mn Ton 10 tp I Ths. L Vn on


"Cn c b thng minh" Page - 85 -
o/
2
2
2
2
y 3
3y
x
x 2
3x
y
'
1

1
=
1
1
1
!
1
1
=
1
1
1+
q/
2
2
3
2x y
x
3
2y x
y
'
1
1
=
1
1
1
!
1
1
=
1
1
1+

Bi 431. Bi 431. Bi 431. Bi 431. Gii cc h phng trnh sau
a/
x 1 y 2 1
x y 10
'
1
=
1
1
!
1
=
1
1+
b/
x y 3 0
x y xy 7
'
1 =
1
1
!
1
=
1
1+

c/
x 4 y 1 4
x y 15
'
1
=
1
1
!
1
=
1
1+
d/
x y 3 4
y x 2 3
'
1
=
1
1
!
1
=
1
1+

e/
2x y 2 4 0
2y x 2 4
'
1
=
1
1
!
1
=
1
1+
f/
2x 2y
3
y x
x y xy 3
'
1
1
1 =
1
!
1
1
=
1
1+

Bi 432. Bi 432. Bi 432. Bi 432. Gii cc phng trnh sau bng cch a v h i xng loi 2
a/
2
2 x 2 x = . b/ 4 4 x x = .
c/
3 2 3
x 4 x 4 = . d/
3 3
x 2 3 3x 2 = .
e/ 5 5 x x = . f/
3 3
x 1 2 2x 1 = .
g/
2
x 2x 2 2x 1 = h/
3 3
x 4 4x 3 3 = .
Bi 433. Bi 433. Bi 433. Bi 433. Cho h phng trnh
( )

2 2
x y xy m
x y y x 3m 8
'
1 =
1
1

!
1 =
1
1+

a/ Gii h
( )
khi
7
m
2
= .
b/ Tm m h
( )
c nghim.
Bi 434. Bi 434. Bi 434. Bi 434. Tm tham s m h phng trnh
2 2
x xy y m
x y xy 1 2m
'
1 =
1
1
!
1 =
1
1+
c ng hai nghim.
Bi 435. Bi 435. Bi 435. Bi 435. Cho h phng trnh
( )

2 2
x my m 0
x y x 0
'
1 =
1
1

!
1 =
1
1+

a/ Tm tham s m h phng trnh
( )
c hai nghim phn bit.
b/ Gi
( ) ( )
1 1 2 2
x ; y , x ; y l cc nghim ca h. Chng minh:
( ) ( )
2 2
2 1 2 1
x x y y 1 .
Bi 436. Bi 436. Bi 436. Bi 436. Gii cc h phng trnh sau
a/
x y 2x y 2 7
3x 2y 23
'
1
=
1
1
!
1
=
1
1+
b/
2 2
x y x y 2
x y 25
'
1
=
1
1
!
1
=
1
1+

c/
2x y 1 x y 1
3x 2y 4
'
1
=
1
1
!
1
=
1
1+
d/
2 2
x y y x 6
x y y x 20
'
1
=
1
1
!
1
=
1
1+

www.MATHVN.com
www.DeThiThuDaiHoc.com
Ths. L Vn on Phn i S


Page - 86 - "All the flower of tomorrow are in the seeks of today"
e/
x 2 y 2
y 2 x 2
'
1
=
1
1
!
1
=
1
1+
f/
x y xy 3
x 1 y 1 4
'
1
=
1
1
!
1
=
1
1+

g/
3
x y x y
x y x y 2
'
1
=
1
1
!
1
=
1
1+
h/
x y y x 30
x x y y 35
'
1
=
1
1
!
1
=
1
1+

i/
2 2
x y 2xy 8 2
x y 4
'
1
= 1
1
!
1
=
1
1+
j/
x y 7
1
y x
xy
x xy y xy 78
'
1
1
1 =
1
1
!
1
1
1
=
1
1+

k/
2 2
xy x y x 2y
x 2y y x 1 2x 2y
'
1
=
1
1
!
1
=
1
1+
l/
( )
( )
2 2 3 3
3
3
2 x y 3 x y xy
x y 6
'
1
= 1
1
!
1
1 =
1
+

Bi 437. Bi 437. Bi 437. Bi 437. Gii cc h phng trnh sau
a/
2 2
x y 8
x 9 y 9 10
'
1 =
1
1
!
1
=
1
1+
b/
( ) ( )
x 1 y 1 3
x 1 y 1 y 1 x 1 6
'
1
=
1
1
!
1
=
1
1+

c/
x 1 2 y 2 x y 1
x y 2
'
1
=
1
1
!
1
=
1
1+
d/
2 2
x 4 y 1 y 3 x 2
x y xy 2y x 12
'
1
=
1
1
!
1
=
1
1+

e/
1
x x y 3 3
y
1
2x y 8
y
'
1
1
1 =
1
1
!
1
1
= 1
1
1
+
f/
2
2 3
2
2 3
2xy
x x y
x 2x 9
2xy
y y x
y 2y 9
'
1
1
=
1
1
1
1
!
1
1
=
1
1
1
1+

g/
2 2
2 2
x x y 1 x y x y 1 y 18
x x y 1 x y x y 1 y 2
'
1
= 1
1
!
1
=
1
1+
h/
( )
( )
2 2
2 2
1
x y 1 5
xy
1
x y 1 49
x y
' 1
1
1
= 1

1

( ) 1
1
!
1
1
1
= 1

1

( )
1
1+

Bi 438. Bi 438. Bi 438. Bi 438. Gii cc h phng trnh sau
a/
2 2
xy 3x 2y 16
x y 2x 4y 33
'
1 =
1
1
!
1 =
1
1+
b/
( ) ( )
2 2
x y x y 4
x x y 1 y y 1 2
'
1
=
1
1
!
1
=
1
1+

c/
( )
2
x xy x y 4
x y xy x y 4
'
1
=
1
1
!
1
=
1
1+
d/
4 3 2 2
3 2
x x y x y 1
x y x xy 1
'
1
=
1
1
!
1
=
1
1+

e/
( )( )
2 2
2
x y 1 x y 1 3x 4x 1
xy x 1 x
'
1
=
1
1
!
1
=
1
1+
f/
( )( )
2
x xy 2x 2y 16 0
x y 4 xy 32
'
1
=
1
1
!
1
=
1
1+

g/
2
1 1
x y
x y
2x xy 1 0
'
1
1
=
1
1
!
1
1
=
1
1+
h/
3
1 1
x y
x y
2y x 1
'
1
1
=
1
1
!
1
1
=
1
1+

i/
( )( )
( )( )
2 2
2 2
x y x y 13
x y x y 25
'
1
=
1
1
!
1
=
1
1+
j/
( )
( )
2 2
3
2 2
x xy y 3 x y
x xy y 7 x y
'
1
=
1
1
!
1
=
1
1+

www.MATHVN.com
www.DeThiThuDaiHoc.com
cng hc tp mn Ton 10 tp I Ths. L Vn on


"Cn c b thng minh" Page - 87 -
k/
( )
2 3 2
4 2
5
x y x y xy xy
4
5
x y xy 1 2x
4
'
1
1
=
1
1
1
!
1
1
=
1
1
1+
l/
( )
3 3
2 2
x 8x y 2y
x 3 3 y 1
'
1
=
1
1
!
1
=
1
1+

m/
( )( )
2
2 2
y 5x 4 4 x
y 5x 4xy 16x 8y 16 0
'
1
=
1
1
!
1
=
1
1+
n/
4 3 2 2
2
x 2x y x y 2x 9
x 2xy 6x 6
'
1
=
1
1
!
1 =
1
1+

Bi 439. Bi 439. Bi 439. Bi 439. Gii cc h phng trnh sau
a/
( )
3
3
x 2 3y 8
xy 2x 6 0
'
1
=
1
1
!
1
=
1
1+
b/
3 2
2 2
x y 2
x xy y y 0
'
1
=
1
1
!
1 =
1
1+

c/
2 2
2 2
1 1
x y 4
x y
1 1
x y 4
x y
'
1
1
=
1
1
1
!
1
1
=
1
1
1+
d/
( ) ( )
( )( )
2
2
x 1 y y x 4y
x 1 y x 2 y
'
1
=
1
1
!
1
=
1
1+

e/
( )
( )
2 2
2
3
4xy 4 x y 7
x y
1
2x 3
x y
'
1
1
=
1
1
1
1
!
1
1
1
=
1
1
1+
f/
( )
2 2
3 2 2
1
x y
2
4x x x x 1 y 2xy 2
'
1
1
=
1
1
!
1
1
=
1
1+

Bi 440. Bi 440. Bi 440. Bi 440. nh tham s m h sau c nghim
a/
x y m
x y xy m
'
1
=
1
1
!
1
=
1
1+
b/
3 2
3 2
1 1
x y 5
x y
1 1
x y 15m 10
x y
'
1
1
=
1
1
1
!
1
1
=
1
1
1+

c/
( )
2 3 2
4 2
x y x y xy xy m
x y xy 1 2x m
'
1
=
1
1
!
1
=
1
1+
d/
x y 1
x x y y 1 3m
'
1
=
1
1
!
1
=
1
1+

Bi 441. Bi 441. Bi 441. Bi 441. Cho h phng trnh
( )
( )
( )

2 2
2
x y 2 1 m
x y 4
'
1
=
1
1

!
1
=
1
1+

a/ Chng minh rng nu
( )
o o
x , y l mt nghim ca h phng trnh
( )
th
( )
o o
x , y cng
l nghim. T tm iu kin cn ca m h phng trnh
( )
c nghim duy nht.
b/ Th li cc gi tr ca m tm cu a c kt lun cui cng.
Bi 442. Bi 442. Bi 442. Bi 442. Trong cc h phng trnh sau
Tm s nguyn m h c nghim duy nht l nghim nguyn.
Khi h c nghim , tm h thc gia x, y c lp vi m.
a/ b/
c/ d/
( )
x, y
mx 2y m 1
2x my 2a 1
'
1 =
1
!
1 =
1
+
mx y 3m
x my 2m 1
'
1 =
1
!
1 =
1
+
x 2y 4 m
2x y 3m 3
'
1 =
1
!
1 =
1
+
2x y 5
2y x 10m 5
'
1 =
1
!
1 =
1
+
www.MATHVN.com
www.DeThiThuDaiHoc.com
Ths. L Vn on Phn i S


Page - 88 - "All the flower of tomorrow are in the seeks of today"
BI TP QUA CC K THI
Bi 443. Bi 443. Bi 443. Bi 443. Cao ng S Phm Tp. H Ch Minh nm 1998
Cho h phng trnh:
( )

x y m
x y xy m
'
1
=
1
1

!
1
=
1
1+

1/ Gii h phng trnh
( )
khi m 4 = .
2/ Tm tham s m h phng trnh
( )
c nghim.
Bi 444. Bi 444. Bi 444. Bi 444. Cao ng S Phm Tp. H Ch minh nm 1999 Cao ng S Phm Vinh nm 2001
Cho h phng trnh:
( )
( )

3 3
x y 1
x y m x y
'
1 =
1
1

!
1 =
1
1+

1/ Gii h phng trnh
( )
khi m 1 = .
2/ Tm tham s m h phng trnh c ba nghim phn bit.
Bi 445. Bi 445. Bi 445. Bi 445. Cao ng S Phm Hng Yn khi A nm 2000
Gii h phng trnh:
2 2
x x y y 6
x y y x 20
'
1
=
1
1
!
1
=
1
1+

Bi 446. Bi 446. Bi 446. Bi 446. Cao ng Y T Nam nh H Cao ng iu Dng chnh qui nm 2000
Gii h phng trnh:
2 2
x xy y 4
x xy y 2
'
1
=
1
1
!
1 =
1
1+

Bi 447. Bi 447. Bi 447. Bi 447. Cao ng Kim St pha Nam nm 2000
Gii h phng trnh:
2 2
x y 2
x y 10
'
1 =
1
1
!
1 =
1
1+

Bi 448. Bi 448. Bi 448. Bi 448. Cao ng Giao Thng nm 2000
Gii h phng trnh:
3
x y 9
3x y 6
'
1
=
1
1
!
1 =
1
1+

Bi 449. Bi 449. Bi 449. Bi 449. Cao ng S Phm H Ni khi A nm 2001
Gii h phng trnh:
2
2
xy 10 20 x
xy 5 y
'
1
=
1
1
!
1
=
1
1+

Bi 450. Bi 450. Bi 450. Bi 450. Cao ng S Phm Nh Tr Mu Gio TW1 nm 2001
Gii h phng trnh:
2 2
5
x y xy
4
1
x y xy
4
'
1
1
=
1
1
1
!
1
1
=
1
1
1+

Bi 451. Bi 451. Bi 451. Bi 451. Cao ng S Phm Vnh Phc khi A nm 2002
www.MATHVN.com
www.DeThiThuDaiHoc.com
cng hc tp mn Ton 10 tp I Ths. L Vn on


"Cn c b thng minh" Page - 89 -
Gi s
( )
x, y l cc nghim ca h phng trnh:
2 2 2
x y 2m 1
x y m 2m 3
'
1 =
1
1
!
1 =
1
1+

Xc nh tham s m biu thc P xy = t gi tr nh nht.
Bi 452. Bi 452. Bi 452. Bi 452. Cao ng Xy Dng s 3 nm 2002 Hc Vin Hng Khng nm 1997 1998
Gii h phng trnh:
( )( )
( )( )
2 2
2 2
x y x y 3
x y x y 15
'
1
=
1
1
!
1
=
1
1+

Bi 453. Bi 453. Bi 453. Bi 453. Cao ng S Phm H Tnh khi A, B nm 2002
Gii h phng trnh:
( )
3 3
2 2
x y 7 x y
x y x y 2
'
1
=
1
1
!
1
=
1
1+

Bi 454. Bi 454. Bi 454. Bi 454. Cao ng S Phm Qung Ngi nm 2002
Gii h phng trnh:
2 2
2 2
2x y 3x 4
2y x 3y 4
'
1
=
1
1
!
1
=
1
1+

Bi 455. Bi 455. Bi 455. Bi 455. Cao ng K Thut H Ty 2002 Hc Vin Ngn Hng Phn Vin TP.HCM nm 2001
Gii h phng trnh:
2 2
2 2
x 2xy 3y 9
2x 13xy 15y 0
'
1
=
1
1
!
1
=
1
1+

Bi 456. Bi 456. Bi 456. Bi 456. Cao ng Kinh T K Thut Thi Bnh nm 2002
Gii h phng trnh:
3 3
x y 2
x y 26
'
1 =
1
1
!
1 =
1
1+

Bi 457. Bi 457. Bi 457. Bi 457. Cao ng S Phm K Thut Vinh nm 2002
Tm a h phng trnh sau c nghim duy nht:
( )
( )
2
2
xy x a y 1
xy y a x 1
'
1
=
1
1
!
1
=
1
1+

Bi 458. Bi 458. Bi 458. Bi 458. Cao ng khi A, D nm 2003
Gii h phng trnh:
( )
2 2
2 2
x x y y
x y 3 x y
'
1
=
1
1
!
1
=
1
1+

Bi 459. Bi 459. Bi 459. Bi 459. Cao ng khi M, T nm 2003
Gii h phng trnh:
( )( )
2
2
x 2x 3x y 18
x 5x y 9 0
'
1
=
1
1
!
1
=
1
1+

Bi 460. Bi 460. Bi 460. Bi 460. Cao ng Nng Lm nm 2003
Gii h phng trnh:
2 2
2 2
2x 3y 4xy 3
2x y 7
'
1
=
1
1
!
1
=
1
1+

Bi 461. Bi 461. Bi 461. Bi 461. Cao ng khi A nm 2004
www.MATHVN.com
www.DeThiThuDaiHoc.com
Ths. L Vn on Phn i S


Page - 90 - "All the flower of tomorrow are in the seeks of today"
Gii h phng trnh:
2 2
3 3
2x y xy 15
8x y 35
'
1
=
1
1
!
1
=
1
1+

Bi 462. Bi 462. Bi 462. Bi 462. Cao ng Kinh T K Hoch Nng nm 2004
Gii h phng trnh:
2 2
xy x y 3
x y x y xy 6
'
1 =
1
1
!
1 =
1
1+

Bi 463. Bi 463. Bi 463. Bi 463. Cao ng S Phm Kom Tum nm 2005
Gii h phng trnh:
x 1 y 1 4
x y 8
'
1
=
1
1
!
1
=
1
1+

Bi 464. Bi 464. Bi 464. Bi 464. Cao ng S Phm H Nam nm 2005 i hc Ngoi Ng nm 2001
Gii h phng trnh:
2 2
3 3
x y 1
x y 1
'
1
=
1
1
!
1
=
1
1+

Bi 465. Bi 465. Bi 465. Bi 465. Cao ng S Phm Sc Trng nm 2005
Gii h phng trnh:
( )
3 3
x y 2
xy x y 2
'
1
=
1
1
!
1
=
1
1+

Bi 466. Bi 466. Bi 466. Bi 466. Cao ng Ti Chnh K Ton IV nm 2005
Gii h phng trnh:
2 2
x y xy 3
x y y x 2
'
1 =
1
1
!
1 =
1
1+

Bi 467. Bi 467. Bi 467. Bi 467. Cao ng Kinh T K Thut I nm 2005
Gii h phng trnh:
2
2
xy x 1 y
xy y 1 x
'
1
=
1
1
!
1
=
1
1+

Bi 468. Bi 468. Bi 468. Bi 468. Cao ng S Phm Tp. H Ch Minh nm 2001
Xc nh tham s m h phng trnh:
( )
( )
2
2
x m 2 x my
y m 2 y mx
'
1
=
1
1
!
1
=
1
1+
c ng hai nghim phn bit
Bi 469. Bi 469. Bi 469. Bi 469. i hc Nng nm 2001
Gii h phng trnh:
2 2
x xy y 1
x xy 6
'
1 =
1
1
!
1 =
1
1+

Bi 470. Bi 470. Bi 470. Bi 470. i hc Thi Nguyn nm 2001
Gii h phng trnh:
3
3
x 1 2y
y 1 2x
'
1
=
1
1
!
1
=
1
1+

Bi 471. Bi 471. Bi 471. Bi 471. i hc Y Hi Phng nm 2001
www.MATHVN.com
www.DeThiThuDaiHoc.com
cng hc tp mn Ton 10 tp I Ths. L Vn on


"Cn c b thng minh" Page - 91 -
Gii h phng trnh:
( )( )
2 2
x y x y 4
xy x 1 y 1 4
'
1
=
1
1
!
1
=
1
1+

Bi 472. Bi 472. Bi 472. Bi 472. Hc Vin Qun Y nm 2001
Gii h phng trnh:
2 2 2 2
x y x y 2
x y x y 4
'
1
=
1
1
!
1
=
1
1+

Bi 473. Bi 473. Bi 473. Bi 473. Hc Vin Hnh Chnh Quc Gia i hc S Phm H Ni khi B, M, T nm 2001
Gii h phng trnh:
3 3
x y 8
x y 2xy 2
'
1
=
1
1
!
1 =
1
1+

Bi 474. Bi 474. Bi 474. Bi 474. i hc Nng Nghip I khi A nm 2001
Gii h phng trnh:
( )
2
3 3
x y y 2
x y 19
'
1
1 =
1
!
1
=
1
1+

Bi 475. Bi 475. Bi 475. Bi 475. i hc Cn Th khi A nm 2001
Tm gi tr ca a h
2
2 2
x 3 y a
y 5 x x 5 3 a
'
1
= 1
1
!
1
=
1
1+
c ng mt nghim.
Bi 476. Bi 476. Bi 476. Bi 476. i hc Vn Ha khi D nm 2001 i hc Dn lp ng nm 1997 1998
Gii h phng trnh:
x 1 7 y 4
y 1 7 x 4
'
1
=
1
1
!
1
=
1
1+

S: x y 8 = = .
Bi 477. Bi 477. Bi 477. Bi 477. Phn Vin Bo Ch v Tuyn Truyn nm 2001
Gii h phng trnh:
2
2
1
2x y
y
1
2y x
x
'
1
1
=
1
1
1
!
1
1
=
1
1
1+

Bi 478. Bi 478. Bi 478. Bi 478. i hc Thy Li nm 2001
Gii h phng trnh:
2
2
3
2x y
x
3
2y x
y
'
1
1
=
1
1
1
!
1
1
=
1
1
1+

Bi 479. Bi 479. Bi 479. Bi 479. Hc Vin Quan H Quc T nm 2001
Gii h phng trnh:
( )( )
2 2 3 3
x y 4
x y x y 280
'
1 =
1
1
!
1 =
1
1+

Bi 480. Bi 480. Bi 480. Bi 480. i hc Hng Hi nm 2001
www.MATHVN.com
www.DeThiThuDaiHoc.com
Ths. L Vn on Phn i S


Page - 92 - "All the flower of tomorrow are in the seeks of today"
Gii h phng trnh:
( )
( )
2
2 2
2 2
x xy y 19 x y
x xy y 7 x y
'
1
1 =
1
!
1
=
1
1+

Bi 481. Bi 481. Bi 481. Bi 481. i hc Thng Mi nm 2001
Gii h phng trnh:
3 3 3
2 2
1 x y 19x
x xy 6x
'
1
=
1
1
!
1
=
1
1+

Bi 482. Bi 482. Bi 482. Bi 482. i hc Ngoi Thng nm 2001
Gii h phng trnh:
3 3
6 6
x 3x y 3y
x y 1
'
1
=
1
1
!
1
=
1
1+

Bi 483. Bi 483. Bi 483. Bi 483. i hc S Phm Tp. HCM khi A, B i hc Lut Tp. HCM khi A nm 2001
Xc nh tham s a h sau y c nghim duy nht:
( )
( )
2
2
x 1 y a
y 1 x a
'
1
1 =
1
!
1
= 1
1
+

Bi 484. Bi 484. Bi 484. Bi 484. i hc S Phm Tp. H Ch Minh khi D, M, T nm 2001
Cho h phng trnh:
x 1 y 2 m
y 1 x 2 m
'
1
=
1
1
!
1
=
1
1+
(vi m l tham s).
1/ Gii h phng trnh khi m 0 = .
2/ Xc nh tham s m h phng trnh c nghim.
Bi 485. Bi 485. Bi 485. Bi 485. Trung Tm Bi Dng Cn B Y T Tp. H Ch Minh nm 2001
Cho h phng trnh:
( )

3
3
x 2y x m
y 2x y m
'
1
=
1
1

!
1
=
1
1+
vi m l tham s.
1/ Gii h phng trnh
( )
khi m 2 = .
2/ Xc nh cc gi tr ca tham s m h
( )
c nghim duy nht.
Bi 486. Bi 486. Bi 486. Bi 486. i hc Kinh T Tp. H Ch Minh nm 2001
Cho h phng trnh:
( )

2
2
xy y 12
x xy 26 m
'
1
=
1
1

!
1
=
1
1+

1/ Gii h phng trnh
( )
khi m 2 = .
2/ Vi nhng gi tr no ca tham s m th h phng trnh cho c nghim.
Bi 487. Bi 487. Bi 487. Bi 487. i hc Nng Lm Tp. H Ch Minh nm 2001
Gii h phng trnh:
3 3
x y 6
x y 126
'
1 =
1
1
!
1 =
1
1+

Bi 488. Bi 488. Bi 488. Bi 488. i hc Dn lp Ngoi Ng Tin hc Tp. H Ch Minh nm 2001
www.MATHVN.com
www.DeThiThuDaiHoc.com
cng hc tp mn Ton 10 tp I Ths. L Vn on


"Cn c b thng minh" Page - 93 -
Gii h phng trnh:
2 2
2 2
x 2xy 2y 5
3x xy y 3
'
1
=
1
1
!
1
=
1
1+

Bi 489. Bi 489. Bi 489. Bi 489. i hc Ti Chnh K Ton H Ni nm 2001
Gii h phng trnh:
4 4
6 6
x y 1
x y 1
'
1
=
1
1
!
1
=
1
1+

Bi 490. Bi 490. Bi 490. Bi 490. i hc M H Ni nm 2001
Gii h phng trnh:
2x 2y
3
y x
x y xy 3
'
1
1
1 =
1
!
1
1
=
1
1+

Bi 491. Bi 491. Bi 491. Bi 491. Trng S Quan Lc Qun 2 Cp phn i nm 1999 2000
Cho h phng trnh:
( )

2 2
x xy y m 6
2x xy 2y m
'
1
=
1
1

!
1 =
1
1+
vi m l tham s.
1/ Gii h phng trnh
( )
khi m 3 = .
2/ Xc nh tt c cc gi tr ca tham s m h phng trnh
( )
c nghim duy nht.
S:
( ) ( )
( )

/ / a S 3, 3 , 3, 3 , 1, 1 b m 21 = = .
Bi 492. Bi 492. Bi 492. Bi 492. i hc Giao Thng Vn Ti nm 1999 2000
Gii h phng trnh:
2 2
xy 3x 2y 16
x y 2x 4y 33
'
1 =
1
1
!
1 =
1
1+

S:
( ) ( )
S 3 3, 2 3 , 3 3, 2 3 = .
Bi 493. Bi 493. Bi 493. Bi 493. i hc Y Dc Tp. H Ch Minh nm 1999 2000 h trung cp
Gii v bin lun theo m h phng trnh:
2 2
x y m
x y 2x 2
'
1 =
1
1
!
1 =
1
1+

Bi 494. Bi 494. Bi 494. Bi 494. i hc An Ninh khi D, G nm 1999 2000
Gii h phng trnh:
2 2
2 2
1 1
x y 4
x y
1 1
x y 4
x y
'
1
1
=
1
1
1
!
1
1
=
1
1
1+

S:
( )
S 1,1 = . C th gii theo BT Bunhiacpxki.
Bi 495. Bi 495. Bi 495. Bi 495. i hc Hng Hi nm 1999 2000
www.MATHVN.com
www.DeThiThuDaiHoc.com
Ths. L Vn on Phn i S


Page - 94 - "All the flower of tomorrow are in the seeks of today"
Gii h phng trnh:
( )

x y 7
1
y x x 0, y 0
xy
x xy y xy 78
'
1
1
1 =
1
1

!
1
1
1 =
1
1+
.
S:
( ) ( )
S 4, 9 , 9, 4 = .
Bi 496. Bi 496. Bi 496. Bi 496. i Hc Ngoi Thng H Ni nm 1999 2000

Gii h phng trnh:
( )
( )
2 2
2 2
1
x y 1 5
xy
1
x y 1 49
x y
' 1
1
1
= 1

1

( ) 1
1
!
1
1
1
= 1

1

( ) 1
1+

S:
7 45 7 45
S , 1 , 1,
2 2
' ' 1 1
1 1
1 1
1 1


=
! !


1 1

( ) ( ) 1 1
1 1 + +
.
Bi 497. Bi 497. Bi 497. Bi 497. i hc H Ni khi A, B nm 1999 2000
Gii h phng trnh:
1 3
2x
y x
1 3
2y
x y
'
1
1
=
1
1
1
!
1
1
=
1
1
1+

S:
( )
( ) ( )
S 1,1 , 2, 2 , 2, 2 = .
Bi 498. Bi 498. Bi 498. Bi 498. i hc S Phm Qui Nhn nm 1999 2000
Cho h phng trnh:
( )

2 2 2
x y m 1
x y y x 2m m 3
'
1 =
1
1

!
1 =
1
1+

1/ Gii h phng trnh
( )
khi m 3 = .
2/ Chng minh rng vi mi m th h phng trnh
( )
lun c nghim.
S:
( ) ( )
S 1, 3 , 3,1 = .
Bi 499. Bi 499. Bi 499. Bi 499. i hc S Phm H Ni 2 nm 1999 2000
Gii h phng trnh:
2 2
2 2
x y 3x 4y 1
3x 2y 9x 8y 3
'
1
=
1
1
!
1
=
1
1+

S:
3 3 3 3
S , 0 , , 4
2 2
' ' 1 1
1 1
1 1
1 1


=
! !


1 1

( ) ( ) 1 1
1 1 + +
.
Bi 500. Bi 500. Bi 500. Bi 500. i hc S Phm Vinh nm 1999 2000
www.MATHVN.com
www.DeThiThuDaiHoc.com
cng hc tp mn Ton 10 tp I Ths. L Vn on


"Cn c b thng minh" Page - 95 -
Tm tham s m h
3 2 2
3 2 2
x y 7x mx
y x 7y my
'
1
=
1
1
!
1
=
1
1+
c nghim duy nht.
S: m 16 .
Bi 501. Bi 501. Bi 501. Bi 501. i hc Quc gia Tp. H Ch Minh t 2 nm 1998 1999
Cho h phng trnh:
( )

2 2
2 2
3x 2xy y 11
x 2xy 3y 17 m
'
1
=
1
1

!
1
=
1
1+

1/ Gii h phng trnh
( )
vi m 0 = .
2/ Vi gi tr no ca tham s m th h phng trnh
( )
c nghim.
S:
( )
/ /
4 5
1 S 1, 2 , , 2 5 11 3 m 5 11 3
3 3
' ' 1
1 1
1 1
= ! !

1 1
( )
1 1
+ +
.
Bi 502. Bi 502. Bi 502. Bi 502. i hc Nng nm 1998 1999
Gii v bin lun theo m s nghim ca h phng trnh:
x y 3m
2y xy 0
'
1 =
1
1
!
1
=
1
1+

S:
( )
( )
y 0 x 3m S 3m, 0 , m
y m x 4m S 4m, m , m 0
= = =
= = =
R

Bi 503. Bi 503. Bi 503. Bi 503. i hc Kin Trc nm 1998 1999
Gii h phng trnh:
3 3
2 2
y x 7
2x y 3xy 16
'
1
=
1
1
!
1
=
1
1+

S:
( )
S 1, 2 = .
Bi 504. Bi 504. Bi 504. Bi 504. i hc Thi Nguyn khi D nm 1998 1999
Gii h phng trnh:
x y y x 30
x x y y 35
'
1
=
1
1
!
1
=
1
1+

S:
( ) ( )
S 4, 9 , 9, 4 = .
Bi 505. Bi 505. Bi 505. Bi 505. i hc Thy Sn nm 1998 1999
Gii h phng trnh:
x y z 6
2 2 2
3
x y z
xy yz zx 12
'
1 =
1
1
1
1
1
=
!
1
1
1
1 =
1
1+

S: x y z 2 = = = . Nhn hai v ca
( )
2 cho xyz.
Bi 506. Bi 506. Bi 506. Bi 506. i hc Hng Hi nm 1998 1999
www.MATHVN.com
www.DeThiThuDaiHoc.com
Ths. L Vn on Phn i S


Page - 96 - "All the flower of tomorrow are in the seeks of today"
Gii h phng trnh:
3 x 5y 9 0
2x y 7 0
'
1
=
1
1
!
1
=
1
1+
.
S:
44 39
S ,
7 7
' ' 1
1 1
1 1
= ! !

1 1
( )
1 1
+ +
. Chia 4 trng hp gii.
Bi 507. Bi 507. Bi 507. Bi 507. i hc M a Cht nm 1998 1999
Gii h phng trnh:
2 2
3 3
x y y x 30
x y 35
'
1
=
1
1
!
1
=
1
1+

S:
( ) ( )
S 2, 3 , 3, 2 = .
Bi 508. Bi 508. Bi 508. Bi 508. i hc M Thut nm 1998 1999
Gii h phng trnh:
2
2
x 3x y
y 3y x
'
1
=
1
1
!
1
=
1
1+

S:
( ) ( )
S 0, 0 , 2, 2 = .
Bi 509. Bi 509. Bi 509. Bi 509. i hc Quc Gia Tp. H Ch Minh nm 1997 1998
Cho h phng trnh:
( ) ( )
( )

2
x y m
x 1 y xy m y 2
'
1 =
1
1

!
1 =
1
1+

1/ Gii h phng trnh
( )
khi m 4 = .
2/ Tm tt c gi tr ca tham s m h phng trnh
( )
c nhiu hn 2 nghim.
S:
( )
( )
/ /
3 6
1 S 2, 2 , 3 5,1 5 2 m
2
= .
Bi 510. Bi 510. Bi 510. Bi 510. i hc Ngoi Thng Tp. H Ch Minh khi A nm 1997 1998
Gii h phng trnh:
2 2
2 2
1 1
x y 5
x y
1 1
x y 9
x y
'
1
1
=
1
1
1
!
1
1
=
1
1
1+

S:
3 5 3 5
S 1, , ,1
2 2
' ' 1 1
1 1
1 1
1 1


=
! !


1 1

( ) ( ) 1 1
1 1 + +
.
Bi 511. Bi 511. Bi 511. Bi 511. i hc M a Cht nm 1997 1998
Gii h phng trnh:
( )
( )
2 2
2 2
2y x y 3x
x x y 10y
'
1
=
1
1
!
1
=
1
1+

www.MATHVN.com
www.DeThiThuDaiHoc.com
cng hc tp mn Ton 10 tp I Ths. L Vn on


"Cn c b thng minh" Page - 97 -
S:
( ) ( )
5 15 3 15
S 0, 0 , 2, 1 , ,
2 2
' ' 1
1 1
1 1

1 1

= ! !

1 1


1 1
( )
1 1
+ +
.
Bi 512. Bi 512. Bi 512. Bi 512. i hc Ngoi Thng H Ni nm 1997 1998
Cho h phng trnh:
( )( )
( )

2 2
x y x y 8
xy x 1 y 1 m
'
1
=
1
1

!
1
=
1
1+

1/ Gii h phng trnh
( )
vi m 12 = .
2/ Vi gi tr no ca tham s m th h phng trnh
( )
cho c nghim.
S:
( ) ( ) ( ) ( ) ( ) ( ) ( ) ( )
/ 1 S 1, 2 , 2,1 , 1, 3 , 3,1 , 2, 2 , 2, 2 , 2, 3 , 3, 2 =
/
33
2 m ,16
16
l
l

l
l

Bi 513. Bi 513. Bi 513. Bi 513. i hc Xy Dng nm 1997 1998
Gii h phng trnh:
( ) ( ) ( )
2 2
2 2
2x y 5 4x y 6 2x y 0
1
2x y 3
2x y
'
1
1 =
1
1
!
1
=
1
1

1
+

S:
3 1 3 1
S , , ,
8 4 4 2
' ' 1 1
1 1
1 1
= ! !


1 1
( ) ( )
1 1
+ +
. Chia hai v PT
( )
1 cho
( )
2
2x y .
Bi 514. Bi 514. Bi 514. Bi 514. i hc Quc Gia H Ni khi B nm 1997 1998
Gii v bin lun theo tham s a h phng trnh:
x y
a
y x
x y 8
'
1
1
=
1
1
!
1
1
=
1
1+

Bi 515. Bi 515. Bi 515. Bi 515. i hc Quc Gia H Ni khi A nm 1997 1998
Gii h phng trnh:
4y
x 3y
x
4x
y 3x
y
'
1
1
=
1
1
1
!
1
1
=
1
1
1+

S: x y 2 = = .
Bi 516. Bi 516. Bi 516. Bi 516. i hc Quc Gia H Ni khi D nm 1997 1998
Gii h phng trnh:
( )
3 3
x y 7
xy x y 2
'
1
=
1
1
!
1
=
1
1+

S:
( ) ( )
S 1, 2 , 2,1 = . Cch 1. H ng cp. Cch 2. t n ph x y a, xy b = = .
Bi 517. Bi 517. Bi 517. Bi 517. i hc Thy Li nm 1997 1998
www.MATHVN.com
www.DeThiThuDaiHoc.com
Ths. L Vn on Phn i S


Page - 98 - "All the flower of tomorrow are in the seeks of today"
Gii h phng trnh:
x y 4
x y xy 4
'
1
=
1
1
!
1
=
1
1+

S:
( ) ( )
S 1, 9 , 9,1 = .
Bi 518. Bi 518. Bi 518. Bi 518. i hc Tng Hp nm 1991 1992
Cho h phng trnh:
( )

2 2
x y xy m
x y m
'
1 =
1
1

!
1 =
1
1+

1/ Gii h phng trnh khi m 5 = .
2/ Vi gi tr no ca tham s m th h phng trnh
( )
c nghim.
S:
( ) ( )
/ / 1 S 2,1 , 1, 2 2 m 0; 8
l
=
l
l
.
Bi 519. Bi 519. Bi 519. Bi 519. i hc Bch Khoa Tp. H Ch Minh nm 1994 1995
Cho h phng trnh:
( )

2 2
x y xy m
x y xy 3m 8
'
1 =
1
1

!
1 =
1
1+

1/ Gii h phng trnh khi
7
m
2
= .
2/ Vi gi tr no ca tham s m th h phng trnh
( )
c nghim.
S: / /
1 1 13 3 33
1 S 2, , , 2 2 m m 8
2 2 8
' ' 1 1
1 1

1 1
= ! !


1 1
( ) ( )
1 1
+ +
.
Bi 520. Bi 520. Bi 520. Bi 520. i hc S Phm K Thut Tp. H Ch Minh nm 1994 1995
Gii h phng trnh:
2 2
x y 2xy 8 2
x y 4
'
1
= 1
1
!
1
=
1
1+

S: x y 4 = = .
Bi 521. Bi 521. Bi 521. Bi 521. i hc Dn lp Vn Hin nm 1995 1996
Gii h phng trnh:
( )
( )
2 2 3 3
3
3
2 x y 3 x y xy
x y 6
'
1
1 =
1
!
1
1 =
1
+

S:
( ) ( )
S 8, 64 , 64, 8 = .
Bi 522. Bi 522. Bi 522. Bi 522. i hc khi B nm 2002
Gii h phng trnh:
3
x y x y
x y x y 2
'
1
=
1
1
!
1
=
1
1+

S:
( )
3 1
S 1;1 , ;
2 2
' ' 1
1 1
1 1
= ! !

( ) 1 1
1 1
+ +
.
www.MATHVN.com
www.DeThiThuDaiHoc.com
cng hc tp mn Ton 10 tp I Ths. L Vn on


"Cn c b thng minh" Page - 99 -
Bi 523. Bi 523. Bi 523. Bi 523. i hc khi A nm 2003
Gii h phng trnh:
3
1 1
x y
x y
2y x 1
'
1
1
=
1
1
!
1
1
=
1
1+

S:
( )
1 5 1 5 1 5 1 5
S 1;1 , ; , ;
2 2 2 2
' ' 1 1
1 1
1 1
1 1


=
! !


( ) ( ) 1 1
1 1
1 1 + +
.
Bi 524. Bi 524. Bi 524. Bi 524. i hc khi B nm 2003
Gii h phng trnh:
2
2
2
2
y 2
3y
x
x 2
3x
y
'
1

1
= 1
1
1
!
1
1
=
1
1
1+

S:
( )
S 1;1 = .
Bi 525. Bi 525. Bi 525. Bi 525. i hc khi D nm 2004
Tm tham s m h phng trnh
x y 1
x x y y 1 3m
'
1
=
1
1
!
1
=
1
1+
c nghim.
S:
1
0 m
4
.
Bi 526. Bi 526. Bi 526. Bi 526. D b 1 i hc khi A nm 2005
Gii h phng trnh:
( ) ( )
2 2
x y x y 4
x x y 1 y y 1 2
'
1
=
1
1
!
1
=
1
1+

S:
( ) ( )
( ) ( )

S 2; 2 , 2; 2 , 1; 2 , 2;1 = .
Bi 527. Bi 527. Bi 527. Bi 527. D b 2 i hc khi A nm 2005
Gii h phng trnh:
2x y 1 x y 1
3x 2y 4
'
1
=
1
1
!
1
=
1
1+

S:
( )
S 2; 1 = .
Bi 528. Bi 528. Bi 528. Bi 528. i hc khi A nm 2006
Gii h phng trnh:
x y xy 3
x 1 y 1 4
'
1
=
1
1
!
1
=
1
1+

S:
( )
S 3; 3 = .
Bi 529. Bi 529. Bi 529. Bi 529. D b 1 i hc khi A nm 2006
www.MATHVN.com
www.DeThiThuDaiHoc.com
Ths. L Vn on Phn i S


Page - 100 - "All the flower of tomorrow are in the seeks of today"
Gii h phng trnh:
( )
( )( )
2
2
x 1 y y x 4y
x 1 y x 2 y
'
1
=
1
1
!
1
=
1
1+

S:
( ) ( )
S 1;2 , 2; 5 = .
Bi 530. Bi 530. Bi 530. Bi 530. D b 2 i hc khi A nm 2006
Gii h phng trnh:
( )
3 3
2 2
x 8x y 2y
x 3 3 y 1
'
1
=
1
1
!
1
=
1
1+

S:
( )
6 6 6 6
S 3; 1 , 4 ; , 4 ;
13 13 13 13
' ' 1 1
1 1
1 1
1 1


=
! !


1 1 ( ) ( )
1 1
1 1 + +
.
S dng:
( ) ( ) ( )( )
3 3 2 2
3 x y 6 4x y x 3y 4x y = = .
Bi 531. Bi 531. Bi 531. Bi 531. D b 2 i hc khi B nm 2006
Gii h phng trnh:
( )( )
( )( )
2 2
2 2
x y x y 13
x y x y 25
'
1
=
1
1
!
1
=
1
1+

S:
( ) ( )
S 3;2 , 2; 3 = .
( )
3 3
x y 19
HPT
xy x y 6
'
1
=
1
1

!
1
=
1
1+

Bi 532. Bi 532. Bi 532. Bi 532. D b 1 i hc khi D nm 2006
Gii h phng trnh:
( )
( )
2 2
3
2 2
x xy y 3 x y
x xy y 7 x y
'
1
=
1
1
!
1
=
1
1+

S:
( ) ( )
S 2;1 , 1; 2 = . t u x y; v xy = = .
Bi 533. Bi 533. Bi 533. Bi 533. i hc khi D nm 2007
Tm gi tr ca tham s m h phng trnh sau c nghim thc
3 3
3 3
1 1
x y 5
x y
1 1
x y 15m 10
x y
'
1
1
=
1
1
1
!
1
1
=
1
1
1+

S:
7
m 2 m 22
4
. t
( )
1 1
v y , u x , u 2, v 2
y x
= = . Dng PP hm s.
Bi 534. Bi 534. Bi 534. Bi 534. D b 2 i hc khi A nm 2007
Gii h phng trnh:
4 3 2 2
3 2
x x y x y 1
x y x xy 1
'
1
=
1
1
!
1
=
1
1+

S:
( ) ( )
S 1;1 , 1; 1 = . t
2 3
u x xy, v x y = = .
www.MATHVN.com
www.DeThiThuDaiHoc.com
cng hc tp mn Ton 10 tp I Ths. L Vn on


"Cn c b thng minh" Page - 101 -
Bi 535. Bi 535. Bi 535. Bi 535. D b 2 i hc khi B nm 2007
Gii h phng trnh:
2
2 3
2
2 3
2xy
x x y
x 2x 9
2xy
y y x
y 2y 9
'
1
1
=
1
1
1
1
!
1
1
=
1
1
1
1+

S:
( ) ( )
S 0; 0 , 1;1 = . Cng 2 PT v theo v, ta c:

( ) ( )
2 2
2 2
3 3
1 1
VT 2xy x y VP
x 1 8 y 1 8
1

= = =




( )

M
2 2
VT 2 xy x y VP = . Du " " = xy ra
x y 0
x y 1

= =

= =

.
Bi 536. Bi 536. Bi 536. Bi 536. D b 2 i hc khi D nm 2007
Tm m h phng trnh sau c nghim duy nht:
2x y m 0
x xy 1
'
1 =
1
1
!
1
=
1
1+

S:
( )

2
x 1
m 2. PT
x 2 m x 1 0
'
1
1
1

!
1 =
1
1+
. Dng tam thc bc hai.
Bi 537. Bi 537. Bi 537. Bi 537. i hc khi A nm 2008
Gii h phng trnh:
( )
2 3 2
4 2
5
x y x y xy xy
4
5
x y xy 1 2x
4
'
1
1
=
1
1
1
!
1
1
=
1
1
1+

S:
3 3
5 25 3
S ; , 1;
4 16 2
' ' 1
1 1
1
1 1
1 1

= ! !


( ) 1 1 ( )
1 1
1 1 + +
. t
2
u x y; v xy = = .
Bi 538. Bi 538. Bi 538. Bi 538. i hc khi B nm 2008
Gii h phng trnh:
4 3 2 2
2
x 2x y x y 2x 9
x 2xy 6x 6
'
1
=
1
1
!
1
=
1
1+

S:
( ) ( )

3
17
S 4; . HPT x x 4 0 x 4 x 0
4
' ' 1
1 1
1 1
= = = ! !

( ) 1 1
1 1
+ +
.
Bi 539. Bi 539. Bi 539. Bi 539. i hc khi D nm 2008
Gii h phng trnh:
2 2
xy x y x 2y
x 2y y x 1 2x 2y
'
1
=
1
1
!
1
=
1
1+

S:
( )
( )( )

x y x 2y 1 0
S 5; 2 . HPT
x 2y y x 1 2x 2y
'
1
=
1
1
=
!
1
=
1
1+
. Lu rng: x y 0 .
www.MATHVN.com
www.DeThiThuDaiHoc.com
Ths. L Vn on Phn i S


Page - 102 - "All the flower of tomorrow are in the seeks of today"
Bi 540. Bi 540. Bi 540. Bi 540. i hc khi B nm 2009
Gii h phng trnh:
2 2 2
xy x 1 7y
x y xy 1 13y
'
1 =
1
1
!
1 =
1
1+

S:
( )
1
S 1; , 3;1
3
' ' 1
1 1
1 1
= ! !

( ) 1 1
1 1
+ +
. t
1 x
u x , v
y y
= = .
Bi 541. Bi 541. Bi 541. Bi 541. i hc khi D nm 2009
Gii h phng trnh:
( )
( )
2
2
x x y 1 3 0
5
x y 1 0
x
'
1
=
1
1
1
!
1
=
1
1
1+

S:
( )
3
S 1;1 , 2;
2
' ' 1
1 1
1 1
= ! !

( ) 1 1
1 1
+ +
.
( )
2
2
3
x y 1 0
x
HPT
5
x y 1 0
x
'
1
1
=
1
1
1

!
1
1
=
1
1
1+
. t
1
u x y; v
x
= = .
Bi 542. Bi 542. Bi 542. Bi 542. i hc khi A nm 2010
Gii h phng trnh:
( ) ( )
2
2 2
4x 1 x y 3 5 2y 0
4x y 2 3 4x 7
'
1
=
1
1
!
1
=
1
1+

S:
1
S ; 2
2
' ' 1
1 1
1 1
= ! !

( ) 1 1
1 1
+ +
.
2
2 2
5
HPT 4x 2x 2 3 4x 7 0
2
1

=

( )
. Dng PP hm s.
Bi 543. Bi 543. Bi 543. Bi 543. i hc khi A nm 2011
Gii h phng trnh:
( ) ( )
2 2 3
2
2 2
5x y 4xy 3y 2(x y) 0
xy x y 2 x y
'
1
=
1
1
!
1
=
1
1+

S:
( ) ( )
2 2 2
S 1;1 , 1; 1 , ,
5 5
' ' 1
1 1
1 1
1 1

=
! !


1 1

( ) 1 1
1 1 + +
.
Bi 544. Bi 544. Bi 544. Bi 544. Tp ch Ton hc v tui tr s 379 thng 1 nm 2009
Gii h phng trnh:
( )( )
2 2
2
x y 1 x y 1 3x 4x 1
xy x 1 x
'
1
=
1
1
!
1
=
1
1+

S:
( )
5
S 1; 1 , 2;
2
' ' 1
1 1
1 1
= ! !

1 1
( )
1 1
+ +
. Chia hai v ca
( )
2 cho x, thay vo
( )
1 .
Bi 545. Bi 545. Bi 545. Bi 545. Tp ch Ton hc v tui tr s 379 thng 1 nm 2009
Gii h phng trnh:
2 2
xy x y x 2y
x 2y y x 1 2x 2y
'
1
=
1
1
!
1
=
1
1+

S:
( )
S 5;2 = . T PT
( ) ( ) ( )( )
2 2
1 x xy 2y x y 0 x y x 2y 1 0 = = .
www.MATHVN.com
www.DeThiThuDaiHoc.com
cng hc tp mn Ton 10 tp I Ths. L Vn on


"Cn c b thng minh" Page - 103 -
Bi 546. Bi 546. Bi 546. Bi 546. Tp ch Ton hc v tui tr s 379 thng 1 nm 2009
Gii h phng trnh:
( )( ) ( )
( )


2
2 2
y 5x 4 4 x 1
y 5x 4xy 16x 8y 16 0 2
'
1
=
1
1
!
1
=
1
1+
.
S:
( ) ( ) ( )

2 2
4
S ; 0 , 0; 4 . PT 2 y 4x 8 y 5x 16x 16 0
5
' ' 1
1 1
1 1
= = ! !

1 1
( )
1 1
+ +
. Xem nh
phng trnh bc hai vi n y.
Bi 547. Bi 547. Bi 547. Bi 547. Tp ch Ton hc v tui tr s 379 thng 1 nm 2009
Gii h phng trnh:
( )
( )( )
2
2
x 1 y y x 4y
x 1 y x 2 y
'
1
=
1
1
!
1
=
1
1+
.
S:
( ) ( )
S 1;2 , 2; 5 = . Chia hai v cho y, t
2
x 1
u , v y x 2
y

= = .
Bi 548. Bi 548. Bi 548. Bi 548. Tp ch Ton hc v Tui tr s 379 thng 1 nm 2009
Gii h phng trnh:
( )
( )
2 2
2
3
4xy 4 x y 7
x y
1
2x 3
x y
'
1
1
=
1
1
1
1
!
1
1
1
=
1
1
1+

S:
( )
S 0;1 = . Bin i v t
( )

1
u x y u 2 , v x y
x y
= =

.

www.MATHVN.com
www.DeThiThuDaiHoc.com
Ths. L Vn on Phn i S


Page - 104 - "All the flower of tomorrow are in the seeks of today"
BI TP N CHNG 3
Bi 549. Bi 549. Bi 549. Bi 549. Gii v bin lun cc phng trnh sau
a/ . b/ .
c/ . d/ .
Bi 550. Bi 550. Bi 550. Bi 550. Tm m cc phng trnh sau c nghim
a/ . b/ .
c/ . d/ .
Bi 551. Bi 551. Bi 551. Bi 551. Gii v bin lun cc phng trnh sau
a/ . b/ .
c/ . d/ .
Bi 552. Bi 552. Bi 552. Bi 552. Tm m phng trnh c mt nghim x
o
. Tnh nghim cn li
a/ . b/ .
Bi 553. Bi 553. Bi 553. Bi 553. Trong cc phng trnh sau, tm m
Phng trnh c hai nghim tri du
Phng trnh c hai nghim m phn bit
Phng trnh c hai nghim dng phn bit
Phng trnh c hai nghim phn bit tho: .
a/ . b/ .
c/ . d/ .
e/ . f/ .
Bi 554. Bi 554. Bi 554. Bi 554. Trong cc phng trnh sau, hy
Gii v bin lun phng trnh
Khi phng trnh c hai nghim , tm h thc gia c lp vi m
a/ . b/ .
c/ . d/ .
Bi 555. Bi 555. Bi 555. Bi 555. Gii cc phng trnh sau
a/ . b/ .
2 2
m x 4m 3 x m =
2 2 2 2
a x 2ab b x a b =
( ) ( ) ( )
2
2 2 2
a b x 2a 2a a b a b x =
( )
2
a ax b 4ax b 5 =
2x m x m 1
1
x 1 x

=

2
m x
m x 2m 1
x 1
=

2mx 1 m 1
2 x 1
x 1 x 1

=

x 1 2x 3 m =
2
2x 12x 15m 0 =
( )
2 2
x 2 m 1 x m 0 =
2
x mx m 1 0 =
( ) ( )
2
x 2 m 2 x m m 3 0 =

2
o
3
x mx m 1 0; x
2
= =
2 2
o
2x 3m x m 0; x 1 = =
1 2
x , x
3 3 2 2
1 2 1 2
x x 0, x x 3 = =
( ) ( )
2
x 2 m 2 x m m 3 0 =
( )
2 2
x 2 m 1 x m 0 =
( )
2 2
x 2 m 1 x m 2 0 =
( ) ( )
2
m 2 x 2 m 1 x m 2 0 =
( ) ( )
2
m 1 x 2 m 4 x m 1 0 =
2
x 4x m 1 0 =
1 2
x , x
1 2
x , x
( )
2
x m 1 x m 0 =
( ) ( )
2
x 2 m 2 x m m 3 0 =
( ) ( )
2
m 2 x 2 m 1 x m 2 0 =
( )
2 2
x 2 m 1 x m 2 0 =
2 2
x x 6 12 =
2 2
x x 11 31 =
www.MATHVN.com
www.DeThiThuDaiHoc.com
cng hc tp mn Ton 10 tp I Ths. L Vn on


"Cn c b thng minh" Page - 105 -
c/ . d/ .
e/ . f/ .
g/ . h/ .
Bi 556. Bi 556. Bi 556. Bi 556. Gii cc phng trnh sau
a/ . b/
c/ . d/ .
e/ . f/ .
g/ . h/ .
Bi 557. Bi 557. Bi 557. Bi 557. Gii cc phng trnh sau
a/ . b/ .
c/ . d/ .
e/ . f/ .
g/ . h/ .
Bi 558. Bi 558. Bi 558. Bi 558. Gii cc h phng trnh sau
a/ b/
c/ d/
e/ f/
Bi 559. Bi 559. Bi 559. Bi 559. Gii cc h phng trnh sau
a/ b/
c/ d/

16x 17 8x 23 =
( )
2
x 2x 8 3 x 4 =
2
3x 9x 1 x 2 0 =
2
51 2x x 1 x =
( )
2 2
x 3 x 4 x 9 = x 3 1 3x 1 =
4 3 10 3x x 2 = x 1 1 x x 8 =
x 5 x 3 2x 4 = 3x 4 2x 1 x 3 =
2 2
x 3x 3 x 3x 6 3 = x 2 2x 3 3x 5 =
3x 3 5 x 2x 4 = 2 x 2 2 x 1 x 1 4 =
x 2 x 1 x 2 x 1 2 =
4 2 2
x x 1 x x 1 2 =
x 3
x 2 x 1 x 2 x 1
2

=
2 2
x x x x 13 7 =
2 2
x 2 x 3x 1 3x 4 =
2 2
2x 3 2x x 1 9 x =
2 2
x x 2x 4 2x 2 =
2 2
2x 5 x 3x 5 23 6x =
2 2
x xy y 1
x y y x 6
'
1 =
1
1
!
1 =
1
1+
2 2
4 2 2 4
x y 5
x x y y 13
'
1
=
1
1
!
1
=
1
1+
2 2
3 3
x y y x 30
x y 35
'
1
=
1
1
!
1
=
1
1+
3 3
5 5 2 2
x y 1
x y x y
'
1
=
1
1
!
1
=
1
1+
2 2
4 4 2 2
x y xy 7
x y x y 21
'
1
=
1
1
!
1
=
1
1+
( )
2 2
x y xy 11
x y 3 x y 28
'
1 =
1
1
!
1 =
1
1+
( )
( )
2 2
2 2
1
x y 1 5
xy
1
x y 1 49
x y
' 1
1
1
= 1

1

( ) 1
1
!
1
1
1
= 1

1

( ) 1
1+
( ) ( )
( )
2 2
2 2
2 2
y x 1 2x y 1
1
x y 1 24
x y
'
1
=
1
1
1
1
!

1
=
1

1
( )
1
+
2 2
2 2
1 1
x y 4
x y
1 1
x y 4
x y
'
1
1
=
1
1
1
!
1
1
=
1
1
1+
( )
2 2
x y 2
3 x 1 y 1
1
x y 1 6
xy
'
1
1
=
1
1

1
!
1
1

1
=
1

1
( )
1
+
www.MATHVN.com
www.DeThiThuDaiHoc.com
Ths. L Vn on Phn i S


Page - 106 - "All the flower of tomorrow are in the seeks of today"
Chng





















































A BT NG THC

C CC C Tnh cht
iu kin Ni dung

Cng hai v vi s bt k

a b a c b c < < ( )
1

Nhn hai v

c 0 (mt s dng)

a b ac bc < < ( )
2a

c 0 < (mt s m)

a b ac bc < ( )
2b

Cng v theo v cc BT cng chiu

a b < v c d < a c b d < ( )
3
Nhn tng v BT khi bit n dng
a 0, c 0

a b < v c d < ac bd <

( )
4

Nng ly tha vi n

Z

M l
2n 1 2n 1
a b a b

< < ( )
5a

M chn
2n 2n
0 a b a b < < < ( )
5b

Ly cn hai v

a 0 a b a b < < ( )
6a

a bt k
3 3
a b a b < < ( )
6b


Nghch o

Nu a, b cng du: ab 0
1 1
a b
a b
<

( )
7a


Nu a, b tri du: ab 0 <
1 1
a b
a b


( )
7b


Cng hai v BT cng chiu
a b
c d
'
1
1
!
1
1
+


a c b d


( )
8a


Nhn hai v BT cng chiu
khi bit chng dng
a b 0
c d 0
'
1
1
!
1
1
+


ac bd


( )
8b



O Lu
Khng v khng c quy tc chia hai v bt ng thc cng chiu.
Ta ch nhn hai v bt ng thc khi bit chng dng.
Cn nm vng cc hng ng thc ng nh v cch bin i.
BT NG THC V BT NG THC V BT NG THC V BT NG THC V BT PHNG TRNH BT PHNG TRNH BT PHNG TRNH BT PHNG TRNH
4
www.MATHVN.com
www.DeThiThuDaiHoc.com
cng hc tp mn Ton 10 tp I Ths. L Vn on


"Cn c b thng minh" Page - 107 -























































C CC C Mt s bt ng thc thng dng
a/ S chnh phng:
2
a 0, a R
2 2
a b 2ab

.
b/ Bt ng thc Cauchy
( )
Arithmetic Means Geometric Means
Vi x, y 0 th
( )
( )


2 2
x y 2 xy 1
x y 2xy 2
'
1

1
1
!
1

1
1+
. Du " " = xy ra khi x y = .
Vi x, y R th
( )
( ) ( )

4
2
2
x y
xy 3
2
x y 4xy
'
1
1
1

1

1

1


( ) !
1
1
1

1
1+
. Du " " = xy ra khi x y = .
Vi x, y, z 0 th
( )
( )


3
3
x y z 3. xyz 5
x y z
xyz 6
3
'
1

1
1
1
1
! 1

1

1

1

( ) 1
1+
. Du " " = xy ra khi x y z = = .
M rng cho n s
1 2 3 n
a , a , a ,..., a khng m ta c:
n
1 2 n 1 2 n
a a ... a n. a .a ...a .
Du " " = xy ra khi
1 2 3 n
a a a ... a = = = = .
H qu
+ Nu x, y 0 c S x y = khng i th P xy = ln nht x y = .
+ Nu x, y 0 c P xy = khng i th S x y = nh nht x y = .
c/ Bt ng thc v gi tr tuyt i
iu kin Ni dung
x R
x 0, x x, x x


x 0
x a a x a
x a
x a
x a


a, b R
a b a b a b

d/ Bt ng thc v cc cnh ca tam gic
Vi a, b, c l di cc cnh ca mt tam gic, ta c
a, b, c 0 . a b c a b < < .
b c a b c < < . c a b c a < < .
e/ Bt ng thc Bunhiacpxki
( )
B.C.S .
Vi x, y bt k, ta lun c:
( ) ( )( ) ( )
( )( ) ( )


2
2 2 2 2
2 2 2 2
a.x b.y a b x y 7
a.x b.y a b x y 8
'
1
1
11
!
1
1
1
1+

www.MATHVN.com
www.DeThiThuDaiHoc.com
Ths. L Vn on Phn i S


Page - 108 - "All the flower of tomorrow are in the seeks of today"












































BI TP P DNG
Bi 605. Bi 605. Bi 605. Bi 605. Cho a, b, c, d, e R . Chng minh cc bt ng thc sau
a/
2 2 2
a b c ab bc ca . b/
2 2
a b 1 ab a b .
c/
( )
2 2 2
a b c 3 2 a b c . d/
( )
2 2 2
a b c 2 ab bc ca .
e/
( )
4 4 2 2
a b c 1 2a ab a c 1 . f/
2
2 2
a
b c ab ac 2bc
4
.
Du " " = xy ra khi
a b x y
hay
x y a b
= = .
Vi x, y, z bt k, ta lun c:
( ) ( )( ) ( )
( )( ) ( )


2
2 2 2 2 2 2
2 2 2 2 2 2
a.x b.y c.z a b c x y z 9
a.x b.y c.z a b c x y z 10
'
1
1
1
1
!
1
1
1
1+

Du " " = xy ra khi =
a b c x y z
hay
x y z a b c
= = = .
f/ Bt ng thc cng mu s
Bt ng thc cng mu s (BT Cauchy Schwarz) l h qu trc tip ca bt ng thc BCS.
Vi a, b R v x, y 0 , ta lun c:
( )
( )

2
2 2
a b
a b
11
x y x y

.
Vi a, b, c R v x, y, z 0 , ta lun c:
( )
( )

2
2 2 2
a b c
a b c
12
x y z x y z



.
Du " " = xy ra khi v ch khi
a b c
x y z
= = .
Dng ton 1. Chng minh BT da vo nh ngha v tnh cht
chng minh mt BT ta c th s dng cc cch sau
Bin i BT cn chng minh tng ng vi mt BT bit.
S dng mt BT bit, bin i dn n BT cn chng minh.
Mt s BT thng dng

2
A 0 .
2 2
A B 0 . A.B 0 vi A, B 0
2 2
A B 2AB .
Lu
Trong qu trnh bin i, ta thng ch n cc hng ng thc.
Khi chng minh BT ta thng tm iu kin du ng thc xy ra.
Khi ta c th tm GTLN, GTNN ca biu thc.
www.MATHVN.com
www.DeThiThuDaiHoc.com
cng hc tp mn Ton 10 tp I Ths. L Vn on


"Cn c b thng minh" Page - 109 -
g/
( ) ( ) ( )
2 2 2 2 2 2
a 1 b b 1 c c 1 a 6abc . h/
( )
2 2 2 2 2
a b c d e a b c d e .
i/
( )
1 1 1 1 1 1
, a, b, c 0
a b c
ab bc ca
. j/
( )
a b c ab bc ca, a, b, c 0 .
Bi 606. Bi 606. Bi 606. Bi 606. Cho a, b, c R . Chng minh cc bt ng thc sau
a/
( )
3
3 3
a b a b
, a, b 0
2 2
1




( )
. b/
4 4 3 3
a b a b ab .
c/
4
a 3 4a . d/
( )
3 3 3
a b c 3abc, a, b, c 0 .
e/
( )
6 6
4 4
2 2
a b
a b , a, b 0
b a
. f/
( )
2 2
1 1 2
, ab 1
1 ab 1 a 1 b


.
g/
2
2
a 3
2
a 2

. h/
( )( ) ( )( ) ( )
5 5 4 4 2 2
a b a b a b a b , ab 0 .
Bi 607. Bi 607. Bi 607. Bi 607. Cho a, b, c, d, e R . Chng minh rng
( )

2 2
a b 2ab 1 . p dng bt ng thc
( )
1
chng minh cc bt ng thc sau
a/
( )( )( )
2 2 2
a 1 b 1 c 1 8abc .
b/
( )( )( )( )
2 2 2 2
a 4 b 4 c 4 d 4 256abcd .
c/
4 4 4 4
a b c d 4abcd .
Bi 608. Bi 608. Bi 608. Bi 608. Cho a, b, c R . Chng minh bt ng thc:
( )

2 2 2
a b c ab bc ca 2 . p dng
bt ng thc
( )
2 chng minh cc bt ng thc sau
a/
( ) ( )
2
2 2 2
a b c 3 a b c . b/
2
2 2 2
a b c a b c
3 3
1



( )
.
c/
( ) ( )
2
a b c 3 ab bc ca . d/
( )
4 4 4
a b c abc a b c .
e/
( )
a b c ab bc ca
, a, b, c 0
3 3

. f/
( )
4 4 4
a b c abc, a b c 1 = .
Bi 609. Bi 609. Bi 609. Bi 609. Cho a, b, c, d 0 . Chng minh rng nu
a
1
b
< th
( )
a a c
b b c

<

. p dng
( )
chng minh
cc bt ng thc sau
a/
a b c
2
a b b c c a
<

.
b/
a b c d
1 2
a b c b c d c d a d a b
< <


c/
a b b c c d d a
2 3
a b c b c d c d a d a b

< <

.
Bi 610. Bi 610. Bi 610. Bi 610. Cho a, b 0 . Chng minh bt ng thc:
( ) ( )

3 3 2 2
a b a b b a ab a b 3 = . p dng
bt ng thc
( )
3 chng minh cc bt ng thc sau
www.MATHVN.com
www.DeThiThuDaiHoc.com
Ths. L Vn on Phn i S


Page - 110 - "All the flower of tomorrow are in the seeks of today"
a/
( )
3 3 3 3 3 3
a b b c c a
2 a b c
ab bc ca

.
b/
3 3 3 3 3 3
1 1 1 1
abc a b abc b c abc c a abc


vi a, b, c 0 .
c/
3 3 3 3 3 3
1 1 1
1
a b 1 b c 1 c a 1


vi a, b, c 0 v abc 1 = .
d/
1 1 1
1
a b 1 b c 1 c a 1


vi a, b, c 0 v abc 1 = .
e/
( ) ( ) ( ) ( )
3 3 3 3 3 3
3 3 3
4 a b 4 b c 4 c a 2 a b c vi a, b, c 0 .
f/
3 3 3
2 2 2 2 2 2
a b c a b c
3 a ab b b bc c c ac a



vi a, b, c 0 .
g/
3 3 3 3 2 2
1 1 1 1
abc a abc b b abc c a abc c


vi a, b, c 0 .
h/
3 3 2 3 2 3
2 2 2
5b a 5c b 5a c
a b c
ab 3b cb 3c ac 3a



vi a, b, c 0 .
Bi 611. Bi 611. Bi 611. Bi 611. Cho a, b, x, y R . Chng minh bt ng thc sau
( )
Min cp xki
( ) ( ) ( )

2 2
2 2 2 2
a x b y a b x y 4
p dng chng minh cc bt ng thc sau:
a/ Cho a, b 0 tho a b 1 = . Chng minh:
2 2
1 a 1 b 5 .
b/ Tm GTNN ca biu thc
2 2
2 2
1 1
P a b
b a
= .
c/ Cho x, y, z 0 tho mn x y z 1 = . Chng minh:

2 2 2
2 2 2
1 1 1
x y z 82
x y z
.
d/ Cho x, y, z 0 tho mn x y z 3 = . Tm GTNN ca biu thc:

2 2 2
P 223 x 223 y 223 z = .
Bi 612. Bi 612. Bi 612. Bi 612. Cho a, b, c l di 3 cnh ca mt tam gic. Chng minh:
a/
( )
2 2 2
ab bc ca a b c 2 ab bc ca < .
b/
( )( )( )
abc a b c b c a a c b .
c/
2 2 2 2 2 2 4 4 4
2a b 2b c 2c a a b c 0 .
d/
( ) ( ) ( )
2 2 2
3 3 3
a b c b c a c a b a b c .
www.MATHVN.com
www.DeThiThuDaiHoc.com
cng hc tp mn Ton 10 tp I Ths. L Vn on


"Cn c b thng minh" Page - 111 -
BI TP RN LUYN
Bi 613. Bi 613. Bi 613. Bi 613. Chng minh cc bt ng thc sau
a/
( )
2
a b 4ab . b/
( ) ( )
2
2 2
2 a b a b .
c/
( ) ( )
2
4 4 2 2
2 a b a b . d/
4 2
a 4 4a .
e/
1
a a
4
. f/
1
a b a b
2
.
g/
3
a b c a b c
4
. h/
( )
2 2 2
a b c 12 4 a b c .
i/
( ) ( )
3
a b c 3 ab bc ca . j/
( ) ( )
2
2 2 2
3 a b c a b c .
k/
( )
2 2 2
2a b c 2a b c . l/
4 4 4 2 2 2 2 2 2
a b c a b b c c a .
m/
( )
4 4 2 2 2 2 2
a b c a b c b a . n/
( )
6 6 2 3 3 3 3
a b c a b c a b .
o/
2 2 2 2 2 2 2 2 2
a b b c c a a bc b ca c ab . q/
( ) ( )
2
ab bc ca 3abc a b c .
Bi 614. Bi 614. Bi 614. Bi 614. Chng minh cc bt ng thc sau
a/
( )
3 3 2 2
a b a b ab ; a, b 0 . b/
( )
5 5 3 2 2 3
a b a b a b ; a, b 0 .
c/
( )
5 5 4 4
a b a b ab ; a, b 0 . d/
6 6 5 5
a b a b ab .
e/
6 6 4 2 2 4
a b a b a b . f/
4
a a 1 0 .
Bi 615. Bi 615. Bi 615. Bi 615. Chng minh cc bt ng thc sau
a/
2
a a 1 0 . b/
2
a a 1 0 .
c/
4
a a 1 0 . d/
2
2
a a 1 1
3 a a 1


.
e/
2
2
a a 1 1
3 a a 1


. f/
2
2
a a 1
3
a a 1


.
g/
2
2
a a 1
3
a a 1


. h/
2 2
a ab b 0 .
i/
2 2
a ab b 0 . j/
2 2
2 2
a ab b 1
3 a ab b


.
k/
2 2
2 2
a ab b 1
3 a ab b


. l/
( )
3
2 2
a 2a b
; a, b 0
3 a ab b



.
Bi 616. Bi 616. Bi 616. Bi 616. Chng minh cc bt ng thc sau
a/
( )
x 2 x 1; x 1 . b/
( )
x 2 2 x 3; x 3 .
www.MATHVN.com
www.DeThiThuDaiHoc.com
Ths. L Vn on Phn i S


Page - 112 - "All the flower of tomorrow are in the seeks of today"
c/
( )
x 5 2 x 6; x 6 . d/
2
2
x 2
2
x 1

.
Bi 617. Bi 617. Bi 617. Bi 617. Cho cc s thc a b c d 0 . Chng minh rng
a/
( )
2
2 2 2
a b c a b c . b/
( )
2
2 2 2 2
a b c d a b c d .
Bi 618. Bi 618. Bi 618. Bi 618. Chng minh cc bt ng thc sau
a/
2
y
2 2
x 4 3z 14 2x 12y 6z .
b/
( )
a b
a b; a, b 0
b a
.
c/
( ) ( ) ( ) ( )
a 1 b b 1 c c 1 a 1; a, b, c 0;1
l
<
l
l
.
e/
( )

2 a b
a ab b; 0 a b
1 1 2
a b

< < < < < <

.
f/
( )

b c a a b c
; a b c 0
a b c b c a
.
g/
( )

ac bd a b c d
. ; a b, c d hay a b, c d : BT Tr bu sep
2 2 2

.
h/
( )

ac bd a b c d
. ; a b, c d hay a b, c d : BT Tr bu sep
2 2 2

.
i/
( )

2 2
2 2
a b a b
; a 0, b 0
b a b a
.
j/
( ) ( )( ) ( )

2
2 2 2 2
ab cd a c b d ; a, b, c, d : BT Bunhiacpxki R .
k/
( ) ( ) ( )
2
3 3
1 1
a b a b ; a, b 0
a b
1



( )
.
l/
( )
3 3
3
a b a b
; a, b 0
2 2

.
m/
( )( ) ( ) ( )

2
ax by ay bx a b xy; a, b, x, y ; a.b 0 R .
n/
( ) ( ) ( ) ( )
4 4 2 2 2 2 2 2 2 2
a b c 1 2a ab a c 1 a 1 b b 1 c c 1 a 6abc .
o/
( ) ( )
2 2 2 2 2
a b c d e a b c d e ; a, b, c, d, e R .
p/
( ) ( ) ( )
2
ab bc ca 3abc a b c ; a, b, c, d R .




www.MATHVN.com
www.DeThiThuDaiHoc.com
cng hc tp mn Ton 10 tp I Ths. L Vn on


"Cn c b thng minh" Page - 113 -







































BI TP P DNG
nh gi t trung bnh cng sang trung bnh nhn v ngc li
Bi 619. Bi 619. Bi 619. Bi 619. Cho a, b, c 0 . Chng minh cc bt ng thc sau
a/
( )
2
a b 4ab . b/
( ) ( )
2
2 2
2 a b a b .
c/
2 2
a b 2ab . d/
1 1 4
a b a b

.
e/
( )
1 1
a b 4
a b
1



( )
. f/
( )( )
a b 1 ab 4ab .
Dng ton 2. Chng minh BT da vo BT Cauchy (AM GM)
Cc dng ca bt ng thc Cauchy (AM GM)
Vi x, y 0 th
( )
( )


2 2
x y 2 xy 1
x y 2xy 2
'
1

1
1
!
1

1
1+
. Du " " = xy ra khi x y = .
Vi x, y R th
( )
( ) ( )

4
2
2
x y
xy 3
2
x y 4xy
'
1
1
1

1

1

1

( ) !
1
1
1

1
1+
. Du " " = xy ra khi x y = .
Vi x, y, z 0 th
( )
( )


3
3
x y z 3. xyz 5
x y z
xyz 6
3
'
1

1
1
1
1
! 1

1

1

1

( ) 1
1+
. Du " " = xy ra khi x y z = = .
M rng cho n s
1 2 3 n
a , a , a ,..., a khng m ta c:
n
1 2 n 1 2 n
a a ... a n. a .a ...a .
Du " " = xy ra khi
1 2 3 n
a a a ... a = = = = .
H qu
Nu x, y 0 c S x y = khng i th P xy = ln nht x y = .
Nu x, y 0 c P xy = khng i th S x y = nh nht x y = .
Khi nim gi tr ln nht v gi tr nh nht ca hm s (biu thc)
Xt hm s
( )
y f x = vi tp xc nh D
M l gi tr ln nht ca hm s
( )
( )
( )
o o
f x M, x D
y f x
x D, f x M
'
1

1
1
=
!
1
=
1
1+

m l gi tr nh nht ca hm s
( )
( )
( )
o o
f x m, x D
y f x
x D, f x m
'
1

1
1
=
!
1
=
1
1+

www.MATHVN.com
www.DeThiThuDaiHoc.com
Ths. L Vn on Phn i S


Page - 114 - "All the flower of tomorrow are in the seeks of today"
g/
( )
1 1 1
a b c 9
a b c
1



( )
. h/
a b c
1 1 1 8
b c a
1 1 1





( )( )( )
.
i/
( )( )( )
a b b c c a 8abc . j/
( )( )( )
2 2 2 2 2 2 2 2 2
a b b c c a 8a b c .
k/
( )( )
2 2 2
a b c a b c 9abc . l/
( )( )
1 a b a b ab 9ab .
m/
( )
( )
8
2
a b 64ab a b . n/
3 3 2
3a 7b 9ab (Cao ng SP Qung Bnh).
o/
1 1 1 1 16
a b c d a b c d


. p/
( )
( )
2
a b 2 2 a b ab .
r/
( )
x 4
2; x 3
x 3

. s/
a, b, c 0
b c 16abc;
a b c 1
1 '
1

1


1 =

1 ( )
+
.
t/
( )( )( )( )
abc 2 bc 2 a d d 1 32abcd . u/
4
2
a bc
ab
2c

.
Bi 620. Bi 620. Bi 620. Bi 620. Cho a, b, c 0 . Chng minh cc bt ng thc sau
a/ a b c ab bc ca .
b/
( )
ab bc ca abc a b c .
a/
ab bc ca
a b c
c a b
.
b/
a b c 1 1 1
bc ca ab a b c
.
c/
b a
ab a b 1
a b
.
d/
2 2 2
a b c
a b c
b c a
.
e/
x x x
x x x
12 15 20
3 4 5
5 4 3
1 1 1





( ) ( ) ( )
, x R (i hc khi B 2005).
f/
2 2 2 2 2 2
1 1 1 9
x y z x y z


vi x, y, z 0 (Cao ng S phm Nh tr TW1 2000).
g/ a b 1 b a 1 ab vi a 1, b 1 (i hc Thi Nguyn D 2001).
h/
2 2 2
2 2 2 1 1 1
bc ca ab a bc b ca c ab


(Cao ng Kinh t Tp. HCM 2007).
i/
3 3 3
a b c
ab bc ca
b c a
(Cao ng C kh luyn kim 2006).

www.MATHVN.com
www.DeThiThuDaiHoc.com
cng hc tp mn Ton 10 tp I Ths. L Vn on


"Cn c b thng minh" Page - 115 -
Tch cp nghch o p dng c Bt ng thc Cauchy
K thut tch nghch o l k thut tch phn nguyn theo mu s chuyn sang trung bnh nhn
th cc phn cha bin s phi trit tiu ch cn li l hng s (hoc bin gn ging bin bn v
phi). thc hin cng vic , ta thng thm bt hng s hoc thm bt bin s.
Trong k thut ny, i khi ta cn kt hp vi k thut nh gi t trung bnh nhn sang trung bnh
cng (Cauchy ngc):
( )
1 2 n
n
1 2 n 1 2 n
a a ... a
a a ...a ; a , a ,..., a 0
n

. Khi kt hp k thut
ny, ta cn lu : "Ch s cn l bao nhiu th s cc s hng trong cn l by nhiu. Nu s cc
s hng nh hn ch s cn th phi nhn thm hay cng vo (hng s) s cc s hng bng s
cn". Chng hn nh:
ab
CM : a b 1 , a, b 1
2
. Ta bin i:
( )
( )
b 1 1
ab
a b 1 a b 1 .1 a.
2 2

= = .
Bi 621. Bi 621. Bi 621. Bi 621. Chng minh cc bt ng thc sau
a/
( )
a b
2, : a, b 0
b a
. b/
( )
x 18
6, : x 0
2 x
.
c/
( )
2
25 27
x 2y 19, x, y 0
x y
. d/
( )
( )
1
x 3, x y 0
x y y

.
e/
( )
x 16
3; x 2
2 x 2

. f/
( )
1 10
a ; a 3
a 3
.
g/
( )
2
1 9
a ; a 2
4 a
. h/
( )
2
2
a 2
2; a
a 1

R .
i/
2 2
a b
a b
2 2;
ab 1
a b
1 '
1

1


1 =

1 ( )
+
. j/
( )
( )

1
a 3; a b 0
b a b

.
k/
( )
x 8
6, x 1
x 1

. l/
( )
( )

2
4
a 2 2; a b 0
b a b

.
m/
( )

3
2a 1 1 a
3; a ; 1
2 b 4b a b
1


( )
. n/
( )( )
( )
2
4
x 3; x y 0
x y y 1


.
Bi 622. Bi 622. Bi 622. Bi 622. Chng minh cc bt ng thc sau
a/
3 3 3
x y z
x y z
yz xz xy
.
b/
( )
1 17
a , : a 4
a 4
.
c/
4 2
2
1
2a 3a 1
1 a

.
d/
( )
a b 1 b a 1 ab, : a 1, b 1 .
www.MATHVN.com
www.DeThiThuDaiHoc.com
Ths. L Vn on Phn i S


Page - 116 - "All the flower of tomorrow are in the seeks of today"
e/
( )( )
( )
2
4
a 3; a b 0
a b b 1


(V ch Nam T nm 1979).
f/
( )( )
( )
1
a 4, a, b, c 0
c a b b c


.
g/
( )( )
( )
3
27 5
a , : a 1
2
2 a 1 a 1


.
g/
( )( )( )
( )
3
1
2a 4, : a b c 0
a b b c a 1


.
i/
( )
2 2 2
x y z x y z
, : x, y, z 0
x y y z z x 2



.
j/
( ) ( ) ( )
2
3 3 3
1 1 1
a b c a b c , a, b, c 0
a b c
1


( )
.
k/
4 1 5
5, x, y 0; x y
x 4y 4
1

=


( )
(Cao ng SP TP.HCM nm 2006).
l/
( )
a b c a b c a b c
9, a, b, c 0
a b c

(Cao ng Kinh t KTCN 1 06).
m/
( )
xy z 4 yz x 2 zx y 3 1 1 1 1
, x 2; y 3; z 4
xyz 2 2
2 3
1




( )
(C 05).
n/
2 2 2
x, y, z 0
x y z 3
,
xyz 1
1 y 1 z 1 x 2
'
1
1

!
1 =

1
+
(D b 2 i hc D 2005).
o/
( ) ( )
2
y 9
1 x 1 1 256, x, y 0
x
y
1
1





( )
( )
(D b 2 i hc A 2005).
p/
x y z
x, y, z
3 4 3 4 3 4 6,
x y z 0
'
1
1

!
1 =
1
+
R
(D b 1 i hc A 2005).
q/
( )( ) ( )( ) ( )( )

3 3 3
x, y, z 0
x y z 3
, :
xyz 1
4 1 y 1 z 1 x 1 z 1 x 1 y
'
1
1

!
1 =

1
+
.
r/
( ) ( ) ( )
( )
3 3 3
a b c a b c
, : a, b, c 0
2 b c a c a b a b c



.
s/
3 3 3
3
x y z
P x 3y y 3z z 3x 3,
4
x, y, z 0
'
1
1
=
1
1
=
!
1
1
1
1+
(D b i hc B 2005).


www.MATHVN.com
www.DeThiThuDaiHoc.com
cng hc tp mn Ton 10 tp I Ths. L Vn on


"Cn c b thng minh" Page - 117 -
S dng BT b suy lun t BT Cauchy (AM GM)

( ) ( )

1 1 1 1 4
x y 4 hay I
x y x y x y
1



( )
. Du " " = xy ra khi v ch khi x y = .

( ) ( )
hay
1 1 1 1 1 1 9
x y z 9 II
x y z x y z x y z
1



( )
. Du " " = xy ra x y z = = .
Rt nhiu bi ton chng minh bt ng thc hoc tm GTLN v GTNN ca hm s quy v hai bt
ng thc c bn ni trn. V th, c th xem cch s dng hai bt ng thc ny l mt trong
nhng cch s dng bt ng thc AM GM (Cauchy) trong cc bi ton c th. Khi s dng, ta
phi chng minh li, vic ny xem nh chng minh BT b cho bi ton.

Bi 623. Bi 623. Bi 623. Bi 623. Cho a, b 0 . Chng minh
( )

1 1 4
I
a b a b

. p dng bt ng thc
( )
I chng minh
cc bt ng thc sau
a/
( )
1 1 1 1 1 1
2 , : a, b, c 0
a b c a b b c c a
1




( )

.
b/
( )
1 1 1 1 1 1
2 , : a, b, c 0
a b b c c a 2a b c a 2b c a b 2c
1




( )

.
c/
1 1 1
1 1 1 4
1,
a b c
2a b c a 2b c a b 2c
a, b, c 0
'
1
1
=
1
1

!
1

1
1
1+
(i hc A 2005).
d/
( )
ab bc ca a b c
, : a, b, c 0
a b b c c a 2



.
e/
x 2y 4z 12
2xy 8yz 4xz
6,
x, y, z 0
x 2y 2y 4z 4z x
'
1 =
1

!
1

1
+
.
g/ Chng minh rng trong mi ABC , ta lun c:
1 1 1 1 1 1
2
p a p b p c a b c
1



( )
.
Trong : a BC, b AC, c AB = = = l di ba cnh v
a b c
p
2

= l na chu vi.
(i hc Ngn Hng Tp. HCM khi A nm 2001)
h/
x t t y y z z x
P 0, : x, y, z, t 0
t y y z z x x t

=

.
Bi 624. Bi 624. Bi 624. Bi 624. Cho a, b, c 0 . Chng minh
( )

1 1 1 9
II
a b c a b c


. p dng bt ng thc
( )
II
chng minh cc bt ng thc sau
a/
2 2 2 9
, : a, b, c 0
x y y z z x x y z


.
www.MATHVN.com
www.DeThiThuDaiHoc.com
Ths. L Vn on Phn i S


Page - 118 - "All the flower of tomorrow are in the seeks of today"
b/
( ) ( )

2 2 2
1 1 1 3
a b c a b c , : a, b, c 0
a b b c c a 2
1




( )

.
c/
x y z 0
x y z 3
, :
x y z 1
x 1 y 1 z 1 4
'
1
1

!
1 =

1
+
.
d/
2 2 2
1 1 1
9, : a, b, c 0
a 2bc b 2ac c 2ab


.
e/
2 2 2
1 1 1 1
30, : a, b, c 0
ab bc ca a b c


.
f/
y z z x x y
6, : x, y, z 6
x y z

.
g/
a b c 3
P , : a, b, c 0
b c c a a b 2
=

.
h/
2 2
x 0, y 0
x y 1 5
P x y , :
x y 1
1 x 1 y x y 2
'
1
1
=
!
1 <

1
+
.
i/
2 2 2
a b c a b c
, : a, b, c 0
b c c a a b 2



.
j/
3
1 1 1 15 x y z
P x y z , :
2
x y z 2
x, y, z 0
'
1
1

1
1
=
!
1
1
1
1+
.


K thut i bin (t n ph) p dng c BT Cauchy
Mc ch chnh ca vic i bin l chuyn bi ton t tnh th kh bin i i s (bin c) sang trng
thi d bin i i s hn (bin mi). Thng thng, vi bi ton bin mi l nhng bi ton quen
thuc. Do , cn phi nm vng cc k thut bin i cng nh vic s dng thnh tho cc BT
thng dng v cn nh rng, nu bi ton c iu kin rng buc th khi i bin cn ch iu kin
bin mi sao cho khi t n th iu kin ban u v cui cng c m bo, chng hn nh: Cho
a, b, c 0 v abc 1 = . Tm GTLN GTNN ca biu thc P ........... = T iu kin, ta c th t

x y z
a , b , c
y z x
= = = nhm m bo iu kin ban u:
x y z
abc . . 1
y z x
= = .
Chng minh cc bt ng thc sau
Bi 625. Bi 625. Bi 625. Bi 625.
( )

b c x 0
a b c 3
, : a, b, c 0 BT : Nesbitt HD : c a y 0
b c c a a b 2
a b z 0
'
1
=
1
1
1
=
!
1

1
= 1
1
+
.
Bi 626. Bi 626. Bi 626. Bi 626.
( )

a x 2011 0
x 2011 x 2012 1 1
, : x 2012 HD:
x 2 x
b x 2012 0
2 2013 2 2012
'
1
=
1
1

!
1

=
1
1+
.
www.MATHVN.com
www.DeThiThuDaiHoc.com
cng hc tp mn Ton 10 tp I Ths. L Vn on


"Cn c b thng minh" Page - 119 -
Bi 627. Bi 627. Bi 627. Bi 627.
( )

a 2x y z 0
x y z 3
, : x, y, z 0 HD : b 2y z x 0
2x y z 2y z x 2z x y 4
c 2z x y 0
'
1
=
1
1
1
=
!
1

1
= 1
1
+
.
Bi 628. Bi 628. Bi 628. Bi 628.
2 2 2
a b c
a b c, : a, b, c
b c a c a b a b c


l di ca ba cnh ABC.
b c a x 0
HD : c a b y 0
a b c z 0
'
1
=
1
1
1
=
!
1
1
= 1
1
+
.
Bi 629. Bi 629. Bi 629. Bi 629. Chng minh rng trong mi ABC, ta lun c:
( )( )( )
abc b a c a c b b c a vi
a,b,c l ba cnh ca tam gic ABC.
x b a c
HD : y a c b
z b c a
'
1
=
1
1
1
=
!
1
1
= 1
1
+
.
Bi 630. Bi 630. Bi 630. Bi 630.
( )

a, b, c 0
1 1 1
a 1 b 1 c 1 1, : IMO 2000
abc 1
b c a
'
1 1 1 1
1
!


1 =
( )( )( )
1
+
.

x y z
HD : a , b , c
y z x
= = = .
Bi 631. Bi 631. Bi 631. Bi 631.
2 2 2 2 2 2
1
x
a
a, b, c 0
b 2a c 2b a 2c 1
3, : HD : y
ab bc ca abc
ab cb ac b
1
z
c
'
1
1
=
1
1
1
' 1
1
1
1 1
=
! !
1 1 =
1 1
+
1
1
1 =
1
1
1+
.
Bi 632. Bi 632. Bi 632. Bi 632.
( ) ( ) ( )

3 3 3
1
x
a
x, y, z 0
1 1 1 3 1
, : HD : y
xyz 1
2 b x y z y x z z y x
1
z
c
'
1
1
=
1
1
1
' 1
1
1
1 1
=
! !
1 1 =

1 1
+
1
1
1 =
1
1
1+
. (IMO 1995).
Bi 633. Bi 633. Bi 633. Bi 633.
1
x
a
x, y, z 0
1
x yz y xz z yx xyz x y z, : HD: y
1 1 1
1 b
x y z
1
z
c
'
1
1
=
1
1
'
1 1
1 1
1 1
1 1
=
! !
1 1
=
1 1
1 1
1+
1
1 =
1
1
1+
.
Bi 634. Bi 634. Bi 634. Bi 634.
x, y, z 0
3 1 1 1
x y z xyz, : HD : a, b, c
xyz x y z 2
2 1 x 1 y 1 z
'
1
1
= = =
!
1 =

1
+
.
Bi 635. Bi 635. Bi 635. Bi 635.
x, y, z 0
1 4 9 a b c
36, : HD : x , y , z
x y z 1
x y z a b c a b c a b c
'
1
1
= = =
!
1 =

1
+
.

www.MATHVN.com
www.DeThiThuDaiHoc.com
Ths. L Vn on Phn i S


Page - 120 - "All the flower of tomorrow are in the seeks of today"
S dng cng thc din tch tam gic p dng BT Cauchy
Trong nhiu bi ton BT tam gic th din tch tam gic l chiu cu ni cc mi quan h gia cc yu
t trong tam gic. Do , ta cn nm vng cc cng thc tnh din tch tam gic, ng thi kt hp
thnh tho vi 4 k thut p dng BT Cauchy trnh by trn.
( )( )( )
a b c
a.h b.h c.h
absinC bc sinA ca sinB abc
S p p a p b p c pr
2 2 2 2 2 2 4R
= = = = = = = = = .
Trong
+ a,b,c ln lt l di ca ba cnh BC, AC, AB trong ABC .
+
a b c
h , h , h l cc chiu cao xut pht ln lt t cc nh A, B, C.
+ R, r ln lt l bn knh ng trn ngoi tip v ni tip tam gic.
+
a b c
p
2

= l na chu vi ABC .
Bi 636. Bi 636. Bi 636. Bi 636. Trong ABC cho bn knh ng trn ngoi tip v ni tip ln lt lR, r . Chng minh:
R 2r .
Bi 637. Bi 637. Bi 637. Bi 637. Cho ABC c din tch bng
3
2
. Gi a, b, c ln lt l di cc cnh BC, CA, AB v
a b c
h , h , h l cc chiu cao xut pht ln lt t cc nh A, B, C. Chng minh rng:
a b c
1 1 1 1 1 1
3
a b c h h h
1 1



( )
( )
. (D b 6 i hc 2002).
Bi 638. Bi 638. Bi 638. Bi 638. Cho ABC c di ba cnh BC, CA, AB ln lt l a, b, c v S l din tch.
Chng minh rng:
2 2 2
a b c 4 3S .
Bi 639. Bi 639. Bi 639. Bi 639. Cho ABC c di cc cnh ln lt l a, b, c vS l din tch. Cc trung tuyn v ng
cao ln lt xut pht t cc nh A, B, C l
a b c
m , m , m v
a b c
h , h , h . Chng minh rng:
2 2 2
a b c
m m m 3 3S v
( )( )
2 2 2 2 2 2 2
a b c a b c
m m m h h h 27S .
Bi 640. Bi 640. Bi 640. Bi 640. Cho ABC c di ba cnh BC, CA, AB ln lt l a, b, c v S l din tch.
Chng minh rng:
4
1 1 1 3. 3
a b c b c a c a b
2 S


.
Bi 641. Bi 641. Bi 641. Bi 641. Cho ABC . Chng minh rng:
( ) ( ) ( )
2 2 2 2
1 1 1 1
r
p a p b p c


. Trong : a, b, c ln
lt l di 3 cnh BC, CA, AB; p l na chu vi v r l bn knh ng trn ni tip
ABC .
Bi 642. Bi 642. Bi 642. Bi 642. p dng Bt ng thc Cauchy tm gi tr nh nht ca cc biu thc sau
a/
( )
x 18
y , : x 0
2 x
= . b/
( )
x 2
y , : x 1
2 x 1
=

.
www.MATHVN.com
www.DeThiThuDaiHoc.com
cng hc tp mn Ton 10 tp I Ths. L Vn on


"Cn c b thng minh" Page - 121 -
c/
( )
3x 1
y , : x 1
2 x 1
=

. d/
x 5 1
y , : x
3 2x 1 2
1

=


( )
.
e/
( )
x 5
y , : 0 x 1
1 x x
= < <

. f/
( )
3
2
x 1
y , : x 0
x

= .
Bi 643. Bi 643. Bi 643. Bi 643. p dng Bt ng Cauchy tm gi tr ln nht ca cc biu thc sau
a/
( )( ) ( )
y x 3 5 x , : 3 x 5 = . b/
( ) ( )
y x 6 x , : 0 x 6 = .
c/
( )( )

5
y x 3 5 2x , : 3 x
2
1

=


( )
. d/
( )( )

5
y 2x 5 5 x , : x 5
2
1

=


( )
.
e/
( )( )

1 5
y 6x 3 5 2x , : x
2 2
1

=


( )
. f/
( )
2 2
y x 9 x , : 3 x 3 = .
g/
( )
2
x
y , : x 0
x 2
=

. h/
( )
2
3
2
x
y
x 2
=

.
Bi 644. Bi 644. Bi 644. Bi 644. Cho hai s thc dng khng m x, y tha mn x y 1 = . Tm gi tr ln nht v gi tr nh
nht ca biu thc:
( )( )
2 2
S 4x 3y 4y 3x 25xy = .
(Trch thi tuyn sinh i hc khi D nm 2009)
Bi 645. Bi 645. Bi 645. Bi 645. Cho hai s thc x, y tha mn xy 0 v
( )
2 2
xy x y x xy y = . Tm gi tr ln nht ca
biu thc:
3 3
1 1
A
x y
= .
(Trch thi tuyn sinh i hc khi A nm 2006)
Bi 646. Bi 646. Bi 646. Bi 646. Cho
a, b, c, d 0
1 1 1 1
3
1 a 1 b 1 c 1 d
'
1
1
1
1
!
1

1
1

1+
. Tm gi tr ln nht ca P abcd = .
Bi 647. Bi 647. Bi 647. Bi 647. Cho ba s dng a, b, c tha mn iu kin
2 2 2
a b c 1 = . Hy tm gi tr nh nht ca
biu thc:
ab bc ca
P
c a b
= .
(Trch thi tuyn sinh i hc Si Gn khi A,B 2007)
Bi 648. Bi 648. Bi 648. Bi 648. Cho x,y,z l ba s thc dng thay i. Tm gi tr nh nht ca biu thc
x 1 y 1 z 1
P x y z
2 yz 2 zx 2 xy
1 1 1

=



( ) ( ) ( )
.
(Trch thi tuyn sinh i hc khi B 2007)





www.MATHVN.com
www.DeThiThuDaiHoc.com
Ths. L Vn on Phn i S


Page - 122 - "All the flower of tomorrow are in the seeks of today"





Cho a, b, c, d R
Cho a, b, c, m, n, p R
C
( )( ) ( )
2
2 2 2 2
a b c d a.c b.d
Du" " = xy ra khi v ch khi
a b
c d
= .
O
( )( ) ( )
2
2 2 2 2 2 2
a b c m n p a.m b.n c.p
Du" " = xy ra khi v ch khi
a b c
m n p
= = .
C
( )( )
2 2 2 2
a b c d a.c b.d
Du" " = xy ra khi v ch khi
a b
c d
= .
O
( )( )
2 2 2 2 2 2
a b c m n p a.m b.n c.p
Du" " = xy ra khi v ch khi
a b c
m n p
= = .
C
( )( )
2 2 2 2
a b c d a.c b.d
Du" " = xy ra khi v ch khi
a b
0
c d
= .
O
( )( )
2 2 2 2 2 2
a b c m n p a.m b.n c.p
Du" " = xy ra khi v ch khi
a b c
0
m n p
= = .

Bi 649. Bi 649. Bi 649. Bi 649. Chng minh cc bt ng thc sau
a/ Nu 2x 3y 4 = th
2 2
16
2x 3y
5
. b/ Nu 6x y 5 = th
2 2
9x y 5 .
c/ Nu 3x 4y 7 = th
2 2
49
x y
25
. d/ Nu 6x 12y 5 = th
2 2
4x 9y 1 .
e/ Nu 3x 4y 10 = th
2 2
x y 4 . f/ Nu x 7y 10 = th
2 2
x y 2 .
g/ Nu 3a 4b 7 = th
2 2
3a 4b 7 . h/ Nu 2a 3b 7 = th
2 2
735
3a 5b
47
.
i/ Nu 3a 5b 8 = th
2 2
2464
7a 11b
137
. j/ Nu a 2b 2 = th
2 2
4
a b
5
.
k/ Nu 2a 3b 5 = th
2 2
2a 3b 5 . l/
( ) ( )
2 2
9
x 2y 1 2x 4y 5
5
.
Bi 650. Bi 650. Bi 650. Bi 650. Chng minh cc bt ng thc sau
a/ Nu
2 2
x y 1 = th 3x 4y 5 . b/ Nu
2 2
x 2y 8 = th 2x 3y 2 17 .
c/ Nu
2 2
x 4y 1 = th
5
x y
2
. d/ Nu
2 2
36x 16y 9 = th
5
y 2x
4
.
e/ Nu
2 2 2 2
x y u v 1 = = th
( ) ( )
x u v y u v 2 .
f/ Nu
( ) ( )
2 2
4 a 1 9 b 2 5 = th 2a 6b 20 5 .
Dng ton 3. Chng minh BT da vo BT Bunhiacopxki (B.C.S)
www.MATHVN.com
www.DeThiThuDaiHoc.com
cng hc tp mn Ton 10 tp I Ths. L Vn on


"Cn c b thng minh" Page - 123 -
Bi 651. Bi 651. Bi 651. Bi 651. Chng minh cc bt ng thc sau
a/ Nu x 1; 3
l

l
l
th A 6 x 1 8 3 x 10 2 = .
b/ Nu x 1; 5
l

l
l
th B 3 x 1 4 5 x 10 = .
c/ Nu x 2;1
l

l
l
th C 1 x 2 x 6 = .
d/ Nu x 4;13
l

l
l
th D 2 x 4 13 x 3 5 = .
e/ Nu x 5; 20
l

l
l
th E 3 x 5 2 20 a 13 = .
f/ Nu x 9; 20
l

l
l
th F 5 x 9 2 20 x 29 = .
Bi 652. Bi 652. Bi 652. Bi 652. Chng minh cc bt ng thc sau
a/ Nu x, y, z 0 v x y z 1 = th 1 x 1 y 1 z 6 .
b/ Nu a, b, c R th
( ) ( )
2
2 2 2
3 a b c a b c .
c/ Nu
2 2 2
a b c 1 = th a 3b 5c 35 .
d/ Nu
2 2 2
a b c 1 = th a 2b 2 5c 5 .
e/ Nu a c 0 v b c 0 th
( ) ( )
c a c c b c ab .
f/ Nu 4a 9b 16c 49 = th
1 25 64
49
a b c
.
g/ Nu a b c 1 = th a 1 b 1 c 1 2 3 .
h/ Nu a b c 12 = th a 3 b 2 c 1 3 6 .
i/ Nu a b c 4 = th a b b c c a 2 6 .
j/ Nu a, b, c l ba s thc thay i tha a b c 6 = th
2 2 2
a b c 12 .
Bi 653. Bi 653. Bi 653. Bi 653. Chng minh cc bt ng thc sau
a/ Nu a b 1 th
2 2
1
a b
2
. b/ Nu a b 1 th
3 3
1
a b
4
.
c/ Nu a b 1 th
4 4
1
a b
8
. d/ Nu a b 2 = th
4 4
a b 2 .
e/ Nu a, b c 0 th
( ) ( )
2
3 3
1 1
a b a b
a b
1



( )
.
f/ Nu 1 x 1 y 2 1 z = th x y 2z .
g/ Nu
( ) ( ) ( )
4
a a 1 b b 1 c c 1
3
th a b c 4 .
www.MATHVN.com
www.DeThiThuDaiHoc.com
Ths. L Vn on Phn i S


Page - 124 - "All the flower of tomorrow are in the seeks of today"
h/ Nu
x, y, z 0
x y z 1
'
1
1
!
1
1
+
th
2 2 2
2 2 2
1 1 1
x y z 82
x y z
(i hc A 2003).
i/ Nu a, b, c 0 th
3 3 3 2 2 2
a b c a bc b ca c ab .
j/ Nu x, y, z 0 th
x y z
1
y 2z z 2x x 2y


.
Bi 654. Bi 654. Bi 654. Bi 654. Tm gi tr ln nht v gi tr nh nht ca cc biu thc sau
a/ A 7 x 2 x = , vi 2 x 7 . b/ B 6 x 1 8 3 x = vi x 1; 3
l

l
l
.
c/ C y 2x 5 = vi
2 2
36x 16y 9 = . d/ D 2x y 2 = vi
2 2
x y
1
4 9
= .
e/ E x 1 3 x = . f/ F 3 x x 5 = .
g/ G 2 x 4 8 x = . h/ H 5 x 1 3 6 x = .
i/ I 4 x 3 5 4 x = . j/ J 1 2x x 8 = .
Bi 655. Bi 655. Bi 655. Bi 655. Tm GTLN v GTNN ca biu thc (nu c)
a/ Cho x, y R v
2 2
x y 5 = . Tm GTLN v GTNN ca biu thc A 2x y = .
b/ Cho x, y R v
2 2
2x 3y 6 = . Tm GTLN v GTNN ca biu thc B 4x 2y = .
c/ Cho x, y R v
2 2
x 4y 10 = . Tm GTLN v GTNN ca biu thc C 3x 5y = .
d/ Cho x, y, z R v xy yz zx 1 = . Tm GTNN ca biu thc
4 4 4
D x y z = .
e/ Cho x, y R v
2 2
x y 1 = . Tm GTLN ca biu thc E x 1 y y 1 x = .
f/ Cho a 1 . Tm GTLN ca biu thc F a sin x a sin x = .
g/ Cho x, y 0 v x y 1 = . Tm GTNN ca biu thc
4 1
G
x 4y
= .
h/ Cho x, y, z 0 v x y z 1 = . Tm GTLN ca H 1 x 1 y 1 z = .
i/ Cho x 2; 2
l

l
l
. Tm GTLN v GTNN ca biu thc
2
I x 4 x = (i hc B 2003).










www.MATHVN.com
www.DeThiThuDaiHoc.com
cng hc tp mn Ton 10 tp I Ths. L Vn on


"Cn c b thng minh" Page - 125 -





















Bi 656. Bi 656. Bi 656. Bi 656. Cho ba s thc dng a, b, c 0 . Chng minh:
( )
2
2 2 2
a b c
a b c
b c c a a b 2



.
Bi 657. Bi 657. Bi 657. Bi 657. Cho a, b, c l di ca ba cnh ABC.
Chng minh rng:
2 2 2
a b c
a b c
b c a c a b a b c


.
Bi 658. Bi 658. Bi 658. Bi 658. Cho ba s a, b, c 0 . Chng minh rng:
a b c 3
b c c a a b 2


.
Bi 659. Bi 659. Bi 659. Bi 659. Cho ba s thc a, b, c bt k. Chng minh:
3 3 3 2 2 2
a b c a b c
b c c a a b 2



.
Bi 660. Bi 660. Bi 660. Bi 660. Cho ba s a, b, c 0 . Chng minh:
( ) ( ) ( )
( )
2 2 2
a b c 9
4 a b c
b c c a a b



.
Bi 661. Bi 661. Bi 661. Bi 661. Cho a, b, c 0 tha iu kin a b c 3 = .
Chng minh rng:
2 2 2
2 2 2
a b c
1
a 2b b 2c c 2a


.
Bi 662. Bi 662. Bi 662. Bi 662.
( )
IMO Shortlist 1993 . Cho bn s thc dng a, b, c, d.
Chng minh rng:
a b c d 2
b 2c 3d c 2d 3a d 2a 3b a 2b 3c 3


.
Dng ton 4. Chng minh BT da vo BT Cauchy Schwarz

Thc cht bt ng thc Cauchy Schwarz l h qu trc tip ca bt ng thc Bunhiacpxki m
y d hnh dung, ti gi tt l bt ng thc cng mu s.
Cho a, b R v x, y 0 . p dng BT Bunhiacpxki cho:
( )
a b
, ; x, y
x y
1


( )
, ta c:
( )
( )
( )


2 2
Bunhiacpxki
2 2 2 2
a b
a b a b a b
x y . x . y I
x y x y x y
x y
1
1







( )
( )


Cho a, b, c R v x, y, z 0 . p dng BT Bunhiacpxki cho b 3 s:
( )
a b c
, , ; x, y, z
x y z
1


( )
, ta c:
( )

2
Bunhiacpxki
2 2 2
a b c a b b
x y z . x . y . z
x y z
x y z
1
1







( )
( )


( )
( )

2
2 2 2
a b c
a b c
II
x y z x y z




www.MATHVN.com
www.DeThiThuDaiHoc.com
Ths. L Vn on Phn i S


Page - 126 - "All the flower of tomorrow are in the seeks of today"





















Bi 663. Bi 663. Bi 663. Bi 663. Cho a, b, c R . Chng minh:
( ) ( )
2 2
2 2 2 2
a c b a c b 2 a b .
HD:
( ) ( )
u a c; b , v a c; b = =
, ,
.
Bi 664. Bi 664. Bi 664. Bi 664. Cho a, b, c R . Chng minh:
2 2 2 2
a 4b 6a 9 a 4b 2a 12b 10 5 .
HD:
( ) ( )
u a 3;2b ; v 1 a; 3 2b = =
, ,
.
Bi 665. Bi 665. Bi 665. Bi 665. Cho a, b, c R . Chng minh:
2 2 2 2 2 2
a ab b a ac c b bc c .
HD: Cch 1.
b 3 c 3
u a ; b , v a ; c
2 2 2 2
1 1
1



= =



( )
( ) ( )
, ,
.
Cch 2.
b 3 3 3 b c
A a ; c , B 0; b c , C ; 0
2 2 2 2 2 2
1 1
1







( )
( ) ( )

(D b Cao ng Giao Thng II 2003)
Bi 666. Bi 666. Bi 666. Bi 666. Cho a, b, c R .
Chng minh:
( ) ( )
2 2 2 2 2 2
4 cos a cos b sin a b 4 sin a sin b sin a b 2 .
HD:
( ) ( ) ( ) ( )
u 2cos a cos b; sin a b , v 2sina sin b; sin a b = =
, ,
.
Bi 667. Bi 667. Bi 667. Bi 667. Cho ba s dng x, y, z tha mn iu kin:
2 2
2 2
x xy y 3
y yz z 16
'
1
=
1
1
!
1 =
1
1+
.
Chng minh: xy yz zx 8 .
Dng ton 5. Chng minh BT da vo phng php ta vct

Trong mt s bi ton, ta c th a v ta tm GTLN v GTNN. Do , ta cn nm vng mt
s kin thc c bn v ta trong mt phng Oxy.
C
( )
2 2
a x, y a x y = =
, ,
.
C
( ) ( ) ( ) ( ) ( )
2 2
A A B B C C B A B A
A x , y , B x , y , C x , y AB x x y y = v AB AC BC .
C u v u v u v
, , , , , ,
Du " " =

xy ra khi v ch khi u, v
, ,
cng hng.

C u v w u v w
, , , , , ,
. Du " " =

xy ra khi v ch khi u, v, w
, , ,
cng hng.
C u.v u . v
, , , ,
.
www.MATHVN.com
www.DeThiThuDaiHoc.com
cng hc tp mn Ton 10 tp I Ths. L Vn on


"Cn c b thng minh" Page - 127 -
HD:
x 3x 3x z
u y ; , v ; y
2 2 2 2
1 1



= =





( ) ( )
, ,
.
Bi 668. Bi 668. Bi 668. Bi 668. Tm GTNN ca
2 2
P x x 1 x x 1 = , x R.
HD: Cch 1.
1 3 1 3
u x; , v x ;
2 2 2 2
1 1









( ) ( )
, ,
. Cch 2.
( )

1 3 1 3
A ; , B ; , C x, 0
2 2 3 2
1 1









( ) ( )
.
Bi 669. Bi 669. Bi 669. Bi 669. Cho x, y, z v x 3y 5z 3 .
Chng minh:
4 4 2
3xy 625z 4 15yz x 4 5zx 81y 4 45 5xyz .
HD:
2 2 2
u x; , v 3y; , w 5z
x 3y 5z
1 1 1

= = =



( ) ( ) ( )
, , ,
.
Bi 670. Bi 670. Bi 670. Bi 670. Cho x, y R . Chng minh:
2 2 2 2
x 4 x 2x y 1 y 6y 10 5 .
HD:
( ) ( ) ( )
u x;2 , v 1 x; y , w 1; 3 y = = =
, , ,
.
Bi 671. Bi 671. Bi 671. Bi 671. Cho x, y R . Chng minh:
( )( )
( )( )
2 2
x y 1 xy
1 1
2 2
1 x 1 y



.
HD:
( ) ( )
2 2
u 2x;1 x , v 1 y ;2y = =
, ,
.
Bi 672. Bi 672. Bi 672. Bi 672. Nu
x, y, z 0
x y z 1
'
1
1
!
1
1
+
th
2 2 2
2 2 2
1 1 1
x y z 82
x y z
(i hc A 2003).
HD:
1 1 1
u x; 2 , v y; 2 , w 2; 2
x y z
1 1 1

= = =



( ) ( ) ( )
, , ,
.












Bi 673. Bi 673. Bi 673. Bi 673. Gii cc phng trnh sau
a/
2
x 4 6 x x 10x 27 = .
Dng ton 6. ng dng BT gii phng trnh
O Loi 1. Tng hai s khng m:
( ) ( )
( )
( )
2 2 f x 0
f x g x 0
g x 0
'
1
=
1
1 l l
=
!
l l
l l
1
=
1
1+

O Loi 2. Phng php i lp
Gii phng trnh:
( ) ( ) ( )
f x g x =



Nu chng minh c
( )
( )
f x M
g x M
'
1

1
1
!
1

1
1+
th
( )
( )
( )
f x M
g x M
'
1
=
1
1

!
1
=
1
1+

www.MATHVN.com
www.DeThiThuDaiHoc.com
Ths. L Vn on Phn i S


Page - 128 - "All the flower of tomorrow are in the seeks of today"
b/
2
x 2 4 x x 6x 11 = .
c/
2
x 6 x 2 x 6x 13 = .
d/
2
2x 3 5 2x 3x 12x 4 = .
e/
2
6
2x 1 19x 2x
x 10x 24
=

.
g/
2 2 2
x 2x 3 2x x 3x 3x 1 = .
h/
2 2 2
x x 1 x x 1 x x 2 = .
i/
4 4 2 4
1 x 1 x 1 x 3 = .
j/
( )
2
3
25x 2x 9 4x
x
= .
Bi 674. Bi 674. Bi 674. Bi 674. Gii cc phng trnh sau
a/
2 2
x 2x 5 x 1 1 x 2x = .
b/
2 2 2
3x 6x 7 5x 10x 14 4 2x x = .
c/
2 2 2
3x 6x 7 2x 4x 3 2 2x x = .
d/
2 2 2
3x 6x 7 5x 10x 14 24 2x x = .
e/
2 2 2
3x 6x 7 5x 10x 14 2 2x x = .
BI TP RN LUYN BT & GTLN (max) GTNN (min)
Bi 675. Bi 675. Bi 675. Bi 675. Chng minh cc bt ng thc sau
a/
( ) ( )
2 2 2 2 2
a b c d e a b c d e , : a, b, c, d, e R .
b/
( )

a b
a b, : a 0, b 0
b a
.
c/
( ) ( ) ( ) ( )
a 1 b b 1 c c 1 a 1, : 0 a, b, c 1 < < < .
d/
( ) ( ) ( ) ( )
4 4 2 2 2 2 2 2 2 2
a b c 1 2a ab a c 1 a 1 b b 1 c c 1 a 6abc .
Bi 676. Bi 676. Bi 676. Bi 676. Tm GTNN (min) ca ca hm s
a/
( )
x 2
y , : x 1
2 x 1
=

. b/
( )
4
y x , : x 1
x 1
=

.
c/
( )
4
y x 1 , : x 3
x 3
=

. d/
( )
x 5
y , : 0 x 1
1 x x
= < <

.
e/
( )
x 3 16
y , : x 1
4 x 1

. f/
( )( )
( )
4 x x 2
y , : x 0
x

= .
www.MATHVN.com
www.DeThiThuDaiHoc.com
cng hc tp mn Ton 10 tp I Ths. L Vn on


"Cn c b thng minh" Page - 129 -
g/
( )
3
1
y 3x , : x 0
x
= . h/
( )
2
x 4x 4
y , : x 0
x

= .
Bi 677. Bi 677. Bi 677. Bi 677. Tm GTNN (min) ca ca hm s
a/
( )
2
x
y , : x 1
x 1
=

. b/
( )
4
2
x 1
y , : x 0
x

= .
c/
( )
2
2
x x 2
y , : x
x x 1

=

R . d/
( ) ( )
2
2
2
x
y x 1 2 , : x 1
x 1
1


=



( )
.
Bi 678. Bi 678. Bi 678. Bi 678. Tm GTNN (min) ca cc biu thc sau
a/
( )
1
P x , : x 0
4x
= . b/
( )
1
P 4x , : x 0
x
= .
c/
( )
2
P x 1 , : x 1
x 1
=

. d/
( )
3
P x 2 , : x 2
x 2
=

.
e/ ( )
( )
( )
1
P 3 x 1 , : x 1
12 x 1
=

. f/ ( )
( )
( )
1
P 4 x 3 , : x 3
4 x 3
=

.
g/
( )
( )
1
P x , : x 2
4 x 2
=

. h/
( )
x 2
P , : x 2
2 x 2
=

.
i/
( )
2
x
P , : x 1
x 1
=

. j/
( )
( )
1
P 5x , : x 1
20 x 1
=

.
Bi 679. Bi 679. Bi 679. Bi 679. Tm GTNN (min) ca cc biu thc sau
a/
( )
180
P 5x , : x 1
x 1
=

. b/
( )
2
x 5x 4
P , : x 0
x

= .
c/
( )
2
x 2x 3
P , : x 0
x

= . d/
( )( )
( )
x 1 x 9
P , : x 0
x

= .
e/
( )( )
( )
2x 5 5x 14
P , : x 0
x

= . f/
( )
2
x x 4
P , : x 1
x 1

=

.
g/
( )
2
x x 9
P , : x 2
x 2

=

. h/
( )
2
1 x
P , : x 2
x 2

.
Bi 680. Bi 680. Bi 680. Bi 680. Chng minh cc bt ng thc
a/
( )
a 1 2a, : a 0 . b/
( )
1
a a, : a 0
4
.
c/
( )
2
a 2 2a 2, : a R . d/
( )
b 2 b 1, : b 1 .
e/
( )
b 4 b 4, : b 4 . f/
( )
b 6 b 9, : b 9 .
g/
( )
b 2 3 b 3, : b 3 . g/
( )
ab 2a 2 b 2, : a 0, b 2 .
www.MATHVN.com
www.DeThiThuDaiHoc.com
Ths. L Vn on Phn i S


Page - 130 - "All the flower of tomorrow are in the seeks of today"
h/
( )
ab 8 b 16, : a 0, b 16 . i/
( )
( )
1
a 3, : a b 0
a b b

.
Bi 681. Bi 681. Bi 681. Bi 681. Tm GTNN (min) ca cc biu thc sau
a/
( )
x 1
P , : x 0
x

= . b/
( )
4x 1
P , : x 0
x

= .
c/
( )
2
x
P , : x 1
x 1
=

. d/
( )
2
x
P , : x 4
x 4
=

.
e/
( )
2
x
P , : x 5
x 5
=

. f/
( )
2
2
x
P , : x 9
x 9
=

.
Bi 682. Bi 682. Bi 682. Bi 682. Tm GTNN (min) ca cc biu thc sau
a/
( )
x
P , : x 1
x 1
=

. b/
( )
x
P , : x 3
x 3
=

.
c/
( )( )
( )
xy
P , : x, y 1
x 1 y 1
=

. d/
( )( )
( )
xy
P , : x 4, y 1
x 4 y 1
=

.
e/
( )( )
( )
xy
P , : x, y 9
x 9 y 9
=

. f/
( ) ( )
( )( )
( )
3 3 2 2
x y x y
P , x, y 1
x 1 y 1

=

.
Bi 683. Bi 683. Bi 683. Bi 683. Tm GTLN (max) ca cc biu thc sau
a/
( )
x 1
P , : x 1
x

= . b/
( )
x 2
P , : x 2
x

= .
c/
( )
x 3
P , : x 3
x 1

. d/
( )
x 2
P , : x 2
x 1

.
e/
( )( )
( )
x 1 y 1
P , : x, y 1
xy

= . f/
( )( )
( )
x 1 y 4
P , : x 1, y 4
xy

= .
Bi 684. Bi 684. Bi 684. Bi 684. Tm GTLN (max) v GTNN (min) ca cc biu thc sau
a/ P 5 3x x 6 = . b/ P 7 2x 3x 4 = .
c/ P 11 4x 2x 5 = . d/ P 14 3x x 5 = .
e/ P 2 3 4x 4 x = . f/ P 3 2x 1 2 8 3x = .
Bi 685. Bi 685. Bi 685. Bi 685. Tm GTLN (max) v GTNN (min) ca cc biu thc sau
a/ P 4 3x 2 9 x = . b/
( )
2 2
P 2x 7y, : 3x 8y 1 = = .
c/
( )
2 2
P 2x y, : 2x 5y 8 = = . d/
( )
2 2
P x 3y, : 4x 3y 7 = = .
Bi 686. Bi 686. Bi 686. Bi 686. Chng minh cc bt ng thc sau
a/
( )( ) ( )
1 x 1 y 4, : x, y 0; xy 1 = .
www.MATHVN.com
www.DeThiThuDaiHoc.com
cng hc tp mn Ton 10 tp I Ths. L Vn on


"Cn c b thng minh" Page - 131 -
b/
( )( )( ) ( )
1 x 1 y 1 z 8, : z, y, z 0; xyz 1 = .
c/
( )( )( ) ( )
x y y z z x 8xyz, : z, y, z 0 .
d/
( )
x y z
1 1 1 8, : x, y, z 0
y z x
1 1 1





( )( )( )
.
Bi 687. Bi 687. Bi 687. Bi 687. Chng minh cc bt ng thc sau
a/
( )( )( )
8 p a p b p c abc vi a,b,c l ba cnh ca ABC v p l na chu vi.
b/
( )( )( )
abc a b c b c a c a b vi a,b,c l ba cnh ca ABC.
c/
1 1 1 1 1 1
a b c b c a c a b a b c


vi a,b,c l ba cnh ca ABC.
d/
a b c
3
a b c b c a c a b


vi a,b,c l ba cnh ca ABC.
Bi 688. Bi 688. Bi 688. Bi 688. Chng minh cc bt ng thc sau
a/
( )( )
( )
( )
x y x y 8 8 xy x y , : x, y 0 .
b/
( ) ( )
( )
( )
2
x y 18 x y 12 xy x y , : x, y 0 .
c/
( ) ( ) ( )
2
x y 4 x y 4x 2y 4y 2x, : x, y 0 .
d/
( ) ( )
2
x y x y 2x 2y 2y 2x, : x, y 0 .
e/
( ) ( )
2
x y
x y 2x y 2y x, : x, y 0
2

.
Bi 689. Bi 689. Bi 689. Bi 689. Chng minh cc bt ng thc sau
a/
( )
x y 1 y x 1 xy, : x, y 1 .
b/
( )
xy
x y 4 y x 4 , : x, y 1
2
.
c/
( )
3xyz
xy z 1 yz x 1 zx y 1 , : x, y, z 1
2
.
d/
( )
11xyz
xy z 1 yz x 4 zx y 9 , : x 4, y 9, z 1
12
.
e/
( )
2 2
x y
2 2, : x, y , xy 1, x y
x y

R .
Bi 690. Bi 690. Bi 690. Bi 690. Cho ba s thc x, y, z 0 . Chng minh cc bt ng thc sau
a/ x y 1 xy x y . b/
( )
x y 4 xy 2 x y .
www.MATHVN.com
www.DeThiThuDaiHoc.com
Ths. L Vn on Phn i S


Page - 132 - "All the flower of tomorrow are in the seeks of today"
c/ xy yz zx x yz y zx z xy . d/
2 2 2
x y z x yz y zx z xy .
e/
( )
4 4 4
x y z xyz x y z . f/
8 8 8
3 3 3
x y z 1 1 1
x y z x y z

.
Bi 691. Bi 691. Bi 691. Bi 691. Chng minh cc bt ng thc sau
a/
( )
x y y z z x
6, : x, y, z 0
z x y

.
b/
( )
2008 x 2008 y 2008 z
6, : x y z 2008
x y z

= .
c/
( )
3 3 3
2 2 2
x y z
x y z , : x, y, z 0
y z x
.
d/
( )
3 3 3
x y z
xy yz zx, : x, y, z 0
y z x
.
e/
( )
4 4 4
2 2 2
x y z
xy yz zx, : x, y, z 0
y z x
.
f/
( ) ( )
2 2 2 2 2 2
2
x y y z z x
2 x y z , : x, y, z 0
z x y

.
g/
( )
2 2 2
x y z 1 1 1
, : x, y, z 0
x y z y z x
.
h/
( )
3 3 3 2 2 2
x y z 1 1 1
, : x, y, z 0
y z x x y z
.
i/
( )
2 2 2
x y 4z
x 3y, : x, y, z 0
y z x
.
Bi 692. Bi 692. Bi 692. Bi 692. Chng minh cc bt ng thc sau
a/
( )
3 2 3 2 3 2 2 2 2
2 y 2 x 2 z 1 1 1
, : x, y, z 0
x y y z z x x y z


.
b/
( )
xy yz zx x y z
, : x, y, z 0
x y y z z x 2



.
c/
( )
2 2 2
x y z x y z
, : x, y, z 0
y z z x x y 2



.
d/
( )
2 2 2
x y z x y z
, : x, y, z 0
y 2z z 2x x 2y 3



.
e/
( )
2 2 2
x y z x y z
, : x, y, z 0
3y 2z 3z 2x 3x 2y 5



.
www.MATHVN.com
www.DeThiThuDaiHoc.com
cng hc tp mn Ton 10 tp I Ths. L Vn on


"Cn c b thng minh" Page - 133 -
f/
( ) ( )
2 2 2
x y 16z 1
64z x y , : x, y, z 0
y z z x x y 9


.
g/
( )
3 3 3 2 2 2
x y z x y z
, : x, y, z 0
y z z x x y 2



.
h/
( )
3 3 3 2 2 2
x y z x y z
, : x, y, z 0
2y z 2z x 2x y 3



.
i/
( )
3 3 3 2 2 2
x y z x y z
, : x, y, z 0
2y 3z 2z 3x 2x 3y 5



.
Bi 693. Bi 693. Bi 693. Bi 693. Chng minh cc bt ng thc sau
a/
a b c 3
b c c a a b 2


. b/
a b c
1
b 2c c 2a a 2b


.
c/
a b c 3
b 3c c 3a a 3b 4


. d/
2a 2b 2c 3
3b 5c 3c 5a 3a 5b 4


.
e/
a b c 1
2b 4c 2c 4a 2a 4b 2


. f/
a b c 1
b 2c 3a c 2a 3b a 2b 3c 2


.
Bi 694. Bi 694. Bi 694. Bi 694. Cho a, b, c 0 v
3 3 3
a b c 3 = . Chng minh rng:
a/ a b c 3 . b/
2 2 2
a b c 3 .
c/
4 4 4
a b c 3 . d/
5 5 5
a b c 3 .
e/
6 6 6
a b c 3 . f/
7 7 7
a b c 3 .
Bi 695. Bi 695. Bi 695. Bi 695. Cho x, y, z 0 . Chng minh rng:
a/
1 2
x y
xy

. b/
2 2
1 2
xy x y

.
c/
2 2 4 4
1 2
x y x y

. d/
2
1 1 2
xy xz x yz

.
e/
1 1 2
xy xz x yz

. f/
( )
2
1 2x
, : 0 x 2
3
3 x
<

.
g/
( )
2
1
2x, : 0 x 1
1 x
<

. h/
( )
2
1 x
, : 0 x 2
2
4 x
<

.
i/
( )
2
1
x, : 0 x 1
2 x
<

. j/
1 2
1 x y
x y

.
Bi 696. Bi 696. Bi 696. Bi 696. Tm GTLN (max) ca cc biu thc
a/
( )
P x 4 x = . b/
( )
P x 9 x = .
c/
( )( )
P x 2 4 x = . d/
( )( )
P x 5 3 x = .
www.MATHVN.com
www.DeThiThuDaiHoc.com
Ths. L Vn on Phn i S


Page - 134 - "All the flower of tomorrow are in the seeks of today"
e/
2
P x 1 x = . f/
2
P x 4 x = .
Bi 697. Bi 697. Bi 697. Bi 697. Tm GTLN (max) ca cc biu thc
a/
x y 1 y x 1
P
xy

= . b/
x y 4 y x 4
P
xy

= .
c/
x y 25 y x 25
P
xy

= . d/
x y 4 y x 1
P
xy

= .
e/
x y 2 y x 3
P
xy

= . f/
x y 5 y x 2
P
xy

= .
Bi tp qua cc k thi
Bi 698. Bi 698. Bi 698. Bi 698. Cao ng S Phm Tp. H Ch Minh nm 1996
Cho x, y tha mn iu kin
0 x 3
0 x 4
'
1
1
!
1
1
+
.
Tm gi tr ln nht ca biu thc
( )( )( )
P 3 x 4 y 2x 3y = .
Bi 699. Bi 699. Bi 699. Bi 699. Cao ng S Phm Nh Tr Mu Gio TW 1 nm 2000
Cho x, y, z 0 . Chng minh rng:
2 2 2 2 2 2
1 1 1 9
x y z x y z


.
Bi 700. Bi 700. Bi 700. Bi 700. Cao ng Kim St nm 2000
1/ Cho a, b 0 . Tm gi tr nh nht ca
4 4 2 2
4 4 2 2
a b a b a b
S
b a b a b a
1


=

( )
.
2/ Cho a, b, c l di ba cnh ca ABC.
Chng minh rng:
( )( )( )
a b c b c a c a b abc .
Bi 701. Bi 701. Bi 701. Bi 701. Cao ng Kinh T K Thut Hi Dng khi A nm 2002
Cho hai s thc x, y tha h thc x y 1 = . Chng minh:
4 4
1
x y
8
.
Bi 702. Bi 702. Bi 702. Bi 702. Cao ng Kinh T K Hoch Nng nm 2004
Cho a, b, c l ba s thc tha
3
a 36
abc 1
'
1

1
1
!
1 =
1
1+
. Chng minh:
2
2 2
a
b c ab bc ca
2
.
Bi 703. Bi 703. Bi 703. Bi 703. Cao ng S Phm Sc Trng khi A nm 2005
Cho hai s thc a, b tha mn iu kin a b 1 . Chng minh:
3 3
a b 1 3ab .
Bi 704. Bi 704. Bi 704. Bi 704. Cao ng S Phm H Ni nm 2005
Cho ba s x, y, z 0 . Chng minh:
3 3 3 2 2 2
3 3 3 2 2 2
x y z x y z
y z x y z x
.
www.MATHVN.com
www.DeThiThuDaiHoc.com
cng hc tp mn Ton 10 tp I Ths. L Vn on


"Cn c b thng minh" Page - 135 -
Bi 705. Bi 705. Bi 705. Bi 705. Cao ng Kinh T K Thut Cn Th nm 2005
Cho ba s dng a, b, c tha mn
1 1 1
1
a b c
= . Chng minh:
abc
ab bc ca
3
.
Khi no ng thc xy ra ?
Bi 706. Bi 706. Bi 706. Bi 706. Cao ng Kinh T K Hoch Nng nm 2005
Cho hai s a,b tha a 4, b 4 . Chng minh rng:
2 2
a ab b
a b
6

.
Bi 707. Bi 707. Bi 707. Bi 707. D b Cao ng Giao Thng II nm 2003
Cho 3 s bt k x, y, z. Chng minh:
2 2 2 2 2 2
x xy y x xz z y yz z .
Bi 708. Bi 708. Bi 708. Bi 708. Cao ng Bn Cng Hoa Sen khi A nm 2006
Cho x, y, z 0 v xyz 1 = . Chng minh rng:
3 3 3
x y z x y z .
Bi 709. Bi 709. Bi 709. Bi 709. Cao ng Kinh T K Thut Cn Th khi A nm 2006
Cho 3 s dng x, y, z tho x y z 1 . Tm gi tr nh nht ca biu thc:
1 1 1
A x y z
x y z
= .
Bi 710. Bi 710. Bi 710. Bi 710. Cao ng Kinh T K Thut Cn Th khi B nm 2006
Choa, b, c, d 0 . Chng minh:
a b c d
2
a b c b c d c d a d a b
<

.
Bi 711. Bi 711. Bi 711. Bi 711. Cao ng K Thut Cao Thng khi A nm 2006
Chng minh rng nu x 0 th
( )
2
2
1 2
x 1 1 16
x x
1



( )
.
Bi 712. Bi 712. Bi 712. Bi 712. Cao ng Kinh T K Thut Cng Nghip 1 khi A nm 2006
Cho 3 s dng a, b, c. Chng minh:
a b c a b c a b c
9
a b c

.
Bi 713. Bi 713. Bi 713. Bi 713. Cao ng Y T 1 nm 2006
Cho cc s thc x, y thay i tho mn iu kin:
2
y 0
x x y 12
'
1
1
1
!
1 =
1
1+
.
Tm gi tr ln nht, nh nht ca biu thc: P xy x 2y 17 = .
Bi 714. Bi 714. Bi 714. Bi 714. Cao ng Bn cng Hoa Sen khi D nm 2006
Cho
x, y, z 0
x y z xyz
'
1
1
!
1 =
1
+
. Tm gi tr nh nht ca biu thc: P xyz = .
Bi 715. Bi 715. Bi 715. Bi 715. Hc Vin Hng Khng Vit Nam nm 1999 2000
Cho a, b, c, u, v 0 . t
2 2 2 2 2 2
a b c b c a
A , B
a b b c c a a b b c c a
= =

.
www.MATHVN.com
www.DeThiThuDaiHoc.com
Ths. L Vn on Phn i S


Page - 136 - "All the flower of tomorrow are in the seeks of today"
1/ Chng minh:
2 2
u v u v
u v 2

v A B = .
2/ Chng minh:
a b c
A
2

. Khi no du " " = xy ra ?
Bi 716. Bi 716. Bi 716. Bi 716. i hc An Ninh khi D, G nm 1999 2000
Cho 3 s x, y, z thay i, nhn gi tr thuc on 0;1
l
l
l
.
Chng minh rng:
( ) ( )
3 3 3 2 2 2
2 x y z x y y z z x 3 .
Bi 717. Bi 717. Bi 717. Bi 717. i hc Hng Hi nm 1999 2000
Cho x, y, z 0 v x y z 3 . Chng minh rng:
2 2 2
x y z 3 1 1 1
2 1 x 1 y 1 z 1 x 1 y 1 z


.
Bi 718. Bi 718. Bi 718. Bi 718. i hc Nng nghip I khi A nm 1999 2000
Cho 3 s dng a, b, c tho iu kin abc 1 = . Tm gi tr nh nht ca biu thc:
2 2 2 2 2 2
bc ca ab
P
a b a c b c b a c a c b
=

.
Bi 719. Bi 719. Bi 719. Bi 719. i hc Nng Nghip I khi D nm 1999 2000
Chng minh rng vi mi a, b ta c:
a b a b
1 a b 1 a b


.
Bi 720. Bi 720. Bi 720. Bi 720. Hc Vin Quan H Quc T nm 1999 2000
Cho cc s x, y tha x 0, y 0 v x y 1 = .
Tm gi tr ln nht v gi tr nh nht ca biu thc:
x y
P
y 1 x 1
=

.
S:
( ) ( ) ( )

2 1
max P 1 khi x, y 0,1 , 1, 0 ; minP khi x y
3 2
= = = = = . PP hm s.
Bi 721. Bi 721. Bi 721. Bi 721. i hc Nng nm 1999 2000
Chng minh rng nu a b 0 th
( )( )( ) ( )
2 2 3 3 6 6
a b a b a b 4 a b .
Bi 722. Bi 722. Bi 722. Bi 722. i hc Hu khi A, V nm 1999 2000
Cho cc s thc a,b,c,d tha mn iu kin a 1, b 4, c 2, d 3 .
Chng minh:
( )( )( )( )
4
a 1 b 4 c 2 d 3
1
a b c d 4


.
Bi 723. Bi 723. Bi 723. Bi 723. i hc Quc Gia H Ni khi D nm 1999 2000
Vi a, b, c l 3 s thc dng tho iu kin: ab + bc + ca = abc. Chng minh rng:
2 2 2 2 2 2
b 2a c 2b a 2c
3
ab bc ca

.
www.MATHVN.com
www.DeThiThuDaiHoc.com
cng hc tp mn Ton 10 tp I Ths. L Vn on


"Cn c b thng minh" Page - 137 -
Bi 724. Bi 724. Bi 724. Bi 724. i hc Bch khoa H Ni khi A nm 1999 2000
Cho 2 s a, b tho iu kin a b 0 . Chng minh rng:
3
3 3
a b a b
2 2
1




( )
.
Bi 725. Bi 725. Bi 725. Bi 725. i hc S Phm Tp. H Ch Minh khi D, E nm 1999 2000
Cho 3 s a, b, c bt k. Chng minh cc BT:
a/
2 2 2
a b c ab bc ca . b/
( ) ( )
2
ab bc ca 3abc a b c .
Bi 726. Bi 726. Bi 726. Bi 726. i hc Thy Li II nm 1999 2000
Chng minh rng vi mi s dng a, b, c ta u c:
( )( )( )
( )
3
3
a 1 b 1 c 1 1 abc .
Bi 727. Bi 727. Bi 727. Bi 727. i hc Y H Ni nm 1999 2000
Gi s x, y l hai s dng tho iu kin
2 3
6
x y
= . Tm gi tr nh nht ca tng x + y.
Bi 728. Bi 728. Bi 728. Bi 728. i hc Ty Nguyn khi A, B nm 1999 2000
CMR vi mi x, y, z dng v x y z 1 = th
18xyz
xy yz zx
2 xyz

.
Bi 729. Bi 729. Bi 729. Bi 729. i hc An Ninh Nhn Dn nm 1999 2000
Cho x 0;1
l

l
l
. Chng minh rng:
4 4
x 1 x x 1 x 2 2 2 .
Khi no du " " = xy ra.
HD: p dng BT Bunhiacpxki. Du " " = xy ra khi
1
x
2
= .
Bi 730. Bi 730. Bi 730. Bi 730. i hc Quc Gia Tp. H Ch Minh t 2 khi D nm 1999 2000
Chng minh:
t
t
3 4 2
t 3 t, t 0; 3
2 1

l

l
l

.
HD: S dng BT Bunhiacpxki
Bi 731. Bi 731. Bi 731. Bi 731. i hc Y Dc Tp. H Ch Minh nm 1998 1999
Cho 3 s a, b, c khc 0. Chng minh:
2 2 2
2 2 2
a b c a b c
b c a b c a
.
Bi 732. Bi 732. Bi 732. Bi 732. i hc Y Dc Tp. H Ch Minh nm 1997 1998
Chng minh trong ABC, ta c: p p a p b p c 3p < .
Trong : p l na chu vi ca tam gic.
Bi 733. Bi 733. Bi 733. Bi 733. i hc Ngoi Thng Tp. H Ch Minh khi A nm 1997 1998
Gi s x,y,z l nhng s dng thay i tha iu kin
3
x y z
2
.
Hy tm gi tr nh nht ca biu thc:
1 1 1
P x y z
x y z
= .
www.MATHVN.com
www.DeThiThuDaiHoc.com
Ths. L Vn on Phn i S


Page - 138 - "All the flower of tomorrow are in the seeks of today"
HD:
15 1
minP khi x y z
2 2
= = = = .
Bi 734. Bi 734. Bi 734. Bi 734. Hc Vin Quan H Quc T khi A nm 1997 1998
Cho
x, y, z 0
x y z 1
'
1
1
!
1 =
1
+
. Chng minh:
5 5 5
4 4 4
x y z
1
y z x
.
HD: S dng Cauchy xoay vng:
5 5 5
4 4 4
x y z
...... x y z 1
y z x
.
Bi 735. Bi 735. Bi 735. Bi 735. i hc Ngoi Thng H Ni nm 1997 1998
1/ Cho a, b, c 0 . Chng minh:
a b c a b c
a b b c c a b c c a a b
<

.
2/ Gi s x, y, z 0 v x y z 1 = . Tm gi tr ln nht ca
x y z
P
x 1 y 1 z 1
=

.
HD: 1/ S dng BT b : "nu
x
1
y
< th
( )
x x
; : x, y, 0
y y

<

".
2/ Bin i
x 1 1 y 1 1 z 1 1 1 1 1
P 3
x 1 y 1 z 1 x 1 y 1 z 1
1


= =


( )
.
Bi 736. Bi 736. Bi 736. Bi 736. i hc Thy Li nm 1997 1998
Cho 4 s dng a,b,c,d. Chng minh:
2 2 2 2
5 5 5 5 3 3 3 3
a b c d 1 1 1 1
b c d a a b c d
.
HD: Tch cp nghch o
2 2 2
5 5 5 3 3
a a a 1 1
, , , ,
b b b a a
. Tng t i vi cp cn li.
Bi 737. Bi 737. Bi 737. Bi 737. i hc Nng khi A nm 2001 t 2
Cho a b c 0 tha:
2 2 2
a b c 1 = .
Chng minh:
2 2 2 2 2 2
a b c 3 3
2 b c c a a b


.
Bi 738. Bi 738. Bi 738. Bi 738. Hc vin Ngn Hng Tp. H Ch Minh khi nm A nm 2000 2001
Cho ABC c 3 cnh l a, b, c v p l na chu vi. Chng minh rng:
1 1 1 1 1 1
2
p a p b p c a b c
1



( )
.
Bi 739. Bi 739. Bi 739. Bi 739. i hc Nng nghip I H Ni khi A nm 2000 2001
Cho 3 s x, y, z 0 . Chng minh rng:
3 2 3 2 3 2 2 2 2
2 y 2 x 2 z 1 1 1
x y y z z x x y z


.
Bi 740. Bi 740. Bi 740. Bi 740. i hc Quc gia H Ni khi D nm 2000 2001
Chng minh rng vi mi x 0 v vi mi 1 ta lun c: x 1 x

.
www.MATHVN.com
www.DeThiThuDaiHoc.com
cng hc tp mn Ton 10 tp I Ths. L Vn on


"Cn c b thng minh" Page - 139 -
T chng minh rng vi 3 s dng a, b, c bt k th:
3 3 3
3 3 3
a b c a b c
b c a b c a

Bi 741. Bi 741. Bi 741. Bi 741. i hc Vinh khi A, B nm 2000 2001
Chng minh rng nu a, b, c l di ba cnh ca mt tam gic c chu vi bng 3 th:
2 2 2
3a 3b 3c 4abc 13 .
Bi 742. Bi 742. Bi 742. Bi 742. i hc nm 2002 d b 1
Gi x, y, z l khong cch t im M thuc min trong ca ABC c 3 gc nhn n cc cnh
BC, CA, AB. Chng minh rng:
2 2 2
a b c
x y z
2R

(a, b, c l cc cnh ca
ABC, R l bn knh ng trn ngoi tip). Du = xy ra khi no ?
Bi 743. Bi 743. Bi 743. Bi 743. i hc nm 2002 d b 6
Cho tam gic ABC c din tch bng
3
2
. Gi a, b, c ln lt l di cc cnh BC, CA, AB v
h
a
, h
b
, h
c
tng ng l di cc ng cao k t cc nh A, B, C. Chng minh rng:
a b c
1 1 1 1 1 1
3
a b c h h h
1
1






( )
( )
.
Bi 744. Bi 744. Bi 744. Bi 744. i hc khi D nm 2005
Cho cc s dng x, y, z tho mn xyz = 1. Chng minh rng:
z x
3 3 3 3 3 3
1 x y 1 y z 1 z x
3 3
xy y z

. Khi no ng thc xy ra ?
Bi 745. Bi 745. Bi 745. Bi 745. i hc khi B nm 2005 d b 1
Cho 3 s dng a, b, c tho mn:
3
a b c
4
= .
Chng minh rng: a
3 3 3
a 3b b 3c c 3 3 . Khi no ng thc xy ra ?
Bi 746. Bi 746. Bi 746. Bi 746. i hc khi B nm 2005 d b 2
Chng minh rng: nu 0 y x 1 th
1
x y y x
4
. ng thc xy ra khi no ?
Bi 747. Bi 747. Bi 747. Bi 747. i hc khi D nm 2005 d b 2
Cho x, y, z l 3 s dng v xyz 1 = . Chng minh:
2 2 2
x y z 3
1 y 1 z 1 x 2


.
Bi 748. Bi 748. Bi 748. Bi 748. i hc khi A nm 2006
Cho 2 s thc x 0, y 0 thay i v tho mn iu kin:
( )
2 2
x y xy x y xy = .
Tm gi tr ln nht ca biu thc:
3 3
1 1
A
x y
=
www.MATHVN.com
www.DeThiThuDaiHoc.com
Ths. L Vn on Phn i S


Page - 140 - "All the flower of tomorrow are in the seeks of today"




Phn II



HNH HC

www.MATHVN.com
www.DeThiThuDaiHoc.com
cng hc tp mn Ton 10 tp I Ths. L Vn on


"Cn c b thng minh" Page - 141 -
Chng






































: thng hng
Hai tia AB, CD cng hng.
Hai tia AB, CD ngc hng.
0
xOy 0 =
0
xOy 180 =
C CC C Cc khi nim m u
Vct l mt on thng c hng
'
1
1
1
1
!
1
1
1
1
+



Vct c gc A, ngn B c k hiu l AB
,
v di ca vct AB
,
c k hiu l AB
,

l khong gia im u v im cui ca vct. Ngoi ra, vct cn c k hiu bi mt
ch ci in thng pha trn c mi tn nh a , b , v , u , ....
, , , ,
di ca a
,
k hiu l a
,
.
Vct khng, k hiu 0
,
l vct c
'
1
1
!
1
1
+

Hai vct cng phng khi chng cng nm trn mt ng thng hoc nm trn hai
ng thng song song. Hai vct AB
,
v CD
,
c gi l cng phng
//
//
AB CD
AB CD
A, B, C, D

, ,


Hng ca hai vct : Hai vct cng phng c th cng hng hoc ngc hng. Ta
ch xt hng ca vct khi chng cng phng
+ Hai vct AB
,
v CD
,
gi l cng hng:
// AB CD
AB CD

, ,

+ Hai vct AB
,
v CD
,
gi l ngc hng:
// AB CD
AB CD

, ,

Gc ca hai vct AB
,
v CD
,
l gc to bi hai tia Ox, Oy, ln lt cng hng vi hai tia
AB v CD. Ngha l :

( )

xOy AB,CD =
, ,
.
+ Khi AB
,
v CD
,
khng cng hng th
0 0
0 xOy 180 .

Mt u c xc nh l gc, cn u kia l ngn.
Hng t gc n ngn gi l hng ca vct.
di ca vct l di on thng xc nh bi
im u v im cui ca vct.
A BO
im gc v im ngn trng nhau.
di bng 0.
Hng bt k.
B
B A
C D
A
D A
C
B
:
A

D C
B A
Khi AB
,
v CD
,
cng hng th
0
xOy 0 = . y
D
C
B
A
0 0
0 xOy 180
A
B
C
D
x
y
O
C D
B A
A VCT V CC PHP TON TRN VCT
1
VCT V PHP TO VCT V PHP TO VCT V PHP TO VCT V PHP TON NN N
www.MATHVN.com
www.DeThiThuDaiHoc.com
Ths. L Vn on Phn Hnh hc


Page - 142 - "All the flower of tomorrow are in the seeks of today"





































Hai vct bng nhau khi v ch khi chng cng hng v c di bng nhau.


AB CD
AB CD hay AB CD
'
1
1
1
=
!
1 = =
1
1+
, ,
, ,

Hai vct i nhau khi v ch chng ngc hng v cng di.


AB CD
AB CD hay AB CD
'
1
1
1
=
!
1 = =
1
1+
, ,
, ,

C CC C Cch php ton trn vct
a/ Tng ca hai vct
Qui tc ba im (Qui tc tam gic hay qui tc Chasles)
Vi ba im bt k A, B, C ta c: AB AC CB =
, , ,
(Xen im C vo gia AB).
Qui tc 3 im cn c gi l h thc Chasles dng cng cc vct lin tip, c
th m rng cho trng hp nhiu vct nh sau:
1 n 1 2 2 3 3 4 n 1 n
A A A A A A A A ... A A

=
, , , , ,
.
Qui tc hnh bnh hnh
Cho ABCD l hnh bnh hnh th
AC AB AD
DB DA DC

, , ,
, , ,
v
AB DC
AD BC

, ,
, ,

Qui tc hnh bnh hnh dng cng cc vct chung gc.
Tnh cht: a b b a =
, , , ,

( ) ( )
a b c a b c =
, , , , , ,
a 0 0 a a = =
, , , , ,
.
b/ Hiu ca hai vct
Vct i
Vct i ca vct a
,
, k hiu l a
,
.
Tng ca hai vct i l vct 0
,
:
( )
a a 0 =
, , ,
.
Qui tc tam gic i vi hiu vct
Vi ba im A, B, C bt k, ta lun c : AB CB CA =
, , ,
.
c/ Tch ca mt s i vi mt vct
nh ngha: Cho mt s thc k 0 v mt vct a 0
, ,
.
Tch k.a
,
l mt vct c
'
1
1
!
1
1

AB
,
v CD
,
cng hng.
AB
,
v CD
,
ngc hng.
A B
C D
A B
D C
A
B
D
C
a k.
,
cng hng vi a
,
nu k 0
a k.
,
ngc hng vi a
,
nu k 0 <
Tnh cht

( )
k a b k.a k.b =
, , , ,
.
( )
k h .a k.a h.a =
, , ,
. 1.a a =
, ,
.

( )
( )
k. h.a kh .a =
, ,
.
( )
1 .a a =
, ,
. 0.a 0 =
, ,
.
a b +


a


Minh ha h thc Chasles
www.MATHVN.com
www.DeThiThuDaiHoc.com
cng hc tp mn Ton 10 tp I Ths. L Vn on


"Cn c b thng minh" Page - 143 -













Bi 1. Bi 1. Bi 1. Bi 1. Cho hnh v bn cnh, bit EF // AB // DC. Hy tm
a/ Hai vct bng nhau.
b/ Hai vct i nhau.
c/ Hai vct khng cng phng.
d/ Hai vct cng phng khng bng nhau cng
khng i nhau.
Bi 2. Bi 2. Bi 2. Bi 2. Cho 4 im A, B, C, D khng thng hng. C 5 h thc vct v 5 mnh c t hai ct
tng ng, hy ni chng li vi nhau to thnh mt suy lun ng ?

Ct I Ct II
1/ AD DB =
, ,

A : "ABCD l hnh bnh hnh"
2/ AB 3AC =
, ,

B: "ABDC l hnh bnh hnh"
3/ AB CD =
, ,

C: "ACBD l hnh bnh hnh"
4/ DC DA DB =
, , ,

D: "D l trung im AB"
5/ AD BC =
, ,

E: "C AB "

Bi 3. Bi 3. Bi 3. Bi 3. Cho hnh bnh hnh ABCD. Hy ch ra cc vct
( )
0
,
c im u v im cui l mt trong
bn im ABCD. Trong s cc vct trn, hy ch ra
a/ Cc vct cng phng.
b/ Cc cp vct cng phng nhng ngc hng.
c/ Cc cp vct bng nhau.
Bi 4. Bi 4. Bi 4. Bi 4. Cho lc gic u ABCDEF c tm O.
a/ Tm cc vct khc cc vct khng
( )
0
,
v cng phng vi AO
,
.
b/ Tm cc vct bng vi cc vct AB
,
v CD
,
.
Dng ton 1. i cng v vct
A B
C D
F E
iu kin hai vct cng phng
iu kin cn v 2 vct a
,
; b
,
( )
b 0
, ,
cng phng l tn ti mt s k a b k. =
, ,
.
d/ Trung im on thng v trng tm tam gic
I l trung im ca AB
1
IA IB 0 hay IA IB AB hay IA IB
2
= = = =
, , , , , , , ,
.
I l trung im AB v M l im bt k 2MI MA MB =
, , ,
.
G l trng tm ABC GA GB GC 0 =
, , , ,
.
G l trng tm ABC v M bt k 3MG MA MB MC =
, , , ,

M
A B
I
www.MATHVN.com
www.DeThiThuDaiHoc.com
Ths. L Vn on Phn Hnh hc


Page - 144 - "All the flower of tomorrow are in the seeks of today"
c/ Hy v cc vct bng vi vct AB
,
v c im u l O, D, C.
d/ Hy v cc vct bng vi vct AB
,
v c im gc l O, D, C.
Bi 5. Bi 5. Bi 5. Bi 5. Cho hnh bnh hnh ABCD. Gi O l giao im ca hai ng cho.
a/ Tm cc vct bng vi vct AB
,
.
b/ Tm cc vct bng vi vct OA
,
.
c/ V cc vct bng vi OA
,
v c im ngn l A, B, C, D.
Bi 6. Bi 6. Bi 6. Bi 6. Cho 3 im A, B, C phn bit. C bao nhiu vct khc vct khng c im u v im cui
l cc im ?
Bi 7. Bi 7. Bi 7. Bi 7. Cho 5 im A, B, C, D, E phn bit. C bao nhiu vct khc vct khng c im u v im
cui l cc im ?
Bi 8. Bi 8. Bi 8. Bi 8. Cho ABC c A', B', C' ln lt l trung im ca cc cnh BC, CA, AB.
a/ Chng minh: BC' C' A A' B' = =
, , ,
.
b/ Tm cc vct bng vi B' C', C' A'
, ,
.
Bi 9. Bi 9. Bi 9. Bi 9. Cho vct AB
,
v mt im C. Hy dng im D sao cho AB CD =
, ,
.
Bi 10. Bi 10. Bi 10. Bi 10. Cho t gic ABCD. Gi M, N, P, Q ln lt l trung im ca cc cnh AB, CD, AD, BC.
Chng minh: MP QN, MQ PN = =
, , , ,
.
Bi 11. Bi 11. Bi 11. Bi 11. Cho hnh bnh hnh ABCD c O l giao im ca hai ng cho. Chng minh:
a/ AC BA AD, AB AD AC = =
, , , , ,
.
b/ Nu AB AD CB CD =
, , , ,
th ABCD l hnh ch nht.
Bi 12. Bi 12. Bi 12. Bi 12. Cho ABC u c cnh l a. Tnh di cc vct AB BC, AB BC
, , , ,
.
Bi 13. Bi 13. Bi 13. Bi 13. Cho hnh vung ABCD cnh l a. Tnh AB AC AD
, , ,
.
Bi 14. Bi 14. Bi 14. Bi 14. Cho hnh bnh hnh ABCD tm O. Hy biu din cc vct AB, BC, CD, DA
, , , ,
theo hai vct
AO, BO
, ,
.
Bi 15. Bi 15. Bi 15. Bi 15. Cho ABC u cnh a, trc tm H. Tnh di ca cc vect HA, HB, HC
, , ,
.
Bi 16. Bi 16. Bi 16. Bi 16. Cho hnh vung ABCD cnh a, tm O. Tnh di ca cc vect
AB AD, AB AC, AB AD
, , , , , ,
.
Bi 17. Bi 17. Bi 17. Bi 17. Cho ABC ni tip ng trn tm O. Gi H l trc tm ca ABC, B' l im i xng vi B
qua O. Chng minh rng AH B' C =
, ,
.
Bi 18. Bi 18. Bi 18. Bi 18. T gic ABCD l hnh g nu c AB CD =
, ,
v AB CD = .
Bi 19. Bi 19. Bi 19. Bi 19. Cho a b 0 =
, ,
. So snh v di, phng v hng ca hai vct a
,
v b
,
.
Bi 20. Bi 20. Bi 20. Bi 20. Cho hai vct a
,
v b
,
l hai vct khc vct khng. Khi no c ng thc xy ra ?
a/ a b a b =
, , , ,
. b/ a b a b =
, , , ,
.
Bi 21. Bi 21. Bi 21. Bi 21. Cho ABC. V D i xng vi A qua B, E i xng vi B qua C v F i xng vi C qua A.
Gi G l giao im gia trung tuyn AM ca ABC vi trung tuyn DN ca DEF. Gi I, K ln
lt l trung im ca GA v GD. Chng minh:
www.MATHVN.com
www.DeThiThuDaiHoc.com
cng hc tp mn Ton 10 tp I Ths. L Vn on


"Cn c b thng minh" Page - 145 -
a/ AM NM =
, ,
. b/ MK NI =
, ,
.
Bi 22. Bi 22. Bi 22. Bi 22. Cho ABC v M l mt im khng thuc cc cnh ca tam gic. Gi D, E, F ln lt l trung
im ca AB, BC, CA. V im P i xng vi M qua D, im Q i xng vi P qua E, im N
i xng vi Q qua F. Chng minh rng MA NA =
, ,
.
Bi 23. Bi 23. Bi 23. Bi 23. Cho hai ABC v AEF c cng trng tm G. Chng minh: BE FC =
, ,
.
Bi 24. Bi 24. Bi 24. Bi 24. Cho hnh bnh hnh ABCD. Gi M, N ln lt l trung im ca BC v CD. E, F ln lt l giao
im ca AM, AN vi BD. Chng minh rng: BE FD =
, ,
.
Bi 25. Bi 25. Bi 25. Bi 25. Cho hnh ch nht ABCD, k AH BD . Gi M, N ln lt l trung im ca DH v BC. K
BK AM v ct AH ti E. Chng minh rng: MN EB =
, ,
.
Bi 26. Bi 26. Bi 26. Bi 26. Cho ABC c G l trng tm.
a/ Hy phn tch AG
,
theo hai vct AB
,
v AC
,
.
b/ Gi E, F l hai im xc nh bi cc iu kin: EA 2EB, 3FA 2FC 0 = =
, , , ,
. Hy phn
tch vct EF
,
theo AB, AC
, ,
.
Bi tp trc nghim
Bi 27. Bi 27. Bi 27. Bi 27. Vct c im u l D im cui l E c k hiu l
A. DE. B. DE
,
. C. ED
,
. D. DE
,
.
Bi 28. Bi 28. Bi 28. Bi 28. Vi vct ED
,
(khc vct khng) th di on thng ED c gi l:
A. Phng ca vct ED
,
. B. Hng ca vct ED
,
.
C. Gi ca vct ED
,
. D. di ca vct ED
,
.
Bi 29. Bi 29. Bi 29. Bi 29. Hai vct c gi l bng nhau khi v ch khi:
A. Gi ca chng trng nhau v di ca chng bng nhau.
B. Chng trng vi mt trong cc cp cnh i ca mt hnh bnh hnh.
C. Chng trng vi mt trong cc cp cnh ca mt tam gic u.
D. Chng cng hng v di ca chng bng nhau.
Bi 30. Bi 30. Bi 30. Bi 30. Trong cc mnh sau, mnh no ng, mnh no sai ?
A. Hai vct i nhau th cng phng. B. Hai vct bng nhau th cng phng.
C. Hai vct cng phng th i nhau. D. Hai vct cng phng th bng nhau.
E. Hai vct bng nhau th cng di. F. Hai vct c cng di th bng nhau.
Bi 31. Bi 31. Bi 31. Bi 31. Pht biu no sau y l ng ?
A. Hai vct khng bng nhau th c di khng bng nhau.
B. Hiu ca hai vct c di bng nhau l vct khng.
C. Tng ca hai vct khc vct khng l mt vct khc vct khng.
D. Hai vct cng phng vi 1 vct ( 0)
,
th hai vct cng phng vi nhau.
Bi 32. Bi 32. Bi 32. Bi 32. Cho hnh bnh hnh ABCD. Mnh no trong cc mnh sau l ng ?
A. AB DC =
, ,
. B. AD BC =
, ,
. C. CA DB =
, ,
.
Bi 33. Bi 33. Bi 33. Bi 33. Cho hnh ch nht ABCD, gi O l giao im ca AC v BD, pht biu no l ng ?
A. OA OB OC OD = = =
, , , ,
. B. AC BD =
, ,
.
C. OA OB OC OD 0 =
, , , , ,
. D. AC AD AB =
, , ,
.
www.MATHVN.com
www.DeThiThuDaiHoc.com
Ths. L Vn on Phn Hnh hc


Page - 146 - "All the flower of tomorrow are in the seeks of today"
Bi 34. Bi 34. Bi 34. Bi 34. Vi ba im phn bit G, H v K th s vct m im u v im cui ly trong s cc im
cho l:
A. 3. B. 6. C. 9. D. V s.
Bi 35. Bi 35. Bi 35. Bi 35. Cho t gic ABCD, s vct (khc vct khng) m im u v im cui ly trong s cc
im l nh ca t gic cho l:
A. 6. B. 12. C. 18. D. 24.
Bi 36. Bi 36. Bi 36. Bi 36. Cho trc vct MN
,
0
,
th s vct cng phng vi vct cho l:
A. 1. B. 2. C. 3. D. V s.
Bi 37. Bi 37. Bi 37. Bi 37. Cho trc vct MN
,
khc vct khng th s vct cng hng vi vct cho l:
A. 1. B. 2. A. 3. A. V s.
Bi 38. Bi 38. Bi 38. Bi 38. Cho trc vct MN
,
khc vct khng th s vct bng vct cho l:
A. 1. A. 2. C. 3. D. V s.
Bi 39. Bi 39. Bi 39. Bi 39. Hai vct ngc hng th phi:
A. Bng nhau. B. Cng phng. C. Cng di. D. Cng im u.
Bi 40. Bi 40. Bi 40. Bi 40. Cho t gic ABCD c AD BC =
, ,
. Mnh no trong cc mnh sau l sai ?
A. ABCD l hnh bnh hnh. B. BDCA l hnh bnh hnh.
C. AC BD =
, ,
. D. AB DC =
, ,
.
Bi 41. Bi 41. Bi 41. Bi 41. Nu hai vct cng ngc hng vi mt vct th ba (v c vct u khc vct khng) th
hai vct :
A. Bng nhau. B. Cng di. C. Cng hng. D. Ngc hng.
Bi 42. Bi 42. Bi 42. Bi 42. Nu 3 im A, B, C thng hng th cc vct AB
,
v AC
,
ch c th xy ra kh nng:
A. Bng nhau. B. Cng phng.
C. Cng hng. D. Cng di.
Bi 43. Bi 43. Bi 43. Bi 43. Nu cAB AC =
, ,
th:
A. Tam gic ABC l tam gic cn. B. Tam gic ABC l tam gic u.
C. A l trung im ca on BC. D. im B trng vi im C.
Bi 44. Bi 44. Bi 44. Bi 44. Cho hnh bnh hnh ABCD. Khi AB AC
, ,
bng:
A. BD
,
. B. CB
,
. C. 0
,
. D. Mt kt qu khc.
Bi 45. Bi 45. Bi 45. Bi 45. Cho lc gic u ABCDEF, gi O l giao im cc ng cho, khi cp vct bng vct
AB
,
l:
A. OC
,
v DE
,
. B. FO
,
v CO
,
. C. OF
,
vED
,
. D. OC
,
v ED
,
.
Bi 46. Bi 46. Bi 46. Bi 46. Cho hnh bnh hnh MNPQ, khi :
A. MN PQ =
, ,
v NP MQ =
, ,
. B. MN PQ =
, ,
v NP QM =
, ,
.
C. MN QP =
, ,
v NP QM =
, ,
. D. MN QP =
, ,
v NP MQ =
, ,
.
Bi 47. Bi 47. Bi 47. Bi 47. Cho tam gic MNP vung ti M v MN 3cm, MP 4cm = = . Khi di ca vct NP
,
l:
A. 3cm. B. 4cm. C. 5cm. A. 6cm.
Bi 48. Bi 48. Bi 48. Bi 48. Cc im D, E, F ln lt l trung im ca cc cnh AB, BC, CA ca ABC. Khi :
A. DF BE CE = =
, , ,
. B. AF FD =
, ,
.
C. EF AD DB = =
, , ,
. D. DE AF FC = =
, , ,
.
Bi 49. Bi 49. Bi 49. Bi 49. Cho t gic ABCD (cc nh ly theo th t ), cc im M, N, E, F ln lt l trung im ca
cc cnh AB, BC, CD, DA. Khi :
A. MN EF =
, ,
. B. NE FM =
, ,
. C. MN EF =
, ,
. D. ME FN =
, ,
.
www.MATHVN.com
www.DeThiThuDaiHoc.com
cng hc tp mn Ton 10 tp I Ths. L Vn on


"Cn c b thng minh" Page - 147 -





















MT S BI TP MU


Bi gii tham kho
Cch gii 1. Thc hin php bin i VT
Ta c:
( ) ( ) ( )
xen D xen B
0
VT AB CD AD DB CB BD AD CB DB BD = = =
,
, , , , , , , , , ,



( )
AD CB VP pcm = =
, ,
.
O OO O Li bnh 1. thc hin vic bin i VT VP, ta thc c rng cn to ra s xut hin ca cc
vct AD
,
v CB
,
, do trong li gii trn vi vct AB
,
ta xen vo im D, cn vct
CD
,
ta xen vo im B. Vi t tng ny, ta tin hnh php bin i VP VT, thng
qua cch gii 2, c th nh sau:
Cch gii 2. Thc hin php bin i VP

Dng ton 2. Chng minh mt ng thc vct
Phng php gii
O S dng
Qui tc 3 im:
AB AC CB =
, , ,
, xen im C.
AB CB CA =
, , ,
, hiu hai vct cng gc.
Quy tc hnh bnh hnh : Vi hnh bnh hnh ABCD l lun c AC AB AD =
, , ,
.
Qui tc trung im: Vi im M ty v I l trung im ca AB ta lun c:

1
2MI MA MB hay IA IB 0 hay IA IB AB hay IA IB
2
1

= = = = =


( )
, , , , , , , , , , ,

Kt hp vi cc tnh cht php cng, tr vct v php nhn mt s vi mt vct thc
hin php bin i tng ng cho biu thc cn chng minh.
O V mc thc hnh, ta c th la chn mt trong cc trng hp bin i sau:
Hng 1. Bin i mt v thnh v cn li (VT VP hoc VP VT). Khi :
+ Nu xut pht t v phc tp, ta cn thc hin vic n gin biu thc.
+ Nu xut pht t v n gin, ta cn thc hin vic phn tch vct.
Hng 2. Bin i ng thc cn chng minh v 1 ng thc bit l lun ng.
Hng 3. Bin i ng thc vct bit l lun ng thnh ng thc cn CM.
Hng 4. To dng hnh ph, da vo tnh cht ca hnh bin i,

A
B
C D
A B
M
I
Bi tp 1 Bi tp 1 Bi tp 1 Bi tp 1. Cho 4 im A, B, C, D. Chng minh rng : AB CD AD CB =
, , , ,
.
www.MATHVN.com
www.DeThiThuDaiHoc.com
Ths. L Vn on Phn Hnh hc


Page - 148 - "All the flower of tomorrow are in the seeks of today"
Ta c:
( ) ( ) ( )


xen B xen D 0
VP AD CB AB BD CD DB AB CD BD DB = = =
, , , , , , , , , ,



( )
AB CD VT pcm = =
, ,
.
Cch gii 3. Thc hin php bin i biu thc
Ta c: AB CD AD CB AB AD CB CD DB DB = = =
, , , , , , , , , ,

(lun ng)
VT VP =
( )
pcm .






Bi gii tham kho
a/ Chng minh: AC BD AD BC 2EF = =
, , , , ,
.
- Chng minh: AC BD AD BC =
, , , ,
.
Ta c :
( ) ( )
AC BD AD DC BC CD =
, , , , , ,


AD BC DC CD AD BC = =
, , , , , ,
( )
AC BD AD BC 1 =
, , , ,

- Chng minh: AD BC 2EF =
, , ,
.
Ta c: F l trung im ca DC v E l im bt k
( )
2EF ED EC 2 =
, , ,

Ta li c: E l trung im ca AB v F l im bt k 2FE FA FB =
, , ,
.

( )
2EF FA FB 2EF FA FB 3 = =
, , , , , ,

Cng v theo v ca
( )
2 cho
( )
3 ta c:
( ) ( ) ( ) ( )
4EF ED EC FA FB EA AD EB BC FD DA FC CB = =
, , , , , , , , , , , , ,


( ) ( ) ( ) ( )
0 0
AD AD BC BC
AD DA BC CB EA EB FD FC 2AD 2BC

= =
, , , ,
, , , , , , , , , ,

.
Hay
( )
( )
2 4EF 2AD BC 2 AD BC AD BC 2EF 4 = = =
, , , , , , , ,

T
( ) ( ) ( )
1 , 4 AC BD AD BC 2EF pcm = =
, , , , ,
.

b/ Chng minh: GA GB GC GD 2EF =
, , , , ,

Ta c: E l trung im ca AB v G l im bt k
( )
2GE GA GB 5 =
, , ,
.
Ta li c: F l trung im ca BC v G l im bt k
( )
2GF GC GD 6 =
, , ,
.
Ly
( ) ( ) ( )
5 6 4GF GA GB GC GD 7 =
, , , , ,
.
Qui tc tr
G
A
E
B
C
F
D

Bi tp 2 Bi tp 2 Bi tp 2 Bi tp 2. Cho t gic li ABCD. Gi E, F ln lt l trung im ca AB v CD
a/ Chng minh rng: AC BD AD BC 2EF = =
, , , , ,
.
b/ Gi G l trung im ca EF. Chng minh rng GA GB GC GD 2EF =
, , , , ,
.
www.MATHVN.com
www.DeThiThuDaiHoc.com
cng hc tp mn Ton 10 tp I Ths. L Vn on


"Cn c b thng minh" Page - 149 -
Do G l trung im ca EF nn:
( )

1
GF EF 8
2
=
, ,

Thay
( )
8 vo
( )
7 ( )
1
4. EF GA GB GC GD GA GB GC GD 2EF pcm
2
= =
, , , , , , , , , ,
.






Bi gii tham kho
a/ Chng minh: GA GB GC 0 =
, , , ,

Gi
1
A l trung im ca BC.
Ta c
1
A l trung im ca BC v G l im bt k
( )

1
GB GC 2GA 1 =
, , ,

Theo tnh cht trng tm th:
( )

1
2GA GA 2 =
, ,
(G chia on AA
1
ra 3 on bng nhau)
T
( ) ( )
1 , 2 GB GC GA GA GB GC 0 = =
, , , , , , ,
(pcm).
b/ Chng minh: MA MB MC 3MG =
, , , ,

Ta c: MA MB MC MG GA MG GB MG GC =
, , , , , , , , ,

( )

0
3MG GA GB GC 3MG pcm = =
, , , , ,

.
O OO O Li bnh 2. Thng qua kt qu cu b/, ta c th khng nh c rng nu MA MB MC 0 =
, , , ,

th M l trng tm ca ABC (dng chng minh 1 im l trng tm ), tht vy:
MA MB MC 0 3MG 0 M G = =
, , , , , ,
.







Bi gii tham kho
a/ Chng minh: AH 2OM =
, ,

Gi A' l im i xng ca A qua O th AA' l ng
knh ca ng trn tm
( )
O .
Nn
o
ABA' 90 = hay A' B AB
CC' AB
V
o
ACA' 90 = hay A' C CA
BB' CA
T
( ) ( )
1 , 2 A' BHC l hnh bnh hnh.
G G G
A

B C
B'
C'
H

M
O

A'
A'B // CH
( )
1
A'C // BH
( )
2
A
B C
A
1

G

Bi tp 3 Bi tp 3 Bi tp 3 Bi tp 3. Cho ABC . Gi G l trng tm ca tam gic v M l im ty trong mt phng.
Chng minh rng:
a/ GA GB GC 0 =
, , , ,
b/ MA MB MC 3MG =
, , , ,

Bi tp 4 Bi tp 4 Bi tp 4 Bi tp 4. Cho ABC c M l trung im ca BC, H l trc tm (giao im ca 3 ng cao), O
l tm ng trn ngoi tip (giao im ca 3 ng trung trc cnh, c bit i vi
tam gic vung th tm ngoi l trung im cnh huyn) ca tam gic.
a/ Chng minh: AH 2OM =
, ,
. b/ Chng minh: HA HB HC 2HO =
, , , ,
.

www.MATHVN.com
www.DeThiThuDaiHoc.com
Ths. L Vn on Phn Hnh hc


Page - 150 - "All the flower of tomorrow are in the seeks of today"
M l trung im ng cho HA.
Do O l trung im AA' nn OM l ng trung bnh AHA'.
OM // AH v
1
OM AH AH 2OM
2
= =
, ,

( )
pcm .
b/ Chng minh: HA HB HC 2HO =
, , , ,
.
Do O l trung im ca AA' v H l im bt k
( )
2HO HA HA' 3 =
, , ,
.
M HBA'C l hnh bnh hnh nn
( )
HA' HB HC 4 =
, , ,

Thay
( )
4 vo
( )
3 HA HB HC 2HO (pcm) =
, , , ,
.





Bi gii tham kho
Dng hnh bnh hnh AB
2
IC
2
c AB
2
// CC
1
v AC
2
// BB
1
.
Ta c:
( )
1
2 2
IA IB IC =
, , ,

Do IC
1
// B
2
A, p dng nh l Thales ta c:
( )

2 1
1
2
2
IB C A b
b
IB C B a
IB IB 2
a
IB IB
'
1
1
= = 1
1
=
!
1
1
1
1
+
, ,
, ,

Tng t:
( )

2 1
1
2
2
IC B A c
c
IC B C a
IC IC 3
a
IC IC
'
1
1
= = 1
1
=
!
1
1
1
1
+
, ,
, ,

Thay
( ) ( )
2 , 3 vo
( )
1 ta c:
( )

b c
IA IB IC a.IA b.IB c.IC 0 pcm
a a
= =
, , , , , , ,
.





Bi gii tham kho
Cch gii 1.
K
( )
// MN AC N AB .
p dng nh l Thales ta c :
( )


AN AB MC
AN .AB
MC
MC BC BC
AN .AB 1
MC
BC
AN AB, 0
BC
= =
=

, ,
, ,

.
V
( )


NM MB MB
NM .AC
MB
AC BC BC
NM .AC 2
MB
BC
NM AC ; 0
BC
= =
=

, ,
, ,

.
B
A
C
I
C
2

B
2
C
1

B
1

A
B C
M
N
Bi tp 5 Bi tp 5 Bi tp 5 Bi tp 5. Cho ABC. Gi I l tm ca ng trn ni tip tam gic (giao im ca 3 ng
phn gic trong). Chng minh rng: a.IA b.IB c.IC 0 =
, , , ,
.
Bi tp 6 Bi tp 6 Bi tp 6 Bi tp 6. Cho ABC. Gi M l im trn cnh BC. Chng minh:
MC MB
AM .AB .AC
BC BC
=
, , ,
.

www.MATHVN.com
www.DeThiThuDaiHoc.com
cng hc tp mn Ton 10 tp I Ths. L Vn on


"Cn c b thng minh" Page - 151 -
Ly
( ) ( ) ( )

MC MB
1 2 AN NM AM .AB .AC pcm
BC BC
= =
, , , , ,
.
Cch gii 2.
Ta c:
( )
( )
( )
( )


MC.AM MC. AB BM 3
AM AB BM
AM AC CM MB.AM MB. AC CM 4
'
1
'
1
1 =
= 1 1
1 1

! !
1 1
= = 1 1
1 1 +
1+
, , ,
, , ,
, , , , , ,
Cng tng v ca hai ng thc
( )
3 v
( )
4 , ta c:
( ) ( )
MC.AM MB.AM MC. AB BM MB. AC CM =
, , , , , ,

( )
( )
BC
0
AM. MB MC MC.AB MB.AC MC.BM MB.CM =
, , , , ,



Do hai vct MC.BM
,
v MB.CM
,
l hai vct i nhau (ngc hng v cng di), nn
MC.BM MB.CM 0 =
, ,
v MB MC BC BC.AM MC.AB MB.AC = =
, , ,

( )

MC MB
AM .AB .AC pcm
BC BC
=
, , ,
.


BI TP P DNG
Bi 50. Bi 50. Bi 50. Bi 50. Cho hnh bnh hnh ABCD c tm l O.
Chng minh rng: DA DB DC 0 =
, , , ,
v OA OB OC OD 0 =
, , , , ,
.
Bi 51. Bi 51. Bi 51. Bi 51. Cho 4 im A, B, C, D ty . Chng minh rng:
a/ AB CD AD CB =
, , , ,
. b/ AC BD AD BC =
, , , ,
.
c/ AB CD AC BD =
, , , ,
.
Bi 52. Bi 52. Bi 52. Bi 52. Cho 5 im A, B, C, D, E ty . Chng minh rng:
a/ AB CD EA CB ED =
, , , , ,
. b/ CD EA CA ED =
, , , ,
.
Bi 53. Bi 53. Bi 53. Bi 53. Cho 6 im: A, B, C, D, E, F. Chng minh rng:
a/ AB CD AD CB =
, , , ,
. b/ AB CD AC DB =
, , , ,
.
c/ AD BE CF AE BF CD =
, , , , , ,
. d/ Nu AC BD =
, ,
th AB CD =
, ,
.
Bi 54. Bi 54. Bi 54. Bi 54. Cho 7 im A, B, C, D, E, F, G. Chng minh rng:
a/ AB CD EA CB ED =
, , , , ,
.
b/ AB CD EF GA CB ED GF =
, , , , , , ,
.
c/ AB AF CD CB EF ED 0 =
, , , , , , ,
.
Bi 55. Bi 55. Bi 55. Bi 55. Cho 8 im A, B, C, D, E, F, G, H
Chng minh rng: AC BF GD HE AD BE GC HF =
, , , , , , , ,
.
www.MATHVN.com
www.DeThiThuDaiHoc.com
Ths. L Vn on Phn Hnh hc


Page - 152 - "All the flower of tomorrow are in the seeks of today"
Bi 56. Bi 56. Bi 56. Bi 56. Cho ABC. Gi M, N, P ln lt l trung im ca BC, CA, AB v O l im bt k.
Chng minh rng : AM BN CP 0 =
, , , ,
v OA OB OC OM ON OP =
, , , , , ,
.
Bi 57. Bi 57. Bi 57. Bi 57. Cho hnh bnh hnh ABCD tm O, M l im bt k. Chng minh rng:
a/ MC MA MB MD =
, , , ,
. b/ MC MD AB =
, , ,
.
c/ BD BA OC OB =
, , , ,
. d/ BC BD BA 0 =
, , , ,
.
Bi 58. Bi 58. Bi 58. Bi 58. Cho hai tam gic ABC v A'B'C'c trng tm tng ng l G v G'.
Chng minh rng: AA' BB' CC' 3GG' =
, , , ,
.
Bi 59. Bi 59. Bi 59. Bi 59. Cho tam gic ABC. Gi E l trung im on BC. Cc im M, N theo th t nm trn cnh
BC sao cho E l trung im on MN. Chng minh rng: AB AC AM AN =
, , , ,
.
Bi 60. Bi 60. Bi 60. Bi 60. Cho ng gic u ABCDE c tm l O. Chng minh rng: OA OB OC OD OE O =
, , , , , ,
.
Bi 61. Bi 61. Bi 61. Bi 61. Cho ABC. Gi A' l im i xng ca B qua A, B l im i xng ca C qua B, C' l im
i xng ca A qua C.
Chng minh rng: OA OB OC OA' OB' OC' =
, , , , , ,
(vi O l im bt k).
Bi 62. Bi 62. Bi 62. Bi 62. Cho lc gic u ABCDEF c tm l O. Chng minh rng: vi M l im ty th ta lun c:
a/ OA OB OC OD OE OF 0 =
, , , , , , ,
. b/ OA OC OE 0 =
, , , ,
.
c/ AB AO AF AD =
, , , ,
. d/ MA MC ME MB MD MF =
, , , , , ,
.
Bi 63. Bi 63. Bi 63. Bi 63. Cho ABC, v bn ngoi cc hnh bnh hnh ABIF; BCPQ; CARS.
Chng minh rng: RF IQ PS 0 =
, , , ,
.
Bi 64. Bi 64. Bi 64. Bi 64. Cho ABC ni tip trong ng trn tm O c trc tm H, k ng knh AD.
a/ Chng mnh rng: HB HC HD =
, , ,
.
b/ Gi H' l im i xng ca H qua O. Chng minh rng: HA HB HC HH' =
, , , ,
.
Bi 65. Bi 65. Bi 65. Bi 65. Cho ABC c trng tm G. Gi M thuc cnh BC sao cho MB 2MC = . Chng minh rng:
a/ AB 2AC 3AM =
, , ,
. b/ MA MB MC 3MG =
, , , ,
.
Bi 66. Bi 66. Bi 66. Bi 66. Cho t gic ABCD. Gi I, J ln lt l trung im ca AB, CD v O, M l im bt k.
Chng minh rng:
a/ AD BC 2IJ =
, , ,
. b/ OA OB OC OD 0 =
, , , , ,
.
c/ MA MB MC MD 4MO =
, , , , ,
.
Bi 67. Bi 67. Bi 67. Bi 67. Cho t gic ABCD. Gi E, F, G, H ln lt l trung im ca AB, BC, CD, DA v G l trung
im ca FH, M l im ty . Chng minh rng:
a/ AF BG CH DE 0 =
, , , , ,
. b/ AB AC AD 4AG =
, , , ,
.
c/ MA MB MC MD ME MF MG MH =
, , , , , , , ,
.
Bi 68. Bi 68. Bi 68. Bi 68. Cho hnh bnh hnh ABCD c tm O v E l trung im ca AD. Chng minh rng:
www.MATHVN.com
www.DeThiThuDaiHoc.com
cng hc tp mn Ton 10 tp I Ths. L Vn on


"Cn c b thng minh" Page - 153 -
a/ OA OB OC OD 0 =
, , , , ,
. b/ EA EB 2EC 3AB =
, , , ,
.
c/ EB 2EA 4ED EC =
, , , ,
.
Bi 69. Bi 69. Bi 69. Bi 69. Cho 4 im A, B, C, D. Gi M, N ln lt l cc trung im ca on thng BC, CD.
Chng minh rng:
3
AB AM NA DA .DB
2
=
, , , , ,
.
Bi 70. Bi 70. Bi 70. Bi 70. Cho ABC. Trn cnh BC ly cc im D, E, F sao cho: BD DE EF FC = = = .
Chng minh rng: AB AD AE AF AC 5AE =
, , , , , ,
.
Bi 71. Bi 71. Bi 71. Bi 71. Cho t gic ABCD c AB khng song song vi CD. Gi M, N, P, Q ln lt theo th t l trung
im ca cc on thng AD, BC, AC, DB.
a/ Chng minh rng:
( )
1
MN AB DC
2
=
, , ,
v
( )
1
PQ AB DC
2
=
, , ,
.
b/ Chng minh cc im M, N, P, Q l 4 nh ca mt hnh bnh hnh.
c/ Gi I l trung im ca on thng MN v O l im bt k.
Chng minh rng: IA IB IC ID 0 =
, , , , ,
v OA OB OC OD 4OI =
, , , , ,
.
Bi 72. Bi 72. Bi 72. Bi 72. Cho hnh bnh hnh ABCD c tm l O. Gi M, N ln lt l trung im ca BC, DC.
Chng minh rng:
a/ OA OM ON 0 =
, , , ,
. b/
( )
1
AM AD 2AB
2
=
, , ,
.
c/
3
AM AN AC
2
=
, , ,
.
Bi 73. Bi 73. Bi 73. Bi 73. Cho ABC c 3 gc nhn. Gi H, G, O ln lt l trc tm, trng tm v tm ng trn ngoi
tip tam gic. D l im i xng vi A qua O.
a/ Chng minh rng BHCD l hnh bnh hnh. T hy tnh tng HB HC
, ,
.
b/ Chng minh rng: HA HB HC 2HO =
, , , ,
v OA OB OC OH =
, , , ,
.
c/ C nhn xt g v 3 im O, G, H ?
Bi 74. Bi 74. Bi 74. Bi 74. Cho ABC. Gi M l trung im ca AB v N l mt im trn cnh AC sao cho NC 2NA = .
Gi K, D ln lt l trung im ca MN v BC. Chng minh rng:
a/
1 1
AK AB AC
4 6
=
, , ,
. b/
1 1
KD AB AC
4 3
=
, , ,
.
Bi 75. Bi 75. Bi 75. Bi 75. Cho t gic ABCD. Gi I v J ln lt l trung im ca hai ng cho AC v BD.
Chng minh rng: AB CD 2IJ =
, , ,
.
Bi 76. Bi 76. Bi 76. Bi 76. Cho u ABC c tm l O. Gi M l im thuc min trong ca tam gic v D, E, F ln lt l
hnh chiu ca M ln 3 cnh ca tam gic. Chng minh rng:
3
MD ME MF MO
2
=
, , , ,
.
Bi 77. Bi 77. Bi 77. Bi 77. Cho ABC. Gi H l trc tm ca tam gic.
Chng minh rng: A B C tan .HA tan .HB tan .HC 0 =
, , , ,
.
Bi 78. Bi 78. Bi 78. Bi 78. Cho ABC. Gi I l tm ng trn ni tip tam gic.
www.MATHVN.com
www.DeThiThuDaiHoc.com
Ths. L Vn on Phn Hnh hc


Page - 154 - "All the flower of tomorrow are in the seeks of today"
Chng minh rng: A B C sin .IA sin .IB sin .IC 0 =
, , , ,
.
Bi 79. Bi 79. Bi 79. Bi 79. Cho ABC. Ly im M ty thuc min trong tam gic.
Chng minh rng:
MBC MAC MAB
S .MA S .MB S .MC 0

=
, , , ,
.
Kt qu trn cn ng khi M ngoi tam gic khng ?
HD: Gi A' l giao im ca ng thng MA vi BC. Ta c:
A' C A' B
MA' .MB .MC
BC BC
=
, , ,
.
V
MA' C MAC
MA' B MAB
S S A' C
BC S S


= = ,
MA' B MA' C MA' B MA' C
MAB MAC MAB MAC
S S S S MA'
MA S S S S

= = =

.
Bi 80. Bi 80. Bi 80. Bi 80. Cho t gic ABCD. Gi M, N ln lt thuc cc on AD v BC sao cho:
MA NB m
MD NC n
= = .
Chng minh rng:
n.AB m.DC
MN
m n

, ,
,
.
Bi 81. Bi 81. Bi 81. Bi 81. Cho on AB. Trn on AB ly im C sao cho
CA m
CB n
= v S l im bt k.
Chng minh rng:
n n
SC .SA .SB
m n m n
=

, , ,
.
Bi 82. Bi 82. Bi 82. Bi 82. Cho hnh ch nht c tm l O v S l im bt k.
Chng minh rng:
2 2 2 2
SA SC SB SD =
, , , ,
HD:
( )
2 2
SA SO OA =
, , ,
.
BI TP RN LUYN
Bi 83. Bi 83. Bi 83. Bi 83. Cho hnh bnh hnh ABCD c tm O v im M ty . Chng minh rng
a/ DO AO AB =
, , ,
. b/ CO OB BA =
, , ,
.
c/ AB BC DB =
, , ,
. d/ DA DB OD OC =
, , , ,
.
e/ MA MC MB MD 2MO = =
, , , , ,
. f/ OA OB OC OD 0 =
, , , , ,
.
Bi 84. Bi 84. Bi 84. Bi 84. Cho ABC. Gi M, N, P ln lt l trung im ca cc cnh BC, CA, AB. Chng minh rng:
a/ AB BC CA 0 =
, , , ,
. b/ MN NP PM 0 =
, , , ,
.
c/ AN CM PB 0 =
, , , ,
. d/ AP BM MP 0 =
, , , ,
.
e/
1
AP BM AC
2
=
, , ,
. f/
( )
1
AM AB AC
2
=
, , ,
.
g/ AM BN CP 0 =
, , , ,
. h/ AP BM AN BP PC =
, , , , ,
.
Bi 85. Bi 85. Bi 85. Bi 85. Cho hnh thang OABC. Gi M, N ln lt l trung im ca OB v OC. Chng minh rng:
a/
1
AM OB OA
2
=
, , ,
. b/
1
BN OC OB
2
=
, , ,
. c/
( )
1
MN OC OB
2
=
, , ,
.
Bi 86. Bi 86. Bi 86. Bi 86. Cho ABC c G l trng tm tam gic v I l im i xng ca B qua G. M l trung im ca
BC. Chng minh rng:
www.MATHVN.com
www.DeThiThuDaiHoc.com
cng hc tp mn Ton 10 tp I Ths. L Vn on


"Cn c b thng minh" Page - 155 -
a/ 2AC AB 3AI =
, , ,
. b/ 2AB 3AC 6IC =
, , ,
. c/ AC 5AB 6MI =
, , ,
.
Bi 87. Bi 87. Bi 87. Bi 87. Cho tam gic ABC. Gi M l trung im ca AB, D l trung im ca BC, N l im thuc AC
sao cho CN 2NA =
, ,
. Gi K l trung im ca MN. Chng minh rng:
a/
1 1
AK AB AC
4 6
=
, , ,
. b/
1 1
KD AB AC
4 3
=
, , ,
.
Bi 88. Bi 88. Bi 88. Bi 88. Cho tam gic ABC, c AM l trung tuyn. I l trung im ca AM.
a/ Chng minh: 2IA IB IC 0 =
, , , ,
.
b/ Vi im O bt k, chng minh: 2OA OB OC 4OI =
, , , ,
.
Bi 89. Bi 89. Bi 89. Bi 89. Cho ABC vi M l im ty .
a/ Chng minh rng a MA 2MB MC =
, , , ,
khng ph thuc vo v tr ca im M.
b/ Dng im D sao cho CD a =
, ,
. CD ct AB ti K. Chng minh: KA KB 0 =
, , ,
v
CD 3CK =
, ,
.
Bi 90. Bi 90. Bi 90. Bi 90. Cho ng trn tm I ni tip trong ABC, tip xc vi cc cnh BC, CA, AB ln lt ti M, N,
P. Gi a, b, c ln lt theo th t l di ca cc cnh BC, CA, AB ca ABC.
Chng minh: a.IM b.IN c.IP 0 =
, , , ,
.
Bi 91. Bi 91. Bi 91. Bi 91. Cho lc gic ABCDEF. Gi M, N, P ln lt l trung im ca AB, CD, EF.
a/ Chng minh:
( )
1
IM IN IP IA IB IC ID IE IF
2
=
, , , , , , , , ,
vi I bt k.
b/ Hy tm im G sao cho GA GB GC GD GE GF 0 =
, , , , , , ,
.
c/ Gi
1 2 3 4 5 6
G , G , G , G , G , G tng ng l trng tm ca ABC, DEF, BCD, EFA, CDE,
FAB. Chng minh rng:
1 2 3 4 5 6
G G , G G , G G cng ng qui ti mt im.




www.MATHVN.com
www.DeThiThuDaiHoc.com
Ths. L Vn on Phn Hnh hc


Page - 156 - "All the flower of tomorrow are in the seeks of today"




















































Dng ton 3. Xc nh im tha mn ng thc vct Cm ng qua im c nh

O Phng php gii
Bi ton. Xc nh im M tha mt ng thc vct cho trc ?
Bc 1. Ta bin i ng thc cho (bng xen im, hiu 2 vct cng gc, qui tc hnh
bnh hnh, tnh cht trung im, trng tm, ) v dng: OM v =
, ,
. Trong
im O bit trc v vct v
,
bit.
Bc 2. Nu mun dng im M, ta ly im O lm gc, dng 1 vct bng 1 vct v
,
,
khi im ngn ca vct ny chnh l im M.
O Lu
Lu 1. Thng thng, biu thc OM v =
, ,
l nhng biu thc c bit (trung im, trng
tm, im chia on theo t l a k.b =
, ,
, hnh bnh hnh, Ta da vo biu thc
ny dng hnh.
Lu 2. Mt s cch chng minh thng dng
chng minh I l trung im ca on thng AB, ta cn chng minh 1 trong cc h thc:
IA BI =
, ,
.
IA IB 0 =
, , ,
.
2IA AB =
, ,
.
2OI OA OB =
, , ,
(O bt k).
chng minh im G l trng tm ca ABC, ta cn chng minh 1 trong cc h thc:
GA GB GC 0 =
, , , ,
.
Vi I l trung im ca cnh BC th
2
AG AI
3
=
, ,
.
Vi O l im bt k trong mt phng th: 3OG OA OB OC =
, , , ,
.
chng minh ABCD l hnh bnh hnh
AB DC
AD BC

, ,
, ,

chng minh hai im A
1
v A
2
trng nhau ta c th chng minh 1 trong cc h thc:

1 2
A A 0 =
, ,
.

1 2
OA OA =
, ,
vi O l im bt k.
iu kin cn v ABC v A'B'C' c cng trng tm l: AA' BB' CC' 0 =
, , ,
.

Nu
( )
MB k.MC k 1 =
, ,
th
AB k.AC
AM
1 k

, ,
,

(hay im M chia on AB theo t s k
( )
k 1 .

www.MATHVN.com
www.DeThiThuDaiHoc.com
cng hc tp mn Ton 10 tp I Ths. L Vn on


"Cn c b thng minh" Page - 157 -
MT S BI TP MU




Bi gii tham kho
Ta c:
( )
2MA 3MB 0 2MA 3 MA AB 0 MA 3AB 0 AM 3AB = = = =
, , , , , , , , , , , ,
.
Do , M c xc nh nh sau:
M nm trn ng thng AB v nm ngoi on AB, gn B. Hai vct AM
,
, AB
,
cng hng.
di AM 3AB = , ngha l im B chia AM ra 3 on bng nhau.








Bi gii tham kho
a/ Xc nh im K tha:
( )
3AB 2AC 12AK 0 1 =
, , , ,

Theo gi thit th:
( )

AB 2AM
AB 2AM 2
AB AM
'
1 =
1
1
=
!
1

1
1+
, ,
, ,


( )

AC 3AN
AC 3AN 3
AC AN
'
1 =
1
1
=
!
1

1
1+
, ,
, ,

Thay
( )
2 v
( )
3 vo
( )
1 ta c:
( )
1
6AM 6AN 12AK 0 AK AM AN
2
= =
, , , , , , ,
.

K l trung im ca MN.
b/ Xc nh im D tha:
( )
3AB 4AC 12KD 0 4 =
, , , ,

Ta c
( )


Xen A
KD AD AK 5

=
, , ,

. M theo
( )
4 ( )

1 1
AK AB AC 6
4 3
=
, , ,

Thay
( )
6 vo
( )
5 ta c: ( )

1 1
KD AD AB AC 7
4 3
=
, , , ,

Thay
( )
7 vo
( )
4 :
( )
1 1 1
3AB 4AC 12 AD AB AC 0 AD AB AC
4 3 2
1

= =


( )
, , , , , , , , ,
.

D l trung im ca BC.






Bi gii tham kho
A M B
C
A
B
N
M
K
D
Bi tp 7 Bi tp 7 Bi tp 7 Bi tp 7. Cho im A, B. Xc nh im M bit: 2MA 3MB 0 =
, , ,
.

Bi tp 8 Bi tp 8 Bi tp 8 Bi tp 8. Cho ABC. Gi M l trung im ca AB v N thuc cnh AC, sao cho NC 2NA = .
Hy xc nh K v D khi
a/ 3AB 2AC 12AK 0 =
, , , ,
. b/ 3AB 4AC 12KD 0 =
, , , ,
.

Bi tp 9 Bi tp 9 Bi tp 9 Bi tp 9. Cho hnh bnh hnh ABCD, hy dng cc im M, N tha mn:
a/ MA MB MC AD =
, , , ,
. b/ NC ND NA AB AD AC =
, , , , , ,
.
c/ Chng minh rng: MN BA =
, ,
.

www.MATHVN.com
www.DeThiThuDaiHoc.com
Ths. L Vn on Phn Hnh hc


Page - 158 - "All the flower of tomorrow are in the seeks of today"
a/ Dng im M tha: MA MB MC AD =
, , , ,

Ta c:
BA
MA MB MC AD BA MC AD = =
,
, , , , , , ,


CM AD BA AD AB = =
, , , , ,

Do ABCD l hnh bnh hnh nn:
AD AB AC =
, , ,
CM AC =
, ,
C l trung im ca CM.
b/ Dng im N tha: NC ND NA AB AD AC =
, , , , , ,

Ta c: NC ND NA AB AD AC =
, , , , , ,
( ) ( )
AC AC
NC NA ND AB AD AC =
, ,
, , , , , ,


AC ND AC AC DN AC = =
, , , , , ,
N l nh th t ca hnh bnh hnh DACN.
c/ Chng minh: MN BA =
, ,

Ta c DACN l hnh bnh hnh (cu b) nn NC DA =
M ABCD l hnh bnh hnh (gi thit) nn DA BC =
T gic ABMN l hnh bnh hnh (do c 2 ng cho NB v AM ct nhau ti trung im ca
mi ng) MN BA =
, ,
(pcm).







Bi gii tham kho
a/ Chng minh: tn ti duy nht im I tha mn: .IA .IB 0 =
, , ,
.
Ta c:
( )
( )
.IA .IB 0 .IA . IA AB 0 .IA .AB 0 = = =
, , , , , , , , , ,

( )
.AI .AB AI .AB

= =

, , , ,
.
V A, B c nh nn vct .AB


,
khng i, do tn ti duy nht im I tha bi.
b/ Chng minh:
( )
M : .MA .MB .MI =
, , ,
.
Ta c:
( ) ( )
( ) ( )
0
.MA .MB . MI IA . MI IB .MI .IA .IB .MI = = =
, , , , , , , , , ,

.
Vy
( )
.MA .MB .MI, : M =
, , ,
(pcm).



A B
C
D
M
N
NC NB = C l trung im BN.
Bi tp 10 Bi tp 10 Bi tp 10 Bi tp 10. Cho trc 2 im A, B v hai s thc , tha mn: 0
a/ Chng minh rng tn ti duy nht im I tha mn: .IA .IB 0 =
, , ,
.
b/ T suy ra vi im M bt k, ta lun c:
( )
.MA .MB .MI =
, , ,
.
www.MATHVN.com
www.DeThiThuDaiHoc.com
cng hc tp mn Ton 10 tp I Ths. L Vn on


"Cn c b thng minh" Page - 159 -




























Bi gii tham kho
a/ Tm k tha: 2.MA MB k.MI =
, , ,
.
V
( )
2.MA MB k.MI 1 =
, , ,
tha vi mi M, do n cng ng vi M I.
Khi : ( )
( )
1
2.IA IB k.II 0 2 2.IA IA AB 0 IA .AB I
3
= = = =
, , , , , , , , , ,
c xc
nh. N nm trn ng thng AB, ngoi on AB, vct IA
,
ngc chiu vi vct AB
,
v c
ln
1
IA AB
3
=
T
( )
2 ta c:
( ) ( )
2.MA MB 2 1 MI 3MI 3 = =
, , , ,
(p dng li bnh 3 v M I)
T
( ) ( )
1 , 3 3MI k.MI k 3 = =
, ,
.
b/ Tm k tha: MA MB 2.MC k.MJ =
, , , ,
.
V
( )
MA MB 2.MC k.MJ 4 =
, , , ,
tha mn vi mi M nn n ng vi M J.
Khi :
( )
JA JB 2.JC k.JJ 0 5 = =
, , , , ,

Gi E l trung im ca AB, t
( )
5 2.JE 2.JC 0 JE JC 0 = =
, , , , , ,
J l trung im ca CE.
T
( )
5 , ta c:
( ) ( )
MA MB 2.MC 1 1 2 .MJ 4.MJ 6 = =
, , , , ,


Nu 1 = = th im I chnh l trung im ca AB.
Bi ton trn c m rng cho ba im A, B, C v b 3 s thc , , cho trc tha mn:
0 , ngha l:
+ Tn ti im I duy nht tha mn: .IA .IB .IC 0 =
, , , ,

+ T suy ra vi im M bt k, ta lun c:
( )
.IA .IB .IC .MI =
, , , ,
. V khi:
1 = = = th I l trng tm ca ABC.
Bi ton trn vn ng vi n im
( )
1
A i 1, n = v b s thc
i

( )
i 1, n = tha mn
n
i
i 1
0
=


Kt qu trn dng gii bi ton: "Cho n im A
i
, i 1, n = v b s thc
i
, i 1, n = tha mn
n
i
i 1
0
=

. Tm s thc k v im c nh I sao cho ng thc vct:


n
i i
i 1
.MA k.MI
=
=

, ,

tha mn vi mi im M".
O OO O Li bnh Li bnh Li bnh Li bnh 3 33 3
Bi tp 11 Bi tp 11 Bi tp 11 Bi tp 11. Cho t gic ABCD, M l im ty . Trong mi trng hp hy tm s k v im c
nh I, J, K sao cho ng thc vct sau tha mn vi mi im M.
a/ 2.MA MB k.MI =
, , ,

b/ MA MB 2.MC k.MJ =
, , , ,

c/ MA MB MC 3.MD k.MK =
, , , , ,

www.MATHVN.com
www.DeThiThuDaiHoc.com
Ths. L Vn on Phn Hnh hc


Page - 160 - "All the flower of tomorrow are in the seeks of today"
T
( ) ( )
4 , 6 k.MJ 4.MJ k 4 = =
, ,
.
c/ Tm k tha: MA MB MC 3.MD k.MK =
, , , , ,
.
V
( )
MA MB MC 3.MD k.MK 7 =
, , , , ,
tha mn : M nn n ng vi M K.
Khi :
( )
KA KB KC 3.KD k.KK 0 8 = =
, , , , , ,

Gi G l trng tm ABC, t
( )
8 3.KG 3.KD 0 KG KD = =
, , , , ,
K l trung im ca GD.
T
( )
8 , ta c:
( ) ( )
MA MB MC 3.MD 1 1 1 3 .MK 6.MK 9 = =
, , , , , ,
.
T
( ) ( )
7 , 9 k.MK 6.MK k 6 = =
, ,
.




Bi gii tham kho
Gi G
1
, G
2
ln lt l trng tm ca ANP v CMQ , O l mt im ty .
Ta c: ( )

1
2
OA ON OP 3OG
1
OC OM OQ 3OG
'
1
= 1
1
!
1
=
1
1+
, , , ,
, , , ,

Mt khc:
( ) ( ) ( )
( ) ( ) ( )
( )


1 1 1
OA ON OP OA OB OC OC OD OA OC OB OD
2 2 2
2
1 1 1
OC OM OQ OC OA OB OA OD OA OC OB OD
2 2 2
'
1
1
= =
1
1
1
!
1
1
= =
1
1
1+
, , , , , , , , , , , ,

, , , , , , , , , , , ,


T
( ) ( )
1 2
1 1 2
, 2 OG OG G G =
, ,
ANP v CMQ c cng trng tm (pcm).


BI TP P DNG
Bi 93. Bi 93. Bi 93. Bi 93. Cho ABC. Hy dng hnh v
a/ Tm im I sao cho: IA 2IB 0 =
, ,
.
b/ Tm im K sao cho: KA 2KB CB =
, , ,
.
c/ Tm im M sao cho: MA MB 2MC 0 =
, , , ,
.
d/ Tm im N sao cho: NA 2NB 0 =
, ,
.
e/ Tm im P sao cho: PA PB 2PC 0 =
, , , ,
.
f/ Tm im Q sao cho: QA QB QC BC =
, , , ,
.
g/ Tm im L sao cho: 2LA LB 3LC AB AC =
, , , , ,
.
h/ Tm im H sao cho: 2HA 3HB 3BC =
, , ,
.
i/ Tm im R sao cho: 2RA RB 2BC CA =
, , , ,
.
B
C
A
D
M
N
P
Q

Bi tp 12 Bi tp 12 Bi tp 12 Bi tp 12. Cho t gic li ABCD. Gi M, N, P, Q ln lt l trung im ca AB, BC, CD, DA.
Chng minh ANP v CMQ c cng trng tm.
www.MATHVN.com
www.DeThiThuDaiHoc.com
cng hc tp mn Ton 10 tp I Ths. L Vn on


"Cn c b thng minh" Page - 161 -
j/ Tm im S sao cho: SA SB SC BC =
, , , ,
.
k/ Tm im T sao cho: TA TB TC AB AC =
, , , , ,
.
l/ Tm im U sao cho: 3UA UB UC 0 =
, , , ,
.
m/ Tm im X sao cho: 3XA 2.XB XC 0 =
, , , ,
.
Bi 94. Bi 94. Bi 94. Bi 94. Cho hnh bnh hnh ABCD v ACEF.
a/ Dng cc im M, N sao cho EM BD =
, ,
, FN BD =
, ,
.
b/ Chng minh CD MN =
, ,
.
Bi 95. Bi 95. Bi 95. Bi 95. Cho ABC, hai im D v E.
1/ Chng minh rng nu OA OB OC 0 =
, , , ,
th O l trng tm ABC.
2/ Xc nh im M tha: (+ dng hnh)
a/ MA 2MB 0 =
, , ,
. b/ MA MB 2MC 0 =
, , , ,
.
c/ MA MB MD MD ME =
, , , , ,
. d/ 2MA 3MB MC 0 =
, , , ,

3/ Xc nh im N tha: (+ dng hnh)
a/ NA 3NB 0 =
, , ,
. b/ NA NB NC AB AC =
, , , , ,
.
c/ 2NA 3NB 4NC 0 =
, , , ,
. d/
( )
NA NB NC 3 ND NE 0 =
, , , , , ,
.
4/ Gi P l im xc nh bi 5PA 7PB PI 0 =
, , , ,
v G l trng tm ca ABC.
a/ Chng minh: GP 2AB =
, ,
.
b/ Vi AP BG Q = . Hy tnh t s
QA
QP
.
5/ Gi A' l im i xng ca A qua B, B' l im i xng ca B qua C v C' l im i
xng ca C qua A. Chng minh rng hai tam gic ABC v A'B'C' c cng trng tm J.
Bi 96. Bi 96. Bi 96. Bi 96. Cho ABC.
a/ Xc nh cc im D v E sao cho: AD AB AC =
, , ,
v BE BA BC =
, , ,
.
b/ Chng minh C l trung im ca on thng ED.
Bi 97. Bi 97. Bi 97. Bi 97. Cho hnh bnh ABCD.
a/ Hy xc nh cc im M, P sao cho AM DB , MP AB = =
, , , ,
.
b/ Chng minh rng P l trung im ca on thng DP.
Bi 98. Bi 98. Bi 98. Bi 98. Cho 4 im A, B, C, D. Chng minh rng: AB CD =
, ,
AB v CD c cng trung im.
Bi 99. Bi 99. Bi 99. Bi 99. Cho O, A, B, C l 4 im bt k trong mt phng. t OA u , OB v , OC w = = =
, , , , , ,
.
a/ Hy dng cc im D, E, F sao cho:
, OD u v w , OE u v w OF u v w = = =
, , , , , , , , , , , ,
.
www.MATHVN.com
www.DeThiThuDaiHoc.com
Ths. L Vn on Phn Hnh hc


Page - 162 - "All the flower of tomorrow are in the seeks of today"
b/ Chng minh rng A l trung im ca on thng DE v C l trung im ca on FD.
c/ Chng minh h thc: OD OE OF OA OB OC =
, , , , , ,
.
Bi 100. Bi 100. Bi 100. Bi 100. Cho hai im A v B.
a/ Dng cc im E, F sao cho
2 3
AE AB , AF AB
5 5
= =
, , , ,
.
b/ Chng minh hai on thng AB v EF c cng trung im.
Bi 101. Bi 101. Bi 101. Bi 101. Cho ABC.
a/ Chng minh rng vi mi im M , ta lun c: MA 2MB 3MC CA 2CB =
, , , , ,
.
b/ Hy dng im D sao cho: DA 2DB 3DC CA 2CB =
, , , , ,
.
Bi 102. Bi 102. Bi 102. Bi 102. Cho ABC.
a/ Dng im P sao cho 3PA 2PB PC 0 =
, , , ,
.
b/ Chng minh rng vct v 3MA 5MB 2MC =
, , , ,
khng ph thuc vo v tr im M.
Bi 103. Bi 103. Bi 103. Bi 103. Cho ABC c trng tm l G v ABC c trng tm l G'.
a/ Chng minh h thc: AA' BB' CC' 3GG' =
, , , ,
.
b/ Suy ra iu kin cn v hai tam gic c cng trng tm.
Bi 104. Bi 104. Bi 104. Bi 104. Cho hai tam gic ABC v A'B'C' c cng trng tm l G. Gi G
1
, G
2
, G
3
theo th t l trng
tm ca cc tam gic: BCA' , CAB', ABC'. Chng minh rng G cng l trng tm ca
G
1
G
2
G
3
.
Bi 105. Bi 105. Bi 105. Bi 105. Cho ABC. Gi A', B', C' ln lt l trung im ca cc cnh BC, CA v AB. Chng minh
rng hai tam gic ABC v A'B'C' c chung trng tm.
Bi 106. Bi 106. Bi 106. Bi 106. Cho hnh bnh hnh ABCD v mt im E thuc min trong ca hnh bnh hnh. Chng minh
rng hai ACE v BDE c cng trng tm. iu cn ng khi E nm ngoi hnh bnh
hnh khng ?
Bi 107. Bi 107. Bi 107. Bi 107. Cho ABC, cc ng cao AA', BB', CC'.
Chng minh rng nu AA' BB' CC' 0 =
, , , ,
th ABC l tam gic u.
Bi 108. Bi 108. Bi 108. Bi 108. Cho ABC v D l im bt k. DA, DB, DC theo th t ct BC, CA, AB ti A', B', C'.
Chng minh rng nu ta c: BA' A' C CB' B' A AC' C' B 0 =
, , , , , , ,
th D l trng tm
ABC.
Bi 109. Bi 109. Bi 109. Bi 109. Cho lc gic ABCDEF. Gi M, N, P, Q, R, S ln lt l trung im ca AB, BC, CD, DE, EF,
FA. Chng minh rng hai tam gic MPR v NQS c cng trng tm.
Bi 110. Bi 110. Bi 110. Bi 110. Cho ABC v s thc k. Gi A', B', C' ln lt c xc nh bi AA' kAB; BB' kBC = =
, , , ,

v CC' k.CA =
, ,
. Chng minh: ABC v A'B'Cv c cng trng tm.
HD: Gi G l trng tm ABC GA GB GC 0 =
, , , ,

www.MATHVN.com
www.DeThiThuDaiHoc.com
cng hc tp mn Ton 10 tp I Ths. L Vn on


"Cn c b thng minh" Page - 163 -

( )
( )
( )

AA' k.AB GA' GA k. GB GA
BB' k.BC GB' GB k. GC GB GA' GB' GC' 0 G G'
CC' k.CA GC' GC k. GA GC
'
1
1 = =
1
1
1
1
= = =
!
1
1
1
1
= =
1
1
+
, , , , , ,
, , , , , , , , , ,
, , , , , ,

Bi 111. Bi 111. Bi 111. Bi 111. Cho t gic ABCD.
1/ Tm im c nh I cc h thc sau tha mn.
a/ MA MB 2MC k.MI =
, , , ,
. b/ 2MA 3MB MD k.MI =
, , , ,
.
c/ MA MB 2MC k.MI =
, , , ,
. d/ MA 2MB 3MC 4MD k.MI =
, , , , ,
.
2/ Nu tn ti OA OB OC OD 0 =
, , , , ,
. Chng minh O xc nh duy nht.
3/ Nu ABCD l hnh bnh hnh. Vi mi M, hy tm k v im c nh I tha:
a/ MA MB MC 3MD k.MI =
, , , , ,
. b/ MA 2MB k.MI =
, , ,
.
c/ 2MA MB MC k.MI =
, , , ,
.
4/ Xc nh im S : SA SB SC SD 0 =
, , , , ,
.
Bi 112. Bi 112. Bi 112. Bi 112. Cho ABC, im M trong mt phng tha mn ng thc: MN MA 5MB MC =
, , , ,
.
a/ Chng minh rng: MN lun i qua mt im c nh khi M thay i.
b/ Gi P l trung im ca CN. Chng minh: MP lun i qua mt im c nh khi M thay i.
HD: a/
( )
MN 1 5 1 MI =
, ,
. b/
( )
1
MP MA 5MB 3MJ
2
= =
, , , ,
.
Bi 113. Bi 113. Bi 113. Bi 113. Cho t gic li ABCD, im M trong mt phng tha: MN MA 2MB 3MC 4MD =
, , , , ,
.
a/ Chng minh: MN lun i qua mt im c nh khi M thay i.
b/ Gi P l trng tm ABN. Chng minh: MP lun i qua im c nh khi M thay i.
Bi 114. Bi 114. Bi 114. Bi 114. Cho ABC. Cc im M, N, P ln lt thuc cc cnh BC, CA, AB.
Chng minh rng ABC v MNP c cng trng tm khi v ch khi:
BM CN AP
MC NA PB
= = .
Bi 115. Bi 115. Bi 115. Bi 115. Cho ABC, M l mt im trong tam gic. Gi H, I, K ln lt l hnh chiu ca M trn cc
cnh BC, CA, AB.
Chng minh rng M l trng tm ca ABC khi v ch khi:
2 2 2
a .MH b .MI c .MK 0 =
, , , ,
vi a, b, c l di 3 cnh BC, AC, AB.
Bi 116. Bi 116. Bi 116. Bi 116. Cho ABC. Gi O, H ln lt l tm ng trn ngoi tip v trc tm ca tam gic.
Chng minh rng: OH OA OB OC =
, , , ,

Bi 117. Bi 117. Bi 117. Bi 117. Cho lc gic ABCDEF c AB EF v hai tam gic ACE v BDF c cng trng tm.
Chng minh rng:
2 2 2
AB EF CD = .


www.MATHVN.com
www.DeThiThuDaiHoc.com
Ths. L Vn on Phn Hnh hc


Page - 164 - "All the flower of tomorrow are in the seeks of today"

















































Dng ton 4. Phn tch (tnh) vct Chng minh 3 im thng hng Song song

C CC C Tnh 1 vct theo cc vct khng cng phng (biu din vct).
t vn . Trong dng ton ny chng ta gii quyt i hi " Biu din vct thnh t hp
vct " da vo nh l: "Cho trc 2 vct a
,
, b
,

( )
a, b 0
, , ,
v khng cng
phng. Vi mi vct c
,
bao gi cng tm c 1 cp s thc , duy nht, sao
cho: c .a .b =
, , ,
".
Phng php gii. Ta c th chn 1 trong 2 hng gii sau:

Hng 1. T gi thit xc nh c tnh cht hnh hc, ri t khai trin vct cn biu
din bng phng php xen im, hiu 2 vct cng gc, qui tc hnh bnh hnh,
tnh cht trung im, trng tm,
Hng 2. T gi thit lp c mi quan h vct gia cc i tng, ri t khai trin
biu thc ny bng phng php xen im, hiu 2 vct cng gc, qui tc hnh
bnh hnh, tnh cht trung im, trng tm,
Lu
Lu 1. Ta dng xen im, hiu vct cng gc, vo vct c
,
da vo vct a
,
v b
,
.
Lu 2. Nu vct cn bin i (hay tnh) c th bin i bng nhiu cch khc nhau.
Lc ta n vic cng (tr, lp t s,) v theo v.
C CC C Chng minh ba im thng hng
chng minh 3 im A, B, C thng hng, ta chng minh:
( )
1 AB k.AC =
, ,

nhn c
( )
1 , ta la chn mt trong hai hng sau:
S dng cc qui tc bin i vct.
Xc nh (tnh) vct AB
,
v AC
,
thng qua mt t hp trung gian.
Lu
Da vo li bnh 3, ta c th suy lun c pht biu sau: " Cho ba im A, B, C. iu
kin cn v A, B, C thng hng l:
( )
MC .MA 1 .MB =
, , ,
vi im M ty
v s thc bt k". c bit khi: 0 1 th C AB . Kt qu trn cn c s
dng tm iu kin ca tham s k (hoc m) cho 3 im A, B, C thng hng.
Nu khng d nhn thy k trong biu thc AB k.AC =
, ,
, ta nn quy ng
biu thc phn tch vct AB
,
v AC
,
tm ra s k.
C CC C chng minh AB // DC ta cn chng minh AB k.DC =
, ,
.

www.MATHVN.com
www.DeThiThuDaiHoc.com
cng hc tp mn Ton 10 tp I Ths. L Vn on


"Cn c b thng minh" Page - 165 -
MT S BI TP MU





Bi gii tham kho
Theo gi thit th AB
1
CG l hnh bnh hnh.
a/ Tnh vct
1
CB
,
v
1
AB
,
theo AB, AC
, ,
.
Ta c
( )

1
2
CB GA AG AM 1
3
= = =
, , , ,

M M l trung im ca on BC nn:
( )
1
AM AB AC
2
=
, , ,

Thay vo
( )
1 ta c:
( ) 1
1
CB . AB AC
3
=
, , ,
.
Ta li c:
1
2
AB GC AC AG AC .AM
3
= = =
, , , , , ,
( )
2 1
AC . AB AC
3 2
=
, , ,
.

1
1 2
AB AB .AC
3 3
=
, , ,
.
b/ Tnh vct
1
MB
,
theo AB
,
, AC
,
.
Ta c:
( ) 1 1 1
1 2 1 5 1
MB AB AM AB AC AB AC MB AB AC
3 3 2 6 6
= = =
, , , , , ,
.






Bi gii tham kho
T gi thit ta c:
( ) ( )

2CI 3BI 5JB 2JC
2IC 3IB 1 & 5JB 2JC 2
IC IB JB JC
' '
1 1 = =
1 1
1 1
= =
! !
1 1

1 1
1 1 + +
, , , ,
, , , ,

a/ Tnh cc vct AI
,
, AJ
,
theo AB
,
v AC
,
.
Tnh AI
,
theo vct AB
,
v AC
,
.
T
( )
1 ta c :
( ) ( )
2IC 3IB 2 AC AI 3 AB AI = =
, , , , , ,

( )

3 2
5AI 3AB 2AC AI AB AC 3
5 5
= =
, , , , , ,

Tnh AJ
,
theo vct AB
,
v AC
,
.
C
B
B
1

A
G
M
B I C J
G
A
M
Bi tp 13 Bi tp 13 Bi tp 13 Bi tp 13. Cho ABC, gi G l trng tm ca tam gic v B
1
l im i xng ca B qua G. Gi
M l trung im ca BC. Hy biu din cc vct (tnh)
a/
1
CB
,
v
1
AB
,
theo AB, AC
, ,
. b/
1
MB
,
theo AB, AC
, ,
.
Bi tp 14 Bi tp 14 Bi tp 14 Bi tp 14. Cho ABC. Gi I l im trn cnh BC sao cho 2CI 3BI = v J l im trn BC
ko di sao cho 5JB 2JC = . Gi G l trng tm ABC.
a/ Tnh Tnh AI
,
, AJ
,
theo AB
,
v AC
,
. b/ Tnh AG
,
theo AI
,
v AJ
,
.
www.MATHVN.com
www.DeThiThuDaiHoc.com
Ths. L Vn on Phn Hnh hc


Page - 166 - "All the flower of tomorrow are in the seeks of today"
T
( )
2 ta c:
( ) ( )
5JB 2JC 5 AI AJ 2 AC AJ = =
, , , , , ,


( )

5 2
3AJ 5AB 2AC AJ AB AC 4
3 3
= =
, , , , , ,

b/ Tnh cc vct AG
,
theo AI
,
v AJ
,

Gi M l trung im ca BC, ta c:
( ) ( )
( )
2 2 1 1
AG AM . AB AC AB AC 5
3 3 2 3
= = =
, , , , , ,
.
Mt khc t h to bi
( ) ( )
3 , 4 , ta nhn c:
( )


5 3
AB AI AJ
8 8
6
25 9
AC AI AJ
16 16
'
1
1
=
1
1
1
!
1
1
=
1
1
1+

Thay
( )
6 vo
( )
5 ta nhn c:
35 1
AG AI AJ
48 16
=
, , ,
.










Bi gii tham kho
a/ Chng minh: 3AP 2AC 0 =
, ,
.
Ta c:
( )
( )
1
OP OA 3OP OA 3 OA AP OA 4OA 3AP 0 1
3
= = = =
, , , , , , , , , ,

Do O l trung im ca AC nn:
1
OA AC
2
=
, ,
.
Thay vo
( ) ( )
1 3AP 2AC 0 pcm =
, ,
.
b/ Chng minh: B, P, N thng hng.
Tnh BP
,
theo BA, BC
, ,
.
Ta c: BP BA AP =
, , ,
, m ( )

2 2
AP AC BP BA AC 2
3 3
= =
, , , , ,

Do ABCD l hnh bnh hnh nn AC AB AD AB BC = =
, , , , ,
, thay vo
( )
2 , ta c:
( )

2 2 1 2
BP BA AB BC BA BC 3BP BA 2BC 3
3 3 3 3
= = =
, , , , , , , , ,

Tnh BN
,
theo BA, BC
, ,
.
Ta c:
( ) ( )
1 1 1 1 1 1
BN BD BC BD BC BA BC BC BC BA
2 2 2 2 2 2
= = = =
, , , , , , , , , ,

( )
2BN BA 2BC 4 =
, , ,
.
A
B
D N C
P
M
O
Bi tp 15 Bi tp 15 Bi tp 15 Bi tp 15. Cho hnh bnh hnh ABCD, tm O. Gi M, N theo th t l trung im ca AB, CD
v P l im tha mn h thc:
1
OP OA
3
=
, ,
.
a/ Chng minh h thc: 3AP 2AC 0 =
, ,
.
b/ Chng minh 3 im B, P, N thng hng.
c/ Chng minh 3 ng AC, BD v MN ng quy.
www.MATHVN.com
www.DeThiThuDaiHoc.com
cng hc tp mn Ton 10 tp I Ths. L Vn on


"Cn c b thng minh" Page - 167 -
So snh: t ( ) ( )
2
3 , 4 3PB 2BN PB BN P, B, N
3
= =
, , , ,
thng hng (pcm).
c/ Chng minh: AC, BD v MN ng quy (ngha l cm AC BD O MN = M, O, N thng hng)
Do MO l ng trung bnh ABD
1
OM DA
2
=
, ,

ON l ng trung binh ACB
1 1
ON BC DA
2 2
= =
, , ,

Hay O l trung im ca MN O cng thuc 3 ng AC, BD, MN hay AC, BD v MN ng quy








Bi gii tham kho
a/ Chng minh M, N, J thng hng.
Ta c: 2JA 5JB 3JC 0 2JA 2JB 2JB 3JC 0 = =
, , , , , , , , ,


( ) ( )
( )
2 JA JB 3 JB JC 0 1 =
, , , , ,

Do M l trung im ca AB v J l im bt k nn: 2JM JA JB =
, , ,

N l trung im ca BC v J l im bt k nn: 2JN JB JC =
, , ,

Thay
( )
2 vo
( )
1 ta c:
3
4JM 6JN 0 JM JN J, M, N
2
= =
, , , ,
thng hng (pcm)
b/ Chng minh J l trung im ca BI.
Ta c:
( ) ( ) ( )
2JA 5JB 3JC 0 2 IA IJ 5 IB IJ 3 IC IJ 0 = =
, , , , , , , , , , ,

( )
( )
2IA 3IC 5IB 10IJ 0 2 =
, , , , ,

M 2IA 3IC 0 =
, , ,
, thay vo
( )
3 ta c: IC 2.IJ =
, ,
J l trung im ca BI (pcm).
c/ Tm k tha AE k.AB =
, ,
C, E, J thng hng. (ngha l tnh AE
,
theo AB
,
v so snh k).
Do I l trung im ca BI v C l im bt k
( )
( )

1
CJ CB CI 4
2
=
, , ,

M theo gi thit: 2IA 3IC 0 I =
, , ,
chia CA ra lm 5 on v I gn C
( )

2
CI CA 5
5
=
, ,

Thay
( )
5 vo
( )
4 ta c:
1 2 1 1
CJ CB CA CB CA
2 5 2 5
1

= =


( )
, , , , ,
.
CJ ct AB ti im E tha
1 1
EB EA 0
2 5
=
, , ,
( )
1 1 5 5
EA AB EA 0 AE AB k
2 5 7 7
= = =
, , , , ,
.
OM ON 0 =
, , ,

C
A
B
E
M N
I
J
( )
2
Bi tp 16 Bi tp 16 Bi tp 16 Bi tp 16. Cho ABC, trng tm G. Gi M, N ln lt theo th t l trung im ca hai on
thng AB v BC. Ly 2 im I, J sao cho: 2IA 3IC 0, 2JA 5JB 3JC 0 = =
, , , , , , ,
.
a/ Chng minh rng: M, N, J thng hng.
b/ Chng minh J l trung im ca BI.
c/ Gi E AB tha AE k.AB =
, ,
. Xc nh k C, E, J thng hng.
www.MATHVN.com
www.DeThiThuDaiHoc.com
Ths. L Vn on Phn Hnh hc


Page - 168 - "All the flower of tomorrow are in the seeks of today"















Bi gii tham kho
a/ Chng minh rng OE, OF
, ,
l 2 vct i nhau
Ta c:
( )

1 1 1 1
AE AB AO OE AB OE AB AO AB OA
k k k k
1
1 1 1 1
CF CD CO OF CD OF CD CO CD OC
k k k k
'
1
1
= = = =
1
1
1
!
1
1
= = = =
1
1
1+
, , , , , , , , , ,
, , , , , , , , ,

Do ABCD l hnh bnh hnh nn
( )

AB CD
2
OA OC
'
1
= 1
1
!
1
=
1
1+
, ,
, ,

T
( ) ( )
1 , 2 OE OF OE, OF =
, , , ,
l 2 vct i nhau.
OE OF 0 =
, , ,
O l trung im EF (pcm).
b/ Chng minh t gic AECF l hnh bnh hnh
T
1 1
AE AB; CF CD; AB DC
k k
= = =
, , , , , ,
ta suy ra AE FC AECF =
, ,
l hnh bnh hnh.










Bi gii tham kho
a/ Biu din cc vct AD; AE; DE
, , ,
theo cc vctAB; AC
, ,
.
Tnh AD
,
theo AB; AC
, ,
.
( )
( )

DB AB AD
k 1
DC AC AD AB AD k AC AD AD AC AB 1
k 1 k 1
DB k.DC
=
= = =

=
, , ,
, , , , , , , , , ,
, ,

O Li bnh Li bnh Li bnh Li bnh 5 55 5. Trong li gii cu c/ chng ta s dng kt qu: "Nu MN .MA .MB =
, , ,
th
ng thng MN s ct ng thng AB ti im I tha mn .IA .IB 0 =
, , ,
".
Bi tp 17 Bi tp 17 Bi tp 17 Bi tp 17. Cho hnh bnh hnh ABCD, c tm l O v E, F c xc nh bi cc h thc
( )

1 1
AE AB ; CF CD, k 0
k k
= =
, , , ,
.
a/ Chng minh rng OE ; OF
, ,
l 2 vct i nhau. Chng minh O, E, F thng hng
v O l trung im ca EF.
b/ Chng minh t gic AECF l hnh bnh hnh.
Bi tp 18 Bi tp 18 Bi tp 18 Bi tp 18. Cho ABC v hai im D, E tha mn h thc:
( )

1
DB k.DC, EB EC, k 1
k
= =
, , , ,

a/ Biu din cc vct AD; AE; DE
, , ,
theo cc vct AB; AC
, ,
.
b/ Chng minh ABC v ADE c cng trng tm
c/ im F, I tha FA k.FB; IC k.IA = =
, , , ,
. Chng minh: AD BI CF 0 =
, , , ,
.
www.MATHVN.com
www.DeThiThuDaiHoc.com
cng hc tp mn Ton 10 tp I Ths. L Vn on


"Cn c b thng minh" Page - 169 -
Tnh AE
,
theo AB; AC
, ,

( )
( )

EB AB AE
1 k
EC AC AE AB AE k AC AE AE AC AB 2
k 1 k 1
1
EB EC
k
'
1
1
1 =
1
1
1
1
= = =
!
1

1
1
1
=
1
1
1+
, , ,
, , , , , , , , , ,
, ,

Tnh DE
,
theo AB; AC
, ,

Ta c
( )
DE AE AD 3 =
, , ,

Thay
( ) ( )
1 , 2 vo
( )
3 v rt gn, ta c:
( )
k 1
DE AB AC
k 1

, , ,
.
b/ Chng minh ABC v ADE c cng trng tm
T DB k.DC =
, ,
ta bin i c:
k
DB .BC
1 k
=

, ,

T
1
EB .EC
k
=
, ,
ta bin i c:
k
EC .CB
1 k
=

, ,

Ta c: GB GD DG ; GC GE EC = =
, , , , , ,

GA GB GC GA GD GE EC DB =
, , , , , , , ,

V GA GB GC 0 =
, , , ,
v EC DB 0 =
, , ,
GA GD GE 0 =
, , , ,

G l trng tm ca ADE
( )
pcm .
c/ im F, I tha mn h thc: FA k.FB ; IC k.IA = =
, , , ,
. Chng minh:

AD BI CF 0 =
, , , ,

Ta c:
1
IC k.IA AI .AC
k 1
= =

, , , ,

M
1
BI BA AI BI .AC AB
k 1
= =

, , , , , ,

T gi thit:
k
FA k.FB AF .AB
k 1
= =

, , , ,
v ta tnh ra
k
CF .AB AC
k 1
=

, , ,

Nn
k 1 k 1
AD BI CF AC AC AC .AB AB AB
k 1 k 1 k 1 k 1

=

, , , , , , , , ,

( )
AD BI CF 0 pcm =
, , , ,
.
EC DB 0 =
, , ,

www.MATHVN.com
www.DeThiThuDaiHoc.com
Ths. L Vn on Phn Hnh hc


Page - 170 - "All the flower of tomorrow are in the seeks of today"
BI TP P DNG
Biu din vct Chng minh 3 im thng hng ng qui ng qua
im
Bi 118. Bi 118. Bi 118. Bi 118. Cho ABC c M, D ln lt l trung im ca AB, BC v N l im trn cnh AC sao cho
1
AN .NC
2
=
, ,
. Gi K l trung im ca MN. Hy tnh cc vct AK, KD
, ,
theo AB , AC
, ,
.
HD :
1 1 1 1
AK .AB .AC ; KD .AB .AC
4 6 4 3
= =
, , , , , ,
.
Bi 119. Bi 119. Bi 119. Bi 119. Cho ABC. Trn hai cnh AB v AC ly hai im D v E sao cho AD 2DB ; CE 3EA = =
, , , ,
.
Gi M, I ln lt l trung im ca DE v BC. Hy tnh vct AM ; MI
, ,
theo AB , AC
, ,
.
HD :
1 1 1 3
AM .AB .AC ; MI .AB .AC
3 8 6 8
= =
, , , , , ,
.
Bi 120. Bi 120. Bi 120. Bi 120. Cho 4 im A, B, C, D tha: 2AB 3AC 5AD =
, , ,
. Chng minh: B, C, D thng hng.
HD: Tnh BC ; BD
, ,
theo AB , AC
, ,
.
Bi 121. Bi 121. Bi 121. Bi 121. Cho ABC, ly im M, N, P sao cho MB 3MC ; NA 3NC 0 ; PA PB 0 = = =
, , , , , , , ,
.
a/ Tnh PM ; PN
, ,
theo AB ; AC
, ,
.
b/ Chng minh ba im: M, N, P thng hng.
Bi 122. Bi 122. Bi 122. Bi 122. Cho ABC c hai ng trung tuyn BN, CP. Hy biu th cc vct AB ; BC; CA
, , ,
theo cc
vct BN ; CP
, ,
.
HD:
2 2 2 4 4 2
BC .CP .BN ; CA .BN .CP ; AB .BN .CP
3 3 3 3 3 3
= = =
, , , , , , , , ,
.
Bi 123. Bi 123. Bi 123. Bi 123. Cho ABC. Gi I, J nm trn cnh BC v BC ko di sao cho 2CI 3BI , 5JB 2JC = = . Gi
G l trng tm ca tam gic.
a/ Tnh AI ; AJ
, ,
theo AB ; AC
, ,
. b/ Tnh AG
,
theo AB ; AC
, ,
.
HD:
3 2 5 2
AI .AB .AC ; AJ .AB .AC
5 5 3 3
= =
, , , , , ,
.
Bi 124. Bi 124. Bi 124. Bi 124. Cho ABC c G l trng tm tam gic v I l im i xng ca B qua G. M l trung im ca
BC. Hy tnh AI ; CI ; MI
, , ,
theo AB ; AC
, ,
.
HD:
1 2 1 1 5 1
AI .AB .AC ; CI .AB .AC ; MI .AB .AC
3 3 3 2 6 6
= = =
, , , , , , , , ,
.
Bi 125. Bi 125. Bi 125. Bi 125. Cho ABC c trng tm l G v cc ng trung tuyn AM, BP. Gi G' l im i xng vi
im G qua P.
a/ Hy biu din cc vct AG' ; CG'
, ,
theo AB ; AC
, ,
.
b/ Chng minh h thc: 5AC 6AB 6MG' =
, , ,
.
www.MATHVN.com
www.DeThiThuDaiHoc.com
cng hc tp mn Ton 10 tp I Ths. L Vn on


"Cn c b thng minh" Page - 171 -
HD:
2 1 1 1
AG' .AC .AB ; CG' .AB AC
3 3 3 3
= =
, , , , , ,
. Tnh MG'
,
theo AB ; AC
, ,
.
Bi 126. Bi 126. Bi 126. Bi 126. Cho hnh bnh hnh ABCD. Gi M, N theo th t l trung im ca cc cnh BC, CD. Hy
biu din cc vct BC ; CD
, ,
theo cc vct AM; AN
, ,
.
HD:
2 4 4 2
BC AM AN ; CD AM AN
3 3 3 3
= =
, , , , , ,
.
Bi 127. Bi 127. Bi 127. Bi 127. Cho t gic ABCD c M, N theo th t l trung im ca cc cnh AD, BC. Hy biu din
vct MN
,
theo AB ; DC
, ,
v theo AC; DB
, ,
.
HD:
1 1
MN AB DC
2 2
=
, , ,
v
1 1
MN AC DB
2 2
=
, , ,
.
Bi 128. Bi 128. Bi 128. Bi 128. Cho ABC. Gi I l im i xng ca trng tm G qua B.
a/ Chng minh: IA 5IB IC 0 =
, , , ,
.
b/ t AG a ; AI b = =
, , , ,
. Tnh AB ; AC
, ,
theo a ; b
, ,
.
HD:
( )
1
AB a b
2
=
, , ,
v
5 1
AC a b
2 2
=
, , ,
.
Bi 129. Bi 129. Bi 129. Bi 129. Cho ABC. Gi M, N, P ln lt l trung im ca BC, CA, AB. Tnh cc vct AB, BC, CA
, , ,

theo cc vct BN, CP
, ,
.
Bi 130. Bi 130. Bi 130. Bi 130. Cho ABC. Gi I l im trn cnh BC ko di sao cho IB 3IC = .
a/ Tnh AI
,
theo AB ; AC
, ,
.
b/ Gi J v K ln lt l cc im thuc cnh AC, AB sao cho JA 2JC = v KB 3KA = .
Tnh JK
,
theo AB ; AC
, ,
.
c/ Tnh BC
,
theo AI
,
v JK
,
.
Bi 131. Bi 131. Bi 131. Bi 131. Cho hnh bnh hnh ABCD c tm l O. Hy tnh cc vct sau theo vct AB
,
v AD
,
.
a/ AI
,
vi I l trung im ca BO
,
. b/ BG
,
vi G l trng tm OCD.
Bi 132. Bi 132. Bi 132. Bi 132. Cho lc gic u ABCDEF. Hy biu din cc vct AC ; AD; AF ; EF
, , , ,
theo cc vct
v AB ; u AE = =
, , , ,
.
Bi 133. Bi 133. Bi 133. Bi 133. Cho ABC v im D tha h thc 3DB 2DC 0 =
, , ,
.
a/ Hy biu din vct AD
,
theo cc vct AB ; AC
, ,
ri nu cch dng D.
b/ Xc nh im E tha mn h thc EA 3EB 2AC 0 =
, , , ,
.
HD: AD 3AB 2AC =
, , ,
, E nm trn ng thng BC v c EM BC = .
Bi 134. Bi 134. Bi 134. Bi 134. Cho ABC. Cc im D, E, G c xc nh bi h thc: 2AD AB =
, ,
, AE 2CE =
, ,
,
2GD GC =
, ,
.
a/ Chng minh BE // CD.
b/ Gi M l trung im ca cnh BC. Chng minh: A, G, M thng hng.
www.MATHVN.com
www.DeThiThuDaiHoc.com
Ths. L Vn on Phn Hnh hc


Page - 172 - "All the flower of tomorrow are in the seeks of today"
HD: a/ Tnh BE; CD
, ,
theo BA
,
, AC
,
b/
2
GA MA
3
=
, ,
.
Bi 135. Bi 135. Bi 135. Bi 135. Cho hnh bnh hnh ABCD v 2 im E, F tha cc h thc: 2CE EB 0 =
, , ,
, 3DF BD 0 =
, , ,
.
a/ Chng minh ba im A, E, F thng hng.
b/ Xc nh v tr im M h thc sau c tha mn: 2AM 3AF 0 =
, , ,
.
Bi 136. Bi 136. Bi 136. Bi 136. Cho ABC. Gi M l trung im ca cnh BC v P, Q l hai im c xc nh bi h thc:
BP BC 2AB =
, , ,
, CQ k.AC BC =
, , ,
, trong k l mt s thc.
a/ Biu din cc vct AP
,
, AQ
,
theo cc vct AB ; AC
, ,
.
b/ Xc nh gi tr ca k A, P, Q thng hng.
c/ Tnh gi tr ca k ng thng PQ i qua im M.
HD: AP 2AB AC =
, , ,
, AQ k.AC AB =
, , ,
,
1
k
2
= ,
2
k
5
= .
Bi 137. Bi 137. Bi 137. Bi 137. Cho ABC.
a/ Dng cc im E, F, G tha cc h thc: BE 3AB, BF 3AC, BG BE BF = = =
, , , , , , ,
.
b/ Chng minh im G nm trn ng thng BC.
HD: BG 3BC B, G, C =
, ,
thng hng.
Bi 138. Bi 138. Bi 138. Bi 138. Cho ABC.
a/ Dng cc im E, F, M, N sao cho cc ng thc sau c tha:
2
AE AB
3
=
, ,
,
1
BF AB
3
=
, ,
, EM 2BC =
, ,
, FN 4BC =
, ,
.
b/ Cc im A, M, N c thng hng khng ? Ti sao ?
HD:
1
AM AN
2
=
, ,
thng hng.
Bi 139. Bi 139. Bi 139. Bi 139. Cho ABC v hai im I, F c xc nh bi: IA 3IC 0 =
, ,
, FA 2FB 3FC 0 =
, , ,
.
Chng minh rng ba im I, F, B thng hng.
HD:
1
FB FI
2
=
, ,
I, F, B thng hng.
Bi 140. Bi 140. Bi 140. Bi 140. Cho ABC.
a/ Dng cc im E v D sao cho: BE 2AB 2AC =
, , ,
, 5AD 3AB 2AC =
, , ,
.
b/ Chng minh cc im A, D, E thng hng.
HD: 5AD AE =
, ,
A, D, E thng hng.
Bi 141. Bi 141. Bi 141. Bi 141. Cho EDF.
a/ Dng im H sao cho EH 4ED 3EF =
, , ,
.
b/ Chng minh im H nm trn DF.
HD: DH 3FD D, H, F =
, ,
thng hng H nm trn DF hay H DF .
Bi 142. Bi 142. Bi 142. Bi 142. Cho ABC c I l trung im ca trung tuyn AM v D l im tha h thc: 3AD AC =
, ,
.
a/ Biu din vct BD
,
, BI
,
theo AB ; AC
, ,
.
b/ Chng minh ba im B, I, D thng hng.
www.MATHVN.com
www.DeThiThuDaiHoc.com
cng hc tp mn Ton 10 tp I Ths. L Vn on


"Cn c b thng minh" Page - 173 -
HD:
1
BD AC AB
3
=
, , ,
,
1 3
BI AC AB
4 4
=
, , ,
,
3
BI BD B, I, D
4
=
, ,
thng hng.
Bi 143. Bi 143. Bi 143. Bi 143. Cho hnh bnh hnh ABCD.
a/ Dng cc im E, F sao cho: BE 2AB =
, ,
, AF 3AD =
, ,
.
b/ Dng im G sao cho t gic AEGF l hnh bnh hnh.
c/ Chng t 3 im A, C, G thng hng.
HD: AG 3AC A, C, G =
, ,
thng hng.
Bi 144. Bi 144. Bi 144. Bi 144. Cho hnh bnh hnh ABCD. Gi I l trung im ca AB v I l im tha h thc: 3IE ID =
, ,
.
Chng minh ba im A, C, E thng hng.
HD: AC 3AE A, C, E =
, ,
thng hng.
Bi 145. Bi 145. Bi 145. Bi 145. Cho ABC.
a/ Dng cc im K, L sao cho: KA 2KB 2KC 0 =
, , ,
, 2LB 3LC 0 =
, ,
.
b/ Chng minh ba im A, K, L thng hng.
HD: Biu din AK
,
, AL
,
theo AB ; AC
, ,
ri so snh.
Bi 146. Bi 146. Bi 146. Bi 146. Cho ABC. Gi M l trung im ca cnh AB, N v P l hai im tha mn h thc:
NA 2NC 0 =
, , ,
, PB 2PC 0 =
, , ,
. Chng minh ba im M, N, P thng hng.
HD: Tnh MN
,
, MP
,
theo AB ; AC
, ,
ri so snh.
Bi 147. Bi 147. Bi 147. Bi 147. Cho ABC. Hai im M, N c xc nh bi: 3MA 4MB 0 =
, , ,
, NB 3NC 0 =
, , ,
. Chng
minh MN i qua trng tm ABC.
HD: Tnh GM
,
, GN
,
2
GM GN G, M, N
7
=
, ,
thng hng pcm.
Bi 148. Bi 148. Bi 148. Bi 148. Cho ABC.
a/ Dng cc im D, E tha cc h thc:
3
AD AB
2
=
, ,
,
3
DE BC
2
=
, ,
.
b/ Chng minh ba im A, C, E thng hng.
HD:
3
AE AC A, C, E
2
=
, ,
thng hng.
Bi 149. Bi 149. Bi 149. Bi 149. Cho hnh bnh hnh ABCD. Gi I l trung im ca cnh BC v E l im xc nh bi
2
AE AC
3
=
, ,
. Chng minh ba im D, E, I thng hng.
HD: Tnh DE
,
v DI
,
theo DA
,
, DC
,
.
Bi 150. Bi 150. Bi 150. Bi 150. Cho ABC c trung tuyn AD v M l trung im AD. im N c ly trn AC sao cho
3AN AC =
, ,
. Chng minh ba im B, M, N thng hng.
HD: Tnh BM
,
v BN
,
theo AB ; AC
, ,
ri so snh.
Bi 151. Bi 151. Bi 151. Bi 151. Cho ABC c M l trung im BC v O l trung im ca AM. Trn BC ly im I sao cho
2
AI AB
3
=
, ,
,
2
AJ AC
5
=
, ,
. Chng minh ba im I, J, O thng hng.
HD: Tnh IJ
,
, IO
,
theo theo AB ; AC
, ,
ri so snh.
www.MATHVN.com
www.DeThiThuDaiHoc.com
Ths. L Vn on Phn Hnh hc


Page - 174 - "All the flower of tomorrow are in the seeks of today"
Bi 152. Bi 152. Bi 152. Bi 152. Cho t gic ABCD. Gi M, N l hai im di ng trn AB, CD sao cho
MA ND
MB NC
= v I, J ln
lt l trung im ca AD, BC.
a/ Tnh IJ
,
theo AB
,
v DC
,
.
b/ Chng minh trung im P ca MN nm trn ng thng IJ.
Bi 153. Bi 153. Bi 153. Bi 153. Cho ABC. Gi M, N, P l cc im tha: MA MB =
, ,
, BC 3BN =
, ,
, 4AP 3AC =
, ,
.
a/ Tnh AN
,
, MP
,
theo AB
,
v DC
,
.
b/ Chng minh rng M, I, P thng hng vi im I tha: 16AI 9AN =
, ,
.
Bi 154. Bi 154. Bi 154. Bi 154. Cho ABC vi I, J, K c xc nh bi: IB m.IC =
, ,
, JC n.JA =
, ,
, KA p.KB =
, ,
.
a/ Tnh IJ
,
v IK
,
theo m, n, p, AB
,
v AC
,
.
b/ Tm mi lin h gia m, n, p I, J, K thng hng.
Bi 155. Bi 155. Bi 155. Bi 155. Cho t gic ABCD vi I, J l trung im ca AB v CD. Gi im M, N tha mn cc h thc:
MA k.MC 0 =
, , ,
, NB k.ND 0 =
, , ,

( )
k 1 v O l trung im ca MN.
a/ Chng minh O, I, J thng hng
b/ Chng minh O cng l trung im ca RS vi R v S c xc nh bi cc h thc:
RA RD =
, ,
, SB k.SC =
, ,
.
Bi 156. Bi 156. Bi 156. Bi 156. Cho ABC c J chia BC
,
theo t s bng 3, N chia AC
,
theo t s bng 1, K chia BA
,
theo
t s bng 3.
a/ Chng minh rng: K, N, J thng hng.
b/ Tnh t s
IB
IN
,
AJ
AI
vi I AJ BN = .
Bi 157. Bi 157. Bi 157. Bi 157. Cho hnh bnh hnh ABCD c tm O. M, N trn cnh AB, CD tha 3AM AB, 2CN CD = = .
a/ Tnh AN
,
theo AB
,
v AC
,
.
b/ Gi G l trng tm BMN. Chng minh rng
11 1
AG BA BC
8 3
=
, , ,
.
c/ Gi I tha mn ng thc 11BI 6BC =
, ,
. Chng minh rng A, I, G thng hng.
d/ Tm im M tha: MA MB MC MD 4AB =
, , , , ,
.
Bi 158. Bi 158. Bi 158. Bi 158. Cho ABC. Ly im I tha 3AI AB =
, ,
, 4AJ 3AC =
, ,
v M l giao im ca ng thng IJ
v BC. t BM m.MC =
, ,
.
a/ Chng minh rng: 12IJ 9BC 6BA =
, , ,
.
b/ Tnh IM
,
theo BA
,
, BC
,
.
c/ Tm gi tr m ?
Bi 159. Bi 159. Bi 159. Bi 159. Cho ABC. Gi P, Q, R ln lt l cc im tha cc ng thc: 3PB 4PC 0 =
, ,
,
AQ 2QC =
, ,
, k.RA RB =
, ,
vi
( )
k 1 .
a/ Chng minh rng: 21PQ 2BC 7BA =
, , ,
.
b/ Chng minh rng:
k 4
RP BA BC
1 k 7
=

, , ,
.
c/ Tm k sao cho P, Q, R thng hng.
www.MATHVN.com
www.DeThiThuDaiHoc.com
cng hc tp mn Ton 10 tp I Ths. L Vn on


"Cn c b thng minh" Page - 175 -
Bi 160. Bi 160. Bi 160. Bi 160. Cho hnh bnh hnh ABCD.
a/ Gi I, F, K ln lt l cc im tha cc h thc AI .AB =
, ,
, AF .AC =
, ,
, AK .AD =
, ,
.
Chng minh iu kin cn v I, F, K thng hng l
( )

1 1 1
, , 0 =

.
b/ Gi M, N l hai im ln lt trn on AB v CD sao cho
AM 1
AB 3
= ,
CN 1
CD 2
= . Gi G l
trng tm ca BMN. Tnh AN
,
, AG
,
theo AB
,
v AC
,
. Gi H l im xc nh bi
BH k.BC =
, ,
. Tnh AH
,
theo AB
,
, AC
,
v k. Tm k ng thng AH qua im G.
Bi 161. Bi 161. Bi 161. Bi 161. Cho t gic ABCD. Ly cc im M, N theo th t thuc AB v CD sao cho AM k.AB =
, ,
v
DN k.DC =
, ,
.
a/ Chng minh rng:
( )
MN 1 k AD k.BC =
, , ,
.
b/ Gi E, F, I ln lt theo th t thuc cc AD, BC v MN sao cho AE .AD = l
, ,
, BF .BC = l
, ,

v MI .MN = l
, ,
. Chng minh rng: E, F, I thng hng.
Bi 162. Bi 162. Bi 162. Bi 162. Cho ABC. Gi O, G, H ln lt theo th t l tm ng trn ngoi tip, trng tm, trc tm
ca ABC. Chng minh rng: O, H, G thng hng.
Chng minh song song
Bi 163. Bi 163. Bi 163. Bi 163. Cho hnh thang ABCD, y AB. Gi M, N theo th t l cc trung im ca AD, BC.
a/ Chng minh:
( )
1
MN AB AC
2
=
, , ,
.
b/ Chng minh: MN // DC.
HD: b/ V AB // DC
k
AB k.DC MN DC
2
= =
, , , ,
.
Bi 164. Bi 164. Bi 164. Bi 164. Cho ABC c trng tm G. Gi M l trung im ca cnh BC v I l im tha mn h thc
4CI AC 0 =
, , ,
. Chng minh: MP // BG.
HD: Hy chng minh
3
MI MG
4
=
, ,
.
Bi 165. Bi 165. Bi 165. Bi 165. Cho t gic ABCD. Gi E v F ln lt l trng tm ca ABD v BCD.
Chng minh: EF // AC.
HD: Gi I l trung im BD, hy chng minh
1
EF CA
3
=
, ,
.
Bi 166. Bi 166. Bi 166. Bi 166. Cho ABC c M l trung im ca cnh BC. Cc im D, E tha mn cc ng thc
BD 4BA =
, ,
, AE 3AC =
, ,
. Chng minh: DE // AM.
HD: Hy chng minh
1
AM DE
6
=
, ,
.
Bi 167. Bi 167. Bi 167. Bi 167. Cho ABC. Dng cc im M, N sao cho:
2
AM AB
3
=
, ,
,
2
AN AC
3
=
, ,
.
Chng minh: MN // BC.
HD: Hy chng minh
2
MN BC
3
=
, ,
.
www.MATHVN.com
www.DeThiThuDaiHoc.com
Ths. L Vn on Phn Hnh hc


Page - 176 - "All the flower of tomorrow are in the seeks of today"
Bi 168. Bi 168. Bi 168. Bi 168. Cho ABC. Dng cc im I, J sao cho:
1
AI AB
3
=
, ,
, AJ 3AC =
, ,
. Chng minh: IC // BJ.
HD: Hy chng minh
1
CI BJ
3
=
, ,
.
Bi 169. Bi 169. Bi 169. Bi 169. Cho hnh bnh hnh ABCD. Gi I, J ln lt l trung im ca AB, CD. Dng cc im E, F
tha mn:
1
DE DI
4
=
, ,
,
1
BF BJ
4
=
, ,
. Chng minh: EF // CE.
HD: Hy chng minh
AF AB BF
AF CE
CE CD BE
'
1
1 =
1
=
!
1
=
1
1
+
, , ,
, ,
, , , .
Bi 170. Bi 170. Bi 170. Bi 170. Cho t gic ABCD. Qua nh A k ng thng song song vi cnh BC, ng ny ct ng
cho BD ti im E. ng thng qua B song song vi cnh AD, ct ng cho AC ti im
F. Chng minh rng: EF // CD.
HD: Hai vct OC, OA OC m.OA =
, , , ,
.
Tng t
( )
OD n.OB DC OC OD m.OA n.OB = = =
, , , , , , ,
.
V AE // BC nn
OB OC 1
m OE OB
m
OE OA
= = =
, ,
, ,
, , , tng t:
1
OF OA
n
=
, ,
.
Ta c:
( )
1 1 1
FE OE OF OB OA nOB nOA
m n m n
= = =

, , , , , , ,
.
( )
//
1 1
FE mOA nOB FE DC FE DC
m.n m.n
= =
, , , , ,
.
Bi 171. Bi 171. Bi 171. Bi 171. Cho ABC. Cc im D, E, G c xc nh bi h thc: 2AD AB =
, ,
, AE 2CE =
, ,
,
2GD GC =
, ,
.
a/ Chng minh BE // CD.
b/ Gi M l trung im ca cnh BC. Chng minh: A, G, M thng hng.
Bi 172. Bi 172. Bi 172. Bi 172. Cho ABC, M l trung im ca cnh AB v D, E, F theo th t c xc nh bi cc h
thc: 3DB 2DC 0 ; EA 3EB 2EC 0 ; 5AF 2AC 0 = = =
, , , , , , , , , ,
.
a/ Chng minh rng: EM // BC.
b/ Chng minh rng: ba im A, D, E thng hng.
c/ Chng minh rng: ba ng thng AD, BC, MF ng qui ti mt im.

www.MATHVN.com
www.DeThiThuDaiHoc.com
cng hc tp mn Ton 10 tp I Ths. L Vn on


"Cn c b thng minh" Page - 177 -






































Tm mun ( di)
Bi 173. Bi 173. Bi 173. Bi 173. Cho ABC vung ti A c
( )
AB AC 2 cm = = . Tnh AB AC
, ,
?
Bi 174. Bi 174. Bi 174. Bi 174. Cho ABC u cnh a, trng tm G. Hy tnh
a/ AB AC
, ,
. b/ AB AC
, ,
. c/ GB GC
, ,
.
Bi 175. Bi 175. Bi 175. Bi 175. Cho hnh ch nht ABCD c
( ) ( )
AB 5 cm , BC 10 cm = = . Tnh AB AC AD
, , ,
?
Bi 176. Bi 176. Bi 176. Bi 176. Cho ABC vung ti A c

( )
0
B 60 , BC 2 cm = = . Tm AB , AC , AB AC , AC AB
, , , , , ,
?
Dng ton 5. Tm mun ( di) ca vct Qu tch im v im c nh
C CC C Tm mun ( di) ca vct: u v w
, , , ,

Bc 1. Ta bin i (rt gn) biu thc u v w =
, , , , ,
bng cc phng php
hc sao cho vct
,
n gin nht.
Bc 2. Tnh di (mun) ca
,
da vo tnh cht ca hnh cho,
C CC C Qu tch im:
t bi ton: Tm qu tch im M tha mn iu kin cho trc ?
Phng php gii: S dng cc phng php bin i a v cc trng hp sau
Trng hp 1: Nu MA MB =
, ,
vi A, B cho trc (c nh) th im
M thuc ng trung trc ca on thng AB.
Trng hp 2: Nu MC k. AB =
, ,
vi A, B, C cho trc (c nh)
th im M thuc ng trn tm C, bn knh k.AB.
Trng hp 3: Nu MA k.BC =
, ,
vi A, B, C cho trc (c nh) th:

+ Vi k R th im M thuc ng thng qua A song song vi BC.


+ Vi k

R th im M thuc na ng thng qua A song song BC theo hng BC.




A B
M
C
k.A
M
B
C
M A
C B
M A
C B
A M
+ Nu k

R th im M thuc na ng thng qua A song song BC
ngc hng BC.
www.MATHVN.com
www.DeThiThuDaiHoc.com
Ths. L Vn on Phn Hnh hc


Page - 178 - "All the flower of tomorrow are in the seeks of today"
Bi 177. Bi 177. Bi 177. Bi 177. Cho ABC vung ti B c


0
A 30 , AB a = =
,
. Gi I l trung im ca AC. Hy tnh
AC , AI , AB AC , BC
, , , , ,
?
Bi 178. Bi 178. Bi 178. Bi 178. Cho hnh thang vung ti A v D c


0
AB AD a, C 45 = = = . Tnh CD , BD
, ,
?
Bi 179. Bi 179. Bi 179. Bi 179. Cho ABC vung ti A c
( ) ( )
BC 15 cm , AC 5 cm = = . Tnh CA BC , BC BA
, , , ,
?
Bi 180. Bi 180. Bi 180. Bi 180. Cho hnh thang cn ABCD c y nh AD v ng cao cng bng
( )
2 cm v
0
B 45 = . Tnh
AD DB , CB AD AC , AB AD CB
, , , , , , , ,
?
Bi 181. Bi 181. Bi 181. Bi 181. Cho hnh bnh hnh ABCD tm O. Chng minh:
a/ AC BA AD =
, , ,
.
b/ AB AD AC =
, ,
.
c/ Nu AB AD OB CD =
, , , ,
th ABCD l hnh ch nht.
Bi 182. Bi 182. Bi 182. Bi 182. Cho hnh vung ABCD cnh a, ly im M ty . Chng minh cc vct sau y khng i v
tnh di ca chng ?
a/ u 3MA MB MC MD =
, , , , ,
. b/ v 4MA 3MB MC 2MD =
, , , , ,
.
c/ x 2MA MB MC 2MD =
, , , , ,
. d/ y 3MA MB 2MC =
, , , ,
.
Qu tch im
Bi 183. Bi 183. Bi 183. Bi 183. Cho 2 im c nh A, B. Tm tp hp cc im M sao cho:
a/ MA MB MA MB =
, , , ,
. b/ 2MA MB MA 2MB =
, , , ,
.
HD: a/ ng trn ng knh AB b/ Trung trc ca AB.
Bi 184. Bi 184. Bi 184. Bi 184. Cho ABC. Tm tp hp cc im M sao cho:
a/
3
MA MB MC MB MC
2
=
, , , , ,
. b/ MA BC MA MB =
, , , ,
.
c/ 2MA MB 4MB MC =
, , , ,
d/ 4MA MB MC 2MA MB MC =
, , , , , ,
.
HD: a/ Trung trc ca IG (I l trung im ca BC, G l trng tm ABC).
b/ Dng hnh bnh hnh ABCD. Tp hp l ng trn tm D, bn knh BA.
Bi 185. Bi 185. Bi 185. Bi 185. Cho ABC.
a/ Xc nh im I sao cho: 3IA 2IB IC 0 =
, , ,
,
.
b/ Chng minh rng ng thng ni 2 im M, N xc nh bi h thc:
MN 2MA 2MB MC =
, , , ,
lun i qua mt im c nh.
c/ Tm tp hp cc im H sao cho: 3HA 2HB HC HA HB =
, , , , ,
.
d/ Tm tp hp cc im K sao cho: 2 KA KB KC 3 KB KC =
, , , , ,
.
Bi 186. Bi 186. Bi 186. Bi 186. Cho ABC.
a/ Xc nh im I sao cho: IA 3IB 2IC 0 =
, , ,
,
.
b/ Xc nh im D sao cho: 3DB 2DC 0 =
, ,
,
.
c/ Chng minh 3 im A, I, D thng hng.
www.MATHVN.com
www.DeThiThuDaiHoc.com
cng hc tp mn Ton 10 tp I Ths. L Vn on


"Cn c b thng minh" Page - 179 -
d/ Tm tp hp cc im M sao cho: MA 3MB 2MC 2MA MB MC =
, , , , , ,
.
Bi 187. Bi 187. Bi 187. Bi 187. i hc M a Cht nm 1999 2000
Cho ABC, M l im ty trong mt phng.
a/ Chng minh: v 3MA 5MB 2MC =
, , , ,
khng i.
b/ Tm tp hp im M tha mn: 3MA 2MB 2MC MB MC =
, , , , ,
.
HD: a/ Chng minh: v 3BA 2BC =
, , ,
. b/ M thuc ng trn tm I, bn knh
1
BC
3
.
Bi 188. Bi 188. Bi 188. Bi 188. Cho ABC, tm tp hp nhng im M tha mn:
a/
( )
kMA MB kMC, k =
, , ,
R . b/
( )
MA 1 k MB kMC 0 =
, , , ,
.
HD: a/ ng thng qua B, // AC. b/ ng trung bnh // AC.
Bi 189. Bi 189. Bi 189. Bi 189. Cho ABC. Ly hai im M, N di ng trn cc tia AB v AC sao cho
AM CN
AB CA
= . Dng
hnh bnh MNCP. Tm tp hp nhng im P.
Bi 190. Bi 190. Bi 190. Bi 190. Cho ABC, cc im M, N, P di ng trn cc tia BC, CA v AB sao cho
MB NC PA
MC NA PB
= = .
Dng hnh bnh hnh MNPQ. Tm tp hp im Q.
Bi 191. Bi 191. Bi 191. Bi 191. Cho ABC. Hai im M v N thay i sao cho MN 2MA 3MB MC =
, , , ,
.
a/ Dng im I tha mn iu kin: 2IA 3IB IC 0 =
, , , ,
.
b/ Chng minh rng ng thng MN lun i qua mt im c nh.
Bi 192. Bi 192. Bi 192. Bi 192. Cho ABC v ng thng d c nh. Tm im M trn d sao cho
a/ u 2MA MB MC =
, , , ,
c di nh nht.
b/ v MA 3MB 2MC =
, , , ,
c di nh nht.
c/ x MA MB MC =
, , , ,
c di nh nht.
d/ y 5MA 2MB MC =
, , , ,
c di nh nht.
Bi 193. Bi 193. Bi 193. Bi 193. Cho hnh bnh hnh ABCD c tm O, hai im M, N di ng sao cho
MN MA MB MC MD =
, , , , ,
. Chng minh rng MN lun i qua mt im c nh.
Bi 194. Bi 194. Bi 194. Bi 194. Cho hnh bnh hnh ABCD c cc im M, I, N ln lt thuc cc cnh AB, BC, CD sao cho

1 1
AM AB, BI k.BC, CN CD
3 2
= = = . Gi G l trng tm ca BMN. nh k AI qua G.
Bi 195. Bi 195. Bi 195. Bi 195. Cho ABC c I l trung im ca on thng AB. Mt ng thng d lun thay i i qua I
ln lt ct CA, CB ti A', B'. Chng minh giao im ca AB' v A'B nm trn mt ng
thng c nh.
Bi 196. Bi 196. Bi 196. Bi 196. Cho ABC u, tm O, M l im di ng trn ng trn c nh
( )
O, b (nm trong ). Gi
A', B', C' tng ng l chn cc ng vung gc h t M xung cc cnh BC, CA, AB ca
tam gic v G' l trong tm ca A'B'C'.
a/ Chng minh rng:
3
MA' MB' MC' MO
2
=
, , , ,
.
b/ Chng minh rng G' di ng trn mt ng trn c nh.


www.MATHVN.com
www.DeThiThuDaiHoc.com
Ths. L Vn on Phn Hnh hc


Page - 180 - "All the flower of tomorrow are in the seeks of today"






















































B H TRC TA

C CC C Trc to
Trc to (trc) l mt ng thng trn xc nh mt im gc O v mt vect n
v e
,
. K hiu
( )
O; e
,
.
To ca vect trn trc:
( )
u a u a.e = =
, , ,
.
To ca im trn trc:
( )
M k OM k.e =
,
,
.
di i s ca vect trn trc: AB a AB a.e = =
,
,
.
Lu
Nu AB
,
cng hng vi e
,
th AB AB = .
Nu AB
,
ngc hng vi e
,
th AB AB = .
Nu
( ) ( )
A a , B b th AB b a = .
H thc Sacl: Vi A, B, C tu trn trc, ta c: AB BC AC = .
C CC C H trc to
H gm hai trc to Ox, Oy vung gc vi nhau. Vect n v trn Ox, Oy ln lt l
i, j
, ,
. O l gc to , Ox l trc honh, Oy l trc tung.
To ca vect i vi h trc to :
( )
u x; y u x.i y. j = =
, ,
, ,
.
To ca im i vi h trc to :
( )
M x; y OM x.i y. j =
,
, ,
.
Tnh cht:
( ) ( ) ( ) ( ) ( )

A A B B C C
a x; y , b x'; y' , k , A x ; y , B x ; y , C x ; y = =
,
,
R .

x x'
a b
y y'
'
1 =
1
=
!
1 =
1
+
,
,

( )
a b x x'; y y' =
,
,
.

( )
ka kx; ky =
,

x ' kx
x' y'
b a 0 k :
y' ky x y
'
1 =
1
=
!
1 =
1
+
, ,
,
R .

( ) ( ) ( )
2 2
B A B A B A B A
AB x x ; y y AB x x y y = =
,
.
To trung im I ca on thng AB:
A B A B
I I
x x y y
x ; y
2 2

= = .
To trng tm G ca tam gic ABC:
A B C A B C
G G
x x x y y y
x ; y
3 3

= = .
To im M chia on AB theo t s k 1:
A B A B
M M
x kx y ky
x ; y
1 k 1 k

= =

.
( M chia on AB theo t s k MA kMB =
, ,
).
ba im A, B, C thng hng
B A B A
C A C A
x x y y
x x y y

=


www.MATHVN.com
www.DeThiThuDaiHoc.com
cng hc tp mn Ton 10 tp I Ths. L Vn on


"Cn c b thng minh" Page - 181 -
































BI TP P DNG
Bi 197. Bi 197. Bi 197. Bi 197. Vit ta ca cc vect sau
a/
1
a 2i 3 j, b i 5 j, c 3i, d 2 j
3
= = = =
, , , , , , , ,
, ,
.
b/
1 3
a i 3 j, b i j, c i j, d 4 j, e 3i
2 2
= = = = =
, , , , , , , , , ,
, , ,
.
c/ a 5i, b 3j, c 3i 4j, d 0, 3i 2.j = = = =
, , , , , , , , , ,
.
Bi 198. Bi 198. Bi 198. Bi 198. Vit di dng u xi yj =
, ,
,
khi bit to ca vect u
,
l
a/
( ) ( ) ( ) ( )
u 2; 3 , u 1; 4 , u 2; 0 , u 0; 1 = = = =
, , , ,
.
b/
( ) ( ) ( ) ( )
u 1; 3 , u 4; 1 , u 1; 0 , u 0; 0 = = = =
, , , ,
.
c/
( ) ( ) ( ) ( )
u 1, 1 , u 5, 0 , u 0, 2 , u 7, 7 = = = =
, , , ,
.
Bi 199. Bi 199. Bi 199. Bi 199. Cho
( ) ( )
a 1; 2 , b 0; 3 = =
,
,
. Tm to ca cc vect sau
Dng ton 1. Ta vct Biu din vct

C CC C Ta vct
Ta cn nh cc kt qu sau
Vi hai im
( ) ( )
A A B B
A x , y , B x , y , ta c:
( )
( ) ( )
B A B A
2 2
B A B A
AB x x , y y
AB AB x x y y
'
1
= 1
1
1
!
1
= =
1
1
1+
,
,
Vi hai vct
( )
( )
1 1
2 2
a x , y
b x , y
'
1
= 1
1
!
1
=
1
1+
,
, , ta c:
( )
1 1
1 2
2 2
1 2 1 2
a x .i y .j
x x
a b
y y
.a .b x x , y y
'
1
= 1
1
1
' 1
1 =
1
1 1
=
! !
1 1 =
1 1
+
1
1
1
=
1
1+
, , ,
, ,
, ,

C CC C Biu din vct
Bi ton: Hy biu din vct
( )
1 2
c c , c =
,
theo cc vct
( ) ( )
1 2 1 2
a a , a , b b , b = =
, ,
.
Bc 1. Gi s
( )
c .a .b 1 =
, , ,
.
Bc 2. Ta c:
( ) ( ) ( )
1 2 1 2 1 1 2 2
.a .b a , a b , b .a .b , .a .b = =
, ,
.

Vy
( )
1 xy ra khi v ch khi:
( )

1 1 1
2 2 2
c .a .b
c .a .b
'
1 =
1

!
1 =
1
+

Gii h
( )
, ta tm c
( )
, . Thay vo
( )
1 , ta c kt qu bi ton.
www.MATHVN.com
www.DeThiThuDaiHoc.com
Ths. L Vn on Phn Hnh hc


Page - 182 - "All the flower of tomorrow are in the seeks of today"
a/ x a b; y a b; z 2a 3b = = =
, , ,
, , , , , ,
. b/ u 3a 2b; v 2 b; w 4a 0, 5b = = =
, , ,
, , , , ,
.
Bi 200. Bi 200. Bi 200. Bi 200. Cho
( ) ( )

1
a 2; 0 , b 1; , c 4; 6
2
1

= = =


( )
,
, ,
.
a/ Tm to ca vect d 2a 3b 5c =
, ,
, ,
.
b/ Tm 2 s m, n sao cho: ma b nc 0 =
, ,
, ,
.
Bi 201. Bi 201. Bi 201. Bi 201. Cho
( ) ( ) ( )
a 1;2 , b 1; 4 , c 0; 4 = = =
, , ,
. Tm ta v di ca cc vct u, v
, ,
bit:
a/ u 2a 4b c 5j =
, , , , ,
. b/ v a b 3c 2i =
, , , , ,
.
Bi 202. Bi 202. Bi 202. Bi 202. Biu din vct c
,
theo cc vct a, b
, ,
bit
a/
( ) ( )
a 2; 1 , b 3; 4 = =
, ,
v
( )
c 4; 7 =
,
. b/
( ) ( )
a 1;1 , b 2; 3 = =
, ,
v
( )
c 1; 3 =
,
.
c/
( ) ( )
a 4; 3 , b 2; 1 = =
, ,
v
( )
c 0; 5 =
,
. d/
( ) ( )
a 4;2 , b 5; 3 = =
, ,
v
( )
c 2; 0 =
,
.
e/
( ) ( )
a 2; 2 , b 1; 4 = =
, ,
v
( )
c 5; 0 =
,
. f/
( ) ( )
a 1; 3 , b 1;1 = =
, ,
v
( )
c 4; 3 =
,
.
Bi 203. Bi 203. Bi 203. Bi 203. Cho
( ) ( ) ( )
u 2; 5 , v 3; 4 , w 5; 7 = = =
, , ,
.
a/ Tm ta ca vct a u 3v 5w =
, , , ,
.
b/ Tm ta ca vct x
,
sao cho u 2v 3w x 0 =
, , , , ,
.
c/ Phn tch vct
( )
b 7;2 =
,
theo hai vct u
,
v v
,
.
d/ Tm m bit rng
( )
c 6; m =
,
cng phng vi w
,
.
Bi 204. Bi 204. Bi 204. Bi 204. Cho
( ) ( ) ( )
a 2;1 , b 3; 4 , c 7;2 = = =
, , ,
.
a/ Tm ta ca vct u 3a 2b 4c =
, , , ,
.
b/ Tm ta ca vct x
,
sao cho x a b c =
, , , ,
.
c/ Tm cc s k, l c ka lb =
, , ,
.
Bi 205. Bi 205. Bi 205. Bi 205. Cho bn im
( ) ( ) ( )
A 1;1 , B 2; 1 , C 4; 3 v
( )
D 16; 3 . Hy biu din vct AD
,
theo cc vct
AB
,
v AC
,
.
Bi 206. Bi 206. Bi 206. Bi 206. Cho bn im
( ) ( ) ( ) ( )
A 0;1 , B 2; 0 , C 1;2 , D 6; 4 . Hy biu din vct AD
,
theo cc vct
AB
,
v AC
,
.
Bi 207. Bi 207. Bi 207. Bi 207. Cho ba vct
( ) ( ) ( )
a 2;1 , b 3; 4 , c 7;2 = = =
, , ,
.
a/ Tm ta vct 2a 4b 5c
, , ,
.
b/ Tm ta ca vct x
,
sao cho x 2a 5b c =
, , , ,
.
c/ Hy phn tch vct c
,
theo vct a
,
v b
,
.
Bi 208. Bi 208. Bi 208. Bi 208. Cho hai im
( ) ( )
A 1;1 , B 1; 3 .
a/ Tm ta im M sao cho
( )
BM 3; 0 =
,
. b/ Tm ta im N sao cho
( )
NA 1;1 =
,
.
www.MATHVN.com
www.DeThiThuDaiHoc.com
cng hc tp mn Ton 10 tp I Ths. L Vn on


"Cn c b thng minh" Page - 183 -













Bi 209. Bi 209. Bi 209. Bi 209. Trong mt phng ta Oxy cho im
( )
M x, y .
a/ Tm ta im A i xng vi M qua trc Ox.
b/ Tm ta im B i xng vi M qua trc Oy.
c/ Tm ta im C i xng vi M qua O.
Bi 210. Bi 210. Bi 210. Bi 210. Cho hai im
( ) ( )
A 3; 5 , B 1; 0 .
a/ Tm to im C sao cho: OC 3AB =
, ,
.
b/ Tm im D i xng ca A qua C.
c/ Tm im M chia on AB theo t s k 3 = .
Bi 211. Bi 211. Bi 211. Bi 211. Cho hnh bnh hnh ABCD c
( ) ( ) ( )
A 1; 2 , B 3;2 , C 4; 1 . Tm ta nh D.
Bi 212. Bi 212. Bi 212. Bi 212. Cho hai im
( ) ( )
A 1; 1 , B 4; 3 .
a/ Tm ta v mun ca vct AB
,
.
b/ Tm ta trung im I ca AB.
c/ Tm im M chia on thng theo t s k 2 = .
d/ Tm im C sao cho AB OC =
, ,
.
Bi 213. Bi 213. Bi 213. Bi 213. Cho ba im
( ) ( ) ( )
A 1; 2 , B 0; 4 , C 3; 2 .
a/ Tm to cc vect AB, AC, BC
, , ,
.
b/ Tm ta trung im I ca on AB.
c/ Tm ta im M sao cho: CM 2AB 3AC =
, , ,
.
d/ Tm ta im N sao cho: AN 2BN 4CN 0 =
, , ,
,
.
Bi 214. Bi 214. Bi 214. Bi 214. Cho ba im
( ) ( ) ( )
A 1; 2 , B 2; 3 , C 1; 2 .
a/ Tm to im D i xng ca A qua C.
b/ Tm to im E l nh th t ca hnh bnh hnh c 3 nh l A, B, C.
c/ Tm to trng tm G ca tam gic ABC.
Dng 2. Xc nh im tha mn iu kin cho trc

Bi ton: Xc nh im M tha mn mt ng thc vct hay di
Bc 1. Gi
( )
M x, y tha yu cu bi ton.
Bc 2. Ta ha cc vct c trong ng thc hoc s dng cng thc v khong cch gia hai
im, chuyn biu thc v biu thc i s.
Bc 3. Gii phng trnh hoc h trn, ta nhn c ta im M.
www.MATHVN.com
www.DeThiThuDaiHoc.com
Ths. L Vn on Phn Hnh hc


Page - 184 - "All the flower of tomorrow are in the seeks of today"
Bi 215. Bi 215. Bi 215. Bi 215. Cho ba im
( ) ( ) ( )
A 2;1 , B 3; 2 , C 0; 3 .
a/ Tm ta ca u AB 3BC 2CA =
, , , ,
.
b/ Chng minh A, B, C l ba nh ca mt tam gic v tm trng tm G ca ABC.
c/ Tm ta im D sao cho CD 2AB 3BC =
, , ,
.
d/ Tm im E sao cho ABCE l hnh bnh hnh. Tm tm ca hnh bnh hnh .
Bi 216. Bi 216. Bi 216. Bi 216. Cho ba im
( ) ( ) ( )
A 2; 3 , B 1;1 , C 6; 0 .
a/ Tm ta ca vct u 4AB 3AC 2BC =
, , , ,
.
b/ Chng minh rng A, B, C khng thng hng v tm ta trng tm G ca ABC.
c/ Tm ta im D t gic ABCD l hnh bnh hnh. Tm tm ca hnh bnh hnh .
Bi 217. Bi 217. Bi 217. Bi 217. Trong mt phng Oxy cho
( ) ( )
A 3; 0 , B 3; 0 . Xc nh ta im C v D sao cho
a/ CA 3CB 0 =
, , ,
. b/ DA 3DB 0 =
, , ,
.
Bi 218. Bi 218. Bi 218. Bi 218. Trong mt phng Oxy cho ba im
( ) ( ) ( )
A 3; 6 , B 1; 2 ,C 6; 3 .
a/ Tm ta im D ABCD l hnh bnh hnh v tm ta trng tm G ca ABC.
b/ Tm ta im E tha biu thc vct CE 2AB 3AC =
, , ,
.
c/ Tm ta im F tha biu thc vct AF 2BF 4CF 0 =
, , , ,
.
d/ Tm im K tha biu thc vct 4KA 3BK CK 0 =
, , , ,
.
e/ Tm vct trung tuyn
4
AA
,
(lm theo ba cch).
f/ Tm tm I v bn knh ca ng trn ngoi tip ABC.
g/ Tm cc im
1 2 3
A , A , A sao cho ABC nhn cc im lm trung im cc cnh.
h/ Tm cc im M, N, P sao cho MNP nhn cc im A, B, C lm trung im cc cnh.
i/ Tm hai im chia on AC lm ba phn bng nhau.
j/ Tm cc t s m im A chia on BC, im B chia on AC, im C chia on AB
k/ Tm din tch ca ABC v din tch ng trn ngoi tip ABC.
Bi 219. Bi 219. Bi 219. Bi 219. Trong mt phng Oxy cho ba im
( ) ( ) ( )
A 0;2 , B 6; 4 , C 1; 1 . Cu hi tng t bi 218.
Bi 220. Bi 220. Bi 220. Bi 220. Trong mt phng Oxy cho ba im
( ) ( ) ( )
A 1; 3 , B 2; 4 , C 0;1 . Cu hi tng t bi 218.
Bi 221. Bi 221. Bi 221. Bi 221. Trong mt phng Oxy cho ba im
( ) ( ) ( )
A 2;2 , B 5;1 , C 3; 5 . Cu hi tng t bi 218.
Bi 222. Bi 222. Bi 222. Bi 222. Trong mt phng Oxy cho ba im
( ) ( ) ( )
A 1; 6 , B 4; 4 , C 4; 0 . Cu hi tng t bi 218.
Bi 223. Bi 223. Bi 223. Bi 223. Trong mt phng Oxy cho ba im
( ) ( ) ( )
A 1;1 , B 1; 3 , C 2; 0 . Cu hi tng t bi 218.
Bi 224. Bi 224. Bi 224. Bi 224. Tm im M sao cho
2 2
M M
x y nh nht khi bit ta M c dng
a/
( )
M 1 2t;1 3t . b/
( )
M 1 2t;1 t .
Bi 225. Bi 225. Bi 225. Bi 225. Cho hai im
( ) ( )
A 4; 4 , B 0;1 . Tm im C trn Oy sao cho trung trc ca on AC i qua B.
Bi 226. Bi 226. Bi 226. Bi 226. Trong mt phng Oxy cho ABC c
( ) ( )
A 1;1 , B 5; 3 , nh C nm trn trc tung Oy v trng
tm G ca thuc trc honh Ox. Tm ta im C v tnh din tch ABC.
Bi 227. Bi 227. Bi 227. Bi 227. Tm hai im
( )
2
M, N P : y x = . Bit rng IM 4IN =
, ,
v
( )
I 0;2 .

www.MATHVN.com
www.DeThiThuDaiHoc.com
cng hc tp mn Ton 10 tp I Ths. L Vn on


"Cn c b thng minh" Page - 185 -


























Bi 228. Bi 228. Bi 228. Bi 228. Cho
1
a i 5j, b ki 4j
2
= =
, , , , , ,
. Tm gi tr ca k hai vct a, b
, ,
cng phng.
Bi 229. Bi 229. Bi 229. Bi 229. Trong mt phng Oxy cho
( ) ( )
2
a x 1; 3x 2 , b 2;1 = =
, ,
v im
( )
A 0;1 .
a/ Tm x vct a
,
cng phng vi vct b
,
.
b/ Tm ta im M vct AM
,
cng phng vi b
,
v c di bng 5 .
Bi 230. Bi 230. Bi 230. Bi 230. Cho ba im
( ) ( ) ( )
A 1;1 , B 1; 2 , C 2; 0 . Chng minh hai vct AB
,
v AC
,
cng phng,
t suy ra ba im A, B, C thng hng.
Bi 231. Bi 231. Bi 231. Bi 231. Trong mt phng ta Oxy, cho cc im sau v chng minh chng thng hng
a/
( ) ( ) ( )
A 1; 4 , B 1; 6 , C 1; 2 . b/
( ) ( ) ( )
A 6;2 , B 2;2 , C 0;2 .
c/
( ) ( ) ( )
A 1; 3 , B 2; 5 , C 4; 9 . d/
( ) ( ) ( )
A 0; 4 , B 3;2 ,C 9;10 .
Bi 232. Bi 232. Bi 232. Bi 232. Cho ba im
( ) ( ) ( )
A x; 3 , B 4;2 , C 3; 5 . Tm x A, B, C thng hng.
Bi 233. Bi 233. Bi 233. Bi 233. Cho ba im
( ) ( ) ( )
A 4; y , B 2; 3 , C 6; 3 . Tm y A, B, C thng hng.
Bi 234. Bi 234. Bi 234. Bi 234. Cho ba im
( ) ( ) ( )
A 1;1 , B 2;1 , C m 1;2m 3 . Tm m ba im A, B, C thng hng.
Bi 235. Bi 235. Bi 235. Bi 235. Trong Oxy cho bn im
( ) ( )

1 1
A 1; 5 , B 2; , C 3; 1 , D ; 3
2 3
1 1






( ) ( )
. Chng minh rng:
a/ D nm trn ng thng AB. b/ B on AC.
Bi 236. Bi 236. Bi 236. Bi 236. Trong mt phng Oxy cho ba im
( ) ( ) ( )
A 3; 4 , B 1;1 , C 9; 5 .
a/ Chng minh ba im A, B, C thng hng.
b/ Tm ta im D sao cho A l trung im ca BD.
Dng 3. Vct cng phng v ng dng

Cn nh cc kt qu sau
Vi hai vct
( ) ( )
1 2 1 2
a a ; a , b b ; b = =
, ,
. hai vct a, b
, ,
cng phng
1 2
1 2
a a
b b
= .
Vi ba im
( ) ( ) ( )
A A B B C C
A x ; y , B x ; y , C x ; y . A, B, C thng hng th
B A C A
B A C A
x x x x
AC AB
y y y y

=

, ,
.
Vi ABC bt k th CA CB AB . Du " " = xy ra A, B,C thng hng.
u v u v u v
, , , , , ,
Du " " =

xy ra u, v
, ,
cng phng v hng.
u v w u v w
, , , , , ,
. Du " " =

xy ra u, v, w
, , ,
cng phng v hng.
Lu

( )
2 2
a x, y a x y = =
, ,
v
( ) ( )
2 2
B A B A
AB AB x x y y = =
,
.
Nm vng cng thc tnh din tch , cc bt ng thc c bn (Cauchy, B.C.S).
chng minh ba im l ba nh , ta chng minh ba im khng thng hng.
www.MATHVN.com
www.DeThiThuDaiHoc.com
Ths. L Vn on Phn Hnh hc


Page - 186 - "All the flower of tomorrow are in the seeks of today"
c/ Tm ta im E trn trc honh Ox sao cho A, B, E thng hng.
Bi 237. Bi 237. Bi 237. Bi 237. Trong mt phng Oxy cho ba im
( ) ( ) ( )
A 1; 4 , B 3; 2 , C 2; 3 .
a/ Chng minh A, B, C l ba nh ca mt tam gic v tm cc vct trung tuyn tng ng.
b/ Tm ta im D sao cho t gic ABCD l hnh bnh hnh.
c/ Tm im E trn trc tung Oy sao cho ba im A, C, E thng hng.
Bi 238. Bi 238. Bi 238. Bi 238. Trong mt phng Oxy cho ba im
( ) ( ) ( )
A 1; 4 , B 3; 2 , C 4; 2 .
a/ Chng minh A, B, C l ba nh ca mt tam gic.
b/ Tm ta im D sao cho ABCD l hnh bnh hnh.
c/ Tm ta im
( )
E x; 6 sao cho A, B, E thng hng.
Bi 239. Bi 239. Bi 239. Bi 239. Trong mt phng Oxy cho ba im
( ) ( ) ( )
A 6;2 , B 2; 6 , C 7; 8 .
a/ Chng minh rng ba im khng thng hng.
b/ Tm ta trng tm G v tm I ng trn ngoi tip ABC.
c/ Tm ta im H sao cho ABGH l hnh bnh hnh.
Bi 240. Bi 240. Bi 240. Bi 240. Trong mt phng Oxy cho ba im
( ) ( ) ( )
A 2; 5 , B 1;2 , C 4; 7 .
a/ Chng minh A, B, C l ba nh ca mt tam gic.
b/ Tm ta im M sao cho AM 2AB 3BC 5i =
, , , ,
.
c/ Tm im N trn trc honh Ox sao cho A, B, N thng hng.
Bi 241. Bi 241. Bi 241. Bi 241. Trong mt phng Oxy cho ba im
( ) ( ) ( )
A 0; 4 , B 3;2 , D 3; 0 .
a/ Chng minh rng ba im A, B, C thng hng, bit rng
( )
C 6 3t; 8 2t , t R.
b/ Chng minh rng A, B, D khng thng hng. T tnh chu vi ca ABD.
S: a/
( )
AC 2 t AB =
, ,
. b/ 2p 7 13 = .
Bi 242. Bi 242. Bi 242. Bi 242. Trong mt phng Oxy, cho hai im
( ) ( )
A 2;1 , B 6; 1 .
a/ Tm im M Ox sao cho ba im A, B, M thng hng.
b/ Tm im N Oy sao cho ba im A, B, N thng hng.
c/ Tm im P khc B sao cho A, B, P thng hng v PA 2 5 = .
S:
( ) ( ) ( )
M 4; 0 , N 0;2 , P 2; 3 .
Bi 243. Bi 243. Bi 243. Bi 243. Trong mt phng Oxy, cho hai im
( ) ( )
A 1; 4 , B 3; 4 .
a/ Tm im M Ox sao cho ba im A, B, M thng hng.
b/ Tm im N Oy sao cho ba im A, B, N thng hng.
c/ Tm im P khc B sao cho A, B, P thng hng v PA 3 5 = .
S:
( ) ( ) ( ) ( )

1 2
M 1; 0 , N 0; 2 , P 2;2 P 4; 10 .
Bi 244. Bi 244. Bi 244. Bi 244. i hc Nng Nghip nm 1996 1997
Trong mt phng ta Oxy cho ba im
( ) ( ) ( )
A 1;1 , B 3; 3 , C 2; 0 .
a/ Tnh din tch ABC.
b/ Hy tm tt c cc im M trn trc honh Ox sao cho gc

AMB nh nht.
S:
( )

ABC
S 2 .v.d.t

= v M O .
Bi 245. Bi 245. Bi 245. Bi 245. Trong mt phng Oxy cho ba im
( ) ( ) ( )
A 1; 3 , B 3;1 , C 2; 4 .
a/ Tnh din tch ABC.
www.MATHVN.com
www.DeThiThuDaiHoc.com
cng hc tp mn Ton 10 tp I Ths. L Vn on


"Cn c b thng minh" Page - 187 -
b/ Tm tt c cc im M Ox sao cho gc

AMB nh nht.
Bi 246. Bi 246. Bi 246. Bi 246. Trch b tuyn sinh i hc Cao ng 97 cu Va.
Tm trn trc honh Ox im P sao cho tng cc khong cch t P n cc im A v B l nh
nht
( ) ( )
min
hay PA PB . Bit rng:
a/
( ) ( )
A 1;1 , B 2; 4 . b/
( ) ( )
A 1;2 , B 3; 4 .
HD: a/ A, B khc pha
( )
o
Ox P x; 0 Ox AB = . A, P
o
, B thng hng
o
6
P ; 0 P
5
1


( )
.
b/ A, B cng pha Ox. Ly A
1
i xng vi A qua
( )
1
Ox A 1; 2
o
5
P P ; 0
3
1




( )
.
Bi 247. Bi 247. Bi 247. Bi 247. Tm trn ng thng d : x y 0 = im M sao cho tng cc khong cch t M n cc im
A v B l nh nht trong cc trng hp sau
a/
( ) ( )
A 1;1 , B 2; 4 . b/
( ) ( )
A 1;1 , B 3; 2 .
Bi 248. Bi 248. Bi 248. Bi 248. Cho im
( )
M 4;1 v hai im
( ) ( )
A a; 0 , B 0; b vi a, b 0 sao cho A, B, M thng hng.
Xc nh ta im A, B sao cho
a/ Din tch tam gic OAB l nh nht
( )
OABmin
S

.
b/ OA OA nh nht.
c/
2 2
1 1
OA OB
nh nht.
HD: A, B, M thng hng
( )

4 1
1
a b
= .
a/
( )

Cauchy
4 1 4
1 ab 16 S 8
a b
ab
= . Vy
( ) ( )
A 8; 0 , B 0;2 .
b/
( )
Cauchy
4b 4
a OA OB b 1 5 9
b 1 b 1
= =

. Vy
( ) ( )
A 6; 0 , B 0; 3 .
c/
( )

2
B.C.S
2 2
2 2 2 2
1 1 4 1 1 1 1
4 1 1
a b 17 a b a b
1 1

=



( ) ( )
. Vy
( )
17
A ; 0 , B 0;17
4
1


( )
.
Bi 249. Bi 249. Bi 249. Bi 249. Cho im
( )
M 2;1 v hai im
( ) ( )
A a; 0 , B 0; b vi a, b 0 sao cho A, B, M thng hng.
Xc nh ta im A, B sao cho
a/ Din tch tam gic OAB l nh nht
( )
OABmin
S

.
b/ OA OA nh nht.
c/
2 2
1 1
OA OB
nh nht.
www.MATHVN.com
www.DeThiThuDaiHoc.com
Ths. L Vn on Phn Hnh hc


Page - 188 - "All the flower of tomorrow are in the seeks of today"
Bi 250. Bi 250. Bi 250. Bi 250. Tm gi tr nh nht ca hm s sau a
2 2 2 2
y x 2ax 2 x 2bx 2b = vi a, b l
cc hng s tha iu kin a 0, b 0 < .
S:
( )
min
y 2 b a = khi A, B, M thng hng M O .
Bi 251. Bi 251. Bi 251. Bi 251. Tm gi tr nh nht ca hm s
2 2
y x x 1 x x 1 = .
Bi 252. Bi 252. Bi 252. Bi 252. Tm gi tr nh nht ca hm s
2 2
y x 4x 8 x 2x 2 = .
Bi 253. Bi 253. Bi 253. Bi 253. Tm gi tr nh nht ca hm s a
2 2 2 2
y x 2ax 2 x 2bx 2b = vi a, b R.
Bi 254. Bi 254. Bi 254. Bi 254. Tm gi tr nh nht ca biu thc:
2 2 2 2
P x y 2x 4y 5 x y 6x 4y 13 = .
S:
min
1 x 3
P 4
y 2
'
1
1
=
!
1 =
1
+
.
Bi 255. Bi 255. Bi 255. Bi 255. Tm gi tr ln nht v gi tr nh nht ca hm s
2 2
y cos 2cos 2 cos 6cos 13 = .
Bi 256. Bi 256. Bi 256. Bi 256. Cho ba im
( ) ( ) ( )
A 0; 6 , B 2; 5 , M 2t 2; t . Tm ta im M sao cho
a/
( )
min
MA MB . b/
max
MA MB .
Bi 257. Bi 257. Bi 257. Bi 257. Cho ba im
( ) ( ) ( )
A 1;2 , B 2; 5 , M 2t 2; t . Tm ta im M sao cho
a/
( )
min
MA MB . b/
max
MA MB
, ,
.
c/
max
MA MB . d/
mim
MA MB .
Bi 258. Bi 258. Bi 258. Bi 258. Cho a, b, c R . Chng minh:
( ) ( )
2 2
2 2 2 2
a c b a c b 2 a b .
HD:
( ) ( )
u a c; b , v a c; b = =
, ,
.
Bi 259. Bi 259. Bi 259. Bi 259. Cho a, b, c R . Chng minh:
2 2 2 2
a 4b 6a 9 a 4b 2a 12b 10 5 .
HD:
( ) ( )
u a 3;2b ; v 1 a; 3 2b = =
, ,
.
Bi 260. Bi 260. Bi 260. Bi 260. Cho a, b, c R . Chng minh:
2 2 2 2 2 2
a ab b a ac c b bc c .
HD: Cch 1.
b 3 c 3
u a ; b , v a ; c
2 2 2 2
1 1
1



= =



( )
( ) ( )
, ,
.
Cch 2.
b 3 3 3 b c
A a ; c , B 0; b c , C ; 0
2 2 2 2 2 2
1 1
1







( )
( ) ( )

www.MATHVN.com
www.DeThiThuDaiHoc.com
cng hc tp mn Ton 10 tp I Ths. L Vn on


"Cn c b thng minh" Page - 189 -
(D b Cao ng Giao Thng II 2003)
Bi 261. Bi 261. Bi 261. Bi 261. Cho a, b, c R .
Chng minh:
( ) ( )
2 2 2 2 2 2
4 cos a cos b sin a b 4 sin a sin b sin a b 2 .
HD:
( ) ( ) ( ) ( )
u 2cos a cos b; sin a b , v 2sina sin b; sin a b = =
, ,
.
Bi 262. Bi 262. Bi 262. Bi 262. Cho ba s dng x, y, z tha mn iu kin:
2 2
2 2
x xy y 3
y yz z 16
'
1
=
1
1
!
1
=
1
1+
.
Chng minh: xy yz zx 8 .
HD:
x 3x 3x z
u y ; , v ; y
2 2 2 2
1 1



= =





( ) ( )
, ,
.
Bi 263. Bi 263. Bi 263. Bi 263. Tm GTNN ca
2 2
P x x 1 x x 1 = , x R.
HD: Cch 1.
1 3 1 3
u x; , v x ;
2 2 2 2
1 1









( ) ( )
, ,
. Cch 2.
( )

1 3 1 3
A ; , B ; , C x, 0
2 2 3 2
1 1









( ) ( )
.
Bi 264. Bi 264. Bi 264. Bi 264. Cho x, y, z v x 3y 5z 3 .
Chng minh:
4 4 2
3xy 625z 4 15yz x 4 5zx 81y 4 45 5xyz .
HD:
2 2 2
u x; , v 3y; , w 5z
x 3y 5z
1 1 1

= = =



( ) ( ) ( )
, , ,
.
Bi 265. Bi 265. Bi 265. Bi 265. Cho x, y R . Chng minh:
2 2 2 2
x 4 x 2x y 1 y 6y 10 5 .
HD:
( ) ( ) ( )
u x;2 , v 1 x; y , w 1; 3 y = = =
, , ,
.
Bi 266. Bi 266. Bi 266. Bi 266. Cho x, y R . Chng minh:
( )( )
( )( )
2 2
x y 1 xy
1 1
2 2
1 x 1 y



.
HD:
( ) ( )
2 2
u 2x;1 x , v 1 y ;2y = =
, ,
.
Bi 267. Bi 267. Bi 267. Bi 267. Nu
x, y, z 0
x y z 1
'
1
1
!
1
1
+
th
2 2 2
2 2 2
1 1 1
x y z 82
x y z
(i hc A 2003).
HD:
1 1 1
u x; 2 , v y; 2 , w 2; 2
x y z
1 1 1

= = =



( ) ( ) ( )
, , ,
.

www.MATHVN.com
www.DeThiThuDaiHoc.com
Ths. L Vn on Phn Hnh hc


Page - 190 - "All the flower of tomorrow are in the seeks of today"
0
0
30
0
45
0
60
0
90
0
180
0

sin 0
1
2

2
2

3
2

1 0
cos 1
3
2

2
2

1
2

0 1
tan 0
3
3

1
3
|| 0
cot ||
3
1
3
3

0 ||

Chng





















































A GI TR LNG GIC CA MT GC BT K

C CC C nh ngha
Ly M trn na ng trn n v tm O. Xt gc nhn

xOM. Gi s
( )
M x; y . Khi
sin y = (tung ).
cos x = (honh ).



y
tan ,
x
1

=


( )
,
( )
x 0 .



x
cot ,
y
1

=


( )
,
( )
y 0 .
Lu
Nu t th cos 0, tan 0, cot 0 < < < .
tan ch xc nh khi 90
0
, cot ch xc nh khi 0
0
v 180
0
.
C CC C Tnh cht "c "c "c "cos i os i os i os i sin b sin b sin b sin b ph cho" ph cho" ph cho" ph cho"
Gc ph nhau
( )
( )
( )
( )
0
0
0
0
sin 90 cos
cos 90 sin
tan 90 cot
cot 90 tan
'
1
=
1
1
1
1
=
1
1
!
1
=
1
1
1
1
=
1
1+
Gc b nhau
( )
( )
( )
( )
0
0
0
0
sin 180 sin
cos 180 cos
tan 180 tan
cot 180 cot
'
1
=
1
1
1
1
=
1
1
!
1
=
1
1
1
1
=
1
1+

C CC C Gi tr lng gic ca cc gc c bit













C CC C Cc h thc c bn (cn nh: 0 sin 1 v 1 cos 1 )


Tung

Honh
Honh

Tung
M
x
y

x
y
O 1 1

( )
sin
tan , cos 0
cos

.
( )
cos
cot , sin 0
sin

.

( )
tan . cot 1, sin . cos 0 = .
2 2
sin cos 1 = .

( )
2
2
1
1 tan , cos 0
cos
=

.
( )
2
2
1
1 cot , sin 0
sin
=

.
2
TCH V HNG V TCH V HNG V TCH V HNG V TCH V HNG V NG DNG NG DNG NG DNG NG DNG
www.MATHVN.com
www.DeThiThuDaiHoc.com
cng hc tp mn Ton 10 tp I Ths. L Vn on


"Cn c b thng minh" Page - 191 -
Bi 268. Bi 268. Bi 268. Bi 268. Trong mt phng Oxy cho
( )
M 3; 4 . Hy tm sinx, cosx, tanx, cotx vi

x xOM = .
Bi 269. Bi 269. Bi 269. Bi 269. Trong mt phng Oxy cho
( )
M x; 4 v
0
xOM 120 = . Hy tm x.
Bi 270. Bi 270. Bi 270. Bi 270. Trong mt phng Oxy cho
( )
M x; y v

xOM = . Hy cho bit du ca x v y khi nhn,


0
120 = , t.
Bi 271. Bi 271. Bi 271. Bi 271. Tnh gi tr cc biu thc sau
a/
0 0 0
A a sin 0 bcos 0 c sin90 = . b/
0 0 0
B a cos 90 bsin90 c sin180 = .
c/
2 0 2 0 2 0
C a sin90 b cos 90 c cos180 = . d/
2 0 2 0 2 0
D 3 sin 90 2cos 60 3 tan 45 = .
e/
( ) ( )
2 2
2 2 0 0 0
E 4a sin 45 3 a tan45 2a cos 45 = .
Bi 272. Bi 272. Bi 272. Bi 272. Tnh gi tr ca cc biu thc sau:
a/ sin x cos x khi x bng 0
0
; 45
0
; 60
0
. b/ 2sin x cos2x khi x bng 45
0
; 30
0
.
Bi 273. Bi 273. Bi 273. Bi 273. Cho bit mt gi tr lng gic ca mt gc, tnh cc gi tr lng gic cn li:
a/
1
sin
4
= , nhn. b/
1
cos
3
= .
c/ tan x 2 2 = . d/
( )
0 0
5
cos , 90 180
13
= < < .
e/
( )
0
4
sin , 0 180
5
= < < . f/
1
cot
2
=
( )
0 0
, 0 90 < < .
Bi 274. Bi 274. Bi 274. Bi 274. Bit
0
6 2
sin15
4

= . Tnh
0 0 0 0 0
cos15 , tan15 , cot15 , cos 75 , cos105 .
Bi 275. Bi 275. Bi 275. Bi 275. Cho bit mt gi tr lng gic ca mt gc, tnh gi tr ca mt biu thc:
a/ Bit
0 0
1
sin x , 90 x 180
3
= < < . Tnh
tan x 3cot x 1
A
tan x cot x

=

.
b/ Bit tan 2 = . Tnh
3 3
sin cos
B
sin 3cos 2sin

=

.
c/ Bit tan 2 = . Tnh
3sin cos
C
sin cos

=

.
d/ Bit
3
sin
2
= . Tnh
cot tan
D
cot tan

=

.
e/ Bit cot x 3 = . Tnh
2 2
2 2
sin x 2sin x cos x 2 cos x
E
2sin x 3sin x cos x 4 cos x

=

.
Bi 276. Bi 276. Bi 276. Bi 276. Cho bit
0 0
0 90 < < .
a/ Chng minh rng: sin cos 1 .
b/ t sin cos m = . Hy tnh
A sin . cos = .
4 4
B sin cos = .

4 4
C sin cos = .
6 6
D sin cos = .
www.MATHVN.com
www.DeThiThuDaiHoc.com
Ths. L Vn on Phn Hnh hc


Page - 192 - "All the flower of tomorrow are in the seeks of today"
Bi 277. Bi 277. Bi 277. Bi 277. Cho
4
sin cos
3
= . Hy tm
a/ A sin cos = . b/
3 3
B sin cos = .
c/
4 4
C sin cos = . d/
6 6
D sin cos = .
f/
8 8
F sin cos = . g/
2 2
2 2
cos cot
G
sin tan

=

.
Bi 278. Bi 278. Bi 278. Bi 278. Chng minh cc ng thc sau:
a/
( )
2
sin x cos x 1 2sin x. cos x = . b/
4 4 2 2
sin x cos x 1 2 sin x. cos x = .

c/
2 2 2 2
tan x sin x tan x. sin x = . d/
6 6 2 2
sin x cos x 1 3sin x. cos x = .
e/
1 sin x cos x
cos x 1 sin x

. f/
2 2
2
2 2
cos x cot x
cot x
sin x tan x

.
g/
2
2
sin x sin x cos x
sin x cos x
sin x cos x tan x 1

=

.
h/
( )( )
sin x. cos x 1 tan x 1 cot x 1 2sin x. cos x = .
i/
2
1 1 1
sin x sin x
2 2cos x 2 2cos x 1 tan x
1


( )
, bit
0 0
0 x 180 < < .
Bi 279. Bi 279. Bi 279. Bi 279. n gin (rt gn) cc biu thc sau
a/ A 1 cos x. 1 cos x = . b/
2
B sin 1 tan = .
c/
2
2
1 cos x
C tan x. cot x
1 sin x

. d/
( )
2 2
2
1 4 sin x. cos x
D
sin x cos x

.

e/
( ) ( ) ( )
0 0 2 2 2
E sin 90 x cos 180 x sin x 1 tan x tan x = .
f/
( ) ( ) ( )
( )
0 0 0
0
2cos 180 x cos 90 x cot 180 x
F
sin x 4 cos 90 x

=

.
g/
0 0
0
0 0
cos 36 sin234
G . cos 54
sin144 cos126

. h/
2
1 sin x
H cot x cos x
cos x
1

( )
.
i/
( ) ( ) ( )
( ) ( ) ( )
0 0 0
0 0 0
cos 90 x cos 270 x tan x 180
I
sin 180 x cos 270 x tan x 90

=

.
j/
3 3 7 7
J cos x sin x cos x sin x
2 2 2 2
1 1 1 1


=



( ) ( ) ( ) ( )
.
k/
( )
2 2 0 0
K sin x cot x cos x, 180 x 270 = < < .
www.MATHVN.com
www.DeThiThuDaiHoc.com
cng hc tp mn Ton 10 tp I Ths. L Vn on


"Cn c b thng minh" Page - 193 -
l/
( )
0 0
1 sin x 1 sin x
L , 0 x 90
1 sin x 1 sin x

= < <

.
m/
( )
0 0
2
1
M , 90 x 180
sin x cot x cos x
= < <

.
n/
0 0 0 0 0 0
N cos10 cos 20 cos 30 ... cos160 cos170 cos180 = .
o/
2 0 2 0 2 0 2 0
O cos 12 cos 78 cos 1 cos 89 = .
p/
2 0 2 0 2 0 2 0
P sin 3 sin 15 sin 75 sin 87 = .
Bi 280. Bi 280. Bi 280. Bi 280. Cho A, B, C ln lt l ba gc ca tam gic ABC. Chng minh rng:
a/
( )
sin B C sin A = . b/
( )
cos B C cos A = .
c/
( )
sin A B sinC = . d/
( )
cos A B cos C = .
e/
A B C
sin cos
2 2

= . f/
A B C
tan cotC
2

= .
g/
A B C
tan cot
2 2

= . h/
B C A
cos sin
2 2

= .
i/
( )
tan A B tanC = . j/
A B 3C
sin cosC
2

= .
k/
A B 2C 3C
tan cot
2 2

= . l/
A B 2C 3C
sin cos
2 2

= .
m/
( )
cos A B C cos 2B = . n/
B C 2A 3A
tan cot
2 2

= .
Bi 281. Bi 281. Bi 281. Bi 281. Chng minh cc biu thc sau khng ph thuc vo bin (hay biu thc c lp vi bin s)
a/
4 4 2 2 2
A 2 cos x sin x sin x cos x 3sin x = .
b/
6 4 2 2 4 4
B cos x 2 sin x cos x 3 sin x cos x sin x = .
c/
( ) ( )
2 2
C cot x tan x cot x tan x = .
d/
( ) ( ) ( ) ( )
0 0 0 0
D cos x 60 cos x 45 cos x 30 cos 135 x = .
e/
4 2 4 2
E sin x 4 cos x cos x 4 sin x = .
f/
2 cot x 1
F
tan x 1 cot x 1

=

.
g/
2 2
2
cot x cos x sin x cos x
G
cos x cot x

= .

www.MATHVN.com
www.DeThiThuDaiHoc.com
Ths. L Vn on Phn Hnh hc


Page - 194 - "All the flower of tomorrow are in the seeks of today"






















































B TCH V HNG CA HAI VCT

C CC C Gc gia hai vct
Cho a, b 0
, , ,
. T mt im O bt k v OA a, OB b = =
, , , ,
.
Khi
( )

a, b AOB =
, ,
vi
0 0
0 AOB 180 .
Lu

( )
0
a, b 90 a b =
, , , ,
.
( )
0
a, b 0 a, b =
, , , ,
cng hng.

( )
0
a, b 180 a, b =
, , , ,
ngc hng.
( ) ( )
a, b b, a =
, , , ,
.
C CC C Tch v hng ca hai vct
nh ngha:
( )
a.b a . b . cos a, b =
, , , , , ,
. c bit:
2 2
a.a a a = =
, , , ,
.
Tnh cht: vi a, b, c
, , ,
bt k v k R, ta c:
a.b b.a =
, , , ,
.
( )
a b c a.b a.c =
, , , , , , ,
.

( ) ( ) ( )
ka .b k a.b a. kb = =
, , , , , ,
.
2 2
a 0; a 0 a 0 = =
, , , ,
.

( )
2 2 2
a b a 2a.b b =
, , , , , ,
.
( )
2 2 2
a b a 2a.b b =
, , , , , ,
.

( )( )
2 2
a b a b a b =
, , , , , ,
.
( )
a.b 0 a, b
, , , ,
l gc nhn.

( )
a.b 0 a, b <
, , , ,
l gc t.
( )
a.b 0 a, b =
, , , ,
l gc vung.
C CC C Biu thc ta ca tch v hng
Cho
( ) ( )
1 2 1 2
a a ; a , b b ; b = =
, ,
. Khi :
( ) 1 1 2 2
a.b a b a b a . b . cos a, b = =
, , , , , ,
.
(Honh nhn honh Tung nhn tung = hng s)

( )
1 1 2 2
2 2 2 2
1 2 1 2
a b a b
a.b
cos a, b
a a . b b a . b

= =

, ,
, ,
, , .

( ) 1 1 2 2
a b cos a, b 0 a.b 0 a b a b 0 = = =
, , , , , ,
.
chng minh a
,
v b
,
khng cng phng, ta chng minh
1 2
1 2 2 1
1 2
a a
hay a b a b
b b
.
(Dng chng minh ba nh ca mt tam gic)
Vi
( ) ( ) ( ) ( )
2 2
A A B B B A B A
A x ; y , B x ; y AB x x y y = .
Khi tnh tch v hng 2 vct, ta nn n chiu nhm xc nh ng gc.
www.MATHVN.com
www.DeThiThuDaiHoc.com
cng hc tp mn Ton 10 tp I Ths. L Vn on


"Cn c b thng minh" Page - 195 -


































Bi 282. Bi 282. Bi 282. Bi 282. Cho ABC vung ti A c AB a, BC 2a = = . Tnh cc tch v hng
a/ AB.AC
, ,
. b/ AC.CB
, ,
. c/ AB.BC
, ,
.
Bi 283. Bi 283. Bi 283. Bi 283. Cho ABC u cnh bng a. Tnh cc tch v hng
a/ AB.AC
, ,
. b/ AC.CB
, ,
. c/ AB.BC
, ,
.
Bi 284. Bi 284. Bi 284. Bi 284. Cho ABC vung cn c AB AC a = = c AH l ng cao. Tnh cc tch v hng sau
a/ AB.AC
, ,
. b/ AH.BC
, ,
. c/ AC.CB
, ,
v AB.BC
, ,
.
Bi 285. Bi 285. Bi 285. Bi 285. Cho ABC vung ti A, c AB.CB 4 =
, ,
v AC.BC 9 =
, ,
.
a/ Tnh cc cnh ca ABC.
b/ Gi I, J l cc im tha cc ng thc vct IA 2IB 0, 2JB JC 0 = =
, , , , , ,
. Tnh IJ
,
theo
hai vct BA, BC
, ,
.
Bi 286. Bi 286. Bi 286. Bi 286. Cho ABC vung ti A c AB 3, AC 4 = = .
a/ Tnh cc tch v hng: AB.BC, BC.CA, CA.AB
, , , , , ,
.
b/ Nu
( ) ( ) ( )
BC 5 cm , CA 7 cm , AB 8 cm = = = .
Dng 1. Tnh tch v hng Tnh gc Chng minh & thit lp vung gc

C CC C Tnh tch v hng
Ta c th la chn mt trong cc hng sau y
Hng 1. S dng nh ngha bng cch a hai vct a
,
v b
,
v cng gc xc nh chnh
xc gc
( )
a, b =
, ,
, t : a.b a . b . cos =
, , , ,
.
Hng 2. S dng cc tnh cht v cc hng ng thc ca tch v hng ca hai vct.
Hng 3. Nu bi cho dng ta
( ) ( )
1 2 1 2 1 1 2 2
a a ; a , b b ; b a.b a b a b = = =
, , , ,
.
C CC C Tnh gc:
( )
1 1 2 2
2 2 2 2
1 2 1 2
a b a b
a.b
cos a, b
a a . b b a . b

= =

, ,
, ,
, , .
C CC C Chng minh vung gc
Ta c th la chn mt trong cc hng sau y
Hng 1. Nu bi khng cho ta , ta s dng tnh cht ca tch v hng. c bit:
( )
( )
a 0
a b a b a.b 0 a . b . cos a, b 0 b 0
cos a, b 0

= = =

,
, , , , , , , , ,
, ,

Hng 2. Nu bi cho dng ta
( ) ( )
1 2 1 2
a a ; a , b b ; b = =
, ,
th
1 1 2 2
a b a.b 0 a b a b 0 = =
, , , ,
.
www.MATHVN.com
www.DeThiThuDaiHoc.com
Ths. L Vn on Phn Hnh hc


Page - 196 - "All the flower of tomorrow are in the seeks of today"
Tnh
( )
BC, BA
, ,
v

B.
Trn cnh AB ly im D sao cho
( )
AD 3 cm = . Hy tnh
( )
AD, AC
, ,
.
Bi 287. Bi 287. Bi 287. Bi 287. Cho ABC vung ti A c BC a 3, M = l trung im ca BC. Bit rng
2
a
AM.BC
2
=
, ,
.
Hy tnh AB, AC.
Bi 288. Bi 288. Bi 288. Bi 288. Cho ABC u cnh a v AM l trung tuyn ca tam gic. Tnh cc tch v hng sau
a/
( )
AC AC. 2AB 3
, , ,
. b/
( )
AC. AC AB
, , ,
.
c/ AM.AB
, ,
. d/
( ) ( )
AB AC . AB AC
, , , ,
.
e/
( ) ( )
CA BC . CA CB
, , , ,
. f/ m AB.BC BC.CA CA.AB =
, , , , , ,
.
Bi 289. Bi 289. Bi 289. Bi 289. Cho ABC c BC a, CA b, AB c = = = . Tnh cc tch v hng sau theo a, b, c
a/ BA.BC
, ,
. b/ CB.CA
, ,
. c/ AC.AB
, ,
.
Bi 290. Bi 290. Bi 290. Bi 290. Cho ABC c


0
AB 3a, AC a, A 60 = = = . Tnh AB.AC
, ,
. Suy ra di cnh BC v di
ng trung tuyn AM.
Bi 291. Bi 291. Bi 291. Bi 291. Cho ABC c
a/


0
AB 2, AC 3, A 60 = = = . Hy tnh di cnh BC.
b/


0
AB 3, BC 4, B 45 = = = . Hy tnh di cnh AC.
c/


0
CA 5, BC 6, C 120 = = = . Hy tnh di cnh AB.
Bi 292. Bi 292. Bi 292. Bi 292. Cho ABC c BC a, CA b, AB c = = = . Chng minh rng:
( )
( )
( )
2 2 2
2 2 2
2 2 2
1 : a b c 2bc cos A
2 : b a c 2ac cos B
3 : c a b 2abcos C
=
=
=
(nh l hm cos)
Bi 293. Bi 293. Bi 293. Bi 293. Cho ABC c AB 5, BC 7, CA 9 = = = .
a/ Tnh cosA, cosB, cosC.
b/ Tnh AB.BC BC.CA CA.AB
, , , , , ,
.
c/ Tnh di ba ng trung tuyn AM, BN, CP ca tam gic ABC.
Bi 294. Bi 294. Bi 294. Bi 294. Cho tam gic ABC c AB 5, BC 7, AC 8 = = = .
a/ Tnh AB.AC
, ,
, ri suy ra gi tr ca gc A.
b/ Tnh CA.CB
, ,
.
c/ Gi D l im trn CA sao cho CD 3 = . Tnh CD.CB
, ,
.
Bi 295. Bi 295. Bi 295. Bi 295. Cho hnh vung ABCD cnh a. Tnh gi tr cc biu thc sau
a/ AB.AC
, ,
. b/
( )
AC AB AD
, , ,
.
c/ AB.BD
, ,
. d/
( )( )
AB AD BD BC
, , , ,
.
e/
( )( )
AC AB 2AD AB
, , , ,
. f/
( )( )
AB AC BC BD BA
, , , , ,
.
www.MATHVN.com
www.DeThiThuDaiHoc.com
cng hc tp mn Ton 10 tp I Ths. L Vn on


"Cn c b thng minh" Page - 197 -
g/
( )( )
AB AC AD DA DB DC
, , , , , ,
. h/ OA.AB
, ,
.
Bi 296. Bi 296. Bi 296. Bi 296. Cho ABC c AB c, AC b, AB a = = = . Gi G l trng tm v D, E, F ln lt l chn
ng phn gic trong ca gc A, B, C. Tnh
a/ Tch v hng ca cc vct: AG.BC, BG.AC, CG.AB
, , , , , ,
.
b/ di cc cnh AG, BG, CG.
c/ Tnh gi tr ca S GB.GC GC.GA GA.GB =
, , , , , ,
.
Bi 297. Bi 297. Bi 297. Bi 297. Cho tam gic ABC c AB 2, BC 4, CA 3 = = = .
a/ Tnh AB.AC, BC.BA, CA.CB
, , , , , ,
, ri suy ra cosA, cosB, cosC.
b/ Gi G l trng tm ca ABC. Tnh AG.BC
, ,
.
c/ Tnh gi tr biu thc S GA.GB GB.GC GC.GA =
, , , , , ,
.
d/ Gi AD l phn gic trong ca gc

( )
BAC, D BC . Tnh AD
,
theo AB, AC
, ,
, suy ra AD.
HD: a/
3
AB.AC
2
=
, ,
,
1
cos A
4
= b/
5
AG.BC
3
=
, ,
c/
29
S
6
= .
d/ ng phn gic
AB
DB .DC
AC
=
, ,

3 2
AD AB AC
5 5
=
, , ,
,
54
AD
5
= .
Bi 298. Bi 298. Bi 298. Bi 298. Cho tam gic ABC c
0
AB 2, AC 3, A 60 = = = . Gi M l trung im ca BC.
a/ Tnh BC, AM.
b/ Tnh IJ, trong I, J c xc nh bi: 2IA IB 0, JB 2JC = =
, , , ,
,
.
HD: a/
7
BC 19, AM
2
= = . b/
2
IJ 133
3
= .
Bi 299. Bi 299. Bi 299. Bi 299. Cho hnh thang vung ABCD, ng cao AB 2a, = y ln BC 3a, = y nh a AD 2 = .
a/ Tnh cc tch v hng: AB.CD, BD.BC, AC.BD
, , , , , ,
.
b/ Gi I l trung im ca CD, tnh AI.BD
, ,
. Suy ra gc ca hai vct AI
,
v BD
,
.
Bi 300. Bi 300. Bi 300. Bi 300. Cho hnh thang vung ABCD c ng cao AB a 3 = , canh y a AD a, BC 2 = = .
a/ Tnh AC.BD
, ,
. Suy ra gc nhn to bi hai ng AC v BD.
b/ Gi G l trng tm ca BCD v tnh AG.AB
, ,
.
Bi 301. Bi 301. Bi 301. Bi 301. Cho hnh ch nht ABCD c tm I, cnh AB a, AD b = = . Tnh theo a, b cc tch v hng
a/
( )( )
AB.AC, BD.AC, AC AB AC AD
, , , , , , , ,
.
b/ MA.MC MB.MD
, , , ,
vi M l im thuc ng trn ngoi tip hnh ch nht ABCD.
Bi 302. Bi 302. Bi 302. Bi 302. Cho tam gic ABC c
( ) ( ) ( )
A 1; 2 , B 2; 6 , C 9; 8 .
a/ Tnh AB.AC
, ,
. Chng minh tam gic ABC vung ti A.
b/ Tm tm v bn knh ng trn ngoi tip tam gic ABC.
c/ Tm to trc tm H v trng tm G ca tam gic ABC.
d/ Tnh chu vi, din tch tam gic ABC.
e/ Tm to im M trn Oy B, M, A thng hng.
f/ Tm to im N trn Ox tam gic ANC cn ti N.
www.MATHVN.com
www.DeThiThuDaiHoc.com
Ths. L Vn on Phn Hnh hc


Page - 198 - "All the flower of tomorrow are in the seeks of today"
g/ Tm to im D ABDC l hnh ch nht.
h/ Tm to im K trn Ox AOKB l hnh thang y AO.
i/ Tm to im T tho TA 2TB 3TC 0 =
, , , ,
.
k/ Tm to im E i xng vi A qua B.
l/ Tm to im I chn ng phn gic trong ti nh C ca ABC.
Bi 303. Bi 303. Bi 303. Bi 303. Cho tam gic ABC c
( ) ( ) ( )
A 0; 2 , B 6; 9 , C 4;1 . Cu hi tng t nh bi 302.
Bi 304. Bi 304. Bi 304. Bi 304. Xc nh hnh dng ca tam gic ABC khi bit
a/
( ) ( ) ( )
A 1; 0 , B 5; 0 , C 3; 4 . b/
( ) ( ) ( )
A 1; 2 , B 2; 6 , C 9; 8 .
c/
( ) ( )
( )
A 1; 0 , B 3; 0 , C 1;2 2 . d/
( ) ( ) ( )
A 5; 7 , B 8; 5 , C 0; 7 .
Bi 305. Bi 305. Bi 305. Bi 305. Xc nh hnh dng ca t gic khi bit
a/
( ) ( ) ( ) ( )
A 2; 6 , B 3; 3 , C 3;1 , D 4; 4 . b/
( ) ( ) ( ) ( )
A 2; 2 , B 1; 3 , C 3; 2 , D 2; 2 .
c/
( ) ( ) ( ) ( )
A 2; 6 , B 4; 4 , C 2; 2 , D 1; 3 . d/
( ) ( ) ( ) ( )
A 2;1 , B 3; 6 , C 2; 5 , D 3; 0 .
Bi 306. Bi 306. Bi 306. Bi 306. Trong mt phng ta Oxy, cho
( ) ( ) ( )
a 1; 3 , b 6; 2 , c x;1 = = =
, , ,
.
a/ Chng minh a b
, ,
. b/ Tm x a c
, ,
.
c/ Tm x a
,
cng phng vi c
,
. d/ Tm ta vct d
,
a d
, ,
v b.d 20 =
, ,

Bi 307. Bi 307. Bi 307. Bi 307. Trong mt phng Oxy, cho
( ) ( )
A 1; 4 , B 3; 2 v vct
( )
v 2m 1; 3 4m =
,
.
a/ Tm m v
,
cng phng vi AB
,
. b/ Tm m v AB
, ,
.
Bi 308. Bi 308. Bi 308. Bi 308. Trong mt phng Oxy cho bn im:
( ) ( ) ( ) ( )
A 2; 3 , B 9; 4 , C 5; y , D x; 2 .
a/ Tm y ABC vung ti C. b/ Tm x 3 im A, B, D thng hng.
Bi 309. Bi 309. Bi 309. Bi 309. Trong mt phng Oxy cho hai im
( ) ( )
A 3; 3 , B 4; 4 .
a/ Tm M Oy
0
AMB 90 = . b/ Tm N Ox A, B, N thng hng.
Bi 310. Bi 310. Bi 310. Bi 310. Tnh gc gia hai vct a
,
v b
,
trong cc trng hp sau
a/
( ) ( )
a 4; 3 , b 1; 7 = =
, ,
. b/
( ) ( )
a 2; 5 , b 3; 7 = =
, ,
.
c/
( ) ( )
a 6; 8 , b 12; 9 = =
, ,
. d/
( ) ( )
a 2; 6 , b 3; 9 = =
, ,
.
Bi 311. Bi 311. Bi 311. Bi 311. Cho ABC vi
( ) ( ) ( )
A 1; 6 , B 2; 6 , C 1;1 .
a/ Tm ta trc tm H. b/ V AK BC . Xc nh ta im K.
Bi 312. Bi 312. Bi 312. Bi 312. Cho tam gic ABC c
( ) ( ) ( )
A 1; 1 , B 5; 3 , C 2; 0 .
a/ Tnh chu vi v nhn dng tam gic ABC.
b/ Tm to im M bit CM 2AB 3AC =
, , ,
.
c/ Tm tm v bn knh ng trn ngoi tip tam gic ABC.
Bi 313. Bi 313. Bi 313. Bi 313. Cho ABC cso
( ) ( ) ( )
A 4; 3 , B 0; 5 , C 6; 2 .
a/ Chng minh ABC vung ti B.
b/ Tm tm ca ng trn ngoi tip tam gic ABC.
c/ Tm tm ca ng trn ni tip tam gic ABC.
www.MATHVN.com
www.DeThiThuDaiHoc.com
cng hc tp mn Ton 10 tp I Ths. L Vn on


"Cn c b thng minh" Page - 199 -
Bi 314. Bi 314. Bi 314. Bi 314. Cho ABC bit
( ) ( )
A 1; 2 , B 3; 4 .
a/ Tm ta hnh chiu ca A ln BC. b/ Tm din tch tam gic ABC.
Bi 315. Bi 315. Bi 315. Bi 315. Cho ba im
( ) ( ) ( )
A 7; 4 , B 0; 3 , C 4; 0 . Tm ta hnh chiu vung gc H ca A ln BC. T
suy ra ta im A
1
l im i xng vi A qua BC.
Bi 316. Bi 316. Bi 316. Bi 316. Cho ABC, bit
( ) ( ) ( )
A 1; 2 , B 1;1 , C 5; 1 .
a/ Tnh AB.AC
, ,
. b/ Tnh cos v sin gc A.
c/ Tm ta chn ng cao A
1
ca ABC. d/ Tm ta trc tm H ca ABC.
e/ Tm ta trng tm G ca ABC. f/ Tm ta tm I ng trn ngoi tip .
g/ Chng minh rng I, H, G thng hng.
Bi 317. Bi 317. Bi 317. Bi 317. Cho
( ) ( ) ( ) ( )
A 0; 2 , B 6; 9 , C 4;1 , D 2;10 .
a/ Chng minh: ABC vung. b/ Chng minh: ABCD l hnh ch nht.
c/ Gi C' tha CC' AB =
, ,
. Tm C', suy ra D i xng vi C' qua B.
Bi 318. Bi 318. Bi 318. Bi 318. Cho ABC c a AB a, AC 2 = = . Gi D l trung im cnh AC, M l im tha
1
BM BC
3
=
, ,
. Chng minh BD vung gc vi AM.
Bi 319. Bi 319. Bi 319. Bi 319. Cho ABC c gc A nhn. V bn ngoi ABC cc tam gic vung cn nh A l ABD,
ACE. Gi M l trung im ca BC. Chng minh rng: AM DE .
Nu gc A t hoc vung th kt qu trn cn ng khng ? Ti sao ?
Bi 320. Bi 320. Bi 320. Bi 320. Cho ABC cn ti A, H l trung im ca BC v D l hnh chiu ca H ln AC, M l trung
im ca HD. Chng minh AM BD .
Bi 321. Bi 321. Bi 321. Bi 321. Cho bn im A, B, C, D bt k.
a/ Chng minh: DA.BC DB.CA DC.AB 0 =
, , , , , ,
.
b/ T suy ra mt cch chng minh nh l: "Ba ng cao trong tam gic ng qui".
Bi 322. Bi 322. Bi 322. Bi 322. Cho tam gic ABC vi ba trung tuyn AD, BE, CF.
Chng minh: BC.AD CA.BE AB.CF 0 =
, , , , , ,
.
Bi 323. Bi 323. Bi 323. Bi 323. Cho ABC u, trn BC, CA, AB ly cc im D, E, F tha 3DB BC, 3CE 2CA = =
, , , ,
v
15AF 4AB =
, ,
. Chng minh: AD EF .
Bi 324. Bi 324. Bi 324. Bi 324. Cho hnh vung OACB v mt im M thuc OC. K ng PP' qua M v vung gc vi OA,
ng QQ' qua M v vung gc vi OB.
a/ Chng minh: AM PQ = . b/ Chng minh: AM PQ .
Bi 325. Bi 325. Bi 325. Bi 325. Cho ba im A, B, M. Gi O l trung im ca AB.
Chng minh rng:
2 2
4OM AB MA MB = .
Bi 326. Bi 326. Bi 326. Bi 326. Cho ABC c AB c, BC a, AC b = = = . Chng minh iu kin cn v hai trung
tuyn BM v CN vung gc nhau l
2 2 2
b c 5a = .
Bi 327. Bi 327. Bi 327. Bi 327. Chng minh rng iu kin cn v tam gic ABC vung ti A l
2
BA.BC AB =
, ,
.
Bi 328. Bi 328. Bi 328. Bi 328. Cho ABC ni tip ng trn tm
( )
O . Gi H l im xc nh bi OH OA OB OC =
, , , ,
.
a/ Tnh AG.BC
, ,
. Suy ra H l trc tm ca tam gic ABC.
b/ Tm h thc gia di ba cnh tam gic ABC l a, b, c sao cho AH AM vi M l trung
im ca BC.
www.MATHVN.com
www.DeThiThuDaiHoc.com
Ths. L Vn on Phn Hnh hc


Page - 200 - "All the flower of tomorrow are in the seeks of today"
Bi 329. Bi 329. Bi 329. Bi 329. Cho hnh vung ABCD.
a/ Gi M, N ln lt l trung im ca BC, CD. Chng minh AM BN .
b/ Gi P, Q tng ng trn BC, CD sao cho
1 1
BP BC, CQ CD
4 4
= = .
Chng minhAP BQ .
Bi 330. Bi 330. Bi 330. Bi 330. Cho hnh ch nht ABCD c
a/ AB a, AD a 2 = = . Gi K l trung im ca AD. Chng minh: BK AC .
b/ AB a, AD b = = . Gi K l trung im ca AD v L trn tia DC sao cho
2
b
DL
2a
= .
Chng minh: BK AL .
Bi 331. Bi 331. Bi 331. Bi 331. Cho t gic ABCD c AC BD ti M. Gi P l trung im ca AD. Chng minh rng:
MP BC MA.MC MB.MD =
, , , ,
.
Bi 332. Bi 332. Bi 332. Bi 332. Cho hnh vung ABCD, im M nm trn AC sao cho
AC
AM
4
= . Gi N l trung im ca
DC. Chng minh tam gic BMN l tam gic vung cn.
Bi 333. Bi 333. Bi 333. Bi 333. Cho hnh thang vung ABCD c ng cao AB h = , cnh y AD a, BC b = = . Tm iu
kin gia a, b, h :
a/ AC BD . b/
0
AIB 90 = vi I l trung im ca CD.
Bi 334. Bi 334. Bi 334. Bi 334. Cho hnh thang vung ABCD, ng cao a AB 2a, AD a, BC 4 = = = .
a/ Tnh AC.BD
, ,
. Suy ra gc gia AC v BD.
b/ Gi I l trung im ca CD, J l im di ng trn cnh BC. Dng tch v hng tnh BJ
sao cho AJ v BI vung gc.
Bi 335. Bi 335. Bi 335. Bi 335. Cho hnh thang vung ABCD, hai y AD a, BC b = = , ng cao AB h = . Tm h thc
lin h gia a, b, h
a/ BD CI vi I l trung im ca AB. b/ AC DI .
c/ BM CN vi M, N ln lt theo th t l trung im ca AC v BD.
S: a/
2
h 2ab = . b/
2
h 2ab = . c/
2 2
h 2b ab = .
Bi 336. Bi 336. Bi 336. Bi 336. Cho t gic ABCD.
a/ Chng minh:
2 2 2 2
AB BC CD DA 2AC.DB =
, ,
.
b/ Suy ra iu kin cn v t gic c hai ng cho vung gc l:

2 2 2 2
AB CD BC DA =

Bi 337. Bi 337. Bi 337. Bi 337. Cho ABC vung ti A, gi M l trung im ca BC. Ly cc im B
1
, C
1
trn AB v AC sao
cho
1 1
AB.AB AC.AC = . Chng minh:
1 1
AM B C .
Bi 338. Bi 338. Bi 338. Bi 338. Cho ABC cn nh A, O l tm ng trn ngoi tip ABC, M l trung im ca AB, E l
trng tm ca ACM. Chng minh: OE CM .
Bi 339. Bi 339. Bi 339. Bi 339. Cho ABC cn nh A, O l tm ng trn ngoi tip, gi BB
1
v CC
1
l ng cao ca tam
gic ABC. Chng minh:
1 1
OA B C .
Bi 340. Bi 340. Bi 340. Bi 340. Cho ng trn tm O v mt im P thuc min trong ca ng trn. Qua P, k hai dy AB,
CD vung gc nhau. Gi M l trung im ca dy BD. Chng minh: PM AC .

www.MATHVN.com
www.DeThiThuDaiHoc.com
cng hc tp mn Ton 10 tp I Ths. L Vn on


"Cn c b thng minh" Page - 201 -






















































Dng 2. Chng minh ng thc v tm qu tch im tha biu thc v tch v
hng hay di. S dng tch v hng gii bi ton cc tr.

C CC C Chng minh ng thc tch v hng hay di
Vi cc biu thc v tch v hng, ta s dng nh ngha hoc tch cht ca tch v hng.
Cn c bit lu php phn tch vct bin i (quy tc ba im , , quy tc trung
im, quy tc hnh bnh hnh, ).
Vi cc cng thc v di, ta thng s dng:
2
2
AB AB AB.AB = =
, , ,
. Cn nm vng
cc hnh tnh ca nhng hnh c bn.
C CC C Xc nh im, qu tch im tha mn ng thc v tch v hng hay di
Bi ton: Tm im M tha mn ng thc
( )
f ,
,
v tch v hng hay di.
Ta bin i biu thc ban u v mt trong cc dng sau:
O Dng 1.
2
AM k 0 = , th im M thuc ng trn tm A, bn kinh R k = .
O Dng 2. MA.MB k =
, ,
vi A, B c nh v k khng i. Khi :
Gi I l trung im ca AB, ta c:
( )( ) ( )( )
2 2
k MA.MB MI IA MI IB MI IA MI IA MI IA = = = =
, , , , , , , , , ,
.

2
2 2
AB
IM k IA k
4
= = =
2
2 2
AB
IM k IA k
4
= = = l .
Khi :
Nu 0 < l th M khng tn ti.
Nu 0 = l th M I : l trung im ca AB.
Nu 0 l th M thuc ng trn tm I bn knh R = l .
O Dng 3. MA.MB k =
, ,
vi A, B c nh. Khi :
Gi
o o
M , A theo th t l hnh chiu vung gc ca M, A len BC, ta c:
o o o o
k
k MA.MB M A .BC M A
BC
= = =
, ,
, c gi tr khng i v do A
o
c
nh nn M
o
c nh.
Vy im M thuc ng vung gc vi BC ti M
o
. c bit, khi k 0 = th M
thuc ng thng qua A v vung gc vi BC.
C CC C S dng tch v hng gii bi ton cc tr
t
Phng php: S dng tch v hng bin i biu thc cn tm cc tr v biu thc
di, chng hn nh:

2
S MI c = vi c l hng s v I c nh.

min
S c = t c khi MI 0 M I = .
Lu : Cn nm vng cch tm cc tr phn i s (BT Cauchy, BCS,)

www.MATHVN.com
www.DeThiThuDaiHoc.com
Ths. L Vn on Phn Hnh hc


Page - 202 - "All the flower of tomorrow are in the seeks of today"
Chng minh ng thc tch v hng hay v di
Bi 341. Bi 341. Bi 341. Bi 341. Cho hai im A v B. Gi O l trung im ca AB v M l mt im ty . Chng minh rng:
2 2
MA.MB OM OA =
, ,
.
Bi 342. Bi 342. Bi 342. Bi 342. Cho ABC, gi M l trung im ca BC. Chng minh:
2 2
AB.AC MA MB =
, ,
.
Bi 343. Bi 343. Bi 343. Bi 343. Cho ABC u, cnh a, c ng cao AH v trng tm G. Tnh GB.GC, AB.GC, GC.BH
, , , , , ,
.
Bi 344. Bi 344. Bi 344. Bi 344. Cho ABC v mt im M ty . Chng minh: AB.CM BC.AM CA.BM 0 =
, , , , , ,
.
Bi 345. Bi 345. Bi 345. Bi 345. Chng minh rng:
a/ MA.BC MB.CA MC.AB 0 =
, , , , , ,
vi mi im M, A, B, C. (gi l h thc Euler).
b/
( )
2 2 2
1
AB.AC AB AC BC
2
=
, ,
vi mi im A, B, C.
c/
( )
2 2 2 2
1
MN.PQ MQ NP MP NQ
2
=
, ,
vi mi im M, N, P, Q.
Bi 346. Bi 346. Bi 346. Bi 346. Cho ABC vi ba ng trung tuyn AD, BE, CF.
Chng minh: BC.AD CA.BE AB.CF 0 =
, , , , , ,
.
Bi 347. Bi 347. Bi 347. Bi 347. Cho I l trung im ca on AB, M l mt im ty . Gi H l hnh chiu ca M ln ng
thng AB. Chng minh rng:
a/
( )
2 2
1
MI.AB MB MA
2
=
, ,
. b/
2 2
1
MA.MB MI AB
4
=
, ,
.
c/
2 2 2 2
1
MA MB 2MI AB
2
= . d/
2 2
MA MB 2IH.AB = .
Bi 348. Bi 348. Bi 348. Bi 348. Cho hnh ch nht ABCD v im M ty . Chng minh:
a/ MA.MC MB.MD =
, , , ,
. b/
2 2 2 2
MA MC MB MD = .
Bi 349. Bi 349. Bi 349. Bi 349. Cho hai im M, N nm trn ng trn ng knh AB 2R = . Gi I l giao im ca hai
ng thng AM v BN.
a/ Chng minh: AM.AI AB.AI, BN.BI BA.BI = =
, , , , , , , ,
.
b/ Tnh AM.AI BN.BI
, , , ,
theo R.
Bi 350. Bi 350. Bi 350. Bi 350. Cho ABC c trc tm H, M l trung im ca BC. Chng minh:
2
1
MH.MA BC
4
=
, ,
.
Bi 351. Bi 351. Bi 351. Bi 351. Cho hnh ch nht ABCD, M l mt im bt k. Chng minh:
a/
2 2 2 2
MA MC MB MD = b/ MA.MC MB.MD =
, , , ,

c/
2
MA MB.MD 2MA.MO =
, , , ,
(O l tm ca hnh ch nht).
www.MATHVN.com
www.DeThiThuDaiHoc.com
cng hc tp mn Ton 10 tp I Ths. L Vn on


"Cn c b thng minh" Page - 203 -
Bi 352. Bi 352. Bi 352. Bi 352. Cho ABC c AA', BB', CC' l cc trung tuyn, G l trng tm, M l im ty . Chng
minh rng:
a/ AA'.BC BB'.CA CC'.AB 0 =
, , , , , ,
.
b/ MA'.BC MB'.CA MC'.AB 0 =
, , , , , ,
.
c/
( )
2 2 2 2 2 2
1
MA.MB MB.MC MC.MA MA MB MC AB BC CA
2
=
, , , , , ,
.
d/
( )
2 2 2 2 2 2
1
MA.MB MB.MC MC.MA MA' MB' MC' AB BC CA
4
=
, , , , , ,
.
e/
( )
2 2 2 2 2 2 2 2 2
1
MA MB MC MA' MB' MC' AB BC CA
4
= .
Bi 353. Bi 353. Bi 353. Bi 353. Cho ABC c G l trng tm v M l im ty . Chng minh rng:
a/
( )
2 2 2 2 2 2
1
GA GB GC AB BC CA
3
= .
b/
( )
2 2 2 2 2 2 2
1
MA MB MC 3MG AB BC CA
3
= .
c/
2 2 2 2 2 2 2
MA MB MC 3MG GA GB GC = (ng thc Leibnizt).
Bi 354. Bi 354. Bi 354. Bi 354. Cho ABC, M l trung im ca BC, I l trung im ca AM.
Chng minh rng:
2 2 2 2 2 2 2
2MA MB MC 4MI 2IA IB IC = .
Bi 355. Bi 355. Bi 355. Bi 355. Cho ABC, H l trc tm, M l trung im ca BC, I l trung im AM. Chng minh rng:
a/
2
1
MH.MA BC
4
=
, ,
. b/
2 2 2 2
1
MH MA AH BC
2
= .
Bi 356. Bi 356. Bi 356. Bi 356. Cho ABC u ni tip trong ng trn tam O bn knh R,
( )
M O . Chng minh rng:
a/
2 2 2 2
MA MB MC 6R = . b/
2 2
MA 2MB.MC 3R =
, ,
.
c/ MA MB MC = (M thuc cung nh BC).
Bi 357. Bi 357. Bi 357. Bi 357. Cho ng thng AB ct ng thng d M v mt im C trn d (C khc M). Chng minh
rng d l tip tuyn ca ng trn
( )
ABC khi v ch khi
2
MC MA.MB = .
Bi 358. Bi 358. Bi 358. Bi 358. Cho t gic ABCD. Chng minh:
2 2 2 2
AB CD BC AD 2AC.BD =
, ,
. T suy ra iu
kin cn v t gic c hai ng cho vung gc nhau.
Bi 359. Bi 359. Bi 359. Bi 359. Cho ABC v hai im M, M' bt k. Gi I v I', Hv H', K v K' theo th t l hnh chiu ca
M v M' ln BC, CA, AB. Chng minh: BC.II ' CA.HH' AB.KK' 0 =
, , , , , ,
.
Bi 360. Bi 360. Bi 360. Bi 360. Cho hnh thoi ABCD c cnh a v gc
0
A 60 = . Chng minh rng vi mi im M, ta c
2 2 2 2 2
MA MB MC MD a = .
www.MATHVN.com
www.DeThiThuDaiHoc.com
Ths. L Vn on Phn Hnh hc


Page - 204 - "All the flower of tomorrow are in the seeks of today"
Bi 361. Bi 361. Bi 361. Bi 361. Cho ABC ni tip ng trn
( )
O, R , H l trc tm ca ABC. Chng minh rng:
( )
2 2 2 2 2
OH 9R a b c = vi a, b, c l di tng ng ca ABC.
Bi 362. Bi 362. Bi 362. Bi 362. Cho MM
1
l ng knh bt k ca ng trn tm O, bn knh R v A l mt im c nh,
t OA d = . Gi s AM ct
( )
O ti N.
a/ Chng minh rng tch v hng
1
AM.AM
, ,
c gi tr khng ph thuc vo im M.
b/ Chng minh rng tch v hng AM.AN
, ,
c gi tr khng ph thuc vo v tr im M.
Bi 363. Bi 363. Bi 363. Bi 363. Cho na ng trn ng knh AB, c AC, BD l hai dy cung thuc na ng trn, ct
nhau ti E. Chng minh:
2
AE.AC BE.BD AB =
, , , ,
.
Bi 364. Bi 364. Bi 364. Bi 364. Cho ABC u cnh bng a. Gi M l im ty trn ng trn ngoi tip ABC. Chng
minh rng:
4 4 4 4
MA MB MC 2a = .
Tp hp im v cc tr
Bi 365. Bi 365. Bi 365. Bi 365. Cho ABC c AB 3 = . Tm tp hp im M tha iu kin
a/ MA.MB 6 =
, ,
. b/ AM.AB 8 =
, ,
.
Bi 366. Bi 366. Bi 366. Bi 366. Cho AB a = c trung im I. Tm tp hp im M tha iu kin
a/
2 2 2
2MA MB a = . b/
2 2
2MA MB k = (k cho trc).
c/
2 2 2
MA 3MB a = . d/ MA.MB k =
, ,
.
d/
2
MA MA.MB 0 =
, ,
. e/
2
2MA MA.MB k =
, ,
(k cho trc)
Bi 367. Bi 367. Bi 367. Bi 367. Cho hnh vung ABCD cnh a. Tm tp hp im M sao cho
a/
2
MA.MC MB.MD a =
, , , ,
. b/
2 2 2 2
MA MB MC a = .
c/
( )( )
2
MA 2MB MC MA MC 2a =
, , , , ,
. d/ MA.MB MC.MD k =
, , , ,
(k cho trc).
e/
2
MA.MB MC.MD 5a =
, , , ,
. f/
2 2 2 2
MA MB MC 3MD = .
g/
( )( )
2
MA MB MC MC MB 3a =
, , , , ,
.
Bi 368. Bi 368. Bi 368. Bi 368. Cho ABC. Tm tp hp im M tha iu kin
a/
2 2 2 2
MA MB CA CB 0 = . b/
2 2 2
3MA 2MB MC 0 = .
c/
2 2
2MB MB.MC BC =
, ,
. d/ AM.BC k =
, ,
(k cho trc).
e/
( )( )
MA MB MC MB 0 =
, , , ,
. f/
2 2 2
MA.MB MA.MC MC MB BC =
, , , ,
.
g/ MA.MB AB.MC =
, , , ,
. h/
2
MA MB.MC =
, ,
.
i/
2 2 2 2 2
MA MB MC AB AC = . k/
2
2MA MA.MB MA.MC =
, , , ,
.
www.MATHVN.com
www.DeThiThuDaiHoc.com
cng hc tp mn Ton 10 tp I Ths. L Vn on


"Cn c b thng minh" Page - 205 -
l/
( )( )
MA MB 2MB MC 0 =
, , , ,
. m/
2
2MA MA.MB MA.MC =
, , , ,
.
Bi 369. Bi 369. Bi 369. Bi 369. Cho t gic ABCD, I, J ln lt l trung im ca AB v CD. Tm tp hp im M sao cho:
2
1
MA.MB MC.MD IJ
2
=
, , , ,
.
Bi 370. Bi 370. Bi 370. Bi 370. Cho ABC u cnh a. Tm tp im M tha iu kin
a/
2 2 2 2
MA MB MC 2a = . b/ MA.MC MC.MB =
, , , ,
.
c/
2
MA MA.MB MA.MC 0 =
, , , ,
. d/
2
2
a
MB.MC MA
2
=
, ,
.
e/
2
5a
MA.MB MB.MC MC.MA
2
=
, , , , , ,
. f/
2 2 2
MA 3MB 2MC 0 = .
g/
2 2 2 2
2MA MB MC a = . h/
2
a
MA.MB MB.MC MC.MA
4
=
, , , , , ,
.
Bi 371. Bi 371. Bi 371. Bi 371. Cho hnh bnh hnh ABCD. Bin lun theo k tp hp nhng im tha mn:
2 2 2 2
MA MB MC MD k = .
Bi 372. Bi 372. Bi 372. Bi 372. Cho ABC c G l trng tm v M l im ty .
a/ Chng minh rng: MA.MB MB.CA MC.AB 0 =
, , , , , ,
.
b/ Chng minh rng:
2 2 2 2 2 2 2
MA MB MC GA GB GC 3MG = . T suy ra v
tr ca im M
2 2 2
MA MB MC t gi tr nh nht.
S: M G .
Bi 373. Bi 373. Bi 373. Bi 373. Cho hnh bnh hnh ABCD, tm O, M l im bt k.
a/ Chng minh rng:
( )
2 2 2 2 2 2
MA MB MC MD 2 OB OA = .
b/ Gi s M di ng trn ng trn
( )
D , tm cc v tr ca M
2 2 2
MA MB MC t gi
tr nh nht.
S: M l hnh chiu vung gc ca D ln
( )
D .
Bi 374. Bi 374. Bi 374. Bi 374. Cho ABC u, cnh bng
( )
6 cm . Ly M l mt im thuc ng trn ngoi tip ABC.
t
2 2 2
S MA MB MC = . Tm v tr ca im M S t gi tr nh nht, ln nht ?
S:
2
2
min max
2a
S a , S
3
= = .
Bi 375. Bi 375. Bi 375. Bi 375. Cho ABC, G l trng tm v M l im ty .
a/ Chng minh rng vct v MA MB 2MC =
, , , ,
khng ph thuc vo v tr M.
b/ Gi O l tm ng trn ngoi tip ABC. Chng minh rng:
2 2 2
MA MB 2MC 2MO.v =
, ,
.
www.MATHVN.com
www.DeThiThuDaiHoc.com
Ths. L Vn on Phn Hnh hc


Page - 206 - "All the flower of tomorrow are in the seeks of today"
c/ Gi s M di ng trn ng trn ngoi tip ABC. Tm v tr ca im M
2 2 2
MA MB 2MC t gi tr nh nht, ln nht.
Bi 376. Bi 376. Bi 376. Bi 376. Cho ABC nhn. Tm im M sao cho MA MB MC t gi tr nh nht.
Bi 377. Bi 377. Bi 377. Bi 377. Cho ABC c
0
A 120 = . Tm im M sao cho MA MB MC t gi tr nh nht.
Bi 378. Bi 378. Bi 378. Bi 378. Cho ABC nhn. Tm trn cc ng thng BC, CA, AB cc im X, Y, Z sao cho chu vi
XYZ t gi tr nh nht.
Bi 379. Bi 379. Bi 379. Bi 379. Cho ABC c M l im ty . Tm v tr M trong cc trng hp sau
a/
2 2 2
MA MB MC t gi tr nh nht.
b/ M thuc ng trn ngoi tip ABC v
2 2 2
MA 3MB MC t gi tr ln nht.
Bi 380. Bi 380. Bi 380. Bi 380. Cho hnh vung ABCD cnh a, tm O.
a/ Chng minh rng:
2 2 2 2 2
MA MB MC MD a = M nm trn ng trn ngoi tip
hnh vung ABCD.
b/ Chng minh rng:
( )
2 2 2 2
MA MB MC 3MD 2MO MA MB MC 3MD =
, , , , ,
.
c/ Tm gi tr ln nht v gi tr nh nht ca
2 2 2 2
MA MB MC 3MD khi M di ng
trn ng trn ngoi tip hnh vung ABCD.

www.MATHVN.com
www.DeThiThuDaiHoc.com
cng hc tp mn Ton 10 tp I Ths. L Vn on


"Cn c b thng minh" Page - 207 -
A
B C H




Cho ABC c:
di cc cnh: BC a, CA b, AB c = = = .
di cc ng trung tuyn v t cc nh A, B, C: m
a
, m
b
, m
c

di cc ng cao v t cc nh A, B, C: h
a
, h
b
, h
c

Bn knh ng trn ngoi tip, ni tip tam gic: R, r
Na chu vi tam gic: p
Din tch tam gic: S
1. nh l csin:
2 2 2
2 2 2
2 2 2
a b c 2bc. cos A
b c a 2ca. cos B
c a b 2ab. cos C
'
1
=
1
1
1
=
!
1
1
1 =
1
+
.
2. nh l sin:
a b c
2R
sinA sinB sinC
= = =
3. di trung tuyn:
2 2 2
2
a
2 2 2
2
b
2 2 2
2
c
2(b c ) a
m
4
2(a c ) b
m
4
2(a b ) c
m
4
'
1

1
=
1
1
1
1
1
1
=
!
1
1
1

1
1 =
1
1
1+

4. Din tch tam gic:
a b c
1 1 1
ah bh ch
2 2 2
1 1 1
bc sin A ca sinB absinC
S 2 2 2
abc
pr
4R
p(p a)(p b)(p c)
'
1
1
= = =
1
1
1
1
1
1
= = =
1
=
!
1
1
1
= =
1
1
1
1
1=
1
+

Gii tam gic l tnh cc cnh v cc gc ca tam gic khi bit mt s yu t cho trc.
5. H thc lng trong tam gic vung (nhc li)
Cho ABC vung ti A, AH l ng cao.

2 2 2
BC AB AC = (nh l Pitago)

2
AB BC.BH = ,
2
AC BC.CH =

2
AH BH.CH = ,
2 2 2
1 1 1
AH AB AC
=
AH.BC AB.AC =
b a. sinB a. cosC c tanB ccotC = = = = ; c a. sinC a. cos B btanC bcotC = = = =

C H THC LNG TAM GIC
www.MATHVN.com
www.DeThiThuDaiHoc.com
Ths. L Vn on Phn Hnh hc


Page - 208 - "All the flower of tomorrow are in the seeks of today"
O
M
A
B
C
D
T
R
6. H thc lng trong ng trn (b sung)
Cho ng trn (O; R) v im M c nh.
T M v hai ct tuyn MAB, MCD.
P
M/(O)
=
2 2
MA.MB MC.MD MO R = =
, , , ,

Nu M ngoi ng trn, v tip tuyn MT.
P
M/(O)
=
2 2 2
MT MO R =

BI TP P DNG
Bai 1. Chng minh rng trong mi tam gic ABC ta c;
a) a b. cosC c. cos B = b) sinA sinBcosC sinCcos B =
c)
a
h 2RsinBsinC = d)
2 2 2 2 2 2
a b c
3
m m m (a b c )
4
=
e)
( )
2
2 2
ABC
1
S AB .AC AB.AC
2

=
, ,

Bai 2. Cho tam gic ABC. Chng minh rng:
a) Nu b + c = 2a th
a b c
2 1 1
h h h
= b) Nu bc = a
2
th
2 2
b c a
sinBsinC sin A, h h h = =
c) A vung
2 2 2
b c a
m m 5m =
Bai 3. Cho t gic li ABCD, gi l gc hp bi hai ng chp AC v BD.
a) Chng minh din tch S ca t gic cho bi cng thc:
1
S AC.BD. sin
2
= .
b) Nu kt qu trong trng hp t gic c hai ng cho vung gc.
Bai 4. Cho ABC vung A, BC = a, ng cao AH.
a) Chng minh
2 2
AH a. sinB. cos B, BH a. cos B, CH a. sin B = = = .
b) T suy ra
2 2
AB BC.BH, AH BH.HC = = .
Bai 5. Cho AOB cn nh O, OH v OK l cc ng cao. t OA = a,

AOH = .
a) Tnh cc cnh ca OAK theo a v .
b) Tnh cc cnh ca cc tam gic OHA v AKB theo a v .
c) T tnh sin2 , cos2 , tan2 theo sin , cos , tan .
Bai 6. Gii tam gic ABC, bit:
a)
0 0
c 14; A 60 ; B 40 = = = b)
0 0
b 4, 5; A 30 ; C 75 = = =
c)
0 0
c 35; A 40 ; C 120 = = = d)
0 0
a 137, 5; B 83 ; C 57 = = =
Bai 7. Gii tam gic ABC, bit:
a)
0
a 6, 3; b 6, 3; C 54 = = = b)
0
b 32; c 45; A 87 = = =
c)
0
a 7; b 23; C 130 = = = d)
0
b 14; c 10; A 145 = = =
Bai 8. Gii tam gic ABC, bit:
a) a 14; b 18; c 20 = = = b) a 6; b 7, 3; c 4, 8 = = =
c) a 4; b 5; c 7 = = = d) a 2 3; b 2 2; c 6 2 = = =

www.MATHVN.com
www.DeThiThuDaiHoc.com

You might also like